Вы находитесь на странице: 1из 583

PROFESSIONAL COMPETENCE COURSE

STUDY MATERIAL

COST ACCOUNTING AND FINANCIAL MANAGEMENT


PAPER

Cost Accounting and Financial Management

BOARD OF STUDIES THE INSTITUTE OF CHARTERED ACCOUNTANTS OF INDIA


This study material has been prepared by the faculty of the Board of Studies. The
objective of the study material is to provide teaching material to the students to
enable them to obtain knowledge and skills in the subject. Students should also
supplement their study by reference to the recommended text books. In case
students need any clarifications or have any suggestions to make for further
improvement of the material contained herein, they may write to the Director of
Studies. All care has been taken to provide interpretations and discussions in a
manner useful for the students. However, the study material has not been
specifically discussed by the Council of the Institute or any of its Committees
and the views expressed herein may not be taken to necessarily represent the views
of the Council or any of its Committees. Permission of the Institute is essential
for reproduction of any portion of this material.

The Institute of Chartered Accountants of India

All rights reserved. No part of this book may be reproduced, stored in a retrieval
system, or transmitted, in any form, or by any means, electronic, mechanical,
photocopying, recording, or otherwise, without prior permission, in writing, from
the publisher.

Website : www.icai.org

E-mail : bosnoida@icai.org

Published by Dr. T.P. Ghosh, Director of Studies, ICAI, C-1, Sector-1, NOIDA-
201301 Typeset and designed at Board of Studies, The Institute of Chartered
Accountants of India. Printed at VPS Engineering Impex Pvt. Ltd. Phase � II Noida.
August, 2006, 25,000 copies
PREFACE
The recent surge in globalisation and the massive cross border flow of capital has
increased the significance of Cost Accounting and Financial Management for
management and control purposes. The study of these two important subjects opens
new opportunities for Chartered Accountancy students. It provides them with an
opportunity to draw upon previous experiences and education to apply various
business concepts and analytical tools to complex problems and issues in
organizational settings. This study material provides the basic concepts, theories
and techniques relating to Cost Accounting and Financial Management and aims to
develop the students� ability in understanding the different concepts and their
application in the real life situations. The study material is divided into two
parts. Part I relates to Cost Accounting and Part II deals with Financial
Management. The Cost Accounting portion has ten chapters having an in depth
analysis of concepts relating to Material, Labour, Overheads and other important
costing techniques. Standard Costing, Marginal Costing and Budgeting have been
included in the syllabus at an introductory level. The syllabus has been designed
in such a way that it helps students understand the traditional concepts, their
applications, advantages and disadvantages. Contemporary changes in the subject
shall be dealt with in the Final stage . The portion on Financial Management is
divided into seven chapters. Chapter 1 describes the scope, objectives and
importance of financial management and its relationship with other disciplines.
Time value of money is discussed in Chapter 2. Chapter 3 explains various tools
and techniques of financial management namely Ratio Analysis and Cash Flow
Analysis and their application in practical situations. Chapters 4, 6 and 7 deal
with the theories, concepts and assumptions underlying financial decisions,
namely, Financing, Investment and Working Capital Management. Chapter 5 describes
the various Sources of Finance available to business enterprises to cater their
different types of requirements. The entire study material has been written in a
simple language. A number of self-examination questions are given at the end of
each chapter for practice by students. There are also a number of illustrations in
each chapter to help students to have a better grasp of the subjects.
SYLLABUS
PAPER � 4 : COST ACCOUNTING AND FINANCIAL MANAGEMENT
(One paper - Three hours � 100 Marks) Level of Knowledge: Working knowledge

Part I � Cost Accounting (50 Marks)


Objectives: (a) To understand the basic concepts and processes used to determine
product costs, (b) To be able to interpret cost accounting statements, (c) To be
able to analyse and evaluate information for cost ascertainment, planning, control
and decision making, and (d) To be able to solve simple cases. Contents 1.
Introduction to Cost Accounting (a) Objectives and scope of Cost Accounting (b)
Cost centres and Cost units (c) Cost classification for stock valuation, Profit
measurement, Decision making and control (d) Coding systems (e) Elements of Cost
(f) Cost behaviour pattern, Separating the components of semi-variable costs (g)
Installation of a Costing system (h) Relationship of Cost Accounting, Financial
Accounting, Management Accounting and Financial Management
2.

Cost Ascertainment (a) Material Cost (i) (ii) Procurement procedures - Store
procedures and documentation in respect of receipts and issue of stock, Stock
verification Inventory control - Techniques of fixing of minimum, maximum and
reorder levels, Economic Order Quantity, ABC classification; Stocktaking and
perpetual inventory

(iii) Inventory accounting (iv) Consumption - Identification with products of cost


centres, Basis for consumption entries in financial accounts, Monitoring
consumption. (b) Employee Cost (i) (ii) Attendance and payroll procedures,
Overview of statutory requirements, Overtime, Idle time and Incentives Labour
turnover

(iii) Utilisation of labour, Direct and indirect labour, Charging of labour cost,
Identifying labour hours with work orders or batches or capital jobs (iv)
Efficiency rating procedures (v) Remuneration systems and incentive schemes. (c)
Direct Expenses Sub-contracting - Control on material movements, Identification
with the main product or service. (d) Overheads (i) Functional analysis - Factory,
Administration, Selling, Distribution, Research and Development Behavioural
analysis - Fixed, Variable, Semi variable and Step cost (ii) Factory Overheads -
Primary distribution and secondary distribution, Criteria for choosing suitable
basis for allotment, Capacity cost adjustments, Fixed absorption rates for
absorbing overheads to products or services

(iii) Administration overheads - Method of allocation to cost centres or products


(iv) Selling and distribution overheads - Analysis and absorption of the expenses
in products/customers, impact of marketing strategies, Cost effectiveness of
various methods of sales promotion.
3.

Cost Book- keeping Cost Ledgers - Non-integrated accounts, Integrated accounts,


Reconciliation of cost and financial accounts.

4.

Costing Systems (a) Job Costing Job cost cards and databases, Collecting direct
costs of each job, Attributing overhead costs to jobs, Applications of job
costing. (b) Batch Costing (c) Contract Costing Progress payments, Retention
money, Escalation clause, Contract accounts, Accounting for material, Accounting
for plant used in a contract, Contract profit and Balance sheet entries. (d)
Process Costing Double entry book keeping, Process loss, Abnormal gains and
losses, Equivalent units, Inter-process profit, Joint products and by products.
(e) Operating Costing System

5.

Introduction to Marginal Costing Marginal costing compared with absorption


costing, Contribution, Breakeven analysis and profit volume graph.

6.

Introduction to Standard Costing Various types of standards, Setting of standards,


Basic concepts of material and Labour standards and variance analysis.

Part II � Financial Management (50 Marks)


Objectives: (a) To develop ability to analyse and interpret various tools of
financial analysis and planning, (b) To gain knowledge of management and financing
of working capital, (c) To understand concepts relating to financing and
investment decisions, and (d) To be able to solve simple cases.
Contents 1. Scope and Objectives of Financial Management (a) Meaning, Importance
and Objectives (b) Conflicts in profit versus value maximisation principle (c)
Role of Chief Financial Officer. 2. Time Value of Money Compounding and
Discounting techniques- Concepts of Annuity and Perpetuity. 3. Financial Analysis
and Planning (a) Ratio Analysis for performance evaluation and financial health
(b) Application of Ratio Analysis in decision making (c) Analysis of Cash Flow
Statement. 4. Financing Decisions (a) Cost of Capital - Weighted average cost of
capital and Marginal cost of capital (b) Capital Structure decisions - Capital
structure patterns, Designing optimum capital structure, Constraints, Various
capital structure theories (c) Business Risk and Financial Risk - Operating and
financial leverage, Trading on Equity. 5. Types of Financing (a) Different sources
of finance (b) Project financing - Intermediate and long term financing (c)
Negotiating term loans with banks and financial institutions and appraisal thereof
(d) Introduction to lease financing (e) Venture capital finance. 6. Investment
Decisions (a) Purpose, Objective, Process (b) Understanding different types of
projects
(c) Techniques of Decision making: Non-discounted and Discounted Cash flow
Approaches - Payback Period method, Accounting Rate of Return, Net Present Value,
Internal Rate of Return, Modified Internal Rate of Return, Discounted Payback
Period and Profitability Index (d) Ranking of competing projects, Ranking of
projects with unequal lives. 7. Management of Working Capital (a) Working capital
policies (b) Funds flow analysis (c) Inventory management (d) Receivables
management (e) Payables management (f) Management of cash and marketable
securities (g) Financing of working capital.
CONTENTS
PART I � COST ACCOUNTING
CHAPTER 1 � BASIC CONCEPTS 1. 2. 3. 4. 5. 6. 7. 8. 9. 10. 12. 13. 14. 15. 16.
Evolution of Cost Accounting
......................................................................... 1.1 Cost
Accounting and Inventory Valuation
....................................................... 1.2 Objectives of Cost
Accounting .......................................................................
1.3 Importance of Cost Accounting
...................................................................... 1.5
Advantages of a Cost Accounting system .... ����������������1.7 Essential factors
for. establishing a Cost Accounting system��������..1.8 Relationship between Cost
Accounting, Financial Accounting Management Accounting and Financial Management ..
����������. 1.10 Cost concepts and terms ...................
������������������.1.12 Elements of
cost.....................................................................
������..1.17 Classification of costs
..................................................................................
1.18 Coding systems
..................................................................................
......... 1.26 Types of Costing
..................................................................................
....... 1.27 Methods of
Costing...........................................................................
........... 1.28 Direct expenses
..................................................................................
........ 1.29 Self examination questions
.......................................................................... 1.31

CHAPTER 2 � MATERIAL 1. 2. 3. 4. Introduction


..................................................................................
................ 2.1 Material control
..................................................................................
.......... 2.2 Material procurement procedure
..................................................................... 2.4 Material
storage
..................................................................................
........ 2.14
5. 6. 7. 8. 9. 10. 11. 12.

Inventory
control...........................................................................
............... 2.19 Valuation of material receipts
....................................................................... 2.44
Valuation of material issues
......................................................................... 2.47
Valuation of returns and shortages
............................................................... 2.64 Treatment of
Normal and Abnormal loss of material ...................................... 2.65
Accounting and control: waste,scrap,spoilage & defective
............................. 2.65 Consumption of material
..............................................................................
2.71 Self examination questions
.......................................................................... 2.73

CHAPTER 3 � LABOUR 1 2 3 4 5 6 7. 8. 9. 10. 11. 12 Introduction


..................................................................................
................ 3.1 Labour cost control
..................................................................................
...... 3.1 Attendance & payroll procedures
................................................................... 3.3 Idle
time..............................................................................
........................ 3.11 Overtime
..................................................................................
................... 3.13 Labour turnover
..................................................................................
......... 3.18 Incentive system
..................................................................................
....... 3.25 Labour utilisation
..................................................................................
....... 3.28 System of wage payment and incentive
...................................................... 3.31 Absorption of wages
..................................................................................
.. 3.66 Efficiency rating procedures
........................................................................ 3.74 Self
examination questions
.......................................................................... 3.76

CHAPTER 4 � OVERHEADS 1 2 3 Introduction


..................................................................................
................ 4.1 Classification of
Overheads.........................................................................
... 4.2 Accounting and control of Manufacturing Overheads.
...................................... 4.8

ii
4 5 6 7. 8 9. 10 11

Distribution of overheads
............................................................................. 4.10
Methods of absorbing Overheads
................................................................. 4.35 Treatment
of Overheads in Cost Accounting .................................................
4.46 Accounting and control of Administrative Overhead
....................................... 4.59 Accounting and control of Selling
&Distribution Overhead.............................. 4.64 Concepts related to
Capacity .......................................................................
4.68 Treatment of certain items in Cost Accounting
.............................................. 4.69 Self examination questions
.......................................................................... 4.73

CHAPTER 5 � NON INTEGRATED ACCOUNTS 1. 2. 3. 4. 5. Introduction


..................................................................................
................. 5.1 Non Integrated Accounting System
................................................................ 5.1 Integrated
Accounting System
...................................................................... 5.34
Reconciliation of Cost and Financial Accounts
.............................................. 5.49 Self examination questions
.......................................................................... 5.61

CHAPTER 6 � METHOD OF COSTING(I) 1. 2. 3. 4. 5. 6. Introduction


..................................................................................
................. 6.1 Job Costing
..................................................................................
................. 6.2 Contract Costing
..................................................................................
......... 6.9 Batch
Costing...........................................................................
................... 6.31 Operating Costing
..................................................................................
..... 6.33 Self examination questions
......................................................................... 6. 41

CHAPTER 7 � METHOD OF COSTING (II) 1 2 Meaning of Process


Costing...........................................................................
7.1 Operation Costing
..................................................................................
....... 7.2

iii
3 7 8 9 10

Treatment of Process losses


.......................................................................... 7.4
Costing of Equivalent production units
............................................................ 7.6 Inter Process
Profits.
..................................................................................
. 7.13 Joint products and by
products..................................................................... 7.15
Self examination questions
.......................................................................... 7.31

CHAPTER 8 � STANDARD COSTING 1. 2 3 4 5 6 7 8 9. 10. Introduction


..................................................................................
................. 8.1 Definition of standard cost
............................................................................. 8.3
Setting up of standard cost
............................................................................ 8.3
Types of standards
..................................................................................
...... 8.8 Need for standard
cost..............................................................................
..... 8.9 Process of standard costing
......................................................................... 8.10
Types of variances
..................................................................................
.... 8.11 Accounting procedure for standard cost
........................................................ 8.23 Advantages and
criticisms
........................................................................... 8.26
Self examination questions
.......................................................................... 8.29

CHAPTER 9 � MARGINAL COSTING 1 2 3 4 5 6 7 Introduction


..................................................................................
................. 9.1 Theory of Marginal Costing
............................................................................ 9.2
Ascertainment of Marginal costing
.................................................................. 9.5 Distinction
between Marginal and Absorption Costing...................................... 9.7
Advantages and Limitation of Marginal Costing
............................................. 9.11 Cost Volume Profit Analysis
......................................................................... 9.13
Self Examination Questions
................................................................................

iv
CHAPTER 10 � BUDGETS AND BUDGETARY CONTROL 1 2 3 4 5 6 Introduction
..................................................................................
............... 10.1 Objectives of budgeting
...............................................................................
10.2 Budgetary control
..................................................................................
...... 10.3 Different types of budgets
............................................................................ 10.7
Preparation of budgets
................................................................................
10.8 Self examination questions
.................................................................................

v
CONTENTS
PART II � FINANCIAL MANAGEMENT
CHAPTER 1 � SCOPE AND OBJECTIVES OF FINANCIAL MANAGEMENT 1. 2. 3. 4. 5. 6. 7. 8.
9. Introduction
..................................................................................
................ 1.1 Meaning of Financial Management
................................................................ 1.2 Evolution of
Financial Management
............................................................... 1.4 Importance of
Financial Management ............................................................
1.5 Scope of Financial Management ���������������������.1.5 Objectives of
Financial Management�������������������..1.6 Conflicts in Profit versus Value
Maximisation Principle ����������. 1.10 Role of Chief Financial Officer (CFO)
������������������.1.12 Relationship of Financial Management with Related
Disciplines������..1.13

CHAPTER 2 � TIME VALUE OF MONEY 1. 2. 3. 4. 5. 6. 7. 8. 9. 10. Concept of Time


Value of Money ...................................................................
2.1 Simple Interest
..................................................................................
........... 2.2 Compound Interest
..................................................................................
..... 2.3 Effective Rate of Interest
.............................................................................. 2.8
Present Value
..................................................................................
............. 2.8 Annuity
..................................................................................
..................... 2.10 Perpetuity
..................................................................................
................ 2.12 Sinking Fund
..................................................................................
............. 2.14 Techniques of Discounting
........................................................................... 2.14
Techniques of Compounding
........................................................................ 2.18
CHAPTER 3 � FINANCIAL ANALYSIS AND PLANNING UNIT I : APPLICATION OF RATIO ANALYSIS
FOR PERFORMANCE EVALUATION, FINANCIAL HEALTH AND DECISION MAKING 1.1 1.2 1.3 1.4
1.5 1.6 2.1 2.2 2.3 2.4 2.5 2.6 2.7 2.8 2.9 Introduction
..................................................................................
................ 3.1 Ratio Analysis
..................................................................................
............ 3.1 Types of Ratios
..................................................................................
.......... 3.2 Application of Ratio Analysis in Financial Decision Making
............................ 3.17 Limitations of Financial Ratios
...................................................................... 3.18
Summary of Ratios
..................................................................................
.. 3.19 Introduction
..................................................................................
.............. 3.42 Utility of Cash Flow Analysis
...................................................................... 3.42
Limitations of Cash Flow Analysis
.............................................................. 3.43 Benefits of
Cash Flow Information
............................................................... 3.44 Definitions
..................................................................................
................ 3.44 Cash and Cash Equivalents
........................................................................ 3.44
Presentation of Cash Flow Statement ������������������.3.45 Procedure in
Preparation of Cash Flow Statement ...........................................
3.49 Funds Flow Statement vs. Cash Flow Statement ��������������..3.54

UNIT II : CASH FLOW ANALYSIS

CHAPTER 4 � FINANCING DECISIONS UNIT I : COST OF CAPITAL 1.1 1.2 1.3 1.4 1.5
Introduction
..................................................................................
................ 4.1 Definition of Cost of Capital
.......................................................................... 4.3
Measurement of Cost of Capital .
................................................................... 4.4 Weighted
Average Cost of Capital (WACC) ...................................................
4.17 Marginal Cost of Capital
..............................................................................
4.21

vii
UNIT II : CAPITAL STRUCTURE DECISIONS 2.1 2.2 2.3 2.4 2.5 2.6 2.7 2.8 Meaning of
Capital Structure
....................................................................... 4.26
Choice of Capital Structure
......................................................................... 4.26
Significance of Capital Structure
................................................................. 4.28 Optimal
Capital Structure
............................................................................ 4.30
EBIT-EPS Analysis
..................................................................................
... 4.30 Cost of Capital, Capital Structure and Market Price of Share
........................ 4.33 Capital Structure Theories
.......................................................................... 4.34
Capital Structure and Taxation
.................................................................... 4.46

UNIT III : BUSINESS RISK AND FINANCIAL RISK 3.1 3.2 3.3 Introduction
..................................................................................
............... 4.47 Debt versus Equity Financing
...................................................................... 4.48 Types
of Leverage
..................................................................................
.... 4.50

CHAPTER 5 � TYPES OF FINANCING 1. 2. 3. 4. 5. 6. 7. 8. 9. 10. Introduction


..................................................................................
................. 5.1 Financial Needs and Sources of Finance of a Business
.................................. 5.1 Long Term Sources of Finance
........................................................................ 5.4
Venture Capital Financing
........................................................................... 5.12
Debt Securitisation
..................................................................................
... 5.14 Lease Financing
..................................................................................
...... 5.16 Short Term Sources of Finance
................................................................... 5.16 Other
Sources of Financing
......................................................................... 5.25 New
Instruments
..................................................................................
..... 5.27 International Financing
................................................................................
5.29

viii
CHAPTER 6 � INVESTMENT DECISIONS 1. 2. 3. 4. 5. 6. 7. 8. Introduction
..................................................................................
................. 6.1 Purpose of Capital Budgeting������������������������� 6.2
Capital Budgeting Process
............................................................................ 6.2
Types of capital Investment Decisions
............................................................ 6.3 Project Cash
flows.............................................................................
............ 6.4 Basic Principles for Measuring Project Cash Flows
....................................... 6. 5 Capital Budgeting Techniques
...................................................................... 6. 9
Capital Rationing
..................................................................................
....... 6.18

CHAPTER 7 � MANAGEMENT OF WORKING CAPITAL UNIT I : MEANING, CONCEPT AND POLICIES


OF WORKING CAPITAL 1.1 1.2 1.3 1.7 1.8 1.9 2.1 2.2 2.3 2.4 2.5 2.6 2.7 2.8
Introduction
..................................................................................
................ 7.1 Meaning and Concept of Working Capital
...................................................... 7.1 Management of Working
Capital ................................................................... 7.5
Issues in the Working Capital Management
................................................... 7.6 Estimating Working Capital
Needs ................................................................. 7.9
Operating or Working Capital Cycle
............................................................... 7.9 Treasury
Management: Meaning
................................................................. 7.30 Functions
of Treasury Department
............................................................... 7.30 Management of
Cash
..................................................................................
7.31 Methods of Cash Flow Budgeting
................................................................ 7.34 Cash
Management Models �����������������������.7.48 Recent Developments in Cash
Management ���������������.7.50 Cash Management Services � The ICICI Bank Way
������������7.54 Management of Marketable Securities������������������.7.55

UNIT II : TREASURY AND CASH MANAGEMENT

ix
UNIT III : MANAGEMENT OF INVENTORY 3.1 4.1 4.2 4.3 4.4 4.5 4.6 4.7 4.8 5.1 5.2 5.3
5.4 Inventory Management
................................................................................
7.56 Introduction
..................................................................................
.............. 7.57 Role to be Played by the Finance Manager ����������������7.57
Aspects of Management of Debtors
............................................................ 7.57 Factors
Determining Credit Policy
............................................................... 7.58 Factors under
the Control of the Finance Manager ....................................... 7.59
Financing Receivables
................................................................................
7.67 Innovations in Receivable Management
...................................................... 7.69 Monitoring of
Receivables
.......................................................................... 7.74
Introduction
..................................................................................
.............. 7.76 Sources of Finance���������������������������7.77 Working
Capital Finance from Banks
........................................................... 7.81 Factors
Determining Credit Policy
............................................................... 7.82 UNIT IV :
MANAGEMENT OF RECEIVABLES

UNIT V : FINANCING OF WORKING CAPITAL

x
PART I
COST ACCOUNTING
CHAPTER

BASIC CONCEPTS
Learning Objectives When you have finished studying this chapter, you should be
able to ? Understand the objective and importance of Cost Accounting. ? Understand
the cost accounting terminology. ? Differentiate between cost accounting and
financial accounting ? Understand the relationship between Cost Accounting,
Management Accounting and Financial Management. ? Understand the concept of codes
and the process of codification. ? Understand the various types and methods of
cost accounting. 1.1 EVOLUTION OF COST ACCOUNTING Financial Accounting,

Prior to the industrial revolution, businesses were small and characterised by


simple market exchanges between individuals and organisations. In those times
there was a need of accurate book keeping though not that much of cost accounting.
However, by the seventeenth century in France, the Royal Wallpaper Manufactory had
a Cost Accounting System. Some iron masters and potters in eighteenth century in
England too began to produce Cost Accounting information before the Industrial
Revolution. Subsequently, with the advent of the industrial revolution, large
sized process industries performing single activities (e.g. textiles , railways
etc)came into being. During this period, there was a lack of market for
intermediary products because of which cost information gained importance as a
tool for measuring efficiency of different processes. The period, 1880 AD � 1925
AD saw the development of complex product designs and the emergence of multi
activity diversified corporations like Du Pont, General Motors etc. It was during
this period that scientific management was developed which led accountants to
convert physical standards into cost standards, the latter being used for variance
analysis and control. During World War I and II the social importance of cost
accounting grew with the growth of each country�s defence expenditure. In the
absence of competitive markets for most of the material required to fight war, the
Governments in several countries placed cost-plus contracts
Cost Accounting

under which the price to be paid was the cost of production plus an agreed rate of
profit. The reliance on cost information by the parties to defence contracts
continued after World War II as well. Even today, most of the government contracts
are decided on a cost plus basis. 1.2 COST ACCOUNTING AND INVENTORY VALUATION

The spurt in the industrial growth, as mentioned above, also resulted in the
increased importance of financial accounting and audit (1900 AD onwards).One of
the fundamental issues to be resolved by the accountants during this period was
the measurement of the value of inventory while preparing financial statements.
The valuation had a deep impact over the projected profitability of a company,
which in turn affected the willingness of various stakeholders to inject large
amount of capital in the business. The valuation also directly affected the taxes
which the company was obliged to pay to the government since higher profits meant
higher taxes and vice versa. It was in this context that the need of establishing
rules for inventory valuation was felt. It was then decided that inventory should
be valued at �cost� or �market value� which ever is lower. The term �cost�, being
restricted to the money expended in manufacturing the product till the time the
product was sold. Hence, expenditure incurred in research and development,
distribution, marketing or customer support functions was to be excluded while
computing �costs� for inventory valuation. The computation of the cost incidence
on different types of inventory with different degrees of completion necessitated
the need of accounting for �costs� in order to arrive at the correct values. As
you would have understood by now, �cost accounting� was initiated for
manufacturing organizations and as a field of practice was limited within the
factory premises. However, with the increase in its scope , cost accounting today
is equally important to both manufacturing and service organizations and also does
not restrict itself to inventory valuation alone. It is used in (1)various
decision making scenarios e.g. whether to produce for captive consumption or buy
from outside suppliers,(2) supply of information to the government (cost audit),
(3)planning and control of expenses(variance analysis) , (4)tracking expenses
through a products life cycle (life cycle costing),(5) fixation of selling prices
(cost plus and other approaches) etc. The use of information technology has helped
companies keep /maintain different cost systems for different purposes. Today,
Cost Accounting is popularly known as �Cost and Management Accounting�. Before you
begin your study of Cost Accounting, you must be clear in your mind that you are
going to study a subject, which is immensely useful in all economic activities. It
is a natural instinct with all of us to measure the pros and cons of everything. A
prudent housewife who goes for shopping considers the quality and price of each
product before she buys it. In short, each economic activity, if rationally
viewed, has two aspects - firstly, the costs involved in it
1.2
Basic Concept

and secondly, the benefits obtained out of it. This analysis is technically known
as cost-benefit analysis. It is very important in industrial and commercial
activities. Cost Accounting involves a study of those concepts, tools, and
techniques, which help us in ascertaining and analysing costs. 1.2.1 Definition of
Costing, Cost Accounting and Cost Accountancy: Costing is defined as �the
technique and process of ascertaining costs�. Cost Accounting is defined as "the
process of accounting for cost which begins with the recording of income and
expenditure or the bases on which they are calculated and ends with the
preparation of periodical statements and reports for ascertaining and controlling
costs." Cost Accountancy has been defined as �the application of costing and cost
accounting principles, methods and techniques to the science, art and practice of
cost control and the ascertainment of profitability. It includes the presentation
of information derived there from for the purpose of managerial decision making.�
1.3 (i) (ii) OBJECTIVES OF COST ACCOUNTING Ascertainment of cost. Determination of
selling price.

The main objectives of Cost Accounting are as follows :

(iii) Cost control and cost reduction. (iv) Ascertaining the profit of each
activity. (v) Assisting management in decision-making. 1.3.1 Ascertainment of
Cost: There are two methods of ascertaining costs, viz., Post Costing and
Continuous Costing. Post Costing means, analysis of actual information as recorded
in financial books. It is accurate and is useful in the case of �Cost plus
Contracts� where price is to be determined finally on the basis of actual cost.
Continuous Costing, aims at collecting information about cost as and when the
activity takes place so that as soon as a job is completed the cost of completion
would be known. This involves careful estimates being prepared of overheads. In
order to be of any use, costing must be a continuous process. Cost ascertained by
the above two methods may be compared with the standard costs which are the target
figures already compiled on the basis of experience and experiments.

1.3
Cost Accounting

1.3.2 Determination of selling price: Though the selling price of a product is


influenced by market conditions, which are beyond the control of any business, it
is still possible to determine the selling price within the market constraints.
For this purpose, it is necessary to rely upon cost data supplied by Cost
Accountants. 1.3.3 Cost control and cost reduction: as �The guidance and
regulation, by executive action of the cost of operating an undertaking�. The word
�guidance� indicates a goal or target to be guided; �regulation� indicates taking
action where there is a deviation from what is laid down; executive action denotes
action to �regulate� must be initiated by executives i.e. persons responsible for
carrying out the job or the operation; and all this is to be exercised through
modern methods of costing in respect of expenses incurred in operating an
undertaking. To exercise cost control, broadly speaking the following steps should
be observed: (i) (ii) Determine clearly the objective, i.e., pre-determine the
desired results; Measure the actual performance;

(iii) Investigate into the causes of failure to perform according to plan; and
(iv) Institute corrective action. The target cost and/or targets of performance
should be laid down in respect of each department or operation and these targets
should be related to individuals who, by their action, control the actual and
bring them into line with the targets. Actual cost of performance should be
measured in the same manner in which the targets are set up, i.e. if the targets
are set up operation-wise, then the actual costs should also be collected
operation-wise and not cost centre or department-wise as this would make
comparison difficult. Cost Reduction, may be defined "as the achievement of real
and permanent reduction in the unit cost of goods manufactured or services
rendered without impairing their suitability for the use intended or diminution in
the quality of the product." Cost reduction should not be confused with Cost
control. Cost saving could be a temporary affair and may be at the cost of
quality. Cost reduction implies the retention of the essential characteristics and
quality of the product and thus it must be confined to permanent and genuine
savings in the cost of manufacture, administration, distribution and selling,
brought about by elimination of wasteful and inessential elements from the design
of the product and from the techniques carried out in connection therewith. In
other words, the essential characteristics and quality of the products are
retained through improved methods and techniques and thereby a permanent reduction
in unit cost is achieved. The definition of cost reduction does not, however,
include reduction in expenditure arising from reduction in taxation or similar
Government action or the effect of price agreements.

1.4
Basic Concept

The three-fold assumptions involved in the definition of cost reduction may be


summarised as under : (a) There is a saving in unit cost. (b) Such saving is of
permanent nature. (c) The utility and quality of the goods and services remain
unaffected, if not improved. 1.3.4 Ascertaining the profit of each activity : The
profit of any activity can be ascertained by matching cost with the revenue of
that activity. The purpose under this step is to determine costing profit or loss
of any activity on an objective basis. 1.3.5 Assisting management in decision
making : Decision making is defined as a process of selecting a course of action
out of two or more alternative courses. For making a choice between different
courses of action, it is necessary to make a comparison of the outcomes, which may
be arrived under different alternatives. Such a comparison has only been made
possible with the help of Cost Accounting information. 1.4 IMPORTANCE OF COST
ACCOUNTING TO BUSINESS CONCERNS

Management of business concerns expects from Cost Accounting a detailed cost


information in respect of its operations to equip their executives with relevant
information required for planning, scheduling, controlling and decision making. To
be more specific, management expects from cost accounting - information and
reports to help them in the discharge of the following functions : (a) Control of
material cost : Cost of material usually constitute a substantial portion of the
total cost of a product. Therefore, it is necessary to control it as far as
possible. Such a control may be exercised by (i) Ensuring un-interrupted supply of
material and spares for production. (ii) By avoiding excessive locking up of
funds/capital in stocks of materials and stores. (iii) Also by the use of
techniques like value analysis, standardisation etc. to control material cost. (b)
Control of labour cost : It can be controlled if workers complete their work
within the standard time limit. Reduction of labour turnover and idle time too
help us, to control labour cost. (c) Control of overheads : Overheads consists of
indirect expenses which are incurred in the factory, office and sales department ;
they are part of production and sales cost. Such expenses may be controlled by
keeping a strict check over them. (d) Measuring efficiency : For measuring
efficiency, Cost Accounting department should provide information about standards
and actual performance of the concerned activity. (e) Budgeting : Now�a�days
detailed estimates in terms of quantities and amounts are drawn up before the
start of each activity. This is done to ensure that a practicable course of

1.5
Cost Accounting

action can be chalked out and the actual performance corresponds with the
estimated or budgeted performance. The preparation of the budget is the function
of Costing Department. (f) Price determination: Cost accounts should provide
information, which enables the management to fix remunerative selling prices for
various items of products and services in different circumstances. (g) Curtailment
of loss during the off-season: Cost Accounting can also provide information, which
may enable reduction of overhead, by utilising idle capacity during the offseason
or by lengthening the season. (h) Expansion: Cost Accounts may provide estimates
of production of various levels on the basis of which the management may be able
to formulate its approach to expansion. (i) Arriving at decisions: Most of the
decisions in a business undertaking involve correct statements of the likely
effect on profits. Cost Accounts are of vital help in this respect. In fact,
without proper cost accounting, decision would be like taking a jump in the dark,
such as when production of a product is stopped. 1.5 VARIOUS REPORTS PROVIDED BY
COST ACCOUNTING DEPARTMENT

Following reports may be provided by a Cost Accounting Department for the use of
its executives: (a) Cost sheets setting out the total cost, analysed into various
elements, giving comparative figures for the previous period and for other plants
under the same management. (b) Consumption of material statements, showing total
quantity of materials issued for production, materials actually embodied in
production and wastage. (c) Labour utilisation statements providing details about
the total number of hours paid for, standard hours for the output, idle time (and
amount involved) and causes thereof. (d) Overheads incurred compared with budgets;
overheads actually charged to production and the difference between the amount
actually incurred and the amount so charged. (e) Sales effected compared with
budgets, showing the difference between the two because of quality being different
from those taken into account while budgeting. (f) Reconciliation of actual profit
earned with estimated or budgeted profit. (g) The total cost of abnormally spoiled
work in the factory and abnormal losses in the store. (h) The total cost of
inventory carried, analysed into raw materials in chief stores and other stores.
The number of months for which stocks would be sufficient (on the basis of average
consumption being worked out).
1.6
Basic Concept

(i)

Labour turnover, and the cost of recruitment and training of new employees.

(j) Expenses incurred on Research and Development as compared with the budgeted
amount. Reports about particular departments and operations (like transport or
power generation) may also be compiled and submitted to the departmental manager
concerned. 1.6 1. 2. ADVANTAGES OF A COST ACCOUNTING SYSTEM A good Cost Accounting
System helps in identifying unprofitable activities, losses or inefficiencies in
any form. The application of cost reduction techniques, operations research
techniques and value analysis technique, helps in achieving the objective of
economy in concern�s operations. Continuous efforts are being made by the business
organisation for finding new and improved methods for reducing costs. Cost
Accounting is useful for identifying the exact causes for decrease or increase in
the profit/loss of the business. It also helps in identifying unprofitable
products or product lines so that these may be eliminated or alternative measures
may be taken. It provides information and data to the management to serve as
guides in making decisions involving financial considerations. Guidance may also
be given by the Cost Accountant on a host of problems such as, whether to purchase
or manufacture a given component, whether to accept orders below cost, which
machine to purchase when a number of choices are available. Cost Accounting is
quite useful for price fixation. It serves as a guide to test the adequacy of
selling prices. The price determined may be useful for preparing estimates or
filling tenders. The use of cost accounting technique viz., variance analysis,
points out the deviations from the pre-determined level and thus demands suitable
action to eliminate such deviations in future. Cost comparison helps in cost
control. Such a comparison may be made from period to period by using the figures
in respect of the same unit of firms or of several units in an industry by
employing uniform costing and inter-firm comparison methods. Comparison may be
made in respect of costs of jobs, processes or cost centres. A system of costing
provides figures for the use of Government, Wage Tribunals and other bodies for
dealing with a variety of problems. Some such problems include price fixation,
price control, tariff protection, wage level fixation, etc.

Important advantages of a Cost Accounting System may be listed as below :

3.

4.

5.

6.

7.

8.

1.7
Cost Accounting

9.

The cost of idle capacity can be easily worked out, when a concern is not working
to full capacity.

10. The use of Marginal Costing technique, may help the executives in taking short
term decisions. This technique of costing is highly useful during the period of
trade depression, as the orders may have to be accepted during this period at a
price less than the total cost. 11. The marginal cost has linear relationship with
production volume and hence in formulating and solving �Linear Programming
Problems�, marginal cost is useful. 1.7 ESSENTIAL FACTORS FOR INSTALLING A COST
ACCOUNTING SYSTEM

As in the case of every other form of activity, it should be considered whether it


would be profitable to have a cost accounting system. The benefits from such a
system must exceed the amount to be spent on it. This would depend upon many
factors including the nature of the business and the quality of the management.
Management, which is prone to making decisions on the basis of pre-conceived
notions without taking into account the information and data placed before it,
cannot derive much benefit from a costing system. On the other hand management,
which is in the habit of studying information thoroughly before making decisions,
would require cost accounting system. Before setting up a system of cost
accounting the under mentioned factors should be studied: (i) (ii) The objective
of costing system, for example whether it is being introduced for fixing prices or
for insisting a system of cost control. The areas of operation of business wherein
the managements� action will be most beneficial. For instance, in a concern, which
is anxious to expand its operations, increase in production would require maximum
attention. On the other hand for a concern, which is not able, to sell the whole
of its production the selling effort would require greater attention. The system
of costing in each case should be designed to highlight, in significant areas,
factors considered important for improving the efficiency of operations in that
area.

(iii) The general organisation of the business, with a view of finding out the
manner in which the system of cost control could be introduced without altering or
extending the organisation appreciably. (iv) The technical aspects of the concern
and the attitude and behaviour that will be successful in winning sympathetic
assistance or support of the supervisory staff and workmen. (v) The manner in
which different variable expenses would be affected with expansion or cessation of
different operations.
1.8
Basic Concept

(vi) The manner in which Cost and Financial accounts could be inter-locked into a
single integral accounting system and in which results of separate sets of
accounts, cost and financial, could be reconciled by means of control accounts.
(vii) The maximum amount of information that would be sufficient and how the same
should be secured without too much clerical labour, especially the possibility of
collection of data on a separate printed form designed for each process; also the
possibility of instruction as regards filling up of the forms in writing to ensure
that these would be faithfully carried out. (viii) How the accuracy of the data
collected can be verified? Who should be made responsible for making such
verification in regard to each operation and the form of certificate that he
should give to indicate the verification that he has carried out ? (ix) The manner
in which the benefits of introducing Cost Accounting could be explained to various
persons in the concern, specially those in charge of production department and
awareness created for the necessity of promptitude, frequency and regularity in
collection of costing data. 1.7.1 Essentials of a good Cost Accounting System: The
essential features, which a good Cost Accounting System should possess, are as
follows: (i) (ii) Cost Accounting System should be tailor-made, practical, simple
and capable of meeting the requirements of a business concern. The data to be used
by the Cost Accounting System should be accurate; otherwise it may distort the
output of the system.

(iii) Necessary cooperation and participation of executives from various


departments of the concern is essential for developing a good system of Cost
Accounting. (iv) The Cost of installing and operating the system should justify
the results. (v) The system of costing should not sacrifice the utility by
introducing meticulous and unnecessary details. (vi) A carefully phased programme
should be prepared by using network analysis for the introduction of the system.
(vii) Management should have a faith in the Costing System and should also provide
a helping hand for its development and success.

1.9
Cost Accounting

1.8 RELATIONSHIP BETWEEN COST ACCOUNTING, FINANCIAL ACCOUNTING, MANAGEMENT


ACCOUNTING AND FINANCIAL MANAGEMENT Cost Accounting is a branch of accounting,
which has been developed because of the limitations of Financial Accounting from
the point of view of management control and internal reporting. Financial
accounting performs admirably, the function of portraying a true and fair overall
picture of the results or activities carried on by an enterprise during a period
and its financial position at the end of the year. Also, on the basis of financial
accounting, effective control can be exercised on the property and assets of the
enterprise to ensure that they are not misused or misappropriated. To that extent
financial accounting helps to assess the overall progress of a concern, its
strength and weaknesses by providing the figures relating to several previous
years. Data provided by Cost and Financial Accounting is further used for the
management of all processes associated with the efficient acquisition and
deployment of short, medium and long term financial resources. Such a process of
management is known as Financial Management. The objective of Financial Management
is to maximise the wealth of shareholders by taking effective Investment,
Financing and Dividend decisions. Investment decisions relate to the effective
deployment of scarce resources in terms of funds while the Financing decisions are
concerned with acquiring optimum finance for attaining financial objectives. The
last and very important �Dividend decision� relates to the determination of the
amount and frequency of cash which can be paid out of profits to shareholders. On
the other hand, Management Accounting refers to managerial processes and
technologies that are focused on adding value to organisations by attaining the
effective use of resources, in dynamic and competitive contexts. Hence, Management
Accounting is a distinctive form of resource management which facilitates
management�s �decision making� by producing information for managers within an
organisation. 1.8.1 Limitations of Financial Accounting: There are, however,
serious limitations of financial accounting from the point of view of management.
These stem from the fact that management must have information properly analysed
and continuously flowing to it. Management cannot be satisfied with a broad
picture, and that too available at the end of a period. The limitations of
financial accounts together with procedures that overcome the limitations are
given below :

1.10
Basic Concept

Limitations A. Forecasting and Planning Management requires information so that it


can make effective plans for the coming year and the period after that. In other
words, information about the future is required. Financial Accounts, provide this
information.

Procedures that overcome limitations The technique of budgeting has been evolved.
Starting with the assessment of the limiting factors that may be operating,
careful estimates can be prepared of all the activities that will be undertaken
and these then can cannot be translated in terms of money. (An analysis of cost
into fixed and variable is most useful in this respect). These �estimates�,
properly coordinated, become budgets and plans of action. It is the complete
analysis of cost incurred that can help management in making the type of decisions
mentioned. The analysis has to be two-fold :

B. Decision making Management requires information daily for making decision of


any type. Financial Accounts normally are not able to provide information for this
purpose. Information is required to answer the under mentioned questions amongst
others : (i) What should be the product under normal circumstances and under
special circumstances ? Should a part be produced in the factory itself or bought
from the market ?

(i)

The total cost of each job, product, process etc. as to be ascertained; and The
cost must be further analysed as fixed and variable.

(ii)

(ii)

(iii) Should the production of a product be given up ?

(iii) In other words, management should have information to see what the effect on
the revenues and cost (both) will be if a proposed course of action is taken. This
is the marginal costing technique and most problems can be handled with the aid of
this technique.

(iv) What should be the priority accorded to a product ?

1.11
Cost Accounting

(v) Should investment be made in a new project ? (vi) How much should be produced
to earn a certain profit, given the selling price? C. Control and assessment
Management requires information to assess the performance of various persons and
departments and to see that costs do not exceed a reasonable limit for a given
quantum of work of the requisite quality. Financial accounts cannot provide
information for this purpose. Apart from the generality of the above, management
requires to know : (i) (ii) the profitability of each product ; and the extent of
unnecessary material, labour, facilities etc. The techniques of budgeting and
standard costing enable management to perform this function which is one of most
important one. In this case also the main activity is analysis and comparison. If
standard costing is not adopted, simple comparison of figures over the periods for
each element of cost in terms of quantity if possible is of great help.
Profitability of each waste product can be easily established by comparing its
selling price with the contribution it makes i.e., the difference between the
selling price and its variable cost.

It will be seen from the above that the chief limitation of financial accounting
is lack of analysis of information and absence of measuring rods. Cost accounting
with the aid of budgeting, standard costing and marginal costing has filled the
need in this respect. 1.9 COST CONCEPTS AND TERMS

1.9.1 Cost (a) The amount of expenditure (actual or notional) incurred on or


attributable to a specified article, product or activity. (here the word cost is
used as a noun) (b) To ascertain the cost of a given thing. (here the word cost is
used as a verb) NOTE : The word �Cost� can rarely stand on its own and should be
qualified as to its limitation (e.g., historical, variable etc.) and related to a
particular thing or �object of thought� e.g., a given quantity or unit of goods
made or services performed. 1.9.2 Cost object � Anything for which a separate
measurement of cost is desired. Examples of cost objects include a product, a
service , a project , a customer , a brand category , an activity , a department ,
a programme. 1.9.3 Direct costs � Costs that are related to the cost object and
can be traced in an economically feasible way.
1.12
Basic Concept

1.9.4 Indirect costs � Costs that are related to the cost object but cannot be
traced to it in an economically feasible way. 1.9.5 Pre-determined - A cost which
is computed in advance before production or operations start, on the basis of
specification of all the factors affecting cost, is known as a predetermined cost.
1.9.6 Standard Cost - A pre-determined cost, which is calculated from managements
�expected standard of efficient operation� and the relevant necessary expenditure.
It may be used as a basis for price fixing and for cost control through variance
analysis. 1.9.7 Marginal Cost - The amount at any given volume of output by which
aggregate costs are changed if the volume of output is increased or decreased by
one unit. Note : In this context a unit may be a single article, an order, a stage
of production, a process of a department. It relates to change in output in the
particular circumstances under consideration within the capacity of the concerned
organisation. 1.9.8 Cost of Sales - The cost which is attributable to the sales
made. Note: It is not uncommon to use this in a restricted sense as the production
cost of goods sold. 1.9.9 Total Cost - The sum of all costs attributable to the
cost object under consideration. 1.9.10 Cost Centre - It is defined as a location,
person or an item of equipment (or group of these) for which cost may be
ascertained and used for the purpose of Cost Control. Cost Centres are of two
types, viz., Personal and Impersonal. A Personal cost centre consists of a person
or group of persons and an Impersonal cost centre consists of a location or an
item of equipment (or group of these). In a manufacturing concern there are two
main types of Cost Centres as indicated below : (i) Production Cost Centre : It is
a cost centre where raw material is handled for conversion into finished product.
Here both direct and indirect expenses are incurred. Machine shops, welding shops
and assembly shops are examples of production Cost Centres. Service Cost Centre :
It is a cost centre which serves as an ancillary unit to a production cost centre.
Power house, gas production shop, material service centres, plant maintenance
centres are examples of service cost centres.

(ii)

1.9.11 Cost unit - It is a unit of product, service or time (or combination of


these) in relation to which costs may be ascertained or expressed. We may for
instance determine the cost per tonne of steel, per tonne kilometre of a transport
service or cost per machine hour. Sometime, a single order or a contract
constitutes a cost unit. A batch which consists of a group of

1.13
Cost Accounting

identical items and maintains its identity through one or more stages of
production may also be considered as a cost unit. Cost units are usually the units
of physical measurement like number, weight, area, volume, length, time and value.
A few typical examples of cost units are given below : Industry or Product
Automobile Cement Chemicals Power Steel Cost Unit Basis -Number -Tonne/per bag
etc. -Litre, gallon, kilogram, tonne etc. -Kilo-watt hour -Tonne

Transport -Passenger kilometre 1.9.12 Responsibility Centre - It is defined as an


activity centre of a business organisation entrusted with a special task. Under
modern budgeting and control, financial executives tend to develop responsibility
centres for the purpose of control. Responsibility centres can broadly be
classified into three categories. They are : (a) Cost Centres ; (b) Profit Centres
; and (c) Investment Centres ; 1.9.13 Profit Centres - Centres which have the
responsibility of generating and maximising profits are called Profit Centres.
1.9.14 Investment Centres - Those centres which are concerned with earning an
adequate return on investment are called Investment Centres. 1.9.15 Cost
allocation - It is defined as the assignment of the indirect costs to the chosen
cost object. 1.9.16 Cost absorption - It is defined as the process of absorbing
all indirect costs allocated to or apportioned over a particular cost centre or
production department by the units produced. Hence, while allocating, the relevant
cost objects would be the concerned cost centre or the concerned department,
while, the process of absorption would consider the units produced as the relevant
cost object. For example, the overhead costs of a lathe centre may be absorbed by
using a rate per lathe hour. Cost absorption can take place only after cost
allocation. In other words, the overhead costs are either allocated or apportioned
over different cost centres and afterwards they are absorbed on equitable basis by
the output of the same cost centres.
1.14
Basic Concept

1.9.17 Estimated cost - Kohler defines estimated cost as �the expected cost of
manufacture, or acquisition, often in terms of a unit of product computed on the
basis of information available in advance of actual production or purchase�.
Estimated cost are prospective costs since they refer to prediction of costs.
1.9.18 Differential cost - (Incremental and decremental costs). It represents the
change (increase or decrease) in total cost (variable as well as fixed) due to
change in activity level, technology, process or method of production, etc. For
example if any change is proposed in the existing level or in the existing method
of production, the increase or decrease in total cost or in specific elements of
cost as a result of this decision will be known as incremental cost or decremental
cost. 1.9.19 Imputed costs - These costs are notional costs which do not involve
any cash outlay. Interest on capital, the payment for which is not actually made,
is an example of imputed cost. These costs are similar to opportunity costs.
1.9.20 Capitalised costs � These are costs which are initially recorded as assets
and subsequently treated as expenses. 1.9.21 Product costs - These are the costs
which are associated with the purchase and sale of goods (in the case of
merchandise inventory). In the production scenario, such costs are associated with
the acquisition and conversion of materials and all other manufacturing inputs
into finished product for sale. Hence, under marginal costing, variable
manufacturing costs and under absorption costing, total manufacturing costs
(variable and fixed) constitute inventoriable or product costs. Under the Indian
GAAP, product costs will be those costs which are allowed to be a part of the
value of inventory as per Accounting Standard 2, issued by the Council of the
Institute of Chartered Accountants of India. 1.9.22 Opportunity cost - This cost
refers to the value of sacrifice made or benefit of opportunity foregone in
accepting an alternative course of action. For example, a firm financing its
expansion plan by withdrawing money from its bank deposits. In such a case the
loss of interest on the bank deposit is the opportunity cost for carrying out the
expansion plan. 1.9.23 Out-of-pocket cost - It is that portion of total cost,
which involves cash outflow. This cost concept is a short-run concept and is used
in decisions relating to fixation of selling price in recession, make or buy, etc.
Out�of�pocket costs can be avoided or saved if a particular proposal under
consideration is not accepted. 1.9.24 Shut down costs - Those costs, which
continue to be, incurred even when a plant is temporarily shutdown, e.g. rent,
rates, depreciation, etc. These costs cannot be eliminated with the closure of the
plant. In other words, all fixed costs, which cannot be avoided during the
temporary closure of a plant, will be known as shut down costs.

1.15
Cost Accounting

1.9.25 Sunk costs - Historical costs incurred in the past are known as sunk costs.
They play no role in decision making in the current period. For example, in the
case of a decision relating to the replacement of a machine, the written down
value of the existing machine is a sunk cost and therefore, not considered. 1.9.26
Absolute cost - These costs refer to the cost of any product, process or unit in
its totality. When costs are presented in a statement form, various cost
components may be shown in absolute amount or as a percentage of total cost or as
per unit cost or all together. Here the costs depicted in absolute amount may be
called absolute costs and are base costs on which further analysis and decisions
are based. 1.9.27 Discretionary costs � Such costs are not tied to a clear cause
and effect relationship between inputs and outputs. They usually arise from
periodic decisions regarding the maximum outlay to be incurred. Examples include
advertising, public relations, executive training etc. 1.9.28 Period costs - These
are the costs, which are not assigned to the products but are charged as expenses
against the revenue of the period in which they are incurred. All nonmanufacturing
costs such as general and administrative expenses, selling and distribution
expenses are recognised as period costs. 1.9.29 Engineered costs - These are costs
that result specifically from a clear cause and effect relationship between inputs
and outputs. The relationship is usually personally observable. Examples of inputs
are direct material costs, direct labour costs etc. Examples of output are cars,
computers etc. 1.9.30 Explicit Costs - These costs are also known as out of pocket
costs and refer to costs involving immediate payment of cash. Salaries, wages,
postage and telegram, printing and stationery, interest on loan etc. are some
examples of explicit costs involving immediate cash payment. 1.9.31 Implicit Costs
- These costs do not involve any immediate cash payment. They are not recorded in
the books of account. They are also know as economic costs.

1.16
Basic Concept

1.10 ELEMENTS OF COST A diagram as given below shows the elements of cost
described as under : ELEMENTS OF COST
MATERIALS COST LABOUR COST OTHER EXPENSES

DIRECT MATERIALS COST

INDIRECT MATERIALS COST

DIRECT LABOUR COST

INDIRECT LABOUR COST

DIRECT EXPENSES

INDIRECT EXPENSES

OVERHEADS

PRODUCTION OR WORKS OVERHEADS

ADMINISTRATION OVERHEADS

SELLING OVERHEADS

DISTRIBUTION OVERHEADS

1.10.1 Direct materials : Materials which are present in the finished product(cost
object) or can be economically identified in the product are called direct
materials. For example, cloth in dress making; materials purchased for a specific
job etc. Note: However in some cases a material may be direct but it is treated as
indirect, because it is used in small quantities and it is not economically
feasible to identify that quantity. 1.10.2 Direct labour : Labour which can be
economically identified or attributed wholly to a cost object is called direct
labour. For example, labour engaged on the actual production of the product or in
carrying out the necessary operations for converting the raw materials into
finished product. 1.10.3 Direct expenses : It includes all expenses other than
direct material or direct labour which are specially incurred for a particular
cost object and can be identified in an economically feasible way. 1.10.4 Indirect
materials : Materials which do not normally form part of the finished product
(cost object) are known as indirect materials. These are � Stores used for
maintaining machines and buildings (lubricants, cotton waste, bricks

1.17
Cost Accounting

etc.) Stores used by service departments like power house, boiler house, canteen
etc. 1.10.5 Indirect labour : Labour costs which cannot be allocated but can be
apportioned to or absorbed by cost units or cost centres is known as indirect
labour. Examples of indirect labour includes - charge hands and supervisors;
maintenance workers; etc. 1.10.6 Indirect expenses : Expenses other than direct
expenses are known as indirect expenses. Factory rent and rates, insurance of
plant and machinery, power, light, heating, repairing, telephone etc., are some
examples of indirect expenses. 1.10.7 Overheads : It is the aggregate of indirect
material costs, indirect labour costs and indirect expenses. The main groups into
which overheads may be subdivided are the following : (i) Production or Works
overheads (ii) Administration overheads (iii) Selling overheads 1.11
CLASSIFICATION OF COSTS It means the grouping of costs according to their common
characteristics. The important ways of classification of costs are : (1) By nature
or element (3) As direct and indirect (5) By controllability (2) By functions (4)
By variability (6) By normality (iv) Distribution overheads

1.11.1 By Nature of Element - Under this classification the costs are divided into
three categories i.e., materials cost, labour cost and expenses. This type of
classification is useful to determine the total cost. 1.11.2 By Functions - Under
this classification, costs are divided according to the function for which they
have been incurred. Some of the examples are : Production cost - The cost of
sequence of operations which begins with supplying materials, labour and services
and ends with primary packing of the product. Selling cost - The cost seeking to
create and stimulate demand (sometimes termed �marketing�) and of securing orders.
Distribution cost - The cost of the sequence of operations which begins with
making the packed product available for despatch and ends with making the
reconditioned returned empty package, if any available for re-use.
1.18
Basic Concept

Note - It also includes expenditure incurred in transporting articles to central


or local storage. Distribution costs include expenditure incurred in moving
articles to and from prospective customers as in case of goods on sale or return
basis. In the gas, electricity and water industry distribution means pipes, mains
and services which may be regarded as the equivalent of packing and
transportation. Administrative cost - The cost of formulating the policy,
directing the organisation and controlling the operations of an undertaking which
is not related directly to a production, selling and distribution, research or
development activity or function. Research cost - The cost of researching for new
or improved products, new applications of materials, or improved methods.
Development cost - The cost of the process which begins with the implementation of
the decision to produce a new or improved product or to employ a new or improved
method and ends with commencement of formal production of that product or by that
method. Pre-production cost - The part of development cost incurred in making a
trial production run preliminary to formal product. Note - This term is sometimes
used to cover all activities prior to production including research and
development, but in such cases the usage should be made clear in the context.
Conversion cost - The sum of direct wages, direct expenses and overhead cost of
converting raw materials to the finished stage or converting a material from one
stage of production to the next. Note - In some circumstances this phrase is used
to include any excess material cost or loss of material incurred at the particular
stage of production. Product costs � Please refer to 1.9.21 Purposes for computing
product costs : The three different purposes for computing product costs are as
follows : (i) (ii) Preparation of financial statements: Here focus is on
inventoriable costs for complying with Accounting Standard �2 , issued by the
Council of ICAI. Product pricing: It is an important purpose for which product
costs are used. For this purpose, the cost of the other areas of the value chain
should be included to make the product available to the customer.

(iii) Contracting with government agencies: Normally such contracts are on a cost
plus basis. For this purpose government agencies may not allow the contractors to
recover research and development and marketing costs under cost plus contracts.
1.11.3 As Direct and Indirect � Please refer to 1.9.3 and 1.9.4

1.19
Cost Accounting

1.11.4 By Variability - According to this classification costs are classified into


three groups viz., fixed, variable and semi-variable. (a) Fixed costs - These are
the costs which are incurred for a period, and which, within certain output and
turnover limits, tend to be unaffected by fluctuations in the levels of activity
(output or turnover). They do not tend to increase or decrease with the changes in
output. For example, rent, insurance of factory building etc., remain the same for
different levels of production. A fixed cost can be depicted graphically as
follows, Total Fixed Cost Rs. 1,000 Activity Level The graph shows that the cost
is constant (Rs.1,000) for all activity levels. However, it should be noted that
this is only true for a relevant range of activity. Fixed costs tend to change
beyond the relevant range. Such cost behaviour pattern is described as a stepped
fixed cost:

Total Fixed Cost (Rs.)

Relevant Relevant Range 1 Range 2

Relevant Range 3

Activity Level (b) Variable costs - These costs tend to vary with the volume of
activity. Any increase in the activity results in an increase in the variable cost
and vice-versa. For example, cost of direct labour, etc. Variable costs are
depicted graphically as follows, Total variable Cost (Rs.)

Activity Level
1.20
Basic Concept

(c) Semi-variable costs - These costs contain both fixed and variable components
and are thus partly affected by fluctuations in the level of activity. Examples of
semi variable costs are telephone bills, gas and electricity etc. Such costs are
depicted graphically as follows: Total Cost (Rs.) Variable Cost

Fixed Cost Activity Level Methods of segregating Semi-variable costs into fixed
and variable costs - The segregation of semi-variable costs into fixed and
variable costs can be carried out by using the following methods: (a) Graphical
method (b) High points and low points method (c) Analytical method (d) Comparison
by period or level of activity method (e) Least squares method (a) Graphical
method: Under this method, a large number of observations regarding the total
costs at different levels of output are plotted on a graph with the output on the
X-axis and the total cost on the Y-axis. Then, by judgment, a line of �best-fit�,
which passes through all or most of the points is drawn. The point at which this
line cuts the Y-axis indicates the total fixed cost component in the total cost.
If a line is drawn at this point parallel to the X-axis, this indicates the fixed
cost. The variable cost, at any level of output, is derived by deducting this
fixed cost element from the total cost. The following graph illustrates this:

1.21
Cost Accounting

(b) High points and low points method: - Under this method in the following
illustration the difference between the total cost at highest and lowest volume is
divided by the difference between the sales value at the highest and lowest
volume. The quotient thus obtained gives us the rate of variable cost in relation
to sales value. The fixed cost is the remainder. See the following illustration.
Illustration : Sales value Rs. 1,40,000 80,000 60,000 Total cost Rs. 72,000 60,000
12,000

At the Highest volume At the Lowest volume

Thus, Variable Cost (Rs. 12,000/Rs. 60,000) = 1/5 or 20% of sales value = Rs.
28,000 (at highest volume) Fixed Cost : Alternatively : Rs. 72,000 � Rs. 28,000
i.e., (20% of Rs. 1,40,000) = Rs. 44,000. Rs. 60,000 � Rs. 16,000 (20% of Rs.
80,000) = Rs. 44,000.

(c) Analytical method: Under this method an experienced cost accountant tries to
judge empirically what proportion of the semi-variable cost would be variable and
what would be fixed. The degree of variability is ascertained for each item of
semi-variable expenses. For example, some semi-variable expenses may vary to the
extent of 20% while others may vary to the extent of 80%. Although it is very
difficult to estimate the extent of variability of an expense, the method is easy
to apply. (Go through the following illustration for clarity).

1.22
Basic Concept

Illustration Suppose, last month the total semi-variable expenses amounted to Rs.
3,000. If the degree of variability is assumed to be 70%, then variable cost = 70%
of Rs. 3,000 = Rs. 2,100. Fixed cost = Rs. 3,000 � Rs. 2,100 = Rs. 900. Now in the
future months, the fixed cost will remain constant, but the variable cost will
vary according to the change in production volume. Thus, if in the next month
production increases by 50%, the total semi-variable expenses will be: Fixed cost
of Rs. 900, plus variable cost viz., Rs. 3,150 i.e., (Rs. 2,100(V.C.) plus 50%
increase of V.C. i.e., Rs. 1,050) i.e., Rs. 4,050. (d) Comparison by period or
level of activity method - Under this method, the variable overhead may be
determined by comparing two levels of output with the amount of expenses at those
levels. Since the fixed element does not change, the variable element may be
ascertained with the help of the following formula.

Change in the amount of expense Change in the quantity of output


Suppose the following information is available: Production Units January February
Difference The variable cost : 100 140 40 Semi-variable expenses Rs. 260 300 40

Rs. 40 Change in Semi - variable expenses = = Re. 1/unit 40 units Change in


production volume
Thus, in January, the variable cost will be 100 � Re. 1 = Rs. 100 and the fixed
cost element will be (Rs. 260 � Rs. 100) or Rs. 160. In February, the variable
cost will be 140 � Re. 1 = Rs. 140 whereas the fixed cost element will remain the
same, i.e., Rs. 160. (e) Least squared method : This is the best method to
segregate semi-variable costs into its fixed and variable components. This is a
statistical method and is based on finding out a line of best fit for a number of
observations. The method uses the linear equation y = mx + c, where m represents
the variable element of cost per unit, �c� represents the total fixed cost, �y�
represents the total cost, �x� represents the volume of output. The total cost is
thus split into its

1.23
Cost Accounting

fixed and variable elements by solving this equation. By using this method, the
expenditure against an item is determined at various levels of output and values
of x and y are fitted in the above formula to find out the values of m and c. The
following illustration may be helpful to understand this method. Level of activity
Capacity % Volume (Labour hours) x Semi-variable expenses (maintenance of plant) y
60% 150 Rs. 1,200 80% 200 Rs. 1,275

Substituting the values of x and y in the equation, y = mx + c, at both the levels


of activity, we get 1,200 = 150 m + c 1,275 = 200 m + c On solving the above
equation, we get (c) (Fixed cost) = Rs. 975 and m (Variable cost) = Rs. 1.50 per
labour hour. Advantages of Classification of Costs into Fixed and Variable: The
primary objective of segregating semi-variable expenses into fixed and variable is
to ascertain marginal costs. Besides this, it has the following advantages also.
(a) Controlling expenses: The classification of expenses into fixed and variable
components helps in controlling expenses. Fixed costs are generally policy costs,
which cannot be easily reduced. They are incurred irrespective of the output and
hence are more or less non controllable. Variable expenses vary with the volume of
activity and the responsibility for incurring such an expenditure is determined in
relation to the output. The management can control these costs by giving proper
allowances in accordance with the output achieved. (b) Preparation of budget
estimates: The segregation of overheads into fixed and variable part helps in the
preparation of flexible budget. It enables a firm to estimate costs at different
levels of activity and make comparison with the actual expenses incurred. Suppose
in October, 2006 the output of a factory was 1,000 units and the expenses were:
Fixed Variable Semi-variable (40% fixed) Rs. 5,000 4,000 6,000 15,000

1.24
Basic Concept

In November, 2006 the output was likely to increase to 1,200 units. In that case
the budget or estimate of expenses will be : Rs. Fixed Variable 5,000 4,800

? Rs.4,000 � 1,200 units ? ? ? 1,000 units ? ?


Semi-variable

Fixed, 40% of Rs. 6,000

2,400 6,720 16,520

? Rs.3,600 � 1,200 units ? Variable: ? ? 4,320 1,000 units ? ?

It would be a mistake to think that with the output going up from 1,000 units to
1,200 units the expenses would increase proportionately to Rs. 18,000. (c)
Decision making: The segregation of semi variable cost between fixed and variable
overhead also helps the management to take many important decisions. For example,
decisions regarding the price to be charged during depression or recession or for
export market. Likewise, decisions on make or buy, shut down or continue, etc.,
are also taken after separating fixed costs from variable costs. In fact, when any
change is contemplated, say, increase or decrease in production, change in the
process of manufacture or distribution, it is necessary to know the total effect
on cost (or revenue) and that would be impossible without a correct segregation of
fixed and variable costs. The technique of marginal costing, cost volume profit
relationship and break-even analysis are all based on such segregation.
1.11.5 By Controllability - Costs here may be classified into controllable and
uncontrollable costs.

(a) Controllable costs - These are the costs which can be influenced by the action
of a specified member of an undertaking. A business organisation is usually
divided into a number of responsibility centres and an executive heads each such
centre. Controllable costs incurred in a particular responsibility centre can be
influenced by the action of the executive heading that responsibility centre.
Direct costs comprising direct labour, direct material, direct expenses and some
of the overheads are generally controllable by the shop level management.

1.25
Cost Accounting

(b) Uncontrollable costs - Costs which cannot be influenced by the action of a


specified member of an undertaking are known as uncontrollable costs. For example,
expenditure incurred by, say, the Tool Room is controllable by the foreman
incharge of that section but the share of the tool-room expenditure which is
apportioned to a machine shop is not to be controlled by the machine shop foreman.
The distinction between controllable and uncontrollable costs is not very sharp
and is sometimes left to individual judgement. In fact no cost is uncontrollable;
it is only in relation to a particular individual that we may specify a particular
cost to be either controllable or uncontrollable.
1.11.6 By Normality - According to this basis cost may be categorised as follows:

(a) Normal cost - It is the cost which is normally incurred at a given level of
output under the conditions in which that level of output is normally attained.
(b) Abnormal cost - It is the cost which is not normally incurred at a given level
of output in the conditions in which that level of output is normally attained. It
is charged to Costing Profit and loss Account.
1.12 CODING SYSTEMS Codes

The Chartered Institute of Management Accountants has defined a code as � a system


of symbols designed to be applied to a classified set of items to give a brief
account reference , facilitating entry collation and analysis� Hence cost
classification forms the basis of any cost coding. It helps us understand the
characteristic of any cost through a short symbolised form.
Composite Codes

Let us consider the following example A company has devised a system of


codification in which the first three digits indicate the nature of the
expenditure and the last three digits the cost centre or cost unit to be charged
e.g. if the first digit is 1, the system implies that it refers to raw material
and if the number is 2 it represents a labour cost. The second and third numbers
relating to 1 i.e., raw material, provide details of the type e.g., whether the
raw material is an electronic component (number 4), mechanical component (number
1) consumables(number 2) or packing (number 3) and the name respectively. Hence
the description of a cost with a code 146.729 shall be understood as follows: ?
Since the first number is 1 the cost refers to raw material cost
1.26
Basic Concept

? The second number being 4 indicates that the raw material is an electronic
component. ? The third number 6 refers to the description which according to the
company�s codification refers to Diodes. The last three numbers provide details of
the cost centre e.g. the first number provides details of the location of the
plant, the second number gives detail of the department (machining or assembly or
something else) and the third number indicates whether the cost is direct or
indirect.
Advantages of a coding system

The following are some of the advantages of a well-designed coding system : (a)
Since the code is, most of the times, briefer than a description, it saves time
when systems are worked upon manually and in case the system is computerised it
reduces the data storage capacity. The illustration above demonstrates this
advantage very clearly. (b) A code helps in reducing ambiguity. In case two
professionals understand the same item differently a code will help them
objectively. (c) Unlike detailed descriptions, a code facilitates data processing
in computerised systems.
The requirements for an efficient coding system

(a) Every number used in the code should be unique and certain, i.e. it should be
easily identified from the structure of the code. (b) Elasticity and
comprehensiveness is an absolute must for a well designed coding system. It should
be possible to identify a code for every item and the coding system should be
capable of expanding to accommodate new items. (c) The code should be brief and
meaningful. (d) The maintenance of the coding system should be centrally
controlled. It should not be possible for individuals to independently add new
codes to the existing coding system. (e) Codification systems should be of the
same length. This makes errors easier to spot and it assists computerised data
processing.
1.13 TYPES OF COSTING

For ascertaining cost, following types of costing are usually used.


1.13.1 Uniform Costing: When a number of firms in an industry agree among
themselves to follow the same system of costing in detail, adopting common
terminology for various items and processes they are said to follow a system of
uniform costing. In such a case, a

1.27
Cost Accounting

comparison of the performance of each of the firms can be made with that of
another, or with the average performance in the industry. Under such a system it
is also possible to determine the cost of production of goods which is true for
the industry as a whole. It is found useful when tax-relief or protection is
sought from the Government.
1.13.2 Marginal Costing: It is defined as the ascertainment of marginal cost by
differentiating between fixed and variable costs. It is used to ascertain effect
of changes in volume or type of output on profit. 1.13.3 Standard Costing and
variance analysis: It is the name given to the technique whereby standard costs
are pre-determined and subsequently compared with the recorded actual costs. It is
thus a technique of cost ascertainment and cost control. This technique may be
used in conjunction with any method of costing. However, it is especially suitable
where the manufacturing method involves production of standardised goods of
repetitive nature. 1.13.4 Historical Costing: It is the ascertainment of costs
after they have been incurred. This type of costing has limited utility. 1.13.5
Direct Costing: It is the practice of charging all direct costs to operations,
processes or products leaving all indirect costs to be written off against profits
in which they arise. 1.13.6 Absorption Costing: It is the practice of charging all
costs, both variable and fixed to operations, processes or products. This differs
from marginal costing where fixed costs are excluded. 1.14 METHODS OF COSTING

Different industries follow different methods of costing because of the


differences in the nature of their work. The various methods of costing are as
follows:
1.14.1 Job Costing: In this case the cost of each job is ascertained separately.
It is suitable in all cases where work is undertaken on receiving a customer�s
order like a printing press, motor workshop, etc. In case a factory produces a
certain quantity of a part at a time, say 5,000 rims of bicycle, the cost can be
ascertained like that of a job. The name then given is Batch Costing. 1.14.2 Batch
Costing: It is the extension of job costing. A batch may represent a number of
small orders passed through the factory in batch. Each batch here is treated as a
unit of cost and thus separately costed. Here cost per unit is determined by
dividing the cost of the batch by the number of units produced in the batch.
1.14.3 Contract Costing : Here the cost of each contract is ascertained
separately. It is suitable for firms engaged in the construction of bridges,
roads, buildings etc.

1.28
Basic Concept

1.14.4 Single or Output Costing : Here the cost of a product is ascertained, the
product being the only one produced like bricks, coals, etc. 1.14.5 Process
Costing : Here the cost of completing each stage of work is ascertained, like cost
of making pulp and cost of making paper from pulp. In mechanical operations, the
cost of each operation may be ascertained separately ; the name given is operation
costing. 1.14.6 Operating Costing : It is used in the case of concerns rendering
services like transport, supply of water, retail trade etc. 1.14.7 Multiple
Costing : It is a combination of two or more methods of costing outlined above.
Suppose a firm manufactures bicycles including its components; the parts will be
costed by the system of job or batch costing but the cost of assembling the
bicycle will be computed by the Single or output costing method. The whole system
of costing is known as multiple costing. 1.15. DIRECT EXPENSES 1.15.1. Meaning of
Direct Expenses : Direct Expenses are also termed as �Chargeable expenses�. These
are the expenses which can be allocated directly to a cost object. Direct expenses
are defined as �costs other than material and wages which are incurred for a
specific product or saleable services�.

Examples of direct expenses are : (i) (ii) Hire charges of special machinery or
plant for a particular production order or job. Payment of royalties.

(iii) Cost of special moulds, designs and patterns. (iv) Experimental costs before
undertaking the job concerned. (v) Travelling and conveyance expenses incurred in
connection with a particular job. (vi) Sub-contracting expenses or outside work
costs if jobs are sent out for special processing.
1.15.2 Characteristics of Direct Expenses :

(i) (ii)

Direct expenses are those expenses, which are other than the direct materials and
direct labour. These expenses are either allocated or charged completely to cost
centres or work orders.

(iii) These expenses are included in prime cost of a product.

1.29
Cost Accounting

The nature of direct expenses demands a strict control over such expenses. This
feature of controlling direct expenses in business houses compels their management
to treat some of the direct expenses as indirect expense. Sometime a direct
expense is assumed as indirect due to the convenience. Sometimes a concern may
treat an expense as direct whereas another may treat the same expense as indirect.
Further the amount of these expenses in proportion to the total cost also
influences the decision to treat them as direct expense or as an indirect expense.
1.15.3 Sub-contracting : It is a common business practice followed by business
concerns, under which operations requiring special processing are sub-contracted.
Examples of such operations are painting, cutting, stitching etc. This is done due
to following reasons :

(i)

The operations, which are given to outside sub-contractors, are those operations,
which require the use of special skill, or special equipment, which is not
available with the concern. If the management of a concern intends to engage
available labour hours and machine hours for operations, which require special
skill or special facility available with the concern, the operations, which
require a lower level of skill or general-purpose machine, are given to outsider.

(ii)

(iii) If there is temporary increase in demand of product of a concern, some of


the operations are given to outsiders to bridge the imbalance between the
production capacities available with various work-centres. The payments made to
sub-contractors or outsiders are charged as direct expenses of the specific
jobs/work orders.
1.15.4. Documentation: The basic document which is used for the charging of direct
expenses to products or batches or work order is the invoice received from
suppliers of such service. The payment to supplier of service is made on the basis
of invoice & expenses are booked in financial accounts.

For example, hiring charges of a machine is charged to the product for which it is
hired on the basis of invoice received from supplier of machinery.
1.15.5. Identification of direct expenses with main product or service: For the
identification of direct expense with main product or service it is required that
the code number of product or service should be inscribed on invoice received from
supplier of services.

For example, if a machine is hired to complete a particular product, then the


hiring charging of a machine is direct expense of the particular product. For
charging hiring charges of machine to a particular product it is required that the
invoice received from supplier of machine should be coded with the product code
for ensuring that the hiring charges of machine is charged to the particular
product. Alternatively, if cash is paid, then the cash book analysis will show the
product code which is to be charged with the cost of hiring machinery.
1.30
Basic Concept

1.16 Self Examination Questions Multiple Choice Questions

(a) The main purpose of cost accounting is to : (i) (ii) maximise profits; help in
inventory valuation;

(iii) provide information to management for decision making; (iv) aid in the
fixation of selling price. (b) One of the most important tools in cost planning is
: (i) (ii) direct cost; budget;

(iii) cost sheet; (iv) marginal costing. (c) Increase in total variable cost is
due to : (i) (ii) increase in fixed cost; increase in sales;

(iii) increase in production. (d) An example of fixed cost is : (i) direct


material cost; (ii) works manager�s salary; (iii) depreciation of machinery; (iv)
chargeable expenses. (e) Cost of goods produced includes : (i) (ii) (f) production
cost and finished goods inventory; production cost and work-in-progress;

(iii) production cost, work-in-progress and finished goods inventory. Conversion


cost is equal to the total of : (i) (ii) material cost and direct wages; material
cost and indirect wages;

1.31
Cost Accounting

(iii) direct wages and factory overhead; (iv) material cost and factory overhead.
(g) Costs which are ascertained after they have been incurred are known as (i)
(ii) imputed costs; sunk costs;

(iii) historical costs; (iv) opportunity costs. (h) Prime costs plus variable
overhead is known as : (i) (ii) production cost; marginal costs;

(iii) total cost; (iv) cost of sales. (i) Indirect costs are known as: (i) (ii)
Variable costs; Fixed costs;

(iii) Overheads; (iv) None of the above. (j) Accounting standard 2 recommends the
valuation of inventory on: (i) (ii) Absorption costing; Marginal costing;

(iii) Activity based costing; (iv) None of the above. (k) If Rs 10 is spend on
producing 10 units and Rs 15 for producing 15, then the fixed cost per unit is:
(i) (ii) Rs 0; Rs 1;

(iii) Rs 2; (iv) None of the above. (l)


1.32

The variable cost per unit is:


Basic Concept

(i) (ii)

Variable in nature; Fixed in nature;

(iii) Semivariable in nature; (iv) None of the above. (m) The total cost for
producing 10 items is Rs 15 and that for producing 15 items is Rs 20. What is the
fixed cost? (i) (ii) Rs 10; Rs 15;

(iii) Rs 5; (iv) None of the above. (n) A company presently does not utilise its
available capacity. In case of full capacity utilisation, the cost per unit shall
(i) (ii) Increase; Decrease;

(iii) Remain constant; (iv) None of the above. (o) The following extract is taken
from the distribution cost budget of DC Ltd. Volume delivered (units) Distribution
cost (i) (ii) (iii) (iv) Rs. 6,600 Rs. 9,000 Rs. 9,400 Rs. 10,800 8,000 Rs. 7,200
14,000 Rs. 10,500

The budgeted cost allowance for distribution cost for a delivery volume of 12,000
units is

Answers to Multiple Choice Questions (a) iii; (b) ii; (c) iii; (d) ii; (e) ii; (f)
iii; (g) iii; (h) ii; (i) iii; (j) i; (k) i; (l) ii; (m) i; (n) ii; (o) iii Short
Answer Type Questions

1.

What is Cost Accounting? Explain its important objectives.

1.33
Cost Accounting

2.

Write short note on: (i) (ii) Conversion cost. Sunk cost.

(iii) Differential cost. (iv) Opportunity cost. (v) Out of pocket cost. 3.
Distinguish between: (i) (ii) Cost centre and cost unit. Cost control and cost
reduction.

(iii) Period cost and product cost. (iv) Controllable and non-controllable costs.
(v) Estimated cost and standard cost. (vi) Variable cost and direct cost. 4. Write
brief answers to the following : (a) While introducing a cost accounting system,
to what should most attention be paidproduction or sales ? (b) Are direct expenses
more important than indirect expenses ? (c) Is prompt reporting better than
accurate reporting ? (d) If the selling price for a product is not within the
control of the firm, why should it have a system of cost accounting ? 5. Below is
given a list of industries and also the methods of costing and cost unit. Give the
correct number of methods of costing and unit of cost against each industry.
Industry Methods of Costing Unit of Cost

(i) (ii)

Nursing Home Road Transport iron ore mines)

(a) Process (b) Job (c) Multiple (d) Single (e) Operating (f) Contract

1. Each job 2. Bed per week or per day 3. Per tonne 4. Each contract 5. Each unit
6. Per tonne-kilo-metre

(iii) Steel (owning (iv) Coal (v)


1.34

Bicycle
Basic Concept

(vi) Bridge construction (vii) Interior decoration (viii) Advertising (ix)


Furniture (x) 7. Sugar company having own sugarcane fields A company manufacturing
garments. The following costs are incurred by the company. You are required to
group the costs which are listed below into following classifications: (a) direct
materials (b) direct labour (c) direct expenses (d) indirect production overhead
(e) research and development cost (f) selling & distribution cost (g)
administration costs (h) finance costs. (i) (ii) Lubricant for sewing machines
Floppy disk for general office computer

(iii) Maintenance contact for office photocopying machine (iv) Road licence for
delivery vehicle (v) Market research prior to new product launch (vi) Cost of
painting advertising slogans in delivery vans (vii) Trade magazine (viii) Upkeep
of delivery vehicles (ix) Wages of operative in cutting department (x) Interest on
bank overdraft
Long Answer Type Questions

1.

State and explain the differences between Financial Accounting, Cost Accounting
and Management Accounting.

1.35
Cost Accounting

2. 3. 4. 5.

What do you understand by direct expenses? What are their characteristics? Discuss
the factors which a Cost Accountant should consider before installing a costing
system in a manufacturing concern. Discuss the various methods known to you which
may be utilised for segregating a semi variable cost into its fixed and variable
component. How is Cost Accounting important to business concerns? Discuss.

1.36
CHAPTER
MATERIAL
Learning Objectives

When you have finished studying this chapter, you should be able to ? Understand
the need and importance of material control. ? Describe the procedures involved in
procuring, storing and issuing material. ? Differentiate amongst the various
methods of valuing material. ? Understand the meaning and the accounting treatment
for normal and abnormal loss of material. ? Understand the meaning and the
accounting treatment of waste, scrap, spoilage and defectives. 2.1 INTRODUCTION

We have acquired a basic knowledge about the concepts, objectives, advantages,


methods and elements of cost. We shall now study each element of cost separately.
The first element of cost is �Direct Material Cost�. Materials constitute a very
significant proportion of total cost of finished product in most of the
manufacturing industries. A proper recording and control over the material costs
is essential because of the following : (a) The exact quality of specification of
materials required should be determined according to the required quality of the
finished product. If too superior quality of material is purchased, it would mean
higher cost due to high prices; if the quality of materials purchased is too low,
the product will be of inferior quality. (b) The price paid should be the minimum
possible otherwise the higher cost of the finished products would make the product
uncompetitive in the market. (c) There should be no interruption in the production
process for want of materials and stores, including small inexpensive items like
lubricating oil for a machine. Sometime their out of stock situation may lead to
stoppage of machines.
Cost Accounting

(d) There should be no over stocking of materials because that would result in
loss of interest charges, higher godown charges, deterioration in quality and
losses due to obsolescence (either due to manufacture of certain articles being
given up or the material previously required for the production not being required
any longer due to a change in methods of production). (e) Wastage and losses while
the materials are in store should be avoided as far as possible; and (f) Wastage
during the process of manufacture should be the minimum possible. It may also be
added that information about availability of materials and stores should be
continuously available so that production may be planned properly and the required
materials purchased in time. 2.2 MATERIAL CONTROL

The publication of the Institute of Cost and Management Accountants on Budgetary


Control defines it as �the function of ensuring that sufficient goods are retained
in stock to meet all requirements without carrying unnecessarily large stocks�.
When the functions of indexing buying, receiving, inspection, storing and paying
of the goods are separated it is essential that these should be properly co-
ordinated so as to achieve the advantages of specialisation. 2.2.1 Objectives of
system of material control : The objectives of a system of material control are
the following : (i) Ensuring that no activity, particularly production, suffers
from interruption for want of materials and stores-it should be noted that this
requires constant availability of every item that may be needed howsoever small
its cost may be. Lubricating oil may cost much less than the main raw material
but, from the point of view of uninterrupted production, both have equal
importance. Seeing to it that all the materials and stores are acquired at the
lowest possible price considering the quality that is required and considering
other relevant factors like reliability in respect of delivery, etc.

(ii)

(iii) Minimisation of the total cost involved, both for acquiring stocks (apart
from the price paid to the supplier) and for holding them. (iv) Avoidance of
unnecessary losses and wastages that may arise from deterioration in quality due
to defective or long storage or from obsolescence. It may be noted that losses and
wastages in the process of manufacture, concern the production department.

2.2
Material

(v) Maintenance of proper records to ensure that reliable information is available


for all items of materials and stores that not only helps in detecting losses and
pilferages but also facilitates proper production planning. The fulfilment of the
objectives mentioned above will require that standard lists of all the materials
and stores required for the firm�s work be drawn up with the weekly consumption
figures. Also the lead time for each item has to be determined which will then
enable the firm to ascertain the minimum quantity for each items. It is also
necessary to fix maximum quantity so that capital is not locked up unnecessarily
and the risk of obsolescence is minimised. Costs are minimised through the use of
ABC analysis (which means classification of the various items on the basis of
investment involved into three categories, viz., A, B and C.) 2.2.2 Requirements
of material control - Material control requirements are as follows:� 1. 2. 3. 4.
5. 6. 7. Proper co-ordination of all departments involved viz., finance,
purchasing, receiving, inspection, storage, accounting and payment. Determining
purchase procedure to see that purchases are made, after making suitable
enquiries, at the most favourable terms to the firm. Use of standard forms for
placing the order, noting receipt of goods, authorising issue of the materials
etc. Preparation of budgets concerning materials, supplies and equipment to ensure
economy in purchasing and use of materials. Operation of a system of internal
check so that all transactions involving materials, supplies and equipment
purchases are properly approved and automatically checked. Storage of all
materials and supplies in a well designated location with proper safeguards.
Operation of a system of perpetual inventory together with continuous stock
checking so that it is possible to determine at any time the amount and value of
each kind of material in stock. Operation of a system of stores control and issue
so that there will be delivery of materials upon requisition to departments in the
right amount at the time they are needed. Development of system of controlling
accounts and subsidiary records which exhibit summary and detailed material costs
at the stage of material receipt and consumption.

8.

9.

2.3
Cost Accounting

10. Regular reports of materials purchased, issue from stock, inventory balances,
obsolete stock, goods returned to vendors, and spoiled or defective units. 2.3
MATERIALS PROCUREMENT PROCEDURE

If a concern can afford it, there should be a separate purchase department for all
purchases to be made on behalf of all other departments. Purchasing should be
centralised i.e. all purchases should be done by the purchasing department except
for small purchases which may be done by the user�s department. What is needed is
that there should be staff wholly devoted to purchasing. Such a staff is bound to
become expert in the various matters to be attended to, for examples� units of
materials to be purchased and licences to be obtained, transport, sources of
supply, probable price etc. The concerned officers in this department keep
themselves in constant touch with the markets either by reading various trade
magazines or by direct association, to have the latest information. If a concern
has a number of factories requiring the same material and stores, there will be
advantage in centralising purchases since important economies in prices and even
transport may be obtained by placing large orders at a time. In such a case
inspection at a source may also be possible. Of course, there will be a little
delay in supply because of clerical processes and sometimes, there may be a
misunderstanding resulting in supply of wrong articles. However, there is no
advantage in centralised purchasing if different materials, and stores are
required by different plants. Materials purchase department in a business house is
confronted with the following issues: (i) (ii) What to purchase ? When to purchase
?

(iii) How much to purchase ? (iv) From where to purchase. (v) At what price to
purchase. To overcome the above listed issues, the purchase department follows the
procedure involving following steps : 1. 2.
2.4

Receiving purchase requisitions. Exploring the sources of materials supply and


selecting suitable material suppliers.
Material

3. 4. 5.

Preparation and execution of purchase orders. Receipt and inspection of materials.


Checking and passing of bills for payment.

2.3.1 Receiving purchase requisitions - Since the materials and stores purchased
will be used by the production departments, there should be constant co-ordination
between the purchase and production departments. A purchase requisition is a form
used for making a formal request to the purchasing department to purchase
materials. This form is usually filled up by the store keeper for regular
materials and by the departmental head for special materials (not stocked as
regular items). The requisition form is duly signed by either works manager or
plant superintendent, in addition to the one originating it. At the beginning a
complete list of materials and stores required should be drawn up, the list should
have weekly consumption figures. It should be gone through periodically so that
necessary deletion and addition may be made. If there is any change in the rate of
consumption per week (say, due to extra shift being worked), the purchase
department should be informed about the new figures. Once an item has been
included in the standard list, it becomes the duty of the purchase department to
arrange for fresh supplies before existing stocks are exhausted. But if the
production department requires some new material, it should make out an indent
well in time and send it to the purchase department for necessary action. Control
over buying - For control over buying of regular stores materials it is necessary
to determine their maximum, minimum, reorder level and economic order quantities.
The use of economic order quantities and various levels constitutes an adequate
safeguard against improper indenting of regular materials. In respect of special
materials, required for a special order or purpose, it is desirable that the
technical department concerned should prepare materials specifications list
specifying the quantity, size and order specifications of materials to be drawn
from the store and those to be specially procured. In all cases, the starting
point in the process for purchasing is the issue of a proper purchase requisition
(form is given below). (Students may note that the forms suggested in this booklet
as well as in the text books are illustrative in nature. The actual form may
differ under different circumstances). It may originate either in the stores
department in connection with regular stock of materials or in the production
planning or in other technical departments concerned in respect of special
materials. Its purpose is to request and authorise the purchase department to
order to procure the materials specified in

2.5
Cost Accounting

stated quantities. It should be made out in triplicate. The original copy is sent
to the purchasing department, the duplicate is kept by the storekeeper or the
department which initates the requisition and the triplicate is sent to the
authorising executive. Purchase Requisition (Regular/Special) (Use a separate form
for each item) No.................. Date............. Purchase...................
Description of Materials required Department................................. Date
by which material required............................... Quantity required Exact
specification

........................ Indentor For use in purchase department Firm Quotations


..................................................................................
..... Price (including charges)
..................................................................................
..... Price............................... Date of Delivery
..................................................................................
..... Remarks Purchase Manager Date of dly...................
With...................................... 1. 2. 3. Order No. &
Date...........................

2.6
Material

A specimen form of purchase requisition for reordering regular stock items.


Purchase Requistion No. S.No. Name of article Present stock Minimum stock Maximum
stock Date Remarks

Remarks

.............................. ..............................

.............................. Purchase manager

.............................. Stores Ledger Clerk

2.3.2 Exploring the sources of materials supply and selecting suitable material
suppliers: A source for the supply of each material may be selected after the
receipt of the purchase requisition. Purchase department in each business house
usually maintains a list of suppliers for each group of materials, required by
their concern. Atleast three quotations are invited from such suppliers. On the
receipt of these quotations a comparative statement is prepared. For selecting
material suppliers the factors which the purchase department keeps in its mind
are�price; quantity; quality offered; time of delivery; mode of transportation;
terms of payment; reputation of supplier; etc. In addition to the above listed
factors purchase manager obtains the necessary information from the statement of
quotations; past records, buyer guides etc. for finally selecting material
suppliers. 2.3.3 Preparation and execution of purchase orders : Having decided on
the best quotation that should be accepted, the purchase manager or concerned
officer proceeds to issue the formal purchase order. It is a written request to
the supplier to supply certain specified materials at specified rates and within a
specified period. Copies of purchase order are sent to : (i) (ii) The supplier;
Store or the order indenting department;

(iii) Receiving department; and (iv) Accounting department. A copy of the purchase
order, alongwith relevant purchase requisitions, is held in the file

2.7
Cost Accounting

of the department to facilitate the follow-up, of the delivery and also for
approving the invoice for payment. 2.3.4 Receipt and inspection of materials:
Under every system of stores organisation, a distinction is made between the
function of receiving and storing, so that each acts as a check on the other. The
receiving department or section is responsible for taking charge of the incoming
materials, checking and verifying their quantities, inspecting them as regards
their grade, quality or other technical specifications and if found acceptable,
passing them on to the stores (or other departments for which these might have
been purchased). In large organisation, a special inspection wing is often
attached to the receiving department and, where it is not so, technical appraisal
of the incoming supplies is carried out by the general inspecting staff. In case
the quality is not the same as ordered, the goods are not accepted. If everything
is in order and the supply is considered suitable for acceptance, the Receiving
department prepares a Receiving Report or Material Inward Note or Goods Received
Note. It is prepared in quadruplicate, the copies being distributed as under : (i)
(ii) First copy is sent to the Purchase Department for verifying supplier�s bill
for payment. Second copy is sent to the store or the department that indents the
material.

(iii) Third copy is sent to the stores ledger clerk in the Cost Department. (iv)
Fourth and the last copy is retained for use by the receiving department. A good
plan would be that the receiving clerk sends all the three copies (meant for
others) along with the materials to the store or the department that placed the
order. The materials are then physically inspected and the particulars thereof as
recorded in the Receiving Report are verified. If the quantity and quality are in
order, the delivery of the same is accepted and copies of the report are signed;
two copies of the report are forwarded to the Purchase Department and the third is
kept on the file as documentary evidence of the quantities of stores received for
storage or use, as the case may be. The Purchase Department in turn, enters the
purchase price and forwards one copy to the Accounts Department and the second to
the Cost Department. A specimen form of the receiving report is given below:

2.8
Material

Goods Received Note Received from.............................. Order No.


.................. Amount Quantity Code Description Amount due to supplier Rs.
Charges Rs. Total Rs. Remarks No. .............. Date............................

Inspector....................... Receiver........................

Store Keeper.................................. Store Ledger


Clerk......................

Material outward return note - Sometimes materials have to be returned to


suppliers after these have been received in the factory. Such returns may occur
before or after the preparation of the receiving report. If the return takes place
before the preparation of the receiving report, such material obviously would not
be included in the report and hence not debited in the stores books and ledgers.
In that case, no adjustment in the account books would be necessary. But if the
material is returned after its entry in the receiving report, a suitable document
must be drawn up in support of the credit entry so as to exclude from the Stores
of Material Account the value of the materials returned back. This document
usually takes the form of a Material outward return note. The Material outward
return note is drawn up by the Stores or the Despatch Department. Five copies of
it are usually prepared; two for the supplier (one of which is to be sent back by
the supplier after he has signed the same), one for Store, one for Cost (stores)
Ledger and one copy to be retained in the Material outward return book. (Please
draw up the form yourself and then see the text book). 2.3.5 Checking and passing
of bills for payment : The invoice received from the supplier is sent to the
stores accounting section to check authenticity and mathematical accuracy. The
quantity and price are also checked with reference to goods received note and the
purchase order respectively. The stores accounting section after checking its
accuracy finally certifies and passes the invoice for payment. In this way the
payment is

2.9
Cost Accounting

made to supplier. 2.4 MATERIAL ISSUE PROCEDURE

Issue of material must not be made except under properly authorised requisition
slip; usually it is the foreman of a department who has the authority to draw
materials from the store. Issue of material must be made on the basis of first in
first out, that is, out of the earliest lot on hand. If care is not exercised in
this regard, quality of earliest lot of material may deteriorate for having been
kept for a long period. Material requisition note : It is the voucher of the
authority as regards issue of materials for use in the factory or in any of its
departments. Where a �Materials List� has been prepared, either the whole of the
materials would be withdrawn on its basis or separate materials requisitions would
be prepared by the person or department and the material drawn upto the limit
specified in the list. The Requisition Notes are made out in triplicate. The
copies are distributed in the following manner : One copy for the Store-keeper.
One copy for Cost department. One copy for the Department requiring it. If no
material list has been prepared, it is desirable that the task of the preparation
of Material Requisition Notes be left to the Planning Department. If there is no
Planning Department, (or although in existence, is unable to undertake this task),
the Requisition Notes should be prepared by the person or department that requires
the materials. Usually, a foreman�s authority is enough but, in the case of costly
materials, it would be desirable to have such requisitions duly approved by some
higher authority, like the Superintendent or Works Manager before these are
presented to Stores. A specimen form of the Material Requisition is shown below:
Material Requisition Note Work Order No. ............................ Department
..................................... Item No. Particulars No.
.......................................... Date
....................................... Qnty. Rate Amount Rs. Rs.

Store-keeper

Workman receiving the material

Foreman

S.L. Clerk

2.10
Material

Bill of material : It is also known as Material Specification List or simply


Material List. It is a schedule of standard quantities of materials required for
any job or other unit of production. A comprehensive Materials List should rigidly
lay down the exact description and specifications of all materials required for a
job or other unit of production and also required quantities so that if there is
any deviation from the standard list, it can easily be detected. The materials
List is prepared by the Engineering or Planning Department in a standard form. The
number of copies prepared vary according to the requirement of each business, but
four is the minimum number. A copy of it is usually sent to each of the following
department : (i) (ii) Stores department. Cost Accounts Department.

(iii) Production Control department. (iv) Engineering or Planning department. The


advantages of using �bill of material�, by the above departments may be summed up
as follows:� Stores Department : 1. 2. 3. A bill of material serves as an
important basis of preparing material purchase requisitions by stores department.
It acts as an authorisation for issuing total material requirement. The clerical
activity is reduced as the stores clerk issues the entire/part of the material
requirement to the users if the details of material asked are present in the bill
of materials.

Cost Accounts Department : 1. 2. Bill of material, is used by Cost Accounts


department for preparing an estimate/budget of material cost for the
job/process/operation, it is meant. It may be used as a device for controlling the
excess cost of material used. This is done after determining material variances
and ascertaining the reasons for their occurrence.

Production Control Department : 1. 2. Bill of material, may be used by this


department for controlling usage of materials. Its usage saves time which
otherwise would have been wasted for preparing separate requisitions of material.

2.11
Cost Accounting

Engineering or Planning department : As stated earlier this department prepares


the materials list in a standard form. A copy of list is sent to stores, cost
accounts and production control department. Proforma of Bill of Materials Job No.
............................................. Department authorised
............... Sl. No. Code No. or Description Qty. Date of issue & Qty. issued
Date Qty. No. ...................................... Date
................................... Rate Rs. Amount Rs.

Authorised by .................................................. Store Keeper�s


signature .............................

Received by ......................... Checked by .......................... Cost


clerk .............................

Transfer of material: The surplus material arising on a job or other units of


production may sometime be unsuitable for transfer to Stores because of its bulk,
heavy weight, brittleness or some such reason. It may, however, be possible to
find some alternative use for such materials by transferring it to some other job
instead of returning it to the Store Room. It must be stressed that generally
transfer of material from one job to another is irregular, if not improper, in so
far it is not conducive to correct allocation and control of material cost of jobs
or other units of production. It is only in the circumstances envisaged above that
such direct transfer should be made, at the time of material transfer a material
transfer note should be made in duplicate, the disposition of the copies of this
note being are as follows : One copy for the Cost Department, and One copy for the
department making the transfer. No copy is required for the Store as no entry in
the stores records would be called for. The Cost Department would use its copy for
the purpose of making the necessary entries

2.12
Material

in the cost ledger accounts for the jobs affected. The form of the Material
Transfer Note is shown below: Material Transfer Note From Job No.
.................................. To Job No.
........................................ Item No. Particulars No.
.................................... Date ................................. Rate
Rs. Amount Rs.

Transferred by

Received by .................................... Job Ledger Clerk

Return of material: Sometimes, it is not possible before hand to make any precise
estimate of the material requirements or units of production. Besides, at times
due to some technical or other difficulty, it is not practicable to measure
exactly the quantity of material required by a department. In either case,
material may have to be issued from stores in bulk, often in excess of the actual
quantity required. Where such a condition exists, it is of the utmost importance
from the point of view of materials control that any surplus material left over on
the completion of a job should be promptly hand over to the storekeeper for safe
and proper custody. Unless this is done, the surplus material may be
misappropriated or misapplied to some purpose, other than that for which it was
intended. The material cost of the job against which the excess material was
originally drawn in that case, would be overstated unless the job is given credit
for the surplus arising thereon. The surplus material, when it is returned to the
storeroom, should be accompanied by a document known either as a Shop Credit Note
or alternatively as a Stores Debit Note. This document should be made out, by the
department returning the surplus material and it should be in triplicate to be
used as follows: One copy for the Store Room; One copy for the Cost Department;
and One copy (book copy) for the department returning the surplus material.

2.13
Cost Accounting

The form of Shop Credit Note is given below: Shop Credit Note Job No.
...................................... Department ............................
Item No. Particulars Qnty. No. ........................ Date
...................... Rate Amount

Store-keeper

S.L. Clerk

Foreman of the Returning Department

2.5

MATERIAL STORAGE

Proper storing of materials is of primary importance. It is not enough only to


purchase material of the required quality. If the purchased material subsequently
deteriorates in quality because of bad storage, the loss is even more than what
might arise from purchase of bad quality materials. Apart from preservation of
quality, the store-keeper also must ensure safe custody of the material. It should
be the function of store-keeper that the right quantity of materials always should
be available in stock. 2.5.1 1. 2. 3. 4. Duties of store keeper : These can be
briefly set out as follows : To exercise general control over all activities in
Stores Department To ensure safe keeping both as to quality and quantity of
materials. To maintain proper records. To initiate purchase requisitions for the
replacement of stock of all regular stores items whenever the stock level of any
item of store approaches the minimum limit fixed in respect thereof. To initiate
action for stoppage of further purchasing when the stock level approaches the
maximum limit. To check and receive purchased materials forwarded by the receiving
department and to arrange for the storage in appropriate places. To reserve a
particular material for a specific job when so required. To issue materials only
in required quantities against authorised requisition notes/material lists.

5. 6. 7. 8.

2.14
Material

9.

To check the book balances, with the actual physical stock at frequent intervals
by way of internal control over wrong issues, pilferage, etc.

2.5.2 Minimising the cost of purchasing and store-keeping: There are two types of
costs which are involved in making a purchase and keeping the goods in the store.
For placing each order, a certain amount of labour is required and, therefore, it
will involve a certain sum of money as cost. It should be noted that the cost of
making a purchase not only includes the cost incurred by the purchasing department
but it also includes the cost of receiving and inspecting the goods. These costs
will naturally increase if the number of order is large; there can be saving if
the number of orders is reduced. The other type of cost is concerned with keeping
the goods is stock, it comprises the money invested, the loss which is likely to
take place if the goods are kept, the expenses incurred on looking after the items
etc. Larger the stock, higher will be this type of cost. In order to reduce this
cost, it is necessary to bring down level of the stock. It may be noted that the
number of orders can be cut down only, if the quantity of each order is increased,
but if that is done, the average quantity on hand will increase and, therefore,
interest and the cost of store keeping will be higher. It is necessary, therefore
to have balance between those two costs and to keep total of the two at the
minimum level. With this objective in view, the economic order quantity is worked
out. But different items for stock have to be treated differently. The name given
to such classification is the �ABC� Analysis, or the Selective Inventory Control.
2.5.3 Different classes of stores: Broadly speaking, there are three classes of
stores viz., central or main stores, sub-stores and departmental stores. The
central stores are the most common of all and in practice, factories generally
have only a central store under the control of one store keeper. Such a store is
centrally situated and is easily accessible to all departments. If receipts and
issues of different items of stores are not large, and the various departments are
close to each other, one central store for all purposes is sufficient. In big
organisations, particularly in the case of collieries, tea gardens, etc., where
the work spots are distributed over a large area, sub-stores are created. A sub-
store is in fact a branch of the central store. It is generally created to
facilitate easy accessibility to the various work spots or consumption centres.
Only the essential items, as well as those required urgently, are kept in them.
The issues to sub-stores are not treated as consumption but only as a transfer,
from one store (central) to another sub-store. The control in the matter of
ordering or receiving rests with the central stores and the sub-stores do not
generally receive any item directly.

2.15
Cost Accounting

Departmental stores are created normally to minimise the time spent on drawing
from stores. For example, a week�s supply may be drawn at one time and kept in a
departmental store at a place marked for the purpose. Such stores, however, are
essential where one or more production departments work in multiple shifts and the
central store works for only one shift; also for the storage of work in progress
and semifinished components where these are large in number or in bulk. Unlike a
sub store in the departmental store, the control rests with the department in
charge. The materials are generally issued in bulk to the departmental store and
it is the responsibility of the department-in-charge to keep proper accounts as
regards issues and stock. If the bulk of materials is required for only one
department, it is usually stored near the department under the charge of the super
intendent concerned. 2.5.4 Stores location : The location of store should be
carefully planned. It should be near to the material receiving department so that
transportation charges are minimum. At the same time, it should be easily
accessible to all other departments of the factory, railway siding, roads etc.
Planned location of the stores department avoids delay in the movement of
materials to the departments in which they are needed. 2.5.5 Stores layout : The
store should be adequately provided with the necessary racks, drawers and other
suitable receptacles for storing materials. Each place (for example, a drawer or a
corner) where materials are kept is called a bin. Each bin should be serially
numbered and for every item a bin should be allowed. All receipts of the item of
the same type should be kept in the bin allotted, for convenience of access. The
number of the bin should be entered in the Store Ledger concerned accounts. 2.6
STORE RECORD

The record of stores may be maintained in three forms: (a) Bin Cards (b) Stock
Control Cards, (c) Stores Ledger. The first two forms of accounts are records of
quantities received, issued and those in balance, but the third one is an account
of their cost also. Usually, the account is kept in both the forms, the
quantitative in the store and quantitative-cum-financial in the Cost Department.
Bin Cards and Stock Control Cards : These are essentially similar, being only
quantitative records of stores. The latter contains further information as regards
stock on order. Bin
2.16
Material

cards are kept attached to the bins or receptacles or quite near thereto so that
these also assist in the identification of stock. The Stock Control Cards, on the
other hand, are kept in cabinets or trays or loose binders. Advantages of Bin
Cards : (i) (ii) There would be less chances of mistakes being made as entries
will be made at the same time as goods are received or issued by the person
actually handling the materials. Control over stock can be more effective, in as
much as comparison of the actual quantity in hand at any time with the book
balance is possible.

(iii) Identification of the different items of materials is facilitated by


reference to the Bin Card the bin or storage receptacle. Disadvantages of Bin
Cards : (i) (ii) Store records are dispersed over a wide area. The cards are
liable to be smeared with dirt and grease because of proximity to material and
also because of handling materials.

(iii) People handling materials are not ordinarily suitable for the clerical work
involved in writing Bin Cards. Advantages of Stock Control Cards : (i) (ii)
Records are kept in a more compact manner so that reference to them is
facilitated. Records can be kept in a neat and clean way by men solely engaged in
clerical work so that a division of labour between record keeping and actual
material handling is possible.

(iii) As the records are at one place, it is possible to get an overall idea of
the stock position without the necessity of going round the stores. Disadvantages
of Stock Control Cards : (i) (ii) On the spot comparison of the physical stock of
an item with its book balance is not facilitated. Physical identification of
materials in stock may not be as easy as in the case of bin cards, as the Stock
Control Cards are housed in cabinets or trays.

The specimen forms of these cards may be studied from any text book. Stores
Ledger: A Modern Stores Ledger is a collection of cards or loose leaves specially
ruled for maintaining a record of both quantity and cost of stores received,
issued and those in stock. It being a subsidiary ledger to the main cost ledger,
it is maintained by the Cost Accounts Department. It is posted from Goods Received
Notes and Materials

2.17
Cost Accounting

requisition. The advantages of writing up Stores Ledger mechanically are: (1) It


enables distribution of work among a number of clerks due to which receipts and
issues are posted quickly and regularly. (2) It enables stock records to be
centralised in case of an organisation having a number of depots. (3) The accuracy
of posting can be mechanically tested more conveniently. (4) The records are
clearer and neater. Also the recurring cost of maintaining them is much less than
those kept manually. (5) If up-to-date records are available, the management will
be able to exercise greater control over quantities held in stock from time to
time which may result in a great deal of saving in both the amount of investment
in stock and their cost. Now-a-days, mostly a duplicate record of issues and
receipt of materials is kept one on Bin Cards in the Store and the second in the
Stores Ledger in the stores. The form of ruling the Store Ledger and Bin Cards
should be studied from the text book. 2.6.1 Treatment of shortages in stock taking
: At the time of stock taking generally discrepancies are found between physical
stock shown in the bin card and stores ledger. These discrepancies are in the form
of shortages or losses. The causes for these discrepancies may be classified as
unavoidable or avoidable. Losses arising from unavoidable causes should be taken
care of by setting up a standard percentage of loss based on the study of the past
data. The issue prices may be inflated to cover the standard loss percentage.
Alternatively, issues may be made at the purchase price but the cost of the loss
or shortage may be treated as overheads. Actual losses should be compared with the
standard and excess losses should be analysed to see whether they are due to
normal or abnormal reasons. If they are attributable to normal causes, an
additional charge to overheads should be made on the basis of the value of
materials consumed. If they arise from abnormal causes, they should be charged to
the Costing Profit and Loss account. Avoidable losses are generally treated as
abnormal losses. These losses should be debited to the Costing Profit and Loss
Account. Losses or surpluses arising from errors in documentation, posting etc.,
should be corrected through adjustment entries.

2.18
Material

2.7

INVENTORY CONTROL

The main objective of inventory control is to achieve maximum efficiency in


production and sales with the minimum investment in inventory. Inventory comprises
of stocks of materials, components, work-in-progress, and finished products and
stores and spares. The techniques commonly applied for inventory control are as
follows: Techniques of Inventory control : (i) (ii) Setting of various stock
levels. ABC analysis.

(iii) Two bin system. (iv) Establishment of system of budgets. (v) Use of
perpetual inventory records and continuous stock verification. (vi) Determination
of economic order quantity. (vii) Review of slow and non-moving items. (viii) Use
of control ratios. 2.7.1 Setting of various stock levels: Minimum level - It
indicates the lowest figure of inventory balance, which must be maintained in hand
at all times, so that there is no stoppage of production due to nonavailability of
inventory. The main consideration for the fixation of minimum level of inventory
are as follows: 1. 2. 3. Information about maximum consumption and maximum
delivery period in respect of each item to determine its re-order level. Average
rate of consumption for each inventory item. Average delivery period for each
item. This period can be calculated by averaging the maximum and minimum period.

The formula used for its calculation is as follows: Minimum level of inventory =
Re-order level � (Average rate of consumption � average time of inventory
delivery)

2.19
Cost Accounting

Maximum level : It indicates the maximum figure of inventory quantity held in


stock at any time. The important considerations which should govern the fixation
of maximum level for various inventory items are as follows : 1. The fixation of
maximum level of an inventory item requires information about its-re-order level.
The re-order level itself depends upon its maximum rate of consumption and maximum
delivery period. It in fact is the product of maximum consumption of inventory
item and its maximum delivery period. Knowledge about minimum consumption and
minimum delivery period for each inventory item should also be known. The
determination of maximum level also requires the figure of economic order
quantity. Availability of funds, storage space, nature of items and their price
per unit are also important for the fixation of maximum level. In the case of
imported materials due to their irregular supply, the maximum level should be
high.

2. 3. 4. 5.

The formula used for its calculation is as follows : Maximum level of inventory =
Re-order-level + Re-order quantity - (Minimum consumption � Minimum re-order
period) Re-order level - This level lies between minimum and the maximum levels in
such a way that before the material ordered is received into the stores, there is
sufficient quantity on hand to cover both normal and abnormal consumption
situations. In other words, it is the level at which fresh order should be placed
for replenishment of stock. The formula used for its calculation is as follows :
Re-order level = Maximum re-order period � Maximum Usage (or) = Minimum level +
(Average rate of consumption � Average time to obtain fresh supplies). Average
Inventory Level - This level of stock may be determined by using the following
equal formula : Average inventory level = Minimum level + 1/2 Re-order quantity
(or) =
Maximum level + Minimum level 2

2.20
Material

Danger level - It is the level at which normal issues of the raw material
inventory are stopped and emergency issues are only made. Danger level = Average
consumption � Lead time for emergency purchases Illustration - Two components, A
and B are used as follows : Normal usage Maximum usage Minimum usage Re-order
quantity Re-order period 50 per week each 75 per week each 25 per week each A:
300; B : 500 A : 4 to 6 weeks B : 2 to 4 weeks Calculate for each component (a)
Re-ordering level, (b) Minimum level, (c) Maximum level, (d) Average stock level.
Solution : (a) Re-ordering level : Minimum usage per week � Maximum delivery
period. Re-ordering level for component A = Re-ordering level for component B =
(b) Minimum level : Re-order level � (Normal usage � Average period) Minimum level
for component A = 450 units � 50 units � 5 weeks = 200 units Minimum level for
component B = 300 units � 50 units � 3 weeks = 150 units (c) Maximum level : ROL +
ROQ � (Min. usage � Minimum period) Maximum level for component A = (450 units +
300 units) � (25 units � 4 weeks) = 650 units Maximum level for component B = (300
units + 500 units) � (25 units � 2 weeks) = 750 units (d) Average stock level : �
(Minimum + Maximum) stock level 75 units� 6 75 units� 4 weeks = 450 units weeks =
300 units

2.21
Cost Accounting

Average stock level for component A = � (200 units + 650 units) = 425 units.
Average stock level for component B = � (150 units + 750 units) = 450 units.
Illustration A Company uses three raw materials A, B and C for a particular
product for which the following data apply:
Raw Material Usage per unit of Product (Kgs.) Minimum

Re-order quantity (Kgs.)

Price per Kg.

Delivery period (in weeks)

Reorder level (Kgs)

Minimu m level (Kgs.)

Average

Maximum

Rs.

A B C

10 4 6

10,000 5,000 10,000

0.10 0.30 0.15

1 3 2

2 4 3

3 5 4

8,000 4,750 2,000

Weekly production varies from 175 to 225 units, averaging 200 units of the said
product. What would be the following quantities : (i) (ii) Minimum stock of A ?
Maximum stock of B ?

(iii) Re-order level of C ? (iv) Average stock level of A ? Solution (i) Minimum
stock of A Re-order level � (Average rate of consumption � Average time required
to obtain fresh delivery) = 8,000 � (200 � 10 � 2) = 4,000 kgs. (ii) Maximum stock
of B Re-order level � (Minimum consumption � Minimum re-order period) + Re-order
quantity

2.22
Material

= 4,750 � (175 � 4 � 3) + 5,000 = 9,750 � 2,100 = 7,650 kgs. (iii) Re-order level
of C Maximum re-order period � Maximum usage = 4 � 225 � 6 = 5,400 kgs. OR Re-
order level of C = Minimum stock of C + [Average rate of consumption � Average
time required to obtain fresh delivery] = 2,000 + [(200 � 6) � 3] kgs. = 5,600
kgs. (iv) Average stock level of A = Minimum stock level of A + � Re-order
quantity of A = 4,000 + � � 10,000 = 4,000 + 5,000 = 9,000 kgs. OR Average Stock
level of A

Minimum stock level of A + Maximum stock level of A (Refer to working note) 2


4,000 + 16,250 = 10,125 kgs. 2
Working note Maximum stock of A = ROL+ ROQ � (Minimum consumption � Minimum re-
order period) = 8,000 + 10,000 � [(175 � 10) � 1] = 16,250 kgs. 2.7.2 ABC
Analysis: It is a system of inventory control. It exercises discriminating control
over different items of stores classified on the basis of the investment involved.
Usually the items are divided into three categories according to their importance,
namely,

2.23
Cost Accounting

their value and frequency of replenishment during a period. (i) �A� Category of
items consists of only a small percentage i.e., about 10% of the total items
handled by the stores but require heavy investment about 70% of inventory value,
because of their high prices or heavy requirement or both. (ii) �B� Category of
items are relatively less important; they may be 20% of the total items of
material handled by stores. The percentage of investment required is about 20% of
the total investment in inventories. (iii) �C� Category of items do not require
much investment; it may be about 10% of total inventory value but they are nearly
70% of the total items handled by store. �A� category of items can be controlled
effectively by using a regular system which ensures neither over-stocking nor
shortage of materials for production. Such a system plans its total material
requirements by making budgets. The stocks of materials are controlled by fixing
certain levels like maximum level, minimum level and re-order level. A reduction
in inventory management costs is achieved by determining economic order quantities
after taking into account ordering cost and carrying cost. To avoid shortage and
to minimize heavy investment in inventories, the techniques of value analysis,
variety reduction, standardisation etc., may be used. In the case of �B� category
of items, as the sum involved is moderate, the same degree of control as applied
in �A� category of items is not warranted. The orders for the items, belonging to
this category may be placed after reviewing their situation periodically. For �C�
category of items, there is no need of exercising constant control. Orders for
items in this group may be placed either after six months or once in a year, after
ascertaining consumption requirements. In this case the objective is to economise
on ordering and handling costs. Illustration A factory uses 4,000 varieties of
inventory. In terms of inventory holding and inventory usage, the following
information is compiled: No. of varieties % % value of inventory of inventory
holding (average) 3,875 96.875 20 110 2.750 30 15 0.375 50 4,000 100.000 100
Classify the items of inventory as per ABC analysis with reasons.
2.24

% of inventory usage (in end-product) 5 10 85 100


Material

Solution Classification of the items of inventory as per ABC analysis 1. 15 number


of varieties of inventory items should be classified as �A� category items because
of the following reasons : (i) (ii) Constitute 0.375% of total number of varieties
of inventory handled by stores of factory, which is minimum as per given
classification in the table. 50% of total use value of inventory holding (average)
which is maximum according to the given table.

(iii) Highest in consumption about 85% of inventory usage (in end-product). 2. 110
number of varieties of inventory items should be classified as �B� category items
because of the following reasons : (i) (ii) Constitute 2.750% of total number of
varieties of inventory items handled by stores of factory. Requires moderate
investment of about 30% of total use value of inventory holding (average).

(iii) Moderate in consumption about 10% of inventory usage (in end�product). 3.


3,875 number of varieties of inventory items should be classified as �C� category
items because of the following reasons: (i) (ii) Constitute 96.875% of total
varieties of inventory items handled by stores of factory. Requires about 20% of
total use value of inventory holding (average).

(iii) Minimum inventory consumption i.e. about 5% of inventory usage (in end-
product). Advantages of ABC analysis : The advantages of ABC analysis are the
following : (i) It ensures that, without there being any danger of interruption of
production for want of materials or stores, minimum investment will be made in
inventories of stocks of materials or stocks to be carried. The cost of placing
orders, receiving goods and maintaining stocks is minimised specially if the
system is coupled with the determination of proper economic order quantities.

(ii)

(iii) Management time is saved since attention need be paid only to some of the
items rather than all the items as would be the case if the ABC system was not in
operation.

2.25
Cost Accounting

(iv) With the introduction of the ABC system, much of the work connected with
purchases can be systematized on a routine basis to be handled by subordinate
staff. 2.7.3 Two bin system: Under this system each bin is divided into two parts
- one, smaller part, should stock the quantity equal to the minimum stock or even
the re-ordering level, and the other to keep the remaining quantity. Issues are
made out of the larger part; but as soon as it becomes necessary to use quantity
out of the smaller part of the bin, fresh order is placed. �Two Bin System� is
supplemental to the record of respective quantities on the bin card and the stores
ledger card. 2.7.4 Establishment of system of budgets: To control investment in
the inventories, it is necessary to know in advance about the inventories
requirement during a specific period usually a year. The exact quantity of various
type of inventories and the time when they would be required can be known by
studying carefully production plans and production schedules. Based on this,
inventories requirement budget can be prepared. Such a budget will discourage the
unnecessary investment in inventories. 2.7.5 Use of perpetual inventory records
and continuous stock verification - Perpetual inventory represents a system of
records maintained by the stores department. It in fact comprises: (i) Bin Cards,
and (ii) Stores Ledger. Bin Card maintains a quantitative record of receipts,
issues and closing balances of each item of stores. Separate bin cards are
maintained for each item. Each card is filled up with the physical movement of
goods i.e. on its receipt and issue. Like bin cards, the Store Ledger is
maintained to record all receipt and issue transactions in respect of materials.
It is filled up with the help of goods received note and material issue
requisitions. A perpetual inventory is usually checked by a programme of
continuous stock taking. Continuous stock taking means the physical checking of
those records (which are maintained under perpetual inventory) with actual stock.
Perpetual inventory is essential for material control. It incidentally helps
continuous stock taking. The success of perpetual inventory depends upon the
following: (a) The Stores Ledger-(showing quantities and amount of each item). (b)
Stock Control cards (or Bin Cards). (c) Reconciling the quantity balances shown by
(a) & (b) above. (d) Checking the physical balances of a number of items every day
systematically and by rotation.
2.26
Material

(e) Explaining promptly the causes of discrepancies, if any, between physical


balances and book figures. (f) Making corrective entries where called for after
step (e) and (g) Removing the causes of the discrepancies referred to in step (e)
Advantages - The main advantages of perpetual inventory are as follows : (1)
Physical stocks can be counted and book balances adjusted as and when desired
without waiting for the entire stock-taking to be done. (2) Quick compilation of
Profit and Loss Account (for interim period) due to prompt availability of stock
figures. (3) Discrepancies are easily located and thus corrective action can be
promptly taken to avoid their recurrence. (4) A systematic review of the perpetual
inventory reveals the existence of surplus, dormant, obsolete and show-moving
materials, so that remedial measures may be taken in time. (5) Fixation of the
various stock levels and checking of actual balances in hand with these levels
assist the Store keeper in maintaining stocks within limits and in initiating
purchase requisitions for correct quantity at the proper time. Continuous stock
verification - The checking of physical inventory is an essential feature of every
sound system of material control. Such a checking may be periodical or continuous.
Annual stock-taking, however, has certain inherent shortcomings which tend to
detract from the usefulness of such physical verification. For instance, since all
the items have to be covered in a given number of days, either the production
department has to be shut down during those days to enable thorough checking of
stock or else the verification must be of limited character. Moreover, in the case
of periodical checking there is the problem of finding an adequately trained
contingent. It is likely to be drawn from different departments where stock-taking
is not the normal work and they are apt to discharge such temporary duties
somewhat perfunctorily. The element of surprise, that is essential for effective
control is wholly absent in the system. Then if there are stock discrepancies,
they remain undetected until the end of the period. It means that the figures of
stock during the period continue to be supplied incorrectly. Often, the
discrepancies are not corrected. The system of continuous stock-taking consists of
counting and verifying the number of items daily throughout the year so that
during the year all items of stores are covered three or four times. The stock
verifiers are independent of the stores, and the stores staff have no
foreknowledge as to the particular items that would be checked on any particular

2.27
Cost Accounting

day. But it must be seen that each item is checked a number of times in a year.
Advantages - The advantages of continuous stock-taking are : 1. 2. Closure of
normal functioning is not necessary. Whole time specialised staff can be engaged
for the purpose since the work is spread throughout the year. In smaller concerns,
duties may be assigned to various officers of middle rank by rotation to the
checking, say, of 20 items. This would be easy because the store ledger card and
the bin card will bear the bin number. The officers concerned need only walk up to
the particular bin number, count, weigh or measure the article lying there and
enter the quantity on the form provided for the purpose. The rest of the work
(comparison with book figures) can be done by the stores ledger clerk. Stock
discrepancies are likely to be brought to the notice and corrected much earlier
than under the annual stock-taking system. The system generally has a sobering
influence on the stores staff because of the element of surprise present therein.
The movement of stores items can be watched more closely by the stores auditor so
that chances of obsolescence buying are reduced. Final Accounts can be ready
quickly. Interim accounts are possible quite conveniently.

3. 4. 5. 6.

2.7.6 Economic Order Quantity (EOQ): Purchase department in manufacturing concerns


is usually faced with the problem of deciding the �quantity of various items�
which they should purchase. If purchases of material are made in bulk then
inventory carrying cost will be high. On the other hand if order size is small
each time, then the ordering cost will be high. In order to minimise ordering and
carrying costs it is necessary to determine the order quantity which minimises
these two costs. The size of the order for which both ordering and carrying cost
are minimum is known as economic order quantity. Assumptions underlying E.O.Q.:
The calculation of economic order of material to be purchased is subject to the
following assumptions: (i) (ii) Ordering cost per order and carrying cost per unit
per annum are known and they are fixed. Anticipated usage of material in units is
known.

(iii) Cost per unit of the material is constant and is known as well. (iv) The
quantity of material ordered is received immediately i.e. the lead time is zero.
The famous mathematician Wilson derived the formula which is used for determining
the size of order for each of purchases at minimum ordering and carrying costs.
2.28
Material

The formula given by Wilson for calculating economic order quantity is as follows
: EOQ = where, A S C Illustration Calculate the Economic Order Quantity from the
following information. Also state the number of orders to be placed in a year.
Consumption of materials per annum Order placing cost per order Cost per kg. of
raw materials Storage costs: Solution EOQ = A S C = = = : : : 10,000 kg. Rs. 50
Rs. 2 = = =

2AS C
Annual usage units Ordering cost per order Annual carrying cost of one unit, i.e.,
carrying cost percentage � cost of one unit.

8% on average inventory

2A � S C
Units consumed during year Ordering cost per order Inventory carrying cost per
unit per annum.

EOQ =

2 � 10,000 � 50 = 2�8 100


2,500 kg.

2 � 10,000 � 50 � 25 4

No. of orders to be placed in a year =

Total consumption of materials per annum EOQ

2.29
Cost Accounting

= Illustration

10,000 kg. = 4 Orders per year 2,500 kg.

X Ltd. is committed to supply 24,000 bearings per annum to Y Ltd. on steady basis.
It is estimated that it costs 10 paise as inventory holding cost per bearing per
month and that the set-up cost per run of bearing manufacture is Rs. 324. (a) What
would be the optimum run size for bearing manufacture ? (b) Assuming that the
company has a policy of manufacturing 6,000 bearings per run, how much extra costs
the company would be incurring as compared to the optimum run suggested in (a)
above ? (c) What is the minimum inventory holding cost ? Solution (a) Optimum
production run size (Q) = where, U P I = = = No. of units to be produced within
one year. Set-up cost per production run Carrying cost per unit per annum. =

2UP I

2UP 2 � 24,000 � Rs.324 = I 0.10 � 12

= 3,600 bearings. (b) Total Cost (of maintaining the inventories) when production
run size (Q) are 3,600 and 6,000 bearings respectively Total cost = Total set-up
cost + Total carrying cost. (Total set-up cost) Q = 3,600 = (No. of production
runs ordered) � (Set-up cost per production run) =
24,000 � Rs. 324 3,600

= Rs. 2,160
2.30

...(1)
Material

(Total set�up cost) Q = 6,000 = = 1,296

24,000 � Rs. 324 6,000

...(2)

(Total carrying cost) Q = 3,600 = 1/2 Q � I = 1/2 � 3,600 � 0.10P � Rs. 12 = Rs.
2,160 (Total carrying cost) Q = 6,000 = 1/2 � 6,000 � 0.10P � Rs. 12 = Rs. 3,600
(Total Cost) Q = 3,600 = Rs. 2,160 + Rs. 2,160 [ (1) + (3)] [(2) + (4)] Extra cost
incurred [(6) � (5)] = Rs. 4,320 = Rs. 4,896 = Rs. 4,896 � Rs. 4,320 = Rs. 576 =
1/2 � 3,600 bearings � 0.10P � Rs. 12 ...(5) ...(6) (Total Cost) Q = 6,000 = Rs.
1,296 + Rs. 3,600 ...(4) ...(3)

(c) Minimum inventory holding cost = 1/2 Q � I (when Q = 3,600 bearings) = Rs.
2,160 Illustration Shriram enterprise manufactures a special product �ZED�. The
following particulars were collected for the year 2006: (a) Monthly demand of ZED
- 1,000 units (b) Cost of placing an order Rs. 100. (c) Annual carrying cost per
unit Rs. 15. (d) Normal usage 50 units per week. (e) Minimum usage 25 units per
week. (f) Maximum usage 75 units per week. (g) Re-order period 4 to 6 weeks.
Compute from the above

2.31
Cost Accounting

(1) Re-order quantity (2) Re-order level (3) Minimum level (4) Maximum level (5)
Average stock level. Solution 1. Re-order quantity of units used =

2 AS 2 � 2,600 � Rs.100 = C Rs.15

= 186 units (approximately) (Refer to note) where, A S C 2. 3. Re-order level


Minimum Level = = = = = = = = 4. Maximum Level = = = 5. Average Stock Level = = =
Note: A = = Annual demand of input units Cost of placing an order Annual carrying
cost per unit Maximum re-order period � maximum usage 6 weeks � 75 units = 450
units Re-order level � (normal usage � average re-order period). 450 units � 50
units � 5 weeks. 450 units � 250 units = 200 units. Re-order level + Re-order
quantity � Minimum usage � Minimum order period. 450 units + 186 units � 25 units
� 4 weeks 536 units 1/2 (Minimum stock level + Maximum stock level) 1/2 (200 units
+ 536 units) 368 units. Annual demand of input units for 12,000 units of �ZED� 52
weeks � Normal usage of input units per week

2.32
Material

= = Illustration

52 weeks � 50 units of input per week 2,600 units.

(a) EXE Limited has received an offer of quantity discounts on its order of
materials as under: Price per tonne Rs. 1,200 1,180 1,160 1,140 1,120 Tonnes Nos.
Less than 500 500 and less than 1,000 1,000 and less than 2,000 2,000 and less
than 3,000 3,000 and above.

The annual requirement for the material is 5,000 tonnes. The ordering cost per
order is Rs. 1,200 and the stock holding cost is estimated at 20% of material cost
per annum. You are required to compute the most economical purchase level. (b)
What will be your answer to the above question if there are no discounts offered
and the price per tonne is Rs. 1,500 ? Solution (a)
Total annual requirement (S) Ordersize (Units) (q) No. of orders S/q Cost of
inventory S�Per unit cost (Rs.) Ordering cost S/q�Rs. 1200 (Rs.) 1 500 units 2 400
500 1000 3 12.5 10 5 4 60,00,000 (5,000�Rs. 1200) 59,00,000 (5,000�Rs. 1180)
58,00,000 6,000 12,000 5 15,000 Carrying cost p.u.p.a 1/2�q�20% of cost p.u. (Rs.)
6 48,000 (200 � Rs. 240) 59,000 (250 � Rs. 236) 1,16,000 59,22,000 59,71000 7
60,63,000 Total Cost (4+5+6) (Rs.)

2.33
Cost Accounting (5,000�Rs. 1160) 2000 3000 2.5 1666 57,00,000 (5,000�Rs. 1140)
56,00,000 (5,000�Rs. 1120) 2,000 3,000 (500 � Rs. 232) 2,28,000 (1,000�Rs. 228)
3,36,000 (1,500�Rs. 224) 59,38,000 59,31,000

The above table shows that the total cost of 5,000 units including ordering and
carrying cost is minimum (Rs. 59,22,000) when the order size is 1,000 units. Hence
the most economical purchase level is 1,000 units. (b) EOQ = where

2SC O iC 1
S is the annual inventory requirement, Co, is the ordering cost per order and iC1
is the carrying cost per unit per annum. =
2 � 2,500units � Rs.1,200 20% � Rs.1,500

Illustration From the details given below, calculate: (i) (ii) Re-ordering level
Maximum level

(iii) Minimum level (iv) Danger level. Recording quantity is to be calculated on


the basis of following information: Cost of placing a purchase order is Rs. 20
Number of units to be purchased during the year is 5,000 Purchase price per unit
inclusive of transportation cost is Rs. 50 Annual cost of storage per units is Rs.
5.

2.34
Material

Details of lead time : Rate of consumption :

Average 10 days, Maximum 15 days, Minimum 6 days. For emergency purchases 4 days.
Average : 15 units per day, Maximum : 20 units per day.

Solution BASIC DATA : Co (Ordering cost per order) S (Number of units to be


purchased annually) C 1 (Purchase price per unit inclusive of transportation cost)
iC 1 (Annual cost of storage per unit) Computations : (i) (ii) Re-ordering level =
(ROL) Maximum level = = = = (iii) Minimum level = = = (iv) Danger level = =
Working Notes : 1. ROQ = = Maximum usage per period � Maximum re-order period 20
units per day � 15 days = 300 units ROL + ROQ � [Min. rate of consumption � Min.
re-order period] (Refer to working notes1 and 2) 300 units + 200 units � [10 units
per day � 6 days] 440 units ROL � Average rate of consumption � Average re-
orderperiod 300 units � (15 units per day � 10 days) 150 units Average consumption
� Lead time for emergency purchases 15 units per day � 4 days = 60 units = = = =
Rs. 20 5,000 units Rs. 50. Rs. 5

2SC 0 = iC 1

2 � 5,000units � Rs.1,200 = 200 units 20% � Rs.1,500

2.35
Cost Accounting

2.

Minimum rate of Maximum rate of + Av. rate of consumption consumption =


consumption 2
15 units per day or X = =

Xunits/day + 20units per day 2

10 units per day.

Illustration

About 50 items are required every day for a machine. A fixed cost of Rs. 50 per
order is incurred for placing an order. The Inventory carrying cost per item
amounts to Rs. 0.02 per day. The lead period is 32 days. Compute : (i) (ii)
Economic order quantity. Re-order level.

Solution

Annual consumption (S) = 50 items � 365 days = 18,250 items Fixed cost per order
(Co) or Ordering cost = Rs. 50 Inventory carrying cost per item per annum (iC1) =
Rs. 0.02 � 365 = Rs. 7.30 (i) (ii) Economic order quantity = 2SC 0 iC 1 =

2 � 18,250 units � Rs.50 = 500 units Rs.730

Re-order level = Maximum usage per day � Maximum lead time = 50 items per day � 32
days = 1,600 items

Illustration

M/s. Tubes Ltd. are the manufacturers of picture tubes for T.V. The following are
the details of their operation during 2006: Average monthly market demand Ordering
cost 2,000 Tubes Rs. 100 per order

2.36
Material

Inventory carrying cost Cost of tubes Normal usage Minimum usage Maximum usage
Lead time to supply Compute from the above:

20% per annum Rs. 500 per tube 100 tubes per week 50 tubes per week 200 tubes per
week 6 - 8 weeks

(1) Economic order quantity. If the supplier is willing to supply quarterly 1,500
units at a discount of 5%, is it worth accepting? (2) Maximum level of stock. (3)
Minimum level of stock. (4) Re-order level
Solution

S Co C1 iC1

= = = = = =

Annual usage of tubes = Normal usage per week � 52 weeks 100 tubes � 52 weeks =
5,200 tubes. Ordering cost per order = Rs. 100/- per order Cost per tube = Rs.
500/Inventory carrying cost per unit per annum 20% � Rs. 500 = Rs. 100/- per unit,
per annum 2SC 0 = iC 1
2 � 5,200units � Rs.1,00 = 102 tubes (approx.) Rs.1,00

(1) Economic order quantity E.O.Q =

If the supplier is willing to supply 1500 units at a discount of 5% is it worth


accepting?

Total cost (when order size is 1,500 units) = Cost of 5,200 units + Ordering cost
+ Carrying cost = 5,200 units � Rs. 475 +

5,200 units � Rs. 100 + + 1,500 units � 20% � Rs. 475 1,500 units

2.37
Cost Accounting

= Rs. 24,70,000 + Rs. 346.67 + Rs. 71,250 = Rs. 25,41,596.67 Total cost (when
order size is 102 units) = = = 5,200 units � Rs. 500 +

5,200 units � Rs. 100 + � 102 units � 20% � Rs. 500 102 units

Rs. 26,00,000 + Rs. 5,098.03 + Rs. 5,100 Rs. 26,10,198.03

Since, the total cost under quarterly supply of 1,500 units with 5% discount is
lower than that when order size is 102 units, therefore the offer should be
accepted. While accepting this offer consideration of capital blocked on order
size of 1,500 units per quarter has been ignored.
(2) Maximum level of stock

= Re-order level + Re-order quantity � Min. usage � Min. re-order period = 1,600
units + 102 units � 50 units � 6 weeks = 1,402 units.
(3) Minimum level of stock

= Re-order level � Normal usage � Average reorder period = 1,600 units � 100 units
� 7 weeks = 900 units.
(4) Reorder level

= Maximum consumption � Maximum re-order period = 200 units � 8 weeks = 1,600


units.
Illustration

The complete Gardener is deciding on the economic order quantity for two brands of
lawn fertilizer: Super Grow and Nature�s Own. The following information is
collected:
Fertilizer Super Grow Nature�s Own

Annual Demand Relevant ordering cost per purchase order Annual relevant carrying
cost per bag
2.38

2,000 Bags Rs. 1,200 Rs. 480

1,280 Bags Rs. 1,400 Rs. 560


Material

Required :

(i) (ii)

Compute EOQ for Super Grow and Nature�s Own. For the EOQ, what is the sum of the
total annual relevant ordering costs and total annual relevant carrying costs for
Super Grow and Nature�s Own ?

(iii) For the EOQ Compute the number of deliveries per year for Super Grow and
Nature�s Own.
Solution (i) EOQ =

2SC 0 * iC 1 S = Annual demand of fertilizer bags. C1 = Cost per bag. C0 =


Relevant ordering cost per purchase order iC1 = Annual relevant carrying cost per
bag

* Here

EOQ for Super Grow Fertilizer

EOQ for Nature�s Own Fertilizer

2 � 2,000 bags � Rs.1,200 = 100 bags Rs.480


(ii)

2 � 1,280 bags � Rs.1,400 = 80 bags Rs.560

Total annual relevant costs for Super Grow Fertilizer

= Total annual relevant ordering costs + Total annual relevant carrying costs = =

1 S � C0 + EOQ � iC1 2 EOQ

2000 bags 1 � Rs. 1,200 + � 100 bags � Rs. 480 100 bags 2

= Rs. 24,000 + Rs. 24,000 = Rs. 48,000


Total annual relevant costs for Nature�s Own Fertilizer

1,280 bags 1 � Rs. 1,400 + � 80 bags � Rs. 560 80 bags 2

= Rs. 22,400 + Rs. 22,400 = Rs. 44,800

2.39
Cost Accounting

(iii)

Number of deliveries for Super Grow Fertilizer per year.

2000 bags S = (annual demand of fertiliser bags) = = 20 orders 100 bags EOQ
Number of deliveries for Nature�s Own fertilizers per year.

1,280 bags = 16 orders 80 bags

Illustration

G. Ltd. produces a product which has a monthly demand of 4,000 units. The product
requires a component X which is purchased at Rs. 20. For every finished product,
one unit of component is required. The ordering cost is Rs. 120 per order and the
holding cost is 10% p.a. You are required to calculate: (i) (ii) Economic order
quantity. If the minimum lot size to be supplied is 4,000 units, what is the extra
cost, the company has to incur ?

(iii) What is the minimum carrying cost, the company has to incur ?
Solution (a) (i) Economic order quantity :

S (Annual requirement or Component �X�) C1 (Purchase cost p.u.) C0 (Ordering cost


per order) i (Holding cost) E.O.Q. =

= 4,000 units per month � 12 months = 48,000 units = Rs. 20 = Rs. 120 = 10% per
annum

2 � 48,000 units � Rs.120 S = = 2,400 units 10% � Rs.20 EOQ


= Total ordering cost + Total carrying cost (when order size is 4,000 units)

(ii) Extra cost incurred by the company

Total cost

2.40
Material

= =

S � C0 + q (iC1) Q

1 48,000 units � Rs. 120 + � 4,000 units � 10% � Rs. 20 2 4,000 units
...(a)

= Rs. 1,440 + Rs. 4,000 = Rs. 5,440 Total cost =

1 48,000 units � Rs. 120 + � 2,400 units � 10% � Rs. 20 2 2,400 units
...(b)

(when order size is 2,400 units) = Rs. 2,400 + Rs. 2,400 = Rs. 4,800 Extra cost :
(a) � (b) = Rs. 5,440 � Rs. 4,800 = Rs. 640 (incurred by the company)
(iii) Minimum carrying cost :

Carrying cost depends upon the size of the order. It will be minimum on the least
order size. (In this part of the question the two order sizes are 2,400 units and
4,000 units. Here 2,400 units is the least of the two order sizes. At this order
size carrying cost will be minimum.) The minimum carrying cost in this case can be
computed as under : Minimum carrying cost =
Illustration

1 � 2,400 units � 10% � Rs. 20 = Rs. 2,400. 2

A Ltd. is committed to supply 24,000 bearings per annum to B Ltd. on a steady


basis. It is estimated that it costs 10 paise as inventory holding cost per
bearing per month and that the set-up cost per run of bearing manufacture is Rs.
324. (i) (ii) What should be the optimum run size for bearing manufacture ? What
would be the interval between two consecutive optimum runs ?

(iii) Find out the minimum inventory cost per annum.

2.41
Cost Accounting

Solution (i) Optimum run size for bearing manufacture

= =

2 � Annual supply of bearings � Set - up cost per production run Annual holding
cost per bearing
2 � 24,000 bearings � Rs.324 = 3,600 bearings 12 months � 0.10P.
12 months 12 months = Number of productioon runs per annum ? Annual production ? ?
? ? Optimum run size ? ? ?

(ii) Interval between two consecutive optimum runs

12 months ? 24,000 bearings ? ? ? 3,600 bearings ? ? ? ?

12 months = 1.8 months or 55 days approximately 6.66

(iii) Minimum inventory cost per annum

= Total production run cost + Total carrying cost per annum =

24,000 bearings 1 � Rs. 324 + � 3,600 bearings � 0.10 P � 12 months 3,600 bearings
2

= Rs. 2,160 + Rs. 2,160 = Rs. 4,320


2.7.7 Review of slow and non-moving items : Sometimes, due to high value of slow
moving and non-moving raw materials, it appears that the concern has blocked huge
sum of money unnecessarily in raw materials. To overcome this problem, it is
necessary to dispose-off as early as possible, the non-moving items or make
arrangements for their exchange with the inventories required by the concern.
Besides this no new requisition should be made for the purchase of slow moving
items, till the existing stock is exhausted. Computation of inventory turnover
ratio may help in identifying slow moving items. 2.7.8 Use of control ratios : (i)
Input output ratio : Inventory control can also be exercised by the use of input
output ratio analysis. Input-output ratio is the ratio of the quantity of input of
material to production and the standard material content of the actual output.

2.42
Material

This type of ratio analysis enables comparison of actual consumption and standard
consumption, thus indicating whether the usage of material is favourable or
adverse. (ii) Inventory turnover ratio : Computation of inventory turnover ratios
for different items of material and comparison of the turnover rates, provides a
useful guidance for measuring inventory performance. High inventory turnover ratio
indicates that the material in the question is a fast moving one. A low turnover
ratio indicates over-investment and locking up of the working capital in
inventories. Inventory turnover ratio may be calculated by using the following
formulae:Inventory turnover ratio Average stock = =

Cost of materials consumed durjing the period Cost of average stock held during
the period
1/2 (opening stock + closing stock)

By comparing the number of days in the case of two different materials, it is


possible to know which is fast moving and which is slow moving. On this basis,
attempt should be made to reduce the amount of capital locked up, and prevent
over-stocking of the slow moving items.
Illustration The following data are available in respect of material X for the
year ended 31st March, 2006. Rs.

Opening stock Purchases during the year Closing stock Calculate : (i) (ii)
Inventory turnover ratio, and

90,000 2,70,000 1,10,000

The number of days for which the average inventory is held.

Solution

Inventory turnover ratio (Refer to working note) =

Cost of stock of raw material consumed Average stock of raw material

2.43
Cost Accounting

= = Average number of days for which the average inventory is held =

Rs.2,50,000 Rs.1,00,000

2.5

365days 365 = 2.5 Inventory turnover ratio

= 146
Working Note : Rs. 90,000 2,70,000 1,10,000 2,50,000

Opening stock of raw material Add: Material purchases during the year Less:
Closing stock of raw material Cost of stock of raw material consumed
2.8 VALUATION OF MATERIAL RECEIPTS

The invoice of material purchased from the market sometime contain items such as
trade discount, quantity discount, freight, duty, insurance, cost of containers,
sales tax, excise duty, cash discount etc. Under such a situation, the general
principal is that all costs incurred upto the point of procuring and storing
materials should constitute the cost of materials purchased. The amount of trade
discount, quantity discount and excise duty (under MODVAT credit scheme) being
credit items and are thus deducted from the invoice of material purchased. The
transport charges (carriage and freight), sales tax, insurance, cost of
containers, customs and excise duty (without MODVAT credit) should be included in
the invoice cost of material. The cash discount is considered as financial gain,
so it is kept outside the domain of material cost. In case the containers are
returnable, their resale value should also be taken in the invoice price of
material to correctly ascertain the cost of material purchased. The cost of
material purchased so determined may be used for the entry of material in the
Stores Ledger.

2.44
Material

Illustration

An invoice in respect of a consignment of chemicals A and B provides the following


information:
Rs.

Chemical A: 10,000 lbs. at Rs. 10 per lb. Chemical B: 8,000 lbs. at Rs. 13 per lb.
Sales tax @ 10% Railway freight Total cost

1,00,000 1,04,000 20,400 3,840 2,28,240

A shortage of 500 lbs. in chemical A and 320 lbs. in chemical B is noticed due to
normal breakages. You are required to determine the rate per lb. of each chemical,
assuming a provision of 2% for further deterioration.
Solution Statement showing computation of effective quantity of each chemical
available for use Chemical A lbs. Chemical B lbs.

Quantity purchased
Less : Shortage due to normal breakages Less : Provision for deterioration 2%

10,000 500 9,500 190 9,310


Chemical A Rs.

8,000 320 7,680 53.6 7,526.4


Chemical B Rs.

Quantity available

Statement showing the computation of rate per lb. of each chemical

Purchase price
Add : Sales tax (10%)

1,00,000 10,000

1,04,000 10,400

2.45
Cost Accounting

Railway freight (in the ratio of quantity purchased i.e., 5:4) Total cost Rate per
1b. Rate per lb.
Illustration
Rs.1,12,133 = Rs. 12.04 9,3101bs Rs.1,16,107 = Rs. 15.43 7,526.341bs

2,133 1,12,133

1,707 1,16,107

A: B:

At what price per unit would Part No. A 32 be entered in the Stores Ledger, if the
following invoice was received from a supplier: Invoice 200 units Part No. A 32 @
Rs. 5 Less : 20% discount
Add : Excise duty @ 15% Add : Packing charges (5 non-returnable boxes) Notes: Rs.
1,000.00 200.00 800.00 120.00 920.00 50.00 970.00

(i) (ii)

A 2 per cent discount will be given for payment in 30 days. Documents


substantiating payment of excise duty is enclosed for claiming MODVAT credit.

Solution

200 units net cost after trade discount


Add : Packing charges

Rs. 800 Rs. 50 Rs. 850

Total cost per 200 units Cost per unit =

Rs.850 = Rs. 4.25 200

2.46
Material

2.9

VALUATION OF MATERIAL ISSUES

Materials issued from stores should be priced at the value at which they are
carried in stock. But the value attached to the stock of any item of material, at
any particular point of time may be the result, not of one purchase rate or price
but of different purchase rate or prices. In other words, the same material may
have been acquired at different prices and its stock at any particular time may
comprise materials of more than one lot so that there would be a problem of
determining the appropriate rate at which to price out the issues of materials.
Several methods of pricing material issues have been evolved in an attempt to
satisfactorily answer the problem. These methods may be grouped and explained as
follows :
2.9.1 Cost Price Methods:

(a) Specific price method. (b) First-in First-out method. (c) Last-in-First-out
method. (d) Base stock method.
2.9.2 Average Price Methods :

(e) Simple average price method. (f) Weighted average price method. (g) Periodic
simple average price method. (h) Periodic weighted average price method. (i) (j)
Moving simple average price method. Moving weighted average price method.

2.9.3 Market Price Methods :

(k) Replacement price method. (l) Realisable price method.


2.9.4 Notional Price Methods :

(m) Standard price method.

2.47
Cost Accounting

(n) Inflated price methods. (o) Re-use Price Method. We may now briefly discuss
all the above methods:
(a) Specific Price Method - This method is useful, specially when materials are
purchased for a specific job or work order, and as such these materials are issued
subsequently to that specific job or work order at the price at which they were
purchased. To use this method, it is necessary to store each lot of material
separately and maintain its separate account. The advantages and disadvantage of
this method are : Advantages :

1. The cost of materials issued for production purposes to specific jobs represent
actual and correct costs. 2. This method is best suited for non-standard and
specific products.
Disadvantage : This method is difficult to operate, specially when purchases and
issues are numerous. (b) First-in-First out Method (FIFO) - It is a method of
pricing the issues of materials, in the order in which they are purchased. In
other words, the materials are issued in the order in which they arrive in the
store or the items longest in stock are issued first. Thus each issue of material
only recovers the purchase price which does not reflect the current market price.
This method is considered suitable in times of falling price because the material
cost charged to production will be high while the replacement cost of materials
will be low. But, in the case of rising prices, if this method is adopted, the
charge to production will be low as compared to the replacement cost of materials.
Consequently, it would be difficult to purchase the same volume of material (as in
the current period) in future without having additional capital resources. The
advantages and disadvantages of the method may be stated as follows : Advantages :

1. 2. 3. 4.

It is simple to understand and easy to operate. Material cost charged to


production represents actual cost with which the cost of production should have
been charged. In the case of falling prices, the use of this method gives better
results. Closing stock of material will be represented very closely at current
market price.

2.48
Material

Disadvantages :

1.

If the prices fluctuate frequently, this method may lead to clerical error.

2. Since each issue of material to production is related to a specific purchase


price, the costs charged to the same job are likely to show a variation from
period to period. 3. In the case of rising prices, the real profits of the concern
being low, they may be inadequate to meet the concern�s demand to purchase raw
materials at the ruling price. The application of FIFO method is illustrated below
:
Material Received and Issued Lot Date No. Quantity Lot Kg. Rate No. Amount

1. 2. 3. 4. 5.

July 3 July 13 July 23 August 5 August 6 July 8 July 12 July 22 July 25 August 8

600 800 600 400 1200 400 Kgs. out of (1) 200 Kgs. out of (1) 600 Kgs. out of (2)
200 Kgs. out of (2) 200 Kgs. out of (3) 400 Kgs. out of (3) 400 Kgs. out of (4)
200 Kgs. out of (5)

1.00 1.20 0.90 1.10 0.80 1.00 1.00 1.20 1.20 0.90 0.90 1.10 0.80

600.00 960.00 540.00 440.00 960.00 400.00 200.00 720.00 240.00 180.00 360.00
440.00 160.00

The stock in hand after 8th August will be 1,000 Kgs. This will be out of lot
number (5) and its value will be Rs. 800, i.e., @ Re. 0.80 per Kg.
(c) Last-in-First out method (LIFO) - It is a method of pricing the issues of
materials. This method is based on the assumption that the items of the last batch
(lot) purchased are the first to be issued. Therefore, under this method the
prices of the last batch (lot) is used for pricing the issues, until it is
exhausted, and so on. If however, the quantity of

2.49
Cost Accounting

issue is more than the quantity of the latest lot than earlier (lot) and its price
will also be taken into consideration. During inflationary period or period of
rising prices, the use of LIFO would help to ensure that the cost of production
determined on the above basis is approximately the current one. This method is
also useful specially when there is a feeling that due to the use of FIFO or
average methods, the profits shown and tax paid are too high. The advantages and
disadvantages of LIFO method are as follows :
Advantages :

1. The cost of materials issued will be either nearer to and or will reflect the
current market price. Thus, the cost of goods produced will be related to the
trend of the market price of materials. Such a trend in price of materials enables
the matching of cost of production with current sales revenues. 2. The use of the
method during the period of rising prices does not reflect undue high profit in
the income statement as it was under the first-in-first-out or average method. In
fact, the profit shown here is relatively lower because the cost of production
takes into account the rising trend of material prices. 3. In the case of falling
prices profit tends to rise due to lower material cost, yet the finished products
appear to be more competitive and are at market price. 4. Over a period, the use
of LIFO helps to iron out the fluctuations in profits. 5. In the period of
inflation LIFO will tend to show the correct profit and thus avoid paying undue
taxes to some extent.
Disadvantages :

1. Calculation under LIFO system becomes complicated and cumbersome when frequent
purchases are made at highly fluctuating rates. 2. Costs of different similar
batches of production carried on at the same time may differ a great deal. 3. In
time of falling prices, there will be need for writing off stock value
considerably to stick to the principle of stock valuation, i.e., the cost or the
market price whichever is lower. 4. This method of valuation of material is not
acceptable to the income tax authorities. (d) Base Stock Method - A minimum
quantity of stock under this method is always held at a fixed price as reserve in
the stock, to meet a state of emergency, if it arises. This minimum stock is known
as base stock and is valued at a price at which the first lot of

2.50
Material

materials is received and remains unaffected by subsequent price fluctuations.


Thus, this is more a method of valuing inventory than a method of valuing issues
because, with the base of stock valued at the original cost some other method of
valuing issues should be adopted. The quantity in excess of the base stock may be
valued either on the FIFO or LIFO basis. This method is not an independent method
as it uses FIFO or LIFO. Its advantages and disadvantages therefore will depend
upon the use of the other method viz., FIFO or LIFO.
Illustration :

�AT� Ltd. furnishes the following store transactions for September, 2006 : 1-9-06
Opening balance 4-9-06 Issues Req. No. 85 6-9-06 Receipts from B & Co. GRN No. 26
7-9-06 Issues Req. No. 97 10-9-06 Return to B & Co. 12-9-06 Issues Req. No. 108
13-9-06 Issues Req. No. 110 15-9-06 Receipts from M & Co. GRN. No. 33 17-9-06
Issues Req. No. 12 19-9-06 Received replacement from B & Co. GRN No. 38 20-9-06
Returned from department, material of M & Co. MRR No. 4 22-9-06 Transfer from Job
182 to Job 187 in the dept. MTR 6 26-9-06 Issues Req. No. 146 30-9-06 Shortage in
stock taking 5 units 10 units 2 units 5 units 10 units 25 units value Rs. 162.50 8
units 50 units @ Rs. 5.75 per unit 12 units 10 units 15 units 20 units 25 units @
Rs. 6.10 per unit 10 units

29-9-06 Transfer from Dept. �A� to Dept. �B� MTR 10 5 units Write up the priced
stores ledger on FIFO method and discuss how would you treat the shortage in stock
taking.

2.51
Solution : Date 1 1-9-06 4-9-06 6-9-06 7-9-06 GRN No. MRR No. 2 � � 26 � Qty.
Units 3 � � 50 � � � � 25 � 10 5 � �

10-9-06 � 12-9-06 � 13-9-06 � 15-9-06 33 17-9-06 � 19-9-06 38 20-9-06 4 26-9-06 �


30-9-06 �

Stores Ledger of AT Ltd. for the month of September, 2006 (FIFO Method) RECEIPT
ISSUE BALANCE Rate Amount Requisi- Qty. Rate Amount Qty. Rate Rs. P Rs. P tion No.
Units Rs. P. Rs. P. Units Rs. P. 4 5 6 7 8 9 10 11 � � � � � � 25 6.50 � � 85 8
6.50 52 17 6.50 5.75 287.50 � � � � 17 6.50 50} 5.75} � � 97 12 6.50 78 5 6.50 50}
5.75 � � Nil 10 5.75 57.50 5 6.50 40} 5.75 � � 108 5 6.50 10} 5.75} 90 30 5.75 � �
110 20 5.75 115 10 5.75 6.10 152.50 � � � � 10 5.75 25} 6.10 � � 121 10 5.75 57.50
25 6.10 5.75 57.50 � � � � 25 6.10 10} 5.75} 5 5.75 5.75 28.75 � � � � 25 6.10 10}
7.75} � � 146 5 5.75 20 6.10 5} 6.10} 59.29 10} 5.75} � � Shortage 2 6.10 12.20 18
6.10 10} 5.75}

Amount Rs. P. 12 162.50 110.50 398.00 320.00 262.00 172.50 57.50 210.00 152.50
210.00 258.75 179.50 167.30
Material

Working Notes :

1.

The material received as replacement from vendor is treated as fresh supply.

2. In the absence of information the price of the material received from within on
20-9-06 has been taken as the price of the earlier issue made on 17-9-06. In FIFO
method physical flow of the material is irrelevant for pricing the issues. 3. The
issue of material on 26-9-06 is made out of the material received from within. 4.
The entries for transfer of material from one job and department to other on 22-9-
06 and 29-9-06 are book entries for adjusting the cost of respective jobs and as
such they have not been shown in the stores ledger account. 5. The material found
short as a result of stock taking has been written off.
Illustration :

The following information is provided by SUNRISE INDUSTRIES for the fortnight of


April, 2006 :
Material Exe :

Stock on 1-4-2006 100 units at Rs. 5 per unit. Purchases 5-4-06 8-4-06 12-4-06
Issues 6-4-06 10-4-06 14-4-06 Required : (A) Calculate using FIFO and LIFO methods
of pricing issues : (a) the value of materials consumed during the period (b) the
value of stock of materials on 15-4-06. (B) Explain why the figures in (a) and (b)
in part A of this question are different under the two methods of pricing of
material issues used. You need not draw up the Stores Ledgers. 250 400 500 units
units units 300 500 600 units at Rs. 6 units at Rs. 7 units at Rs. 8

2.53
Cost Accounting

Solution

(A) (a) Value of Material Exe consumed during the period


1-4-06 to 15-4-06 by using FIFO method. Date Description Units Qty. Rs. Rate Rs.
Amount

1-4-06 5-4-06 6-4-06 8-4-06 10-4-06 12-4-06 14-4-06 15-4-06

Opening balance Purchased Issued Purchased Issued Purchased Issued Balance

100 300 100 150 500 150 250 600 250 250 350

5 6 5? ? 6?
7

500 1,800 1,400 3,500 2,650 4,800 3,750 2,800

6? ? 7? 8 7? ? 8? 8

Total value of material Exe consumed during the period under FIFO method comes to
(Rs. 1,400 + Rs. 2,650 + Rs. 3,750) Rs. 7,800 and balance on 15-4-06 is of Rs.
2,800.
Value of Material Exe consumed during the period 1-4-06 to 15-4-06 by using LIFO
method Date Description Qty. Units 100 300 250 500 400 600 500 350 Rate Rs. 5 6 6
7 7 8 8 � Amount Rs. 500 1,800 1,500 3,500 2,800 4,800 4,000 2,300*

1-4-06 5-4-06 6-4-06 8-4-06 10-4-06 12-4-06 14-4-06 15-4-06


2.54

Opening balance Purchased Issued Purchased Issued Purchased Issued Balance


Material

Total value of material Exe issued under LIFO method comes to (Rs. 1,500 + Rs.
2,800 + Rs. 4,000) Rs. 8,300. *The balance 350 units on 15-4-06 of Rs. 2,300,
relates to opening balance on 1-406 and purchases made on 5-4-06, 8-4-06 and 12-4-
06. (100 units @ Rs. 5, 50 units @ Rs. 6, 100 units @ Rs. 7 and 100 units @ Rs.
8). (b) As shown in (a) above, the value of stock of materials on 15-4-06: Under
FIFO method Rs. 2,800 Under LIFO method Rs. 2,300 (B) Total value of material Exe
issued to production under FIFO and LIFO methods comes to Rs. 7,800 and Rs. 8,300
respectively. The value of closing stock of material Exe on 15-406 under FIFO and
LIFO methods comes to Rs. 2,800 and Rs. 2,300 respectively.
The reasons for the difference of Rs. 500 (Rs. 8,300 � Rs. 7,800) as shown by the
following table in the value of material Exe, issued to production under FIFO and
LIFO are as follows : Date Quantity Issued (Units) Value FIFO Rs. Rs. Total Value
LIFO Rs. Rs. Total

6-4-06 10-4-06 1.

250 400

1,400 2,650

1,500 2,800

14-4-06 500 3,750 7,800 4,000 8,300 On 6-4-06, 250 units were issued to
production. Under FIFO their value comes to Rs. 1,400 (100 units � Rs. 5 + 150
units � Rs. 6) and under LIFO Rs. 1,500 (250 � Rs. 6). Hence, Rs. 100 was more
charged to production under LIFO. On 10-4-06, 400 units were issued to production.
Under FIFO their value comes to Rs. 2,650 (150 � Rs. 6 + 250 � Rs. 7) and under
LIFO Rs. 2,800 (400 � Rs. 7). Hence, Rs. 150 was more charged to production under
LIFO. On 14-4-06, 500 units were issued to production. Under FIFO their value
comes to Rs. 3,750 (250 � Rs. 7 + 250 � Rs. 8) and under LIFO Rs. 4,000 (500 � Rs.
8). Hence, Rs. 250 was more charged to production under LIFO.

2.

3.

Thus the total excess amount charged to production under LIFO comes to Rs. 500.

2.55
Cost Accounting

The reasons for the difference of Rs. 500 (Rs. 2,800 � Rs. 2,300) in the value of
350 units of Closing Stock of material Exe under FIFO and LIFO are as follows : 1.
In the case of FIFO, all the 350 units of the closing stock belongs to the
purchase of material made on 12-4-06, whereas under LIFO these units were from
opening balance and purchases made on 5-4-06, 8-4-06 and 12-4-06. Due to different
purchase price paid by the concern on different days of purchase, the value of
closing stock differed under FIFO and LIFO. Under FIFO 350 units of closing stock
were valued @ Rs. 8 p.u. Whereas under LIFO first 100 units were valued @ Rs. 5
p.u., next 50 units @ Rs. 6 p.u., next 100 units @ Rs. 7 p.u. and last 100 units @
Rs. 8 p.u.

2.

Thus under FIFO, the value of closing stock increased by Rs. 500.
Illustration

The following transactions in respect of material Y occurred during the six months
ended 30th June, 2006:
Month Purchase (units) Price per unit Rs. Issued units Nil

January February March April May June Required :

200 300 425 475 500 600

25 24 26 23 25 20

250 300 550 800 400

(a) The Chief Accountant argues that the value of closing stock remains the same
no matter which method of pricing of material issues is used. Do you agree? Why or
why not? Detailed stores ledgers are not required. (b) When and why would you
recommend the LIFO method of pricing material issues ?
Solution

(a) The Closing Stock at the end of six months period i.e., on 30th June, 2006
will be 200 units, whereas up to the end of May 2006, total purchases coincide
with the total
2.56
Material

issues i.e., 2,300 units. It means that at the end of May 2006, there was no
closing stock. In the month of June 2006, 600 units were purchased out of which
400 units were issued. Since there was only one purchase and one issue in the
month of June, 2006 and there was no opening stock on 1st June 2006, the Closing
Stock of 200 units is to be valued at Rs. 20 per unit. In view of this, the
argument of the Chief Accountant appears to be correct. Where there is only one
purchase and one issue in a month with no opening stock, the method of pricing of
material issues becomes irrelevant. Therefore, in the given case one should agree
with the argument of the Chief Accountant that the value of Closing Stock remains
the same no matter which method of pricing the issue is used. It may, however, be
noted that the argument of Chief Accountant would not stand if one finds the value
of the Closing Stock at the end of each month. (b) LIFO method has an edge over
FIFO or any other method of pricing material issues due to the following
advantages : (i) The cost of the materials issued will be either nearer or will
reflect the current market price. Thus, the cost of goods produced will be related
to the trend of the market price of materials. Such a trend in price of materials
enables the matching of cost of production with current sales revenues. The use of
the method during the period of rising prices does not reflect undue high profit
in the income statement, as it was under the first-in-first-out or average method.
In fact, the profit shown here is relatively lower because the cost of production
takes into account the rising trend of material prices.

(ii)

(iii) In the case of falling prices, profit tends to rise due to lower material
cost, yet the finished products appear to be more competitive and are at market
price. (iv) During the period of inflation, LIFO will tend to show the correct
profit and thus, avoid paying undue taxes to some extent.
(c) Simple Average Price Method - Under this method, materials issued are valued
at average price, which is calculated by dividing the total of all units rate by
the number of unit rate. In other words :

Material issue price =

Total of unit prices of each purchase Total number of purchases

This method is useful under the following circumstances : 1. 2. When the materials
are received in uniform lots of similar quantity, otherwise, it will give wrong
results. When purchase prices do not fluctuate considerably.

2.57
Cost Accounting

Advantage : It is simple to understand and easy to operate. Disadvantages :

1. Materials issue cost does not represent actual cost price. Since the materials
are issued at a price obtained by averaging cost prices, a profit or loss may
arise from such type of pricing. 2. In case the prices of material fluctuate
considerably, this method will give incorrect results. 3. The prices of materials
issues used are determined by averaging prices of purchases without giving
consideration to the quantity. Such a price determination is unscientific.
(d) Weighted Average Price Method : This method gives due weights to quantities
purchased and the purchase price, while, determining the issue price. The average
issue price here is calculated by dividing the total cost of materials in the
stock by total quantity of materials prior to each issue. The advantages and
disadvantages of this method are : Advantages :

1.

It smoothens the price fluctuations if at all it is there due to material


purchases.

2. Issue prices need not be calculated for each issue unless new lot of materials
is received.
Disadvantage :

1. Material cost does not represent actual cost price and therefore, a profit or
loss will arise out of such a pricing method.
(e) Periodic Simple Average Price Method : This method is similar to Simple
Average Price Method except that the average price is calculated at the end of the
concerned period. In other words, the price paid during the period for different
lots of materials purchased are added up and the total is divided by the number of
purchases made during the period. The rate so computed is then used to price all
the issues made during the period, and also for valuing the closing inventory of
the period. Advantages :

1.

It is simple to operate, as it avoids calculation of issue price after every


receipt.

2. This method can usefully be employed in costing continuous processes where each
individual order is absorbed into the general cost of producing large quantities
of articles.
Disadvantages :

1. This method cannot be applied in jobbing industry where each individual job
order is to be priced at each stage of its completion. 2. This method is
unscientific as it does not take into consideration the quantities

2.58
Material

purchased at different prices. 3. This method also suffers from all those
disadvantages of simple average cost method.
(f) Periodic Weighted Average Price Method : This method is like weighted average
price method, except that the calculations of issue prices are made periodically
(say, a month). The rate so arrived is used for the issues made during that period
and also for valuing the inventory at the end of the period. Advantage :

1. This method is superior to the periodic simple average price method as it takes
into account the quantities also. 2. It overcomes or evens out the effect of
fluctuations. 3. In addition to above, the method also possesses all the
advantages of the simple weighted average price method.
Disadvantage : This method is not suitable for job costing because each job is to
be priced at each stage of completion. (g) Moving Simple Average Price Method :
Under this method, the rate for material issues is determined by dividing the
total of the periodic simple average prices of a given number of periods by the
numbers of periods. For determining the moving simple average price, it is
necessary to fix up first period to be taken for determining the average. Suppose
a six monthly period is decided upon and moving, average rate for the month of
June is to be calculated. Under such a situation, we have to make a list of the
simple average prices from January to June, add them up, and divide the total by
six. To calculate the moving average rate for July, we have to omit simple average
rate pertaining to January and add the rate relating to July and divide the total
by six. Advantage : This method evens out price fluctuations over a longer period,
thus stabilising the charges to work-in-progress. Thus the cost of production will
be stable to a significant extent. Disadvantage : A profit or loss arises by the
use of moving simple average cost. (h) Moving Weighted Average Price Method :
Under this method, the issue, rate is calculated by dividing the total of the
periodic weighted average price of a given number of periods by the number of
periods. (i) Replacement Price Method: Replacement price is defined as the price
at which it is possible to purchase an item, identical to that which is being
replaced or revalued. Under this method, materials issued are valued at the
replacement cost of the items. This method pre-supposes the determination of the
replacement cost of materials at the time of each issue; viz., the cost at which
identical materials could be currently purchased. The

2.59
Cost Accounting

product cost under this method is at current market price, which is the main
objective of the replacement price method. This method is useful to determine true
cost of production and to value material issues in periods of rising prices,
because the cost of material considered in cost of production would be able to
replace the materials at the increased price.
Advantage: Product cost reflects the current market prices and it can be compared
with the selling price. Disadvantage: The use of the method requires the
determination of market price of material before each issue of material. Such a
requirement creates problems. (j) Realisable Price Method: Realisable price means
a price at which the material to be issued can be sold in the market. This price
may be more or may be less than the cost price at which it was originally
purchased. Like replacement price method, the stores ledger would show profit or
loss in this method too. (k) Standard Price Method: Under this method, materials
are priced at some predetermined rate or standard price irrespective of the actual
purchase cost of the materials. Standard cost is usually fixed after taking into
consideration the following factors:

(i)

Current prices,

(ii) Anticipated market trends, and (iii) Discount available and transport charges
etc. Standard prices are fixed for each material and the requisitions are priced
at the standard price. This method is useful for controlling material cost and
determining the efficiency of purchase department. In the case of highly
fluctuating prices of materials, it is difficult to fix their standard cost on
long-term basis.
Advantages:

(1) The use of the standard price method simplifies the task of valuing issues of
materials. (2) It facilitates the control of material cost and the task of judging
the efficiency of purchase department. (3) It reduces the clerical work.
Disadvantages:

(1) The use of standard price does not reflect the market price and thus results
in a profit or loss. (2) The fixation of standard price becomes difficult when
prices fluctuate frequently.

2.60
Statement of receipts and issues by adopting First-in-First-Out Method Date
Particulars Units No. Jan. 1 Jan. 20 Jan. 22 Jan. 23 Purchase Purchase Issue to
Job W 16 Issue to Job W 17 100 100 � � Receipts Rate Rs. 1 2 � � Value Rs. 100 200
� � Units No. � � 60 40 20 Date Particulars Units No. Jan. 1 Jan. 20 Jan. 22 Jan.
23 Purchase Purchase Issue to Job W 16 Issue to Job W 17 � � � 40 20} 2 1} 80 20}
100 100 � Receipts Rate Rs. 1 2 � Value Rs. 100 200 � Units No. � � 60 Issues Rate
Rs. � � 1 1 2} Issues Rate Rs. � � 2 Value Rs. � � 120 Units No. 100 100 100} 100
40} 80 Value Rs. � � 60 40 40} 80 2 Balance Rate Rs. 1 1 2} 1 2} 1 Value Rs. 100
100 200} 100 80} 80 160 Units No. 100 100 100} 40 100} Balance Rate Rs. 1 1 2} 1
2} Value Rs. 100 100 200} 40 200}

Statement of receipts and issues by adopting Last-In-First-Out method


Statement of Receipt and Issues by adopting Weighted Average method Date
Particulars Units No. Jan. 1 Jan. 20 Jan. 22 Jan. 23 Purchase Purchase Issue to
Job W 16 Issue to Job W 17 100 100 � � 1 2 � � Rate Rs. 100 200 � � Receipts Value
Units Rs. � � 60 60 No. � � 1.50 1.50 Rate Rs. � � 90 90 Issues Value Units Rs.
100 200 140 80 No. 1 1.50 1.50 1.50 Balance Rate Rs. Value Rs. 100 300 210 120

Statement of Material Values allocated to Job W 16, Job 17 and Closing Stock,
under aforesaid methods FIFO Rs. Material for Job W 16 Material for Job W 17
Closing Stock 60 80 160 300 LIFO Rs. 120 100 80 300 Weighted Average Rs. 90 90 120
300
Material

(n) Inflated Price Method - In case material suffers loss in weight due to natural
or climatic factors, e.g., evaporation, the issue price of the material is
inflated to cover up the losses. (o) Re-use Price Method - When materials are
rejected and returned to the stores or a processed material is put to some other
use, then for the purpose it is meant, then such materials are priced at a rate
quite different from the price paid for them originally. There is no final
procedure for valuing use of material. Illustration :

The following information is extracted from the Stores Ledger:


Material X

Opening Stock
Purchases :

Nil

Jan. 1 Jan. 20
Issues :

100 @ Re. 1 per unit 100 @ Rs. 2 per unit 60 for Job W 16 60 for Job W 17

Jan. 22 Jan. 23

Complete the receipts and issues valuation by adopting the First-In-First-Out,


Last-InFirst-Out and the Weighted Average Method. Tabulate the values allocated to
Job W 16, Job W 17 and the closing stock under the methods aforesaid and discuss
from different points of view which method you would prefer.
Solution

From the point of view of cost of material charged to each job, it is minimum
under FIFO and maximum under LIFO (Refer to Tables on previous pages). During the
period of rising prices, the use of FIFO give rise to high profits and that of
LIFO low profits. In the case of weighted average there is no significant adverse
or favourable effect on the cost of material as well as on profits. From the point
of view of valuation of closing stock it is apparent from the above statement that
it is maximum under FIFO, moderate under weighted average and minimum under LIFO.

2.63
Cost Accounting

It is clear from the Tables on previous page that the use of weighted average
evens out the fluctuations in the prices. Under this method, the cost of materials
issued to the jobs and the cost of material in hands reflects greater uniformity
than under FIFO and LIFO. Thus from different points of view, weighted average
method is preferred over LIFO and FIFO.
2.10 VALUATION OF RETURNS & SHORTAGES 2.10.1 Valuation of Materials Returned to
the Vendor : Materials which do not meet quantity, dimensional and other
specifications and are considered to be unfit for production are usually returned
to the vendor. These materials can be returned to the vendor before they are sent
to the stores. In case materials reach store and are noticed to be of sub-standard
quality, then also they can be returned to vendor. The price of the materials to
be returned to vendor should include its invoice price plus freight, receiving and
handling charges etc. Strictly speaking, the materials returned to vendor should
be returned at the stores ledger price and not at invoice price. But in practice
invoice price is only considered, the gap between the invoice price and stores
ledger price is charged as overhead.

In Stores ledger the defective or sub-standard materials are shown in the issue
column at the rate shown in the ledger, and the difference between issue price and
invoice cost is debited to an inventory adjustment account.
2.10.2 Valuation of Materials Returned to Stores: When materials requisitioned for
a specific job or work-in progress are found to be in excess of the requirement or
are unsuitable for the purpose, they are returned to the stores. There are two
ways of treating such returns.

(1) Such returns are entered in the receipt column at the price at which they were
originally issued, and the materials are kept in suspense, to be issued at the
same price against the next requisition. (2) Include the materials in stock as if
they were fresh purchases at the original issue price.
2.10.3 Valuation of Shortages during Physical Verification: Materials found short
during physical verification should be entered in the issue column and valued at
the rate as per the method adopted, i.e., FIFO or any other. 2.11 SELECTION OF
PRICING METHOD

No hard and fast rule or procedure has been laid down to select a method of
pricing
2.64
Material

issues of material. However, the ultimate choice of a method of selection may be


based on the following considerations. (a) The method of costing used and the
policy of management. (b) The frequency of purchases and issues. (c) The extent of
price fluctuations. (d) The extent of work involved in recording, issuing and
pricing materials. (e) Whether cost of materials used should reflect current or
historical conditions?
2.12 TREATMENT OF NORMAL AND ABNORMAL LOSS OF MATERIALS

Whichever method may be adopted for pricing materials, certain differences between
the book balance and the value of physical stock are bound to occur. These
differences, which may be a gain or loss, should be transferred to Inventory
Adjustment Account pending investigation. If, upon investigation, they are
regarded as normal, they should be transferred to Overhead Control Account; if
abnormal, they should be written off to the Costing Profit and Loss Account. In
the case of normal losses, an alternative method used to price per unit of
material so as to cover the normal loss can be understood with the help of the
example considered. Suppose 1,000 metres of gunny cloth are purchased at Rs. 2 per
metre. It is expected that 1% would be the normal loss due to issues being made in
small lots. The inflated price would be Rs. 2.02 p. i.e., (Rs. 2,000 for 990
metres). The rate of Rs. 2.02 per metre of gunny cloth covers the cost a normal
loss as well.
2.13 ACCOUNTING AND CONTROL OF WASTE, SCRAP, SPOILAGE AND DEFECTIVES 2.13.1 Waste
- It represents the portion of basic raw materials lost in processing having no
recoverable value. Waste may be visible - remnants of basic raw materials - or
invisible; e.g., disappearance of basic raw materials through evaporation, smoke
etc. Shrinkage of material due to natural causes may also be a form of a material
wastage.

Normal waste is absorbed in the cost of net output, whereas abnormal waste is
transferred to the Costing Profit and Loss Account. For effective control of
waste, normal allowances for yield and waste should be made from past experience,
technical factors and special features of the material process and product. Actual
yield and waste should be compared with anticipated figures and appropriate
actions should be taken where necessary. Responsibility should be fixed on

2.65
Cost Accounting

purchasing, storage, maintenance, production and inspection staff to maintain


standards. A systematic procedure for feedback of achievement against laid down
standards should be established.
2.13.2 Scrap - It has been defined as the incidental residue from certain types of
manufacture, usually of small amount and low value, recoverable without further
processing. Scrap may be treated in cost accounts in the following ways:-

(i) (ii)

Where the value of scrap is negligible, it may be excluded from costs. In other
words, the cost of scrap is borne by good units and income scrap is treated as
other income. The sales value of scrap net of selling and distribution cost, is
deducted from overhead to reduce the overhead rate. A variation of this method is
to deduct the net realisable value from material cost. This method is followed
when scraps cannot be aggregated job or process-wise.

(iii) When scrap is identifiable with a particular job or process and its value is
significant, the scrap account should be charged with full cost. The credit is
given to the job or process concerned. The profit or loss in the scrap account, on
realisation, will be transferred to the Costing Profit and Loss Account. Control
of scrap really means the maximum effective utilisation of raw material. Scrap
control does not, therefore, start in the production department; it starts from
the stage of product designing. Thus the most suitable type of materials, the
right type of equipment and personnel would help in getting maximum quantity of
finished product from a given raw material. A standard allowance for scrap should
be fixed and actual scrap should be collected, recorded and reported indicating
the cost centre responsible for it. A periodical scrap report would serve the
purpose where two or more departments or cost centres are responsible for the
scrap; the reports should be routed through the departments concerned.
2.13.3 Spoilage - It is the term used for materials which are badly damaged in
manufacturing operations, and they cannot be rectified economically and hence
taken out of process to be disposed of in some manner without further processing.
Spoilage may be either normal or abnormal.

Normal spoilage (i.e., which is inherent in the operation) costs are included in
costs either charging the loss due to spoilage to the production order or by
charging it to production overhead so that it is spread over all products. Any
value realised from spoilage is credited to production order or production
overhead account, as the case may be.
2.66
Material

The cost of abnormal spoilage (i.e., arising out of causes not inherent in
manufacturing process) is charged to the Costing Profit and Loss Account. When
spoiled work is the result of rigid specification, the cost of spoiled work is
absorbed by good production while the cost of disposal is charged to production
overhead. To control spoilage, allowance for normal spoilage should be fixed and
actual spoilage should be compared with standard set. A systematic procedure of
reporting would help control over spoilage. A systematic procedure of reporting
would help control over spoilage. A spoilage report should highlight the normal
and abnormal spoilage, the department responsible, the causes of spoilage and the
corrective action taken, if any.
2.13.4 Defectives - It signifies those units or portions of production which can
be rectified and turned out as good units by the application of additional
material, labour or other service. For example, some mudguards produced in a
bicycle factory may have dents; or there may be duplication of pages or omission
of some pages in a book. Defectives arise due to sub-standard materials, bad-
supervision, bad-planning, poor workmanship, inadequate-equipment and careless
inspection. To some extent, defectives may be unavoidable but usually, with proper
care it should be possible to avoid defect in the goods produced. Reclamation of
loss from defective units - In the case of articles that have been spoiled, it is
necessary to take steps to reclaim as much of the loss as possible. For this
purpose:

(i) (ii)

All defective units should be sent to a place fixed for the purpose; These should
be dismantled;

(iii) Goods and serviceable parts should be separated and taken into stock; (iv)
Parts which can be made serviceable by further work should be separated and sent
to the workshop for the purpose and taken into stock after the defects have been
removed; and (v) Parts which cannot be made serviceable should be collected in one
place for being melted or sold. Printed forms should be used to record quantities
for all purposes aforementioned.
Control - When defectives are found, the Inspector will make out the Defective
Work Report, giving particulars of the department, process or job, defective
units, normal and abnormal defectives, cost of rectification etc. On receipt of
the defective Work Report, it may be decided to rectify the defective work; all
costs of rectification are collected against the rectification work order,
precaution will be taken to see that number of defectives is

2.67
Cost Accounting

within normal limits. Defectives are generally treated in two ways, either they
are brought up to the standard by incurring further costs on additional material
and labour or where possible, they are sold as inferior production (seconds) at
lower prices. Defectives are generally treated in two ways: either they are
brought up to the standard by incurring further costs on additional material and
labour or where possible, they are sold as inferior products (seconds) at lower
prices. The following illustration is given to explain the accounting procedure
followed in either case. Total expenses of manufacture Output Cost of rectifying
defectives Cost per unit of production = Rs. 5,000 Good: 450 units Defective: 50
units Rs. 50

Rs.5,000 + Rs.50 = Rs. 10.10 per unit 500

If the defectives are not rectified but sold as �second�s say, @ Rs. 8 each then
cost of goods produced will be; =

Rs.5,000 - Rs.400 = Rs. 10.22 per unit. 450

Distinction between spoilage and defectives :

The difference between spoilage and defectives is that while spoilage cannot be
repaired or reconditioned, defectives can be rectified and transferred, either
back to standard production or to seconds.
Treatment of spoilage and defectives in Cost Accounting - Under Cost Accounts
normal spoilage costs i.e., (which is inherent in the operation) are included in
cost either by charging the loss due to spoilage to the production order or
charging it to production overhead so that it is spread over all products. Any
value realised from the sale of spoilage is credited to production order or
production overhead account, as the case may be. The cost of abnormal spoilage
(i.e. arising out of causes not inherent in manufacturing process) is charged to
the Costing Profit and Loss Account. When spoiled work is the result of rigid
specifications the cost of spoiled work is absorbed by good production while the
cost of disposal is charged to production overheads.

The problem of accounting for defective work is the problem of accounting of the
costs of
2.68
Material

rectification or rework. The possible ways of treatment are as below: (i)


Defectives that are considered inherent in the process and are identified as
normal can be recovered by using the following methods: (a) Charged to good
products - The loss is absorbed by good units. This method is used when �seconds�
have a normal value and defectives rectified into �seconds� or �first� are normal;
(b) Charged to general overheads - When the defectives caused in one department
are reflected only on further processing, the rework costs are charged to general
overheads; (c) Charged to the department overheads - If the department responsible
for defectives can be identified then the rectification costs should be charged to
that department; (d) Charged to Costing Profit and Loss Account - If defectives
are abnormal and are due to causes beyond the control of organisation, the rework
cost should be charged to Costing Profit and Loss Accounts. (ii) Where defectives
are easily identifiable with specific jobs, the work costs are debited to the job.

Procedure for the control of Spoilage and Defectives - To control spoilage,


allowance for a normal spoilage should be fixed up and actual spoilage should be
compared with standard set. A systematic procedure of reporting would help control
over spoilage. A spoilage report (as below) would highlight the normal and
abnormal spoilage, the department responsible, the causes of spoilage and the
corrective action taken if any. Spoilage Report

Units/Deptt. No................................. Production Order


No......................
Units Produced Units spoiled Normal Spoilage Abnormal Spoilage % Qty. % Qty.

Date.........................
Cost of abnormal spoilage Rs. Reason spoilage Action taken

2.69
Cost Accounting

Control of defectives may cover the following two areas: (a) Control over
defectives produced (b) Control over reworking costs. For exercising effective
control over defectives produced and the cost of reworking, standards, for normal
percentage of defectives and reworking costs should be established. Actual
performance should be compared with the standards set. Defective Work Report (as
shown on below page should be fed back to the respective Centres of Control.)
Defective Work Report

Dept................. Causes of defects................. Nature of


defects.................
Job/ Defective Detail of work to Normal Abnormal be Materials Rs. Labour Rs
Overhead Rs. done Re-work Costs Total Rs.

Date.................

Unit Cost of reworking Rs.

Net good output after rewroking

Losses due to obsolete stores - Obsolescence is defined as �the loss in the


intrinsic value of an asset due to its supersession�. Materials may become
obsolete under any of the following circumstances:

(i) (ii)

where it is a spare part or a component of a machinery used in manufacture and


that machinery becomes obsolete ; where it is used in the manufacture of a product
which has become obsolete ;

(iii) where the material itself is replaced by another material due to either
improved quality or fall in price. In all three cases, the value of the obsolete
material held in stock is a total loss and immediate steps should be taken to
dispose it off at the best available price. The loss

2.70
Material

arising out of obsolete materials on abnormal loss does not form part of the cost
of manufacture. Losses due to obsolescence can be minimised through careful
forethought and reduced stocking of spares, etc. Stores records should be
continuously gone through to see whether any item is likely to become obsolete.
There will be such likelihood if an item has not been used for a long time. (This
does not apply to spare parts of machines still in use).
2.14 CONSUMPTION OF MATERIALS

Any product that is manufactured in a firm entails consumption of resources like


material, labour etc. The management for planning and control must know the cost
of using these resources in manufacturing. The consumption of materials takes
place say when the material is used in the manufacture of the product. It is
important to note that the amount of materials consumed in a period by a cost
object need not be equal to the amount of material available with the concern. For
example, during any period the total of raw material stock available for use in
production may not be equal to the amount of materials actually consumed and
assigned to the cost object of the production. The difference between the material
available and material consumed represents the stock of material at the end of the
period.
2.14.1 Identification of Materials: For the identification of consumption of
materials with products of cost centres the followings points should be noted:

1. 2.

It is required that the concern should follow coding system for all materials so
that each material is identified by unique code number. It is required that each
product of a cost centre should be given a unique code number so that the direct
material issued for production of particular product of a cost centre can be
collected against the code number of that product. However, it may not be possible
to allocate all materials directly to individual product of a cost centre e.g.
maintenance materials, inspection and testing materials etc. The consumption of
these materials are collected for cost centre and then charged to individual
product by adopting suitable overhead absorption rate of cost centre. Overhead
absorption rate of cost centre =

Cost for cost centre Base relating to cost centre (e.g.labou r hrs. or machine
hrs.)

2.71
Cost Accounting

3.

Each issue of materials should be recorded. One way of doing this is to use a
material requisition note. This note shows the details of materials issued for
product of cost centre and the cost centre which is to be charged with cost of
materials. A material return note is required for recording the excess materials
returned to the store. This note is required to ensure that original product of
cost centre is credited with the cost of material which was not used and that the
stock records are updated. A material transfer note is required for recording the
transfer of materials from one product of cost centre to other or from one cost
centre to other cost centre. The cost of materials issued would be determined
according to stock valuation method used.

4.

5. 6.

2.14.2 Monitoring Consumption of Materials: For monitoring consumption of


materials a storekeeper should periodically analyse the various material
requisitions, material return notes and material transfer notes. Based on this
analysis, a material abstract or material issue analysis sheet is prepared, which
shows at a glance the value of material consumed in manufacturing each product.
This statement is also useful for ascertaining the cost of material issued for
each product. Format of Material Abstract

Week Ending............
Material requisition or Transfer Note or Returned Note No. Amount Product Nos.
Total for Product 101 Rs. � Total Rs. � 102 Rs. � 103 Rs. � 104 Rs. � 105 Rs. �
106 Rs. � � Rs. � Overheads (Indirect Material charged)

The material abstract statement serves a useful purpose. It in fact shows the
amount of material to be debited to various products & overheads. The total amount
of stores debited to various products & overheads should be the same as the total
value of stores issued in any period.
2.14.3 Basis for consumption entries in Financial Accounts: Every manufacturing
organisation assigns material costs to products for two purposes. Firstly, for
external financial accounting requirements in order to allocate the material costs
incurred during
2.72
Material

the period between cost of goods produced and inventories; secondly to provide
useful information for managerial decision making requirements. In order to meet
external financial accounting requirements, it may not be necessary to accurately
trace material costs to individual products. Some products costs may be overstated
and others may be understated but this may not matter for financial accounting
purposes as long as total of individual materials costs assigned to cost of
production and inventories are equal to total cost of materials. In Financial
Accounts the external transactions are recorded i.e. transaction between the firm
and other entities are recorded in a manner that facilitates periodical reporting
of assets, liabilities, revenue and expenditures for a firm as a whole or for each
business segment or geographical segment in which firm operates. In Cost
Accounting the internal transactions are recorded i.e., transactions between cost
centre within the firm are recorded in a manner that facilitates analysis of costs
for assigning them to cost units. The consumption entries in financial accounts
are made on the basis of total cost of purchases of materials after adjustment for
opening and closing stock of materials. The stock of materials is taken at cost or
net realisable value whichever is less.
Self-Examination Questions Multiple Choice Questions

(a) Direct material is a (i) Fixed cost (ii) Variable cost (iii) Semi-variable
cost. (b) In most of the industries, the most important element of cost is (i)
Material (ii) Labour (iii) Overheads. (c) Which of the following is considered to
be the normal loss of materials? (i) Loss due to accidents (ii) Pilferage (iii)
Loss due to breaking the bulk (iv) Loss due to careless handling of materials (v)
All of these.

2.73
Cost Accounting

(d) In which of following methods of pricing, costs lag behind the current
economic values? (i) Last-in-first out price (ii) First-in-first out price (iii)
Replacement price (iv) Weighted average price. (e) Continuous stock taking is a
part of (i) Annual stock taking (ii) Perpetual inventory (iii) ABC analysis. (f)
In which of the following methods, issues of materials are priced at pre-
determined rate? (i) Inflated price method (ii) Standard price method (iii)
Replacement price method (iv) Specific price method. (g) When material prices
fluctuate widely, the method of pricing that gives absurd results is (i) Simple
average price (ii) Weighted average price (iii) Moving average price (iv) Inflated
price. (h) When prices fluctuate widely, the method that will smooth out the
effect of fluctuations is (i) Simple average (ii) Weighted average (iii) FIFO (iv)
LIFO. (i) Which of the following is considered to be a normal loss of material?
(i) Loss due to accidents (ii) Pilferage (iii) Loss due to careless handling of
material. (iv) Loss due to breaking the bulk. (j) Continuous stock taking is a
part of, (i) Annual stock taking (ii) Perpetual inventory

2.74
Material

(iii) ABC analysis (iv) None of the above. (k) Lead time 5 weeks, average weekly
consumption 28 units. What should be the reordering level ? (i) 120 units (ii) 130
units (iii) 140 units (iv) 150 units. (l) Price per unit Rs 150, annual
consumption 2,000 units, ordering cost Rs 300 per order and other charges 20% of
cost. What should be the quantity of each order? (i) 150 units (ii) 200 units
(iii) 225 units (iv) None of the above. (m) Bin card is maintained by the (i)
Accounts department (ii) Costing department (iii) Stores (iv) None of the above.
(n) Bin card contains (i) Details of the price of raw material lying in the Bin
(ii) Details of the price and quantity of raw material lying in the Bin (iii)
Details of quantity of material lying in the Bin (iv) None of the above. (o) Which
of the following assumptions hold true for the calculation of Economic Order
Quantity? (i) Anticipated usage of material in units is known (ii) Cost per unit
of material is constant and known (iii) Ordering cost per order is fixed (iv) None
of the above. Answers to multiple choice questions (a) (ii); (b) (i); (c) (ii);
(e) (ii); (f) (ii); (g) (i); (h) (ii);. (i) ii; (j) ii; (k) iii ; (l) ii; (m) iii
; (n) iii ; (o) iv

2.75
Cost Accounting

Short Answer Type Questions 1. What are the main objectives of material control ?
Explain the important requirements to attain these objectives. 2. Give specimen
form of the following : (i) Bill of material. (ii) Purchase requisition. 3. State
how you would treat the following in cost records : (a) Pricing of materials
returned to stores and (b) Pricing of materials returned to suppliers. 4. What do
you mean by waste, scrap, spoilage and defectives ? How they are treated in Cost
Accounts ? 5. How would you deal the following in Cost Accounts : (i) Carriage
inwards on raw materials. (ii) Cost of handling materials. Long Answer Type
Questions 1. Define re-ordering level and explain its relationship to maximum and
minimum stock levels. What factors may be considered in fixing reordering levels?
2. Discuss the use of perpetual inventory records and continuous stock taking. 3.
Explain the concept of �ABC Analysis� as a technique of inventory control. 4.
Explain FIFO and LIFO methods of valuation of material issue. Discuss the effect
of rising prices and falling prices on these two methods of pricing of material
issues. 5. Discuss in detail the procedure followed for procuring material.
Numerical Questions 1. The rate of interest is 12%, the price per unit is Rs. 50,
the number of units required in a year is 5,000 and the cost of placing one order
and receiving the goods once is Rs. 54. How much should be purchased at a time ?
2. The following is the record of an item in a stores ledger Units Amount Units
Amount Rs. Rs. Jan. 1 To Balance b/d 10,000 20,000 Jan. 15 By Issue 14,000 30,000
Jan. 10 To Purchase 5,000 12,000 Feb. 8 By Issue 6,000 15,000 Feb. 3 To Purchase
20,000 55,000 Mar. 15 By Issue 22,000 60,000 Mar. 10 To Purchase 10,000 30,000
Mar. 31 By Bal. C/d 3,000 12,000 45,000 1,17,000 45,000 1,17,000

2.76
Material

3.

4.

5.

6.

7.

Comment upon the method followed to price the issues. Find out the value of
closing stock assuming issue to have been made for period on (i) FIFO basis, (ii)
LIFO basis, and (iii) Weighted average basis. In a printing press, a form of 8
pages was fitted upside down and this was discovered only after 5,000 sheets had
been printed. The total print order was for 10,000 sheets, the cost per 1,000
sheets being Rs. 50. Would it be correct to say that the loss is Rs. 250 ? A
company ordered 54 tonnes of coal from a colliery. The invoice was for Rs. 1,000,
the freight being Rs. 300. The actual quantity received was 52 tonnes. What should
be the price per tonne ? Ten tonnes of coal were destroyed by an accident quantity
received was 52 tonnes. How should be the loss be treated ? In a section of ready-
made garments factory the monthly wages and overhead respectively amounted to Rs.
5,875 and Rs. 3,650. In a period 10,000 metres of cloth were introduced out of
which 600 metres still remained in stock. It takes 2 metres to make the garments
but in the month, the total output was 4,500 garments. The cost of the cloth is
Rs. 3.50 per metre. Ascertain the cost of garment, assuming cuttings were sold for
Rs. 125. The purchase Department of your organisation has received an offer of
quantity discounts on its orders of materials as under : Price per tonne Tonnes
Rs. Nos. 1,400 Less than 500 1,380 500 and less than 1,000 1,360 1,000 and less
than 2,000 1,340 2,000 and less than 3,000 1,320 3,000 and above. The annual
requirement for the material is 5,000 tonnes. The delivery cost per order is Rs.
1,200 and the annual stock holding cost is estimated at 20% of the average
inventory. The Purchase Department wants you to consider the following purchase
options and advise which among them will be the most economical ordering quantity,
presenting the relevant information in a tabular form. The purchase quantity
options to be considered are 400 tonnes, 500 tonnes, 1,000 tonnes, 2,000 tonnes
and 3,000 tonnes. Component �Pee� is made entirely in cost centre 100. Material
cost is 6 paise per component and each component takes 10 minutes to produce. The
machine operator is

2.77
Cost Accounting

8.

paid 72 paise per hour, and the machine hour rate is Rs. 1.50. The setting up of
the machine to produce the component �Pee� takes 2 hours 20 minutes. On the basis
of this information, prepare a cost sheet showing the production and setting up
cost, both in total and per component, assuming that a batch of : (a) 10
components, (b) 100 components, and (c) 1,000 components (d) is produced. From the
details given below, calculate : (i) Re-ordering level (ii) Maximum level (iii)
Minimum level (iv) Danger level Re-ordering quantity is to be calculated on the
basis of following information. cost of placing a purchase order is Rs. 20 Number
of units to be purchased during the year is 5,000. Purchase price per unit
inclusive of transportation cost is Rs. 50 Annual cost of storage per unit is Rs.
5 Details of lead time : Average 10 days, Maximum 15 days, Minimum 6 days. For
emergency purchase 4 days. Rate of consumption Average : 15 units per day. Minimum
: 20 units per day.

2.78
CHAPTER
LABOUR
Learning objectives

When you have finished studying this chapter, you should be able to ? Understand
the need of labour cost control. ? Understand the attendance and the payroll
procedure. ? Describe the meaning and accounting treatment of idle time and
overtime. ? Understand the concept of labour turnover and the various methods of
computing the same. ? Understand various types of systems of wage payment and
incentives. ? Describe the efficiency rating procedures. 3.1 INTRODUCTION

Labour cost after material cost is another significant element of cost not only
because the wage bill in a modern organisation is generally substantial but also
because it has certain peculiar characteristics which other elements of cost do
not have. A good cost accountant must understand the special features of labour
cost, the most important of which is that there is almost no limit to the increase
of output of this most important and wonderful factor of production. 3.2 LABOUR
COST CONTROL

Labour costs are associated with human beings. To control labour costs one has to
understand why human beings work and what factors motivate them to give best
performance. (Here the students may refer to Chapters 2 and 3 in Organisation and
Management booklet). Control over labour costs does not imply control over the
size of the wage bill; it also does not imply that wages of each worker should be
kept as low as possible. Actually if a policy of low wages is adopted, it may turn
out to be expensive since the ill-paid workers will be dissatisfied and turn out
low output. Low wages are, therefore, often dear wages. The aim should be to keep
the wages cost per unit of output as low as possible. This can only be brought
about by giving workers optimum wages and then harnessing their energies to
optimise output. A well
Cost Accounting

motivated team of workers can bring about wonders. Each concern should, therefore,
constantly strive to raise the productivity of labour. The efforts for the control
of labour costs should begin from the very beginning. There has to be a concerted
effort by all the concerned departments. In a large organisation, generally the
following departments are involved in the control of labour costs : 1. Personnel
Department - This department is assigned the duty of recruiting workers, training
them and maintaining their record. It is the duty of this department to ensure
that the persons recruited possess the qualifications and qualities necessary to
perform well the concerned jobs. Engineering and Work Study Department - This
department prepares plans and specifications for each job, supervises production
activities, conducts time and motion studies, undertakes job analysis, etc. Time-
keeping Department - This Department is primarily concerned with the maintenance
of attendance records of the employees and the time spent by them on various jobs,
etc. Payroll Department - This department is responsible for the preparation of
payroll of the employees. Cost Accounting Department - This department is
responsible for the accumulation and classification etc. of all type of costs. All
such data pertaining to labour costs are also collected, analysed and allocated to
various jobs, processes, departments, etc., by this department.

2.

3. 4. 5.

3.2.1 Important Factors for the Control of Labour Cost : To exercise an effective
control over the labour costs, the essential requisite is efficient utilisation of
labour and allied factors. The main points which need consideration for
controlling labour costs are the following : (i) (ii) Assessment of manpower
requirements. Control over time-keeping and time-booking.

(iii) Time & Motion Study. (iv) Control over idle time and overtime. (v) Control
over labour turnover. (vi) Wage systems. (vii) Incentive systems. (viii) Systems
of wage payment and incentives. (ix) Control over casual, contract and other
workers. (x) Job Evaluation and Merit Rating.

3.2
Labour

(xi) Labour productivity. 3.2.2 Collection of Labour Costs : The task of


collecting labour costs is performed by the Cost Accounting Department which
record separately wages paid to direct and indirect labour. It is the duty of this
department to ascertain the effective wages per hour in each department and to
analyse the total payment of wages of each department into : (i) (ii) the amount
included in the direct cost of goods produced or jobs completed; the amount
treated as indirect labour and thus included in overheads ; and

(iii) the amount treated as the cost of idle time and hence loss. Through this
process costs of various jobs are ascertained. Naturally, in this the proper
recording of time spent by the workers is essential. Labour cost per hour may be
collected through the use of the form given below : A.B.C. Co. Ltd.
Department Name of the employee Section Day Work Hrs. O/Time Hours Labour Cost
Report Productive Time Total Hours Paid Idle Time Hrs. Total Time Hrs. Time Hrs.
O/Time Premium Weekended Wages Paid Bonus Total Cost per Production Hours Rs.

Rs.

Rs.

Rs.

Rs.

3.3

ATTENDENCE & PAYROLL PROCEDURES

3.3.1 Attendance Procedure / Time-keeping : It refers to correct recording of the


employees� attendance time. Students may note the difference between �time
keeping� and �time booking�. The latter refers to break up of time on various jobs
while the former implies a record of total time spent by the workers in a factory.
Objectives of Time-keeping : Correct recording of employees� attendance time is of
utmost importance where payment is made on the basis of time worked. Where payment
is made by results viz; straight piece work, it would still be necessary to
correctly record attendance for the purpose of ensuring that proper discipline and
adequate rate of production are maintained. In fact the various objectives of
time-keeping are as follows: (i) (ii) For the preparation of payrolls. For
calculating overtime.

(iii) For ascertaining and controlling labour cost. (iv) For ascertaining idle
time.

3.3
Cost Accounting

(v) For disciplinary purposes. (vi) For overhead distribution. Methods of Time-
keeping : There are two methods of time-keeping. They are the manual methods and
the mechanical methods. The choice of a particular method depends upon the
requirements and policy of a firm. But whichever method is followed, it should
make a correct record of the time incurring the minimum possible expenditure and
should minimise the risk of fraudulent payments of wages. Manual method : The
manual methods of time-keeping are as follows : (a) Attendance Register Method,
and (b) Metal Disc Method. (a) Attendance Register Method : It is the oldest
method of recording time. Under this method, an attendance register is kept in the
time office adjacent to the factory gate or in each department for workers
employed therein. The attendance register contains such columns as the name of the
worker, the worker�s number, the department in which he is working, the rate of
wages, the time of arrival and departure, normal time and the overtime. The time
of a arrival and departure, may be noted down by an employee know as time-keeper.
This method is simple and inexpensive and can be used in small firms where the
number of workers is not large. This method may lead to dishonest practice of
recording wrong time because there is possibility of collusion between some of the
workers and the time-keeper. However, for recording the time of workers who work
at customers� premises and places which are situated at a distance from the
factory, this may be the only suitable method. (b) Metal Disc Method : Under this
method, each worker is allotted a metal disc or a token with a hole bearing his
identification number. A board is kept at the gate with pegs on it and all token
are hung on this board. These boards can be maintained separately for each
department so that the workers could remove their tokens from the board without
undue delay. As the workers enter the factory gate, they remove their respective
discs or tokens and place them in a box or tray kept near the board. Immediately
after the scheduled time for entering the factory, the box is removed and the late
comers will have to give their tokens to the timekeeper personally so that the
exact time of their arrival could be recorded. The discs or tokens still left on
the board represent the absentee workers. Later the time-keeper records the
attendance in a register known as Daily Muster Roll which is subsequently passed
on to the Pay Roll Department. This method is simple because illiterate workers
can very easily recognize their tokens and put them in the box. This method is
better than attendance register method and is useful when the number of employees
is not large. But it has certain disadvantages of its own as given below : 1.
3.4

There are chances that a worker may try to remove his companion�s token from the
Labour

board in order to get his presence marked when he is absent. 2. There are chances
of disputes regarding the exact time of arrival of a worker because the time-
keeper marking the attendance can commit mistakes deliberately or through
carelessness. There is no authentic proof of the presence or absence of the
workers. There are chances of inclusion of dummy or ghost workers by the time-
keeper in the attendance register or Daily Muster Roll.

3.

Mechanical methods : The mechanical methods that are generally used for the
recording of time of workers may be as follows : (a) Time Recording Clocks; and
(b) Dial Time Records. (a) Time Recording Clocks : The time recording clock is
mechanical device which automatically records the time of the workers. This method
has been developed to obviate some of the difficulties experienced in case of
manual methods and this method is useful when the number of workers is fairly
large. Under this method, each worker is given a Time Card usually of one week
duration. Time cards are serially arranged in a tray near the factory gate and as
the worker enters the gate, he picks up his card from the tray, puts it in the
time recording clock which prints the exact time of arrival in the proper space
against the particular day. This process is repeated for recording time of
departure for lunch, return from lunch and time of leaving the factory in the
evening. Late arrivals, early leavings and overtime are printed in red to attract
the attention of the management. A time card may also give such particulars as
hourly rate, total gross wages, less deductions and net wages payable. If these
particulars are included in the time card, it would be known as combined time and
pay-roll card divided into two parts, the upper part being the record of time and
the lower one serving as the wage ticket. Wages are calculated on the basis of
time recorded in the upper portion and are entered in the lower portion by the
pay-roll department. The specimen of a combined time and pay-roll card may be as
given below :

3.5
Cost Accounting

COMBINED TIME AND PAY ROLL CARD Name of the worker................. No. of the
worker............... Department........................ Regular Day In Monday
Tuesday Wednesday Thursday Friday Saturday Sunday A.M. P.M. A.M. P.M. A.M. P.M.
A.M. P.M. A.M. P.M. A.M. P.M. A.M. P.M. Normal Time Calculation of Wages Over Time
Total Time-keeper....................... Foreman.............................
Received the net amount as above...............
3.6

Week ending...............

Over Time In Out

Total Time Normal Time Overtime

Out

Hours worked

Rate

Amount

Deductions

Net Amount

Pay Roll Clerk..................... Worker...............


Labour

The main advantage of this method is that there are no chances of disputes arising
in connection with recording of time of workers because time is recorded by the
time recording clock and not by the time-keeper. There is no scope for partiality
or carelessness of the timekeeper as it is in case of manual methods. But this
method suffers from the following defects : 1. There are chances that a worker may
try to get his friend�s time card from the tray in order to get him marked present
in time when he is actually late or get his presence marked when he is absent.
This drawback can be removed if the time-keeper does not show carelessness.
Sometimes, the time recording clock goes out of order and the work of recording of
time is dislocated.

2.

(b) Dial Time Records : The dial time recorder is a machine which has a dial
around the clock. This dial has a number of holes (usually about 150) and each
hole bears a number corresponding to the identification number of the worker
concerned. There is one radial arm at the centre of the dial. As a worker enters
the factory gate, he is to press the radial arm after placing it at the hole of
his number and his time will automatically be recorded on roll of a paper inside
the dial time recorder against the number. The sheet on which the time is recorded
provides a running account of the worker�s time. This machine allows greater
accuracy and can itself transcribe the number of hours to the wages sheets. This
machine can also calculate the wages of the workers and thus avoids much loss of
time. However, the high installation cost of the dial time recorder and its use
for only a limited of workers are the drawbacks of this method. Requisites of a
Good Time-keeping System : A good time-keeping system should have a following
requisites : 1. 2. System of time-keeping should be such which should not allow
proxy for another worker under any circumstances. There should also be a provision
of recording of time of piece workers so that regular attendance and discipline
may be maintained. This is necessary to maintain uniformity of flow of production.
Time of arrival as well as time of departure of workers should be recorded so that
total time of workers may be recorded and wages may be calculated accordingly. As
far as possible, method of recording of time should be mechanical so that chances
of disputes regarding time may not arise between workers and the time-keeper.
Late-comers should record late arrivals. Any relaxation by the time-keeper in this
regard will encourage indiscipline. The system should be simple, smooth and quick.
Unnecessary queuing at the factory gate should be avoided. Sufficient clocks
should be installed keeping in view the number

3. 4. 5. 6.

3.7
Cost Accounting

of workers so that workers may not have to wait for a long period for recording
their time of arrivals and departures. 7. A responsible officer should pay
frequent visits at the factory gate to see that proper method of recording of time
is being followed.

Time-Booking - The clock card is required, essentially, for the correct


determination of the amount of wages due to a worker on the basis of time he has
put in the factory. It merely records day by day and period by period the total
time spent by each individual worker in the factory. But it does not show how that
time was put to use in the factory�how an individual worker utilised his time in
completing jobs entrusted to him and how long he was kept waiting for one reason
or another due to lack of work, lack of material and supplies, lack of
instructions, machine breakdowns, power failures and the like. These are all vital
pieces of information necessary for the proper collection of cost data and for
effective controlling of costs. For the collection of all such information, a
separate record, generally known as Time (or Job) card, is kept. The time (or job)
card can be of two types�one containing analysis of time with reference to each
job and the other with reference to each worker. In case of job card made out
according to job a separate job card is employed in respect of a job undertaken;
where a job involves several operations, a separate entry is made in respect of
each operation. Thus the job card would record the total time spent on a
particular job or operation. If a number of people are engaged on the same job or
operation, the time of all those workers would be booked on the same card. One
obvious advantage of this method is that it provides complete data on the labour
content of job or operation collectively so that the computation of labour cost is
greatly facilitated. But this method has drawbacks as well. Since a worker�s job
timing is scattered over a number of job cards the time spent on all these jobs
and idle time must be abstracted periodically for finding each worker�s total time
spent on different jobs and the time for which he remained idle during the period.
The total of these two times (job and idle) must obviously equal his total
attendance time, as shown by his clock card or attendance register. Thus, it would
be seen that if the job cards are made out according to job or operation a
separate summary has to be prepared for reconciling each worker�s job and idle
time with his gate time. It would be quite obvious that such a reconciliation is
of great importance from the point of view of labour costs. If on the other hand,
one job (or time) card were to be issued for each worker, it would greatly
facilitate reconciliation of the worker�s job time with his gate time. Under this
system, a card would be issued to each worker for each day or for each week and
the time which he spends on different jobs (and also any idle time) would be
recorded in the same card so that the card would have a complete history on it as
to how his time had been spent during the period. Since all the details would be
on one card the total time accounted for in the job card would be readily tallied
with the total time put in the Gate Card or attendance register. In this case
3.8
Labour

however, a Labour Abstract for different jobs would have to be prepared from the
card of individual worker so that total hours (and/or their value) put in by
different workers on different jobs during the period could be ascertained and
aggregated. It would thus be seen that according to either of the method a process
of abstraction and reconciliation is necessary. Specimens of two types of job
cards are given below: JOB CARD (1st type) Description of
................................. Department .....................................
Date.............................................. Worker�s No. Start Stop Elapsed
time Actual time taken Supervisor�s initial JOB (OR TIME) CARD (2nd type) No.
.................... Name of the worker .................................
No...............................
Department...........................................
Operation............................... Job No. Start Stop Time Elapsed
Supervisor�s initial Reconciliation of gate and job cards - An advantage of the
introduction of job card is that it enables a reconciliation to be made of the
time spent by the worker in each department with the time paid for as per the
attendance record. Reconciliation not only helps in locating wastage of time, but
also in preventing dummy workers being put on the payroll of workers paid for time
not worked by them. The two sets of records serve separate purposes. Where payment
to labour is on the time rate basis, the Gate Card is a record of the hours of
work that should be paid for. Since the Gate Card merely records the hours during
which the worker has been within the premises of the factory and it does not
contain any details as to how those hours have been put to use by the worker in
his department, a job card must be prepared to provide the necessary information.
As we have already seen, the job card may be prepared either worker-wise or job-
wise. Time Rate Amount Date.................................... Ticket Rate Amount
Job No. ........................................

3.9
Cost Accounting

Objectives of Time-Booking - Objectives of time-booking are as follows: 1. 2. 3.


To ensure that time paid for, according to time keeping, has been properly
utilised on different jobs or work orders. To ascertain the cost of each job or
work order. To provide a basis for the apportionment of overhead expenses over
various jobs/work orders when the method for the allocation of overhead depends
upon time spent on different jobs.

3.3.2 Payroll procedure : The hours worked by each employee as reflected on the
completed clock cards are entered by an accounting department employee (or
employees) on the payroll sheet or payroll summary. All employees authorized for
employment by the personnel department are first listed on the payroll sheet.
Hours and hourly rates are then transferred from the clock cards, and total
earnings are computed. Should a clock card for an employee not listed on the
payroll sheet be found, investigation of its propriety is required. Likewise,
there should be an explanation for any missing clock cards. After the gross
earnings (that is, the total amount earned by an employee before any deductions
are taken into consideration) have been calculated for every employee, deduction
are entered on the payroll sheet, and the net pay of each employee is determined.
Under a computerized system, each employee�s payroll data would be input into the
computer and it would prepare the entire payroll sheet. Payroll deductions are of
two kinds, nontax and tax. Nontax deductions are made at the request of the
employee or are required by union contracts. Among the more common examples are
union dues, hospitalization insurance, withholding for the purchase of savings
bonds, and contributions to charities. Tax deductions are made in compliance with
Income Tax Act. Paying the wages : The payroll sheet is the basis for the
preparation of a payroll voucher by the accounting department authorizing
disbursements for the net amounts payable to employees. If the number of employees
is large, payments are usually made from a special payroll account. Using a
separate payroll account contributes to good internal control, since the audit
trail of payroll activities is easier to follow when a separate account for
payroll disbursements is used by a company. In each pay period an amount to cover
the net payroll is transferred from the company�s general account to a special
payroll account. Cheques payable to the individual employees are then drawn
against the payroll account. In a computerized system, the employees� paycheques
would be printed out by the computer based on the information within the computer-
prepared payroll sheet. In addition to providing a better means of control, use of
a separate account for payroll simplifies record keeping by reducing the number of
cheques that will clear through the general account. Precautions to ensure proper
payment of the payroll should be taken. Payments should be made only to the
employees themselves after proper identification. As a control, payroll
3.10
Labour

cheques should not be given to factory supervisors or department heads for


distribution to the employees under their jurisdiction, since they were probably
involved in the process of accumulating the hours worked by their employees.
Rather, an individual (or individuals) having no record-keeping functions
associated with the payroll (such as the time-keeping function and the preparation
of payroll function) should be assigned the job of distributing paycheques.
Unclaimed paycheques should be investigated to determine why they have not been
picked up by employees. 3.3.3 Overview of Statutory requirements : According to
the Factories Act, 1948, every worker is required to work not more than 9 hours a
day or 48 hours in a week. If, due to the urgency of the work, a worker is
required to work for more than 9 hours a day, excess time over 48 hours i.e.
overtime is to be paid to the worker at a higher rate, generally at double the
normal wage rate. The excess rate over normal wage rate is called overtime
premium. 3.4 IDLE TIME

It is a time during which no production is carried out because the worker remains
idle even though they are paid. Idle time can be normal idle or abnormal idle
time. Normal idle time : It is inherent in any work situation and cannot be
eliminated. Abnormal idle time : Apart from normal idle time, there may be factors
which give rise to abnormal idle time. Machines and men cannot be expected to work
continuously. In the midst of operations, a machine may have to be stopped for
some adjustments being made. Each morning and on the resumption of work in the
post-lunch period, some time will be lost before a job can be started. There may
also be some waiting time in between the finishing of one job and the starting of
another. Similarly, even if a plant operates with a reasonable degree of
efficiency, some allowance may be required for loss of time due to occasional
power failure, machines or tool break-down, delay in delivery of material and
stores or of tools etc. All such reasonable time losses are normal idle time.
Normal idle time of direct workers can be treated as a part of the direct cost.
This can be done by inflating the wage rates for costing purposes. Suppose 10
minutes per hour of work are treated as normal idle time. If a worker puts in 3
hours on a job, his wages for 3� hours can be charged to the job. Alternatively
the cost of normal idle time may be treated as a part of factory overheads. Losses
in excess of the normal idle time are not properly chargeable as overheads;
abnormal losses on account of idle time should be written off by being directly
debited to the Costing Profit and Loss Account. It is obvious that for
establishing abnormal idle time, the normal time required for each product or job
will have to be determined. Accurate recording of the idle time in the departments
is of great importance from the point of view of cost finding and cost control.
For controlling costs, careful analysis and recording of

3.11
Cost Accounting

idle time under significant heads is essential. In order to facilitate


identification, the major causes which account for idle time may be grouped under
the following two heads : (i) Normal causes : Some idle time is inherent in every
situation. The time lost between factory gate and the place of work, the interval
between one job and another, the setting up time for the machine, normal fatigue
etc. result in normal idle time. (ii) Abnormal causes : Idle time may also arise
due to abnormal factors like lack of coordination, power failure, breakdown of
machines, non-availability of raw materials, strikes, lockouts, poor supervision,
fire, flood etc. The causes for abnormal idle time should be further analysed into
controllable and uncontrollable. Controllable abnormal idle time refers to that
time which could have been put to productive use had the management been more
alert and efficient. All such time which could have been avoided is controllable
idle time. However, time lost due to abnormal causes, over which management does
not have any control e.g., breakdown of machines, flood etc. may be characterised
as uncontrollable idle time. Treatment of idle time in Cost Accounting : Normal
idle time is treated as a part of the cost of production. Thus, in the case of
direct workers an allowance for normal idle time is built into the labour cost
rates. In the case of indirect workers, normal idle time is spread over all the
products or jobs through the process of absorption of factory overheads. Abnormal
idle time cost is not included as a part of production cost and is shown as a
separate item in the Costing Profit and Loss Account so that normal costs are not
disturbed. This also helps in drawing the attention of the management towards the
exact losses due to abnormal idle time. The cost of abnormal idle time should be
further categorised into controllable and uncontrollable. For each category, the
break-up of cost due to various factors should be separately shown. This would
help the management in fixing responsibility for controlling idle time. Management
should aim at eliminating controllable idle time and on a long-term basis reducing
even the normal idle time. This would require a detailed analysis of the causes
leading to such idle time. Depending upon the particular causes, proper managerial
action would be required to reduce the impact of such idle time. Basic control can
be exercised through periodical reports on idle time showing a detailed analysis
of the causes for the same, the departments where it is occurring and the persons
responsible for it, along with a statement of the cost of such idle time.
Illustration : �X� an employee of ABC Co. gets the following emoluments and
benefits: (a) Basic pay (b) Dearness allowance (c) Bonus (d) Other allowances
3.12

Rs. 1,000 p.m. Rs. 200 p.m. Rs. 20% of salary and D.A. Rs. 250 p.m.
Labour

(e) Employee�s contribution to P.F. 10% of salary and D.A. �X� works for 2,400
hours per annum, out of which 400 hours are non-productive and treated as normal
idle time. You are required to find out the effective hourly cost of employee �X�.
Solution Statement showing computation of effective hourly cost of employee �X�
(i) Earning of Employee �X�: Per month Rs. Basic pay Dearness Allowance Bonus
Employees� contribution to provident fund Other allowance (ii) Effective working
hours : Annual working hours Less : Normal idle time Effective working hours
Effective hourly cost of �X� : Rs. 21,720/2,000 3.5 OVERTIME 2,400 400 2,000 10.86
1,000 200 240 120 250 1,810 Per annum Rs. 12,000 2,400 2,880 1,440 3,000 21,720

Work done beyond normal working hours is known as �overtime work�. Overtime has to
be paid in India at double the rate of wages including dearness allowance and the
value of food concession, according to the Factories Act, 1948. This Act as stated
earlier also lays down that a worker is entitled to overtime when he works for
more than 9 hours on any day or more than 48 hours in a week. Occasional overtime
is a healthy sign since it indicates that the firm has the optimum capacity and
that the capacity is being fully utilised. But persistent overtime is rather a bad
sign because it may indicate either : (a) that the firm needs larger capacity in
men and machines, or (b) that men have got into the habit of postponing their
ordinary work towards the evening so that they can earn extra money in the form of
overtime wages. Overtime work may arise in a department in one of the following
circumstances : (1) The customer may agree to bear the entire charge of overtime
because of urgency of

3.13
Cost Accounting

work. (2) Overtime may be called for to make up any shortfall in production due to
some unexpected development. (3) Overtime work may be necessary to make up a
shortfall in production due to some fault of management. (4) Overtime work may be
resorted to, to secure an out-turn in excess of the normal output to take
advantage of an expanding market or of rising demand. Overtime premium : Overtime
payment is the amount of wages paid for working beyond normal working hours. The
rate for overtime work is higher than the normal time rate; usually it is at
double the normal rates. The extra amount so paid over the normal rate is called
overtime premium. Effect of overtime payment on productivity : Overtime work
should be resorted to only when it is extremely essential because it involves
extra cost. The overtime payment increases the cost of production in the following
ways : 1. 2. 3. 4. The overtime premium paid is an extra payment in addition to
the normal rate. The efficiency of operators during overtime work may fall and
thus output may be less than normal output. In order to earn more the workers may
not concentrate on work during normal time and thus the output during normal hours
may also fall. Reduced output and increased premium of overtime will bring about
an increase in costs of production.

Treatment of overtime premium in Cost Accounting : Under Cost Accounting the


overtime premium is treated as follows : (1) If overtime is resorted to at the
desire of the customer, then overtime premium may be charged to the job directly.
(2) If overtime is required to cope with general production programmes or for
meeting urgent orders, the overtime premium should be treated as overhead cost of
the particular department or cost centre which works overtime. (3) If overtime is
worked in a department due to the fault of another department, the overtime
premium should be charged to the latter department. (4) Overtime worked on account
of abnormal conditions such as flood, earthquake etc., should not be charged to
cost, but to Costing Profit and Loss Account. Steps for controlling overtime : To
keep overtime to its minimum, it is necessary to exercise proper control over the
overtime work. The suitable procedure which may be adopted for
3.14
Labour

controlling overtime comprises the following steps : 1. Watch on the output during
normal hours should be maintained to ensure that overtime is not granted when
normal output is not obtained during the normal hours, without any special
reasons. Statement concerning overtime work be prepared along with justifications,
at appropriate places for putting up before the competent authority. Prior
sanction about overtime should be obtained from competent authority. Actual rate
of output produced during the overtime period should be compared with normal rate
of output. Periodical reports on overtime wages should be sent to top management
for taking corrective action. If possible an upper limit may be fixed for each
category of workers in respect of overtime.

2. 3. 4. 5. 6.

Illustration : It is seen from the job card for repair of the customer�s equipment
that a total of 154 labour hours have been put in as detailed below : Worker �A�
paid at Rs. 2 per day of 8 hours Monday Tuesday Wednesday Thursday Friday Saturday
Total 10 - 1/2 hours 8� 10 - 1/2 hours 9 - 1/2 � 10 - 1/2 � - 49 hours Worker �B�
paid at Re. 1 per day of 8 hours 8 hours 8� 8� 8� 8� 8� 48 hours Supervisory
worker �C� paid of Rs. 3 per day of 8 hours 10 - 1/2 hrs. 8� 10 - 1/2 hours 9 -
1/2 � 10 - 1/2 � 8� 57 hours

In terms of an award in a labour conciliation, the workers are to be paid dearness


allowance on the basis of cost of living index figures relating to each month
which works out @ Rs. 96 for the relevant month. The dearness allowance is payable
to all workers irrespective of wage rate if they are present or are on leave with
wages on all working days. Sunday is a weekly holiday and each worker has to work
for 8 hours on all week days and 4 hours on Saturdays; the workers are however
paid full wages for Saturday (8 hours for 4 hours worked).

3.15
Cost Accounting

Workers are paid overtime according to the Factories Act for hours worked in
excess of normal working hours on each day. Excluding holidays (including 4 hours
work to be put in on Saturday) the total number of hours work out to 172 in the
relevant month. The company�s contribution to Provident Fund and Employees State
Insurance Premium are absorbed into overheads. Work out the wages payable to each
worker. Solution (1) Calculation of hours to be paid for worker A : Normal hours
Monday Tuesday Wednesday Thursday Friday Saturday 8 8 8 8 8 40 Extra hours 1 1 1 1
4 Overtime hours 1� 1� � 1� 5 Equivalent normal overtime 3 3 1 3 10 Total normal
hours 12 8 12 10 12 54

hours of hours

Worker B will get wages for 52 hours i.e., actually worked plus 4 hours worked
extra on Saturday. Worker C will get wages for 66 hours�the same as for A till
Friday plus 12 hours for work on Saturday. (a) Wages payable : Basic wages per
hour (Rs.) Dearness allowance per hour (Rs.) Hourly rate (Rs.) Normal hours
Overtime hours Normal wages (Rs.) Overtime wages (Rs.) Total wages payable : (Rs.)
A 0.250 0.500 0.750 44 5 33.00 7.50 40.50 B 0.125 0.500 0.625 52 32.50 32.50 C
0.375 0.500 0.875 56 5 49.00 8.75 57.75

3.16
Labour

Illustration In a factory, the basic wage rate is Rs. 10 per hour and overtime
rates are as follows : Before and after normal working hours Sundays and holidays
During the previous year, the following hours were worked : Normal time Overtime
before and after working hours Overtime on Sundays and holidays Total The
following hours have been worked on job �Z� : Normal Overtime before and after
working hrs. Sundays and holidays : : : 1000 hours. 100 hours. 25 hours. : : : :
1,00,000 hours 20,000 hours 5,000 hours 1,25,000 hours : : 175% of basic wage rate
225% of basic wage rate

Total : 1125 hours. You are required to calculate the labour cost chargeable to
jobs �Z� and overhead in each of the following instances: (a) Where overtime is
worked regularly throughout the year as a policy due to the labour shortage. (b)
Where overtime is worked irregularly to meet the requirements of production. (c)
Where overtime is worked at the request of the customer to expedite the job.
Solution Workings Computation of average inflated wage rate (including overtime
premium) : Basic wage rate Overtime wage rate before and after working hours
Overtime wage rate for Sundays and holidays Annual wages for the previous year for
normal time Wages for overtime before and after working hours : Rs. 10 per hour :
Rs. 10 � 175% = Rs. 17.50 per hour : Rs. 10 � 225% =Rs. 22.50 per hour : 1,00,000
hrs. � Rs. 10 = Rs. 10,00,000 : 20,000 hrs. � Rs. 17.50=Rs. 3,50,000

3.17
Cost Accounting

Wages for overtime on Sundays and holidays Total wages for 1,25,000 hrs. Average
inflated wage rate

: 5,000 hrs. � Rs. 22.50 = Rs. 1,12,500 = Rs. 14,62,500 : Rs. 14,62,500 = Rs.
11.70 per hour.

= 1,25,000 hrs. (a) Where overtime is worked regularly as a policy due to labour
shortage, the overtime premium is treated as a part of labour cost and job is
charged at an inflated wage rate. Hence, Labour cost chargeable to job Z = Total
hours � Inflated wage rate = 1,125 hrs. � Rs. 11.70 = Rs. 13,162.50 (b) Where
overtime is worked irregularly to meet the requirements of production, basic wage
rate is charged to the job and overtime premium is charged to factory overheads as
under : Labour cost chargeable to Job Z : 1,125 hours @ Rs. 10 per hour Factory
overhead : 100 hrs. � Rs. (17.50 � 10) 25 hrs. � Rs. (22.50 � 10) = = = Rs.
11,250.00 Rs. Rs. 750.00 312.50

Total factory overhead Rs. 1,062.50 (c) Where overtime is worked at the request of
the customer, overtime premium is also charged to the job as under : Rs. Job Z
labour cost Overtime premium Total 3.6 LABOUR TURNOVER 1,125 hrs. @ Rs. 10 100
hrs. @ Rs. (17.50 � 10) 25 hrs. @ Rs. (22.50 � 10) = = = 11,250.00 750.00 312.50
12,312.50

Labour turnover in an organisation is the rate of change in the composition of


labour force during a specified period measured against a suitable index. The
standard of usual labour turnover in the industry or locality or the labour
turnover rate for a past period may be taken as the index or norm against which
actual turnover rate is compared. There are three methods of calculating labour
turnover which are given below :

3.18
Labour

(i) (ii)

Replacement method = Separation method =

Number of employees replaced � 100 Average number of employees on roll

Number of employees separated during the year � 100 Average number of employees on
rolls during the period

(iii) Flux method =

Number of employees separated + number of employees replaced � 100 Average number


of employees on rolls during the period

Labour turnover due to new recruitment: Workers joining a business concern on


account of its expansion do not account for labour turnover. But these newly
recruited workers are certainly responsible for a change in the composition of
labour force, due to this feature, some cost accountants measure workers to the
extent of new (excluding replacements) joining the labour force as follows :

No. of new workers joining in a period (excluding replacements) � 100 Average


number of workers on the roll in a period
The total number of workers joining, including replacements, are called
accessions. The labour turnover rate, in such a case, may also be computed in
respect of total number of workers joining (accessions) the business concern,
during a given period both on account of replacements and because of expansion is
as under :

No.of accessions in a period � 100 Average number of workers in a period


When number of accessions are considered for measuring labour turnover, the labour
turnover rate by flux method may be computed by using any one of the following
expressions :
Labour turnover rate (Flux method) = No. of separations + No. of replacements +
No. of new recruitments � 100 Average number of workers

OR

No. of separations + No. of accessions � 100 Average number of workers


The above rate of labour turnover indicates the total effect of number of workers
separated, number of workers replaced and number of new workers recruited and
joined the concern on account of its expansion, etc. If in the above computations,
the data given is for a period other than a year, the labour turnover rate so
computed may be converted into equivalent annual labour turnover rate by the
following formula:

3.19
Cost Accounting

Equivalent annual labour turnover rate

Turnover rate for the period � 365 Number of days in the period

Causes of labour turnover : The main causes of labour turnover in an


organisation/industry can be broadly classified under the following three heads :
(a) Personal Causes; (b) Unavoidable Causes; and (c) Avoidable Causes. Personal
causes are those which induce or compel workers to leave their jobs; such causes
include the following : (i) (ii) Change of jobs for betterment. Premature
retirement due to ill health or old age.

(iii) Domestic problems and family responsibilities. (iv) Discontent over the jobs
and working environment. In all the above cases the employee leaves the
organisation at his will and, therefore, it is difficult to suggest any possible
remedy in the first three cases. But the last one can be overcome by creating
conditions leading to a healthy working environment. For this, officers should
play a positive role and make sure that their subordinates work under healthy
working conditions. Unavoidable causes are those under which it becomes obligatory
on the part of management to ask one or more of their employees to leave the
organisation; such causes are summed up as listed below: (i) (ii) Seasonal nature
of the business; Shortage of raw material, power, slack market for the product
etc.;

(iii) Change in the plant location; (iv) Disability, making a worker unfit for
work; (v) Disciplinary measures; (vi) Marriage (generally in the case of women).
Avoidable causes are those which require the attention of management on a
continuous basis so as to keep the labour turnover ratio as low as possible. The
main causes under this case are indicated below : (1) Dissatisfaction with job,
remuneration, hours of work, working conditions, etc.,

3.20
Labour

(2) Strained relationship with management, supervisors or fellow workers; (3) Lack
of training facilities and promotional avenues; (4) Lack of recreational and
medical facilities; (5) Low wages and allowances. Proper and timely management
action can reduce the labour turnover appreciably so far as avoidable causes are
concerned. Effects of labour turnover High labour turnover increases the cost of
production in the following ways: (i) (ii) Even flow of production is disturbed;
Efficiency of new workers is low; productivity of new but experienced workers is
low in the beginning;

(iii) There is increased cost of training and induction; (iv) New workers cause
increased breakage of tools, wastage of materials, etc. In some companies, the
labour turnover rates are as high as 100%; it means that on the average, all the
work is being done by new and inexperienced workers. This is bound to reduce
efficiency and production and increases the cost of production. Two types of costs
which are associated with labour turnover are : (a) Preventive costs : These
include costs incurred to keep the labour turnover at a low level, i.e. cost of
medical services, welfare schemes and pension schemes. If a company incurs high
preventive costs, the rate of labour turnover is usually low. (b) Replacement
costs : These are the costs which arise due to high labour turnover. If men leave
soon after they acquire the necessary training and experience of good work,
additional costs will have to be incurred on new workers, i.e., cost of
employment, training and induction, abnormal breakage and scrap and extra wages
and overheads due to the inefficiency of new workers. It is obvious that a company
will incur very high replacement costs if the rate of labour turnover is high.
Similarly, only adequate preventive costs can keep labour turnover at a low level.
Each company must, therefore, work out the optimum level of labour turnover
keeping in view its personnel policies and the behaviour of replacement cost and
preventive costs at various levels of labour turnover rates. Remedial steps to
minimise labour turnover - The following steps are useful for minimising labour
turnover. 1. Exit interview : An interview be arranged with each outgoing employee
to ascertain the

3.21
Cost Accounting

reasons of his leaving the organisation. 2. 3. Job analysis and evaluation :


Before recruiting workers, job analysis and evaluation may be carried out to
ascertain the requirements of each job. Scientific system of recruitment,
placement and promotion - The organisation should make use of a scientific system
of recruitment, selection, placement and promotion for employees. Enlightened
attitude of management - The management should introduce the following steps for
creating a healthy working atmosphere : (i) (ii) 5. Service rules should be
framed, discussed and approved among management and workers, before their
implementation. Provide facilities for education and training of workers.

4.

(iii) Introduce a procedure for settling worker�s grievances. Use of committee :


Issues like control over workers, handling their grievances etc., may be dealt by
a committee, comprising of members from management and workers.

Illustration The management of Bina and Rina Ltd. are worried about their
increasing labour turnover in the factory and before analyzing the causes and
taking remedial steps, they want to have an idea of the profit foregone as a
result of labour turnover in the last year. Last year sales amounted to Rs.
83,03,300 and P/V ratio was 20 per cent. The total number of actual hours worked
by the Direct Labour force was 4.45 Lakhs. As a result of the delays by the
Personnel Department in filling vacancies due to labour turnover, 1,00,000
potentially productive hours were lost. The actual direct labour hours included
30,000 hours attributable to training new recruits, out of which half of the hours
were unproductive. The costs incurred consequent on labour turnover revealed, on
analysis, the following : Settlement cost due to leaving Recruitment costs
Selection costs Rs. 43,820 Rs. 26,740 Rs. 12,750

Training costs Rs. 30,490 Assuming that the potential production lost as a
consequence of labour turnover could have been sold at prevailing prices, find the
profit foregone last year on account of labour turnover.

3.22
Labour

Solution Determination of contribution foregone Actual hours worked (given) Less :


Unproductive training hours Actual productive hours The potentially productive
hours lost are 1,00,000 Sales lost for 1,00,000 hours =
Rs.83,03,300 � 1,00,000 hrs = Rs. 19,31,000 4,30,000 hrs

4,45,000 15,000 4,30,000

Contribution lost for 1,00,000 hrs. Rs.19,31,000 = � 20 = Rs.


3,86,200...........(i) 100 Statement showing profit foregone last year on account
of labour turnover of Bina and Rina Ltd. Rs. Contribution foregone as per (i)
Settlement cost due to leaving Recruitment cost Selection cost Training costs
Profit foregone Illustration The Cost Accountant of Y Ltd. has computed labour
turnover rates for the quarter ended 31st March, 2007 as 10%, 5% and 3%
respectively under �Flux method�, �Replacement method� and �Separation method�
respectively. If the number of workers replaced during that quarter is 30, find
out the number of: (1) workers recruited and joined and (2) workers left and
discharged. 3,86,200 43,820 26,740 12,750 30,490 5,00,000

3.23
Cost Accounting

Solution Working Note: Average number of workers on roll:

Labour turnover rate under replacement method =


5 100

No. of replacements � 100 Average number of workers on roll


30 Average number of workers on roll

Or

Or Average number of workers on roll =

30� 100 = 600 5

(1) Number of workers recruited and joined : Labour turnover rate = (Flux method)
(Refer to Working Note) Or Or

No. of separations * (S) + No. of accessions(A) � 100 Average number of workers on


roll

10 = 100
A =

18 * + A 600
? 6000 ? ? 100 -18? = 42 ? ?

No. of workers recruited and joined 42. (2) Number of workers left and discharged
: Labour turnover rate = (Separation method) (Refer to working note)

No. of separations(S) � 100 Average number of workers on roll

3 S = 100 600
Or S*
3.24

S* = 18
Labour

Hence, number of workers left and discharged comes to 18. 3.7 INCENTIVE SYSTEM

3.7.1 Important factors necessary for introducing an incentive system : An


incentive can be defined as the stimulation for effort and effectiveness by
offering monetary inducement or enhanced facilities. It may be monetary in the
form of a bonus or non-monetary tending to improve living and working conditions.
It may be provided individually or collectively. In the first case, the employee
gets a reward for his efforts directly and in the second, a group of employees
share the reward arising out of their combined effort in equitable production. The
main factors that should be taken into account before introducing a scheme of
incentives are stated below : (i) The need for producing goods of high quality or
those having very good workmanship or finish and the manner this can be ensured.
Only if a system of quality control can be relied upon to maintain the quality of
goods of the standard required, an incentive scheme should be introduced;
otherwise, workers should be paid on time basis. The need to maximise
production�thus required incentives to be given to workers. But sometimes
workmanship is more important than quantity of output; in such cases, incentive
schemes of wage payment are not suitable.

(ii)

(iii) Where the quantity of work done cannot be measured precisely, incentive
schemes cannot be offered. (iv) The role of management and workers in achieving
greater efficiency, if it is unnecessary for the management to constantly plan
work for example, when the work is repetitive, workers should be offered good
incentives to achieve high efficiency; but in case management is constantly
required to plan the work, as in the case of job work, the management should share
the fruits of extra efficiency achieved. This factor determines the choice of a
particular incentive scheme. (v) Whether the quantity of output is within the
control of the worker and if so, to what extent. Sometimes, as in the case of
chain assembly work the output is not dependent on the effort put in by workers;
incentive schemes in such cases are not suitable. (vi) The exactitude with which
standards of performance can be laid down. Fixation of standard is necessary for
the introduction of a scheme of incentives. When this requires heavy expenditure,
incentive schemes may be rather costly. (vii) The effect of an incentive scheme
for one set of workers on other workers. If for instance, an incentive scheme
makes it possible for unskilled workers to earn high wages, the wage rates for
skilled workers must also be raised (if they are paid on time basis) to avoid
dissatisfaction among them. In that event, the incentive scheme may raise

3.25
Cost Accounting

labour cost instead of lowering it. (viii) The system of wage payment prevailing
in other areas and industries or similar occupations. If possible, there should be
uniformity. (ix) The attitude of labour and trade unions towards incentive
schemes. Workers usually like to have a certain guaranteed time-basis wage but
also like to earn extra through an incentive scheme. On the whole, the system of
wage payment should be such as would increase production without lowering quality.
This will increase the surplus and will enable the employer to pay higher wages
which, in turn, will lead to higher output. 3.7.2 Main principles for a sound
system of wage incentive : The objective of wage incentives is to improve
productivity and increase production so as to bring down the unit cost of
production. In order to make the incentive scheme effective and useful, the
following general principles have to be considered while designing a sound system
of wage incentives. (i) The reward for a job should be linked with the effort
involved in that job and the scheme should be just and fair to both employees and
employers. This involves the following : (a) The standard required of the workers
should be carefully set, if possible through proper time and motion studies. (b)
If the work is of repetitive type, the entire benefit of the time saved should be
available to the worker but, in the case of non-standardised work or where precise
standards cannot be set, the benefit of the time saved, if any, should be shared
by the employer, the supervisor and the worker. (ii) The scheme should be clearly
defined and be capable of being understood by the employees easily. The standards
set should be such that they can be achieved even by average employees. While
standards are being set, the workers concerned should be consulted.

(iii) As far as possible, no limit should be placed on the amount of additional


earnings, otherwise it will dampen the initiative of the workers. In this regard,
what is important is not what actually prevails but what the workers think�if they
think, even wrongly, that the employer will stop wages from rising beyond a
certain limit, the incentive scheme may not be really effective. (iv) The scheme
should be reasonable and stable, and should not be changed or modified too often
without consulting the employees. (v) The scheme should take care that the
employees are not penalised for reasons beyond their control. (vi) The scheme
should provide for inspection of output so that only good pieces qualify for
3.26
Labour

incentives. It would even be better not to introduce any incentive scheme if


workmanship is of vital importance in sales. (vii) The management should ensure
that there is no cause for complaint by the workers that they are sitting idle,
say for want of tools or materials. Management has to see that there is, as far as
practicable, no interruption of production. (viii) The operation of the scheme
should not entail heavy clerical costs. In fact the scheme should facilitate the
introduction of budgetary control and standard costing. (ix) It should be capable
of improving the morale of the employees and it should be in conformity with the
local trade union agreements and other government regulations. (x) There should be
a guaranteed wage on time basis which generally works as a good psychological
boost to incentive scheme. (xi) Last, but not least, the effect of incentive
scheme on those who cannot be covered should be gauged and taken note of.
Sometimes, highly skilled workers have perforce to be paid on time basis whereas
semiskilled or unskilled workers may be put on incentive scheme. If the latter
earn more than former, the incentive schemes on the whole prove harmful. 3.7.3
Essential characteristics of a good incentive system : to recapitulate (i) (ii) It
should be just both to the employer and to the employee. It should be positive and
not unnecessarily punitive and so operated as to promote confidence. It should be
strong both ways i.e. it should have a standard task and a generous return. The
latter should be in direct proportion to employee�s efforts. It should reflect the
employer�s contribution to the success of the company.

(iii) It should be unrestricted as to the amount of the earning. (iv) It should be


reasonable, apart from being simple, for employee to figure out his incentive in
relation to his individual performance, as far as practicable. (v) It should be
flexible and intimately related to other management controls. (vi) It should
automatically assist supervision and, when necessary, aid team work. (vii) It
should have employee�s support and in no way should it be paternalistic. (viii) It
should have managerial support in so far as production material, quality control,
maintenance and non-financial incentives are concerned. (ix) It should not be used
temporarily and dropped in recission times as means of wage reduction. 3.7.4
Procedure for laying down an incentive system : An incentive is a reward for
effort made; hence correct measurement of the effort involved is a prerequisite
for any incentive

3.27
Cost Accounting

system. Measurement of effort is made by time and motion study by specialists


appointed for the purpose. Based on its finding and that of job evaluation, the
rates of wages are fixed for different operations. The levels of efficiency that
must be attained to qualify for incentives are then fixed on a consideration of
the factors mentioned above. Having drawn up the broad outlines of the scheme, the
next step is to educate the workers as regards the benefits of the proposed
scheme. This is done through joint consultation with the leading employees or with
union representatives. The scheme is then publicised extensively with the
specimen, calculations of the rewards that would arise under it. After the basic
scheme has been accepted by all, a decision on two vital points will have to be
made viz. how spoiled work will be treated and the intervals at which payment of
wages will be made. An incentive system tends to increase the rate of production
and consequently increases spoilage. But the very purpose of the scheme would be
defeated if spoilage increases beyond a certain limit. It is, therefore, necessary
that the method of treating the spoiled work should be agreed upon in advance. The
prevention of spoiled work can be encouraged either by making the worker do the
job again in his own time or by paying the worker at the time rate for the period
covered by spoiled work not giving him credit at all for the spoiled work; of
these, the first method is more commonly employed since it is both equitable and
deterrent. The incentive should be paid promptly at short intervals of time. This
would give the worker immediate satisfaction of having earned something by the
extra effort he had put in. If payment is delayed the effect of the incentive
would be greatly diminished. 3.8 LABOUR UTILISATION

For identifying utilisation of labour a statement is prepared (generally weekly)


for each department / cost centre. This statement should show the actual time paid
for, the standard time (inlcuding normal idle time) allowed for production and the
abnormal idle time analysed for causes thereof. 3.8.1 Distinction between Direct
and Indirect Labour Cost : Any labour cost that is specifically incurred for or
can be readily charged to or identified with a specific job, contract,. work order
or any other unit of cost, is termed as direct labour cost; it includes : (i) all
labour that is engaged in converting raw materials into manufactured articles in
the case of manufacturing industries, and (ii) other forms of labour, which
although not immediately engaged in converting raw materials into manufactured
articles nonetheless is incurred wholly or specifically for any particular unit of
production and hence, can be readily identified with the unit of production
(Example: A helper attending solely a machine operator in the case of
manufacturing industries ; the entire contingent of labour and staff employed on a
construction job or project). Any labour that does not meet the above test is
indirect, e.g. men generally employed in machine shop such as tool setters,
fitters, workers in tool room, stores, etc. Their wages are charged as indirect
expenses.
3.28
Labour

Page � 26 on Landscape file Blank page for insertion

3.29
Cost Accounting

Thus direct labour is for a specific job or product while indirect labour is for
work in general. In a printing press, for example, wages paid to compositors will
be direct while wages paid to the cleaners of machines will be indirect. The
distinction is one relative to each particular firm or industry. Labour which is
direct in one unit may indeed be indirect in another where the work or process or
method of manufacture is different in nature. The importance of the distinction
lies in the fact that whereas direct labour can be identified with and charged to
the job, indirect labour cannot be so charged and has therefore to be treated as
part of the factory overheads to be included in the cost of production on some
suitable basis of apportionment and absorption. 3.8.2 Identification of
utilisation of labour with cost centres : For the identification of utilisation of
labour with the cost centre a wage analysis sheet is prepared. Wage analysis sheet
is a columnar statement in which total wages paid are analysed according to cost
centre, jobs, work orders etc. The data for analysis is provided by wage sheet,
time card, piece work cards and job cards. The preparation of such sheet serves
the following purposes : (i) (ii) It analyse the labour time into direct and
indirect labour by cost centres, jobs, work orders. It provides details of direct
labour cost comprises of wages, overtime to be charged as production cost of cost
centre, jobs or work orders. Wage Analysis Sheet No. week- Department/ ending cost
centre Total Work in progress Factory control A/c Administration A/c Selling &
Distribution overhead control Account Overhead Overhead control

(iii) It provides information for treatment of indirect labour cost as overhead


expenses.

3.30
Labour

ABC & CO. Wages Analysis Sheet (Sharing break�up of work in progress)
Job No. 1 Clock No. Hrs. Job No. 2 Amount Clock No. Hrs. Amount Job No. 3 Clock
No. Hrs. Amount Job No. Hrs. Summary Cost Ledger Folio Amount Total

Total

3.8.3 Identification of labour hours with work order or batches or capital job :
For identification of labour hours with work order or batches or capital jobs or
overhead work orders the following points are to be noted : (i) The direct labour
hours can be identified with the particular work order or batches or capital job
or overhead work orders on the basis of details recorded on source document such
as time sheet or job cards. The indirect labour hours cannot be directly
identified with the particular work order or batches or capital jobs or overhead
work orders. Therefore, they are traced to cost centre and then assigned to work
order or batches or capital jobs or overhead work orders by using overhead
absorption rate. SYSTEMS OF WAGE PAYMENT AND INCENTIVES

(ii)

3.9

There exist several systems of employee wage payment and incentives, which can be
classified under the following heads :

3.31
Cost Accounting

SYSTEMS OF WAGE PAYMENT & INCENTIVES


Time Rate Systems Payment by Results

Time Rate System

High Wage Plan

Measured Day Work

Differential Time Rate

Piece Work

Combination of Time and Piece Work

Premium Bonus Methods

Group System of wage payment

System of Incentive Schemes for Indirect Workers

Straight Piece Work

Differential Piece Work

Halsey Rowan and Halsey System Weir Systems Merrick System

Barth System

Taylor System Gantt Task and Bonus System

Emerson�s Efficiency System

Points Scheme

Accelerated Premium System

Beadaux Syst.

Haynes Syst.

One should remember that Provident Fund, Employees State Insurance Corporation
Premium and bonus are payable on the basic wages, dearness allowance and value of
food concession. 3.9.1 Time rate system : It is perhaps the oldest system of
remunerating labour. It is also known by other names such as time work, day work,
day wages and day rates. Under this system, the worker is paid by the hour, day,
week, or month. The amount of wages due to a worker are arrived at by multiplying
the time worked (as shown by the gate card) by the appropriate time rate. The time
rate here is fixed after taking into account the rates relevant in the particular
industrial locality for similar trade and skill. The rate may be either fixed or
may be a progressive one, starting from a minimum and rising upto a maximum, in
stages, through periodical increments.

3.32
Labour

Merits :

(i) Simple to understand and to calculate wages. (ii) Reduces temptation on the
part of workers to increase the output at the cost of quality.

Demerits: (i) No monetary incentive to raise the level of production. (ii) No


distinction between the slow and the efficient worker. (iii) The tendency is for
the fall in output; this raises the cost per unit (because both labour and fixed
expenses will be spread over a smaller number of units). (iv) A firm cannot be
sure of labour costs per unit under this method and, hence, may suffer a loss on
quotations if already submitted. In the following cases, time rates are to be
preferred. (1) Persons whose services cannot be directly or tangibly measured,
e.g., general helpers, supervisory and clerical staff etc. (2) Workers engaged on
highly skilled jobs or rendering skilled services, e.g., tool making, inspection
and testing. (3) Where the pace of output is independent of the operator, e.g.,
automatic chemical plants. 3.9.2 High wage plan : This plan was first introduced
by Ford Motor Company (in USA) in order to induce workers to exercise extra effort
in their work. Under this plan a worker is paid a wage rate which is substantially
higher than the rate prevailing in the area or in the industry. In return, he is
expected to maintain a very high level of performance, both quantitative and
qualitative. As a result, high rate men are not as costly or expensive as they
might appear at first sight. High wage plan is suitable where high quality of work
and also increased productivity are required. The advantage which may accrue from
the implementation of this plan are : (1) It is simple and inexpensive to operate.
(2) It helps in attracting highly skilled and efficient workers by providing
suitable incentive. (3) It reduces the extent of supervision. (4) Increased
productivity may result in reduction of unit labour cost. 3.9.3 Measured day work
: According to this method the hourly rate of the time worker consists of two
parts viz, fixed and variable. The fixed element is based on the nature of the job
i.e. the rate for this part is fixed on the basis of job requirements. The
variable portion varies for each worker depending upon his merit rating and the
cost of living index. The aggregate of fixed and variable part for a day is termed
as Measured day�s work rate of a worker.

3.33
Cost Accounting

As the rate is based on two different elements, there are separate time rates not
only for each worker but also for each job. This method does not find much favour
with workers due to the following : 1. 2. The rates fixed are not easily
understood by the workers. Merit rating tends to be arbitrary and unless changed
at rapid intervals, the ratings will not reflect the correct ranking of the
qualities of a worker.

3.9.4 Differential time rate : According to this method, different hourly rates
are fixed for different levels of efficiency. Up to a certain level of efficiency
the normal time or day rate is paid. Based on efficiency level the hourly rate
increases gradually. The following table shows different differential rates : Up
to, say 75% efficiency From 76% to 80% efficiency From 81% to 90% efficiency From
91% to 100% efficiency From 101% to 120% efficiency Normal (say Rs. N per hr.)
1.10 � N 1.20 � N 1.30 � N 1.40 � N

As this method is linked with the output and efficiency of workers, therefore, it
cannot be strictly called as a time rate method of wage payment. This method in
fact is similar to differential piece work system. 3.9.5 Payment by result : Under
this system the payment made has a direct relationship with the output given by a
worker. The attendance of the worker or the time taken by him for doing a job has
no bearing on the payment. The system of payment by results may be classified into
the following four categories : (a) Systems in which the payment of wages is
directly proportionate to the output given by workers. (b) Systems in which the
proportion of the wage payment to the worker increases progressively with increase
in production. (c) Systems in which payment rate decreases with the increase in
output. (d) Systems with earnings varying in proportions which differ at different
levels of production. 3.9.6 Straight piece work system : Under this system of wage
payment, each operation, job or unit of production is termed a piece. A rate of
payment, known as the piece rate or piece work rate is fixed for each piece. The
wages of the worker depend upon his output and rate of each unit of output; it is
in fact independent of the time taken by him. The wages paid to a worker are
calculated as :

3.34
Labour

Wages = Number of units produced � Piece rate per unit. Considerable care and
judgment are called for fixing the piece rate. If the rate fixed is too high or
too low, it would operate to the disadvantage of either the employer or the
employee. Any attempt on the part of the management to revise a piece rate,
erroneously set too high, is likely to lead to friction and conflict with labour.
If on the other hand, it is too low, it would fail in its objective. The only way
all this may be avoided is by employing scientific methods of job evaluation and
time and motion study for the purpose of setting the rates. Advantages : 1. 2. 3.
The system is simple to operate and also easy to understand. The incentive
provided is quite effective as the workers get the full benefit of any increase in
production and the employer also gains by saving on overhead costs. Labour cost
per unit being constant, these can be calculated in advance and quotations can be
confidently submitted. The quality of output usually suffers. Maintenance of
detailed statistics as regards production of individual workers is necessary.
Maintenance of satisfactory discipline in the matter of arrival and departure of
workers becomes somewhat difficult. In the anxiety to produce as large a quantity
as possible, workers may damage the machines and may also increase wastage of
materials. Skilled workers and supervisors (who are often paid on time basis) may
resent higher wages to unskilled workers paid on the piece basis.

Disadvantages : 1. 2. 3. 4. 5.

3.9.7 Differential piece work system : This system provide for higher rewards to
more efficient workers. The main feature of all differential piece-work systems is
that several piece rates on a slab scale are fixed for a job or operation which is
put on piece-work. For different levels of output below and above the standard,
different piece rates are applicable. Taylor Differential Piece Work System and
Merrick Differential Piece Rate System are two important differential piece work
systems discussed briefly as below: (a) Taylor�s differential piece work system -
The Taylor�s Differential Piece Rate System aims at rewarding efficient workers by
providing increased piece rate beyond certain level of output. Under this system
two widely differing piece-rates are prescribed for each job. The lower rate is
83% of the normal piece rate and the higher rate is 125% of the normal piece rate.
In other words the higher rate is 150% of the lower rate. The lower rate is given
to a

3.35
Cost Accounting

worker when his efficiency level is less than 100%. The higher rate is offered at
efficiency level of either 100% or more. Due to the existence of the two piece
rates, the system is known as differential piece rate system. Note: Some authors
also use 80% and 120% of the piece rates as lower and higher rates respectively at
the efficiency levels, as indicated in the above paragraph. Advantages: 1. 2. 3.
It is simple to understand and operate. The incentive is very good and attractive
for efficient workers. It has a beneficial effect where overheads are high as
increased production has the effect of reducing their incidence per unit of
production.

This system is quite harsh to workers, as a slight reduction in output may result
in a large reduction in the wages earned by them. This system is no longer in use
in its original form, though the main idea behind it is used in many wage schemes.
Illustration : Using Taylor�s differential piece rate system, find the earnings of
the Amar, Akbar and Ali from the following particulars: Standard time per piece
Normal rate per hour In a 8 hour day Amar produced Akbar produced Ali produced
Solution Earnings under Differential piece rate system Workers Amar Standard
output per day (units) Actual output per day (units) Efficiency (%) 24 23 95.83%
Akbar 24 24 100% Ali 24 30 125% : : : : : 20 minutes Rs. 9.00 23 units 24 units 30
units

Actual output � 100 Standarad output

23 unit � 100 24 unit

24 unit � 100 24 unit

30 unit � 100 24 unit

3.36
Labour

* Earning rate per unit

83% of the piece rate

125% of the piece rate 3.75 (125% of Rs.3) 90.00

125% of the piece rate 3.75 (125% of Rs.3) 112.50

Earning rate per unit (Rs.) (Refer to working note) Earnings (Rs.)

2.49 (83% of Rs.3) 57.27

(23 units � Rs. 2.49) ( 24 units � Rs. 3.75) (30 units � Rs. 3.75) * Under
Taylor�s Differential price rate system, two widely differing price rates are
prescribed for each job. The lower rate is 83% of the normal piece rate and is
applicable if efficiency of the worker is below 100%. The higher piece rate is
125% of the normal piece rate and is applicable if work completed is at efficiency
level of 100% and above. Working Note: Normal rate per hour Normal rate per unit =
= = Rs. 9.00

Rs.9.00 Rs.9.00 = Standard production per hour 3 units


Rs. 3

(b) Merrick differential piece rate system - Under this system three piece rates
for a job are fixed. None of the fixed rates is below the normal. These three
piece rates are as below: Efficiency Upto 83% Above 83% and upto 100% Above 100%
Piece rate applicable Normal rate, 10% above normal rate. 20% or 30% above normal
rate.

This system is an improvement over Taylor�s Differential Piece Rate System.


Illustration Refer to the statement of previous Illustration and compute the
earnings of workers under Merrick Differential Piece Rate System.

3.37
Cost Accounting

Solution Workers * Earning rate per unit (Refer to previous illustration) Amar 10%
above the normal rate Akbar 10% above the normal rate Ali 20% above the normal
rate or 30% above the normal Earning rate per unit (Rs.) Earnings (Rs.) 3.30 75.90
3.30 79.20 3.60 or 3.90 108 or 117 or (30 units � Rs. 3.90) 3.9.8 Gantt task and
bonus system : This system is a combination of time and piece work system.
According to this system a high standard or task is set and payment is made at
time rate to a worker for production below the set standard. If the standards are
achieved or exceeded, the payment to the concerned worker is made at a higher
piece rate. The piece rate fixed under this system also includes an element of
bonus the extent of 20%. The figure of bonus to such workers is calculated over
the time rate of the workers. Thus in its essence, the system consists of paying a
worker on time basis if he does not attain the standard and on piece basis if he
does. Wages payable to workers under this plan are calculated as under : Output
(i) (ii) Output below standard Output at standard Payment Guaranteed time rate.
Time rate plus bonus of 20% (usually) of time rate. High piece rate on worker�s
whole output. It is so fixed, so as to include a bonus of 20% of the time rate.
Advantages : 1. It provides good incentive for efficient workers and at the same
time protects the less efficient by guaranteeing the time rate.

(23 units � Rs. 3.30)(24 units � Rs. 3.30)(30 units � Rs. 3.60)

(iii) Output above standard

3.38
Labour

2. 3.

It is simple to understand and operate. It encourages better supervision and


planning.

Disadvantage The guaranteed time rate may have the effect of weakening the urge of
slower worker to increase his output. Illustration : In a factory the standard
time allowed for completing a given task (50 units), is 8 hours. The guaranteed
time wages are Rs. 20 per hour. If a task is completed in less than the standard
time, the high rate of Rs. 4 per unit is payable. Calculate the wages of a worker,
under the Gantt system, if he completes the task in (i) 10 hours; (ii) 8 hours,
and (iii) in 6 hours. Also ascertain the comparative rate of earnings per hour
under the three situations. Solution (i) (ii) When the worker performs the task in
10 hours, his earnings will be at the time wage rate i.e. 10 hours � Rs. 20 per
hour = Rs. 200. When the worker performs the task is standard time i.e. in 8
hours, his earning will be: 8 hours � Rs. 20 Total earnings = Rs. 160 Rs. 32 Rs.
192 Bonus @ 20% of time wages =

(iii) When the worker performs the task in less than the standard time his earning
will be at piece rate i.e. 50 units � Rs. 4 per hour = Rs. 200 The comparative
rate of earnings per hour under the above three situations is: (i) (ii) Rs. 200/10
hrs. = Rs. 20 per hour Rs. 192/8 hrs. = Rs. 24 per hour

(iii) Rs. 200/6 hrs. = Rs. 33.33 per hour 3.9.9 Emerson�s efficiency system :
Under this system minimum time wages are guaranteed. But beyond a certain
efficiency level, bonus in addition to minimum day wages is given. A worker who is
able to attain efficiency, measured by his output equal to 2/3rd of the

3.39
Cost Accounting

standard efficiency, or above, is deemed to be an efficient worker deserving


encouragement. The scheme thus provides for payment of bonus at a rising scale at
various levels of efficiency, ranging from 66.67% to 150%. For a performance below
66.67% only time rate wages without any bonus are paid. Above 66?% to 100%
efficiency, bonus varies between 0.01% and 20%. Above 100% efficiency bonus of 20%
of basic wages plus 1% for each 1% increase in efficiency is admissible. This
system is superior to one to the differential piece rate in so far as it
encourages the slow worker to do a little better than before. Also it does not
presuppose a high degree of average performance. Wages on time basis are
guaranteed. Illustration From the following information you are required to
calculate the bonus and earnings under Emerson Efficiency System. The relevant
information is as under: Standard working hours Standard output per hour in units
Daily wage rate Actual output in units Worker A Worker B Worker C Solution Workers
Actual output in units Standard output in units Efficiency level (%)
? Actual output ? ? Standarad output �100? ? ? Rate of bonus

: : :

8 hours a day 5 Rs. 50

25 units 40 units 45 units

Statement showing bonus and earnings under Emerson Efficiency System A 25 40 62.5%
B 40 40 100% C 45 40 112.50%

No bonus 50 Nil 50

20% 50 10 (20% of Rs. 50) 60

Time wages (Rs.) Bonus (Rs.) Total earnings (Rs.)

32.50% (20% + 12.5%) 50 16.25 (32.5% of Rs. 50) 66.25

3.40
Labour

3.9.10 Points scheme or Bedeaux system : Under this scheme, firstly the quantum of
work that a worker can perform is expressed in Bedaux points or B�s. There points
represent the standard time in terms of minutes required to perform the job. The
standard number of points in terms of minutes are ascertained after a careful and
detailed analysis of each operation or job. Each such minute consists of the time
required to complete a fraction of the operation or the job, and also an allowance
for rest due to fatigue. Workers who are not able to complete tasks allotted to
them within the standard time are paid at the normal daily rate. Those who are
able to improve upon the efficiency rate are paid a bonus, equal to the wages for
time saved as indicated by excess of B�s earned (standard minutes for work done)
over actual time. Workers are paid 75% of the time saved. Illustration Calculate
the earnings of worker from the following information under Bedeaux system :
Standard time for a product A-30 seconds plus relaxation allowance of 50%.
Standard time for a product B-20 second plus relaxation allowance of 50%. During 8
hour day for Actual output of product for A Actual output of product B Wage rate
Solution Bedeaux point per unit of product A : 30 seconds + 15 seconds 45 = = 0.75
B�s 60 60 Bedeaux point per unit of product B: 500 units. 300 units Rs. 10 per
hour

15 seconds + 10 seconds 30 = = 0.50 B�s 60 60


Total production in terms of B�s: 500 � 0.75 + 300 � 0.50 = 525 B�s Standard B�s
(8 hours � 60) = 480 B�s No. of B�s saved (525 B�s � 480 B�s) = 45 B�s Earnings =
Hrs. worked � rate per hour + 75/100 �

45 � Rs. 10 = Rs. 80 + Rs. 5.63 = Rs. 85.63 60

3.41
Cost Accounting

3.9.11 Hayne�s system : Under this system also the standard is set in minutes. The
standard time for the job is expressed in terms of the standard man-minutes called
as �MANIT�. Manit stands for man-minute. In the case of repetitive work the time
saved is shared between the worker and the foreman in the ratio 5 : 1. If the work
is of non-repetitive nature, the worker, the employer and the foreman share the
value of time saved in the ratio of 5 : 4 : 1. Each worker is paid according to
hourly rate for the time spent by him on the job. 3.9.12 Accelerated premium
system : Under this system earnings increase with output; the rate of increase of
earnings itself increases progressively with output; in fact the earnings increase
in greater proportion than the increase in production. This system acts as a
strong incentive for skilled workers to earn high wages by increasing output and
for production beyond standard. 3.9.13 Premium bonus methods : Under these
methods, standard time is established for performing a job. The worker is
guaranteed his daily wages (except in Barth System), if his output is below and
upto standard. In case the task is completed in less than the standard time, the
saved time is shared between the employee and the employer. There are two types of
time-sharing plans in use viz., constant sharing plans and variable sharing plans.
3.9.14 Halsey and Halsey Weir systems : Under Halsey system a standard time is
fixed for each job or process. If there is no saving on this standard time
allowance, the worker is paid only his day rate. He gets his time rate even if he
exceeds the standard time limit, since his day rate is guaranteed. If, however, he
does the job in less than the standard time, he gets a bonus equal to 50 percent
of the wages of time saved; the employer benefits by the other 50 percent. The
scheme also is sometimes referred to as the Halsey fifty percent plan. Formula for
calculating wages under Halsey system = Time taken � Time rate + 50% of time saved
� Time rate. The Halsey Weir System is the same as the Halsey System except that
the bonus paid to workers is 30% of the time saved. Advantages: 1. 2. Time rate is
guaranteed while there is opportunity for increasing earnings by increasing
production. The system is equitable in as much as the employer gets a direct
return for his efforts in improving production methods and providing better
equipment. Incentive is not so strong as with piece rate system. In fact the
harder the worker works, the lesser he gets per piece. The sharing principle may
not be liked by employees.

Disadvantages: 1. 2.
3.42
Labour

Illustration Calculate the earning of a worker under Halsey System. The relevant
data is as below : Time Rate (p.h.) Time allowed Time taken Time saved Solution
Calculation of total earnings : 6 hrs. � Re. 0.6 + 1/2 � (2 hrs. � Re. 0.60) or
Rs. 3.60 + Re. 0.60 = Rs. 4.20 Of his total earnings, Rs. 3.60 is on account of
the time worked and Re. 0.60 is on account of his share of the premium bonus.
3.9.15 Rowan system : According to this system a standard time allowance is fixed
for the performance of a job and bonus is paid if time is saved. Under Rowan
System the bonus is that proportion of the time wages as time saved bears to the
standard time. Formula for calculating wages under Rowan system = Time taken �
Rate per hour + Advantages : 1. 2. 3. 1. 2. 3. It is claimed to be a fool-proof
system in asmuch as a worker can never double his earnings even if there is bad
rate setting. It is admirably suitable for encouraging moderately efficient
workers as it provides a better return for moderate efficiency than under the
Halsey Plan. The sharing principle appeals to the employer as being equitable. The
system is a bit complicated. The incentive is weak at a high production level
where the time saved is more than 50% of the time allowed. The sharing principle
is not generally welcomed by employees. Re. 0.6 8 hours 6 hours 2 hours

Time Saved � Time taken � Rate per hour Time allowed

Disadvantages:

3.43
Cost Accounting

Illustration Calculate the earnings of a worker under Rowan System. The relevant
data is given as below: Time rate (per Hour) Time allowed Time taken Time saved
Solution Calculation of total earnings: = Time taken � Rate per hour + = 6 hours �
Rs. 0.60 + Re. 0.6 8 hours. 6 hours. 2 hours.

Time Saved � Time taken � Rate per hour Time allowed

2 hours � 6 hours � 0.60 8 hours

= Rs. 3.60 + Rs. 0.90 = Rs. 4.50 3.9.16 Barth system : The formula used for
calculating the remuneration under this system is as follows : Earnings = Hourly
rate �

Standard hours � Hours worked

The system is particularly suitable for trainees and beginners and also for
unskilled workers. The reason is that for low production efficiency, the earnings
are higher than in the piece-work system but as the efficiency increases, the rate
of increase in the earnings falls. Illustration A factory having the latest
sophisticated machines wants to introduce an incentive scheme for its workers,
keeping in view the following: (i) (ii) The entire gains of improved production
should not go to the workers. In the name of speed, quality should not suffer.

(iii) The rate setting department being newly established are liable to commit
mistakes. You are required to devise a suitable incentive scheme and demonstrate
by an illustrative numerical example how your scheme answers to all the
requirements of the management. Solution

3.44
Labour

Rowan Scheme of premium bonus (variable sharing plan) is a suitable incentive


scheme for the workers of the factory. If this scheme is adopted, the entire gains
due to time saved by a worker will not pass to him. Another feature of this scheme
is that a worker cannot increase his earnings or bonus by merely increasing its
work speed. The reason for this is that the bonus under Rowan Scheme is maximum
when the time taken by a worker on a job is half of the time allowed. As this fact
is known to the workers, therefore, they work at such a speed which helps them to
maintain the quality of output too. Lastly, Rowan System provides a safeguard in
the case of any loose fixation of the standards by the rate-setting department. It
may be observed from the following illustration that in the Rowan Scheme the bonus
paid will be low due to any loose fixation of standards. Workers cannot take undue
advantage of such a situation. The above three features of Rowan Plan can be
discussed with the help of the following illustration: (i) Time allowed Time taken
Time saved Rate Bonus = = = = = = 4 hours 3 hours 1 hour Rs. 5 per hour

Time taken � Time saved � Rate Time allowed

3 hours � 1 hour � Rs. 5 = Rs. 3.75 4 hours In the above illustration time saved
is 1 hour and, therefore, total gain is Rs. 5. Out of Rs. 5 according to Rowan
Plan only Rs. 3.75 is given to the worker in the form of bonus and the remaining
Rs. 1.25 remains with the management. In other words a worker is entitled for 75
percent of the time saved in the form of bonus.
(ii) The figures of bonus in the above illustration when the time taken is 2 hours
and 1 hour respectively are as below: Bonus =

Time taken � Time saved � Rate Time allowed 2 hours � 2 hours � Rs. 5 = Rs. 5 4
hours

3.45
Cost Accounting

1 hours � 3 hours � Rs. 5 = Rs. 3.75 4 hours

The above figures of bonus clearly show that when time taken is half of the time
allowed, the bonus is maximum. When the time taken is reduced from 2 to 1 hour,
the bonus figure fell by Rs. 1.25. Hence, it is quite apparent to workers that it
is of no use to increase speed of work. This feature of Rowan Plan thus protects
the quality of output. (iii) If the rate-setting department erroneously sets the
time allowed as 10 hours instead of 4 hours, in the above illustration; then the
bonus paid will be as follows: Bonus =

3 hours � 7 hours � Rs. 5 = Rs. 10.50 10 hours

The bonus paid for saving 7 hours thus is Rs. 10.50 which is approximately equal
to the wages of 2 hours. In other words the bonus paid to the workers is low.
Hence workers cannot take undue advantage of any mistake committed by the time
setting department of the concern. Illustration (a) Bonus paid under the Halsey
Plan with bonus at 50% for the time saved equals the bonus paid under the Rowan
System. When will this statement hold good ? (Your answer should contain the
proof). (b) The time allowed for a job is 8 hours. The hourly rate is Rs. 8.
Prepare a statement showing: (i) (ii) The bonus earned The total earnings of
labour and

(iii) Hourly earnings. Under the Halsey System with 50% bonus for time saved and
Rowan System for each hour saved progressively. Solution (a) Bonus under Halsey
Plan = Standard wage rate � 50/100 � Time saved ...(i) Bonus under Rowan Plan =
Standard wage rate �

Time taken � Time taken Time allowed

...(ii)

3.46
Labour

Bonus under Halsey Plan will be equal to the Bonus under Rowan Plan when the
following condition holds good = Standard wage rate � 50/100 � Time saved =
Standard wage rate � or or

Time taken � Time saved Time allowed

1 Time taken = 2 Time allowed


Time taken =

1 of time allowed. 2

Hence, when the time taken is 50% of the time allowed, the bonus under Halsey and
Rowan Plans is equal. (b) Statement of Bonus, Total earnings of Labour and hourly
earnings under Halsey and Rowan Systems
Time allowed Time taken Time saved Basic Wages Bonus under Halsey System Bonus
under Rowan system Total earning under Total earning under Hourly earning under
Halsey System Hourly earning under Rowan System

Halsey Rowan System System

B � Rs. 8 C � A B C =(A-B)

50 100

� Rs. 8 E

C � B � Rs. 8 D+E A
F G H I J

hours 8 8 8 8 8 8 8 8

hours 8 7 6 5 4 3 2 1

hours 1 2 3 4 5 6 7

Rs. 64 56 48 40 32 24 16 8

Rs. 4 8 12 16 20 24 28

Rs. 7 12 15 16 15 12 7

Rs. 64 60 56 52 48 44 40 36

Rs. 64 63 60 55 48 39 28 15

Rs. 8.00 8.57 9.33 10.40 12.00 14.67 20.00 36.00

Rs. 8.00 9.00 10.00 11.00 12.00 13.00 14.00 15.00

Illustration Mr. A. is working by employing 10 skilled workers. He is considering


the introduction of some

3.47
Cost Accounting

incentive scheme - either Halsey Scheme (with 50% bonus) or Rowan Scheme - of wage
payment for increasing the labour productivity to cope with the increased demand
for the product by 25%. He feels that if the proposed incentive scheme could bring
about an average 20% increase over the present earnings of the workers, it could
act as sufficient incentive for them to produce more and he has accordingly given
this assurance to the workers. As a result of the assurance, the increase in
productivity has been observed as revealed by the following figures for the
current month : Hourly rate of wages (guaranteed) Average time for producing 1
piece by one worker at the previous performance (This may be taken as time
allowed) No. of working days in the month No. of working hours per day for each
worker Actual production during the month Required : 1. 2. 3. Calculate effective
rate of earnings per hour under Halsey Scheme and Rowan Scheme. Calculate the
savings to Mr. A in terms of direct labour cost per piece under the schemes.
Advise Mr. A about the selection of the scheme to fulfil his assurance. 25 8 1,250
units 2 hours Rs. 2.00

Solution Working Notes: 1. Total time wages of 10 workers per month : = No. of
working days in the month � No. of working hours per day of each worker � Hourly
rate of wages � No. of workers = 2. 25 days � 8 hrs. � Rs. 2 � 10 workers Time
saved per month : Time allowed per piece by a worker No. of units produced during
the month by 10 workers Total time allowed to produce 1,250 pieces (1,250 � 2
hours) 2 hours 1,250 pieces 2,500 hours = Rs. 4,000

3.48
Labour

Actual time taken to produce 1,250 pieces Time saved (2,500 hours � 2,000 hours)
3. Bonus under Halsey scheme to be paid to 10 workers : Bonus = (50% of time
saved) � hourly rate of wages = 50/100 � 500 hours � Rs. 2 = Rs. 500

2,000 hours 500 hours

Total wages to be paid to 10 workers are (Rs. 4,000 + Rs. 500) Rs. 4,500, if Mr. A
considers the introduction of Halsey Incentive Scheme to increase the labour
productivity. 4. Bonus under Rowan Scheme to be paid to 10 workers : Bonus =

Time taken � Time wages = � Rs. 4,000 = Rs. 800 Time allowed

Total wages to be paid to 10 workers are (Rs. 4,000 + Rs. 800) Rs. 4,800, if Mr. A
considers the introduction of Rowan Incentive Scheme to increase the labour
productivity. 1. (i) Effective hourly rate of earnings under Halsey scheme :
(Refer to Working Notes 1, 2 and 3) = =

Total time wages of 10 workers + Total bonus under Halsey scheme Total hours
worked

Rs.4,000 + Rs.500 = Rs. 2.25 2,000 hours

(ii) Effective hourly rate of earnings under Rowan scheme : (Refer to Working
Notes 1, 2 and 4) = = 2. (i)

Total time wages of 10 workers + Total bonus under Rowan scheme Total hours worked

Rs.4,000 + Rs.800 = Rs. 2.40 2,000 hours

Saving in terms of direct labour cost per piece under Halsey scheme : (Refer to
Working Note 3) Labour cost per piece (under time wage scheme) = 2 hours � Rs. 2 =
Rs. 4.

3.49
Cost Accounting

Labour cost per piece (under Halsey scheme) =

Total wages paid under the scheme Rs.4,500 = = Rs. 3.60 Total number of units
produced 1,250
= Re. 0.40.

Saving per piece : (Rs. 4 � Rs. 3.60) (Refer to Working Note 4)

(ii) Saving in terms of direct labour cost per piece under Rowan Scheme : Labour
cost per piece under Rowan scheme = Rs. 4,800/1,250 units = Rs. 3.84 Saving per
piece = Rs. 4 � Rs. 3.84 = Re. 0.16. 3. From the labour cost per piece under
Halsey scheme (Rs. 3.60) and Rowan scheme (Rs. 3.84), it is quite clear that
Halsey scheme brings about more saving than Rowan scheme to the concern. But
Halsey scheme does not fulfil the assurance given to the workers about 20%
increase in their earnings as it secures only 12.5% [500/4,000] � 100 increase.

On the other hand, Rowan scheme secures 20% [800/4,000] � 100 increase in the
earnings and it fulfils the assurance. Therefore, Rowan scheme may be adopted.
Illustration Wage negotiations are going on with the recognised Labour Union and
the Management wants you as the Cost Accountant of the Company to formulate an
incentive scheme with a view to increase productivity. The case of three typical
workers Achyuta, Ananta and Govida who produce respectively 180, 120 and 100 units
of the company�s product in a normal day of 8 hours is taken up for study.
Assuming that day wages would be guaranteed at 75 paise per hour and the piece
rate would be based on a standard hourly output of 10 units, calculate the
earnings of each of the three workers and the labour cost per 100 pieces under (i)
Day wages, (ii) Piece rate, (iii) Halsey scheme, and (iv) The Rowan scheme. Also
calculate under the above schemes the average cost of labour for the company to
produce 100 pieces. Solution Calculation of earnings of each of the three workers
and the labour cost per 100 pieces under different wage schemes. (i) Day wages
Name of workers Day wages Actual output (units) Labour cost per 100 pieces

3.50
Labour

Rs. Achyuta Ananta Govinda Total 6.00 6.00 6.00 18.00 180 120 100 400

Rs. 3.33 5.00 6.00

Average cost of labour for the company to produce 100 pieces :

Rs.18 Total wages paid � 100 = � 100 = Rs. 4.50 400 Total output
(ii) Piece rate Name of workers Actual output (units) Achyuta Ananta Govinda Total
180 120 100 400 Piece rate Rs. 0.075 0.075 0.075 Wages earned Rs. 13.50 9.00 7.50
30.00 Labour cost per 100 pieces Rs. 7.50 7.50 7.50

Average cost of labour for the company to produce 100 pieces : = 7.50 (iii) Halsey
Scheme Name of workers Acutal output (units) Std. time for actual output Hrs.
Achyuta Ananta Govinda 180 120 100 18 12 10 Actual time for actual output Hrs. 8 8
8 10 4 2 Time saved Hrs. Bonus Hrs. (50% of time saved Hrs.) 5 2 1

Rs.30 � 100 = Rs. 400

Total including Bonus* Rs. 9.75 7.50 6.75

Labour 100 pieces Rs. 5.42 6.25 6.75

wages cost per

3.51
Cost Accounting

24.00

Average cost of labour for the company to produce 100 pieces = 100 = Rs.6.00
*Total wages =

Rs.30 (Rs. 24/400) � 400

(Actual hours worked + bonus hours) � Rate per hour

Hence total wages of Achyuta are: (8 + 5) � Rs. 0.75 = Rs. 9.75 Similarly, the
total wages of Ananta and Govinda are Rs. 7.50 and Rs. 6.75 respectively. (iv)
Rowan Scheme:
Name of workers Actual output (units) Std. time for actual output (hrs.) Actual
time taken hours Time saved (hours) Bonus* Wages for hrs. actual hrs. @ 75 P per
hr. Rs. (1) Achyuta Ananta Govinda (2) 180 120 100 (3) 18 12 10 (4) 8 8 8 (5) 10 4
2 (6) 4.44 2.67 1.6 (7) 6.00 6.00 6.00 Bonus @ 75 P per Bonus hour Rs. Rs. Total
earnings Labour cost per 100 pieces Rs

(8) (7)+(8)=(9) (10) 3.33 2.00 1.20 9.33 8.00 7.20 24.53 5.18 6.67 7.20

Average cost of labour to the = company for 100 pieces *Bonus hours = Time taken �
Bonus hours of Achyuta =

Rs.24.53 � 100 = Rs. 6.13 400

Time saved Time allowed

8 hours � 10 hours = 4.44 18 hours

Similarly, bonus hours of Ananta and Govinda are 2.67 hours and 1.6 hours
respectively. 3.9.17 Group System of wage payment : Certain jobs or operations are
required to be performed collectively by a number of workers. Under such cases
each man�s work depends on the work performed by one or more of his colleagues and
as such it is not possible to measure separately the output of each worker.

3.52
Labour

The workers constituting a group or a team here are considered as a composite unit
and the combined output of such a unit is measured for the purpose of wage
calculation. The methods usually used for distributing wages to each worker are
the following : 1. 2. 3. 4. 5. Equally, if all the workers of the group are of the
same grade and skill, same rate of pay and has worked for same duration. Prorata
to the time-rate of each worker where the time spent by the individual worker is
the same. On the basis of the time rates and attendance of each worker. On a
specified percentage basis; the percentage applicable to a worker is predetermined
on the basis of the skill, rate of pay etc. In a group of unskilled and skilled
workers, a method of distribution is to pay the unskilled workers at their time
rates. The balance amount remaining out of the total earnings after payment to the
unskilled workers is distributed among the skilled workers by any of the methods
discussed above.

Group Bonus - Group Bonus refers to the bonus paid for the collective efforts made
by a group of workers. The amount of bonus is distributed among the individual
members of the group on some agreed basis. Group Bonus Schemes - Under a group
bonus scheme, bonus is paid to a team/group of employees working together. Such a
scheme is introduced generally where individual efficiency cannot be established
for the payment of bonus. For example, in the construction work, it is the team
work of masons and labourers which produces results. If any incentive is to be
offered, it should be offered to the team as a whole and not to an individual.
Group bonus is based on the combined output of the team as a whole. The quantum of
bonus is determined on the basis of the productivity of the team and the bonus is
shared by individual workers in specified proportions, often in the proportion of
wages on time basis. Objectives of Group Bonus Schemes: The objectives of a group
bonus scheme are the following :(i) (ii) To create collective interest and team
spirit among workers. To create interest among supervisors to improve performance.

(iii) To reduce wastage in materials and idle time. (iv) To achieve optimum output
at minimum cost. (v) To encourage individual members of the group, team where only
the output of the team as a whole can be measured. Advantages of Group Bonus
Schemes :

3.53
Cost Accounting

1. 2. 3. 4.

They create a sense of team spirit since all the workers in a group realise that
their personal incentives are dependent upon group effort. A spirit of healthy
competition amongst various groups doing identical jobs is also created. This
results is the elimination of excessive waste of materials. The operators and
supervisors also feel interested in raising the production to higher levels. The
scheme is usually easier to understand and entails less costing and accounting
work. It is easier to set up group activity targets, since the performance unit is
large.

Schemes of group bonus - There are five schemes of group bonus as discussed below
: (1) Priestman�s Production Bonus : This method was adopted by Priestman Bros.
Ltd., in 1917. According to this method when the actual production in units or
points exceeds the standard fixed, a bonus is paid to the workers as additional
wages equivalent to the percentage of actual output over the standard output. (2)
Cost Efficiency Bonus : Under this scheme, the amount of bonus is calculated when
the cost is reduced below the normal established targets. Targets of cost, for
example, material cost, labour cost and overhead cost etc., per unit are fixed. If
the measurement of actual performance shows a saving in the total labour and
material cost or a reduction in the total cost per unit, a fair percentage of the
saving is distributed among the staff. Three popular schemes usually used for
calculating the amount to be distributed to workers as Bonus are as below : (i)
Nunn-Bush Plan : According to this plan a norm of direct labour cost is fixed and
expressed as a percentage of the sales value. The amount calculated at this
percentage is credited to a fund. The actual labour cost is debited to this fund
and the balance remaining to the credit of this fund is distributed as bonus to
all the workers and employees. Scanlon Plan : Here also a fund is created for the
normal cost of wages and salaries. This fund is debited with the actual labour
costs. Two-thirds to threefourths of the credit balance, if any, is distributed as
bonus, the balance is kept as reserve for future set-backs.

(ii)

(iii) Rucker Plan : This plan is quite similar to Nunn-Bush Plan except that the
percentage for crediting the fund is based on the total value added by
manufacturer (i.e. the total cost less the value of the material) and not on total
sales value. (3) Town Gain Sharing Plan : Under this plan bonus is dependent upon
a saving in the labour cost as compared to standard. The bonus is calculated at
50% of the saving achieved.

3.54
Labour

(4) Budgeted Expense Bonus : Bonus is determined in advance and paid as a


percentage of savings effected in the actual total expenses as compared to the
budgeted expense. It is payable to indirect workers also. (5) Waste Reduction
Bonus : Bonus becomes payable under this scheme if the team of workers brings
about a reduction in the percentage of material wastages as compared to the
standard set. It is applicable to industries where the material cost assumes a
greater proportion of the total cost. Many times group bonus schemes do not enjoy
the approval of workers. Some workers tend to feel that their personal incentives
are low merely because some members of the group are lazy or inefficient. Such
workers believe that it is better to provide incentives on individual basis, if it
is possible. 3.9.18 System of Incentive schemes for Indirect Workers: Since the
setting of work standards and measurement of output in the case of indirect
workers is not an easy task in respect of maintenance, internal transport,
inspection, packing and cleaning, therefore the introduction of a system of
payment by results for indirect workers is difficult. In spite of the aforesaid
difficulty, it has been felt necessary to provide for incentives to indirect
workers, due to the following reasons: 1. 2. 3. 4. 5. Payment of incentive bonus
to direct workers and time rate to indirect workers leads to dissatisfaction and
labour unrest. Indirect workers are as much entitled to bonus as direct workers.
Bonus payment to indirect workers creates team spirit. An incentive system for
indirect workers assists in maintaining the efficiency of services such as plant
repairs, stores maintenance, material handling etc. The efficiency of direct
workers is reduced where their work is dependent upon the service rendered by the
indirect workers. Incentive to supervisors and foremen : Supervisors and foremen
are an important link between the management and the workers. Incentive payment to
these persons would assist in maintaining all round efficiency. Incentive to
supervisors and foremen may be provided in the form of non-financial benefits.
Incentive can also be provided to these workers in the form of Bonus. The extent
of bonus which will be distributed as incentive will depend on the savings
effected over the standards. (ii) Incentive to maintenance and repairs staff :
Under mass production work, repair and maintenance duties can be considered as
routine and repetitive for which percentage of

A few examples of incentive schemes to indirect workers are as below : (i)

3.55
Cost Accounting

efficiency can be evaluated. In case such an evaluation is not possible or


practicable, a group bonus system may be established, on the basis of reduction in
breakdown or on the number of complaints. Illustration A, B and C were engaged on
a group task for which a payment of Rs. 725 was to be made. A�s time basis wages
are Rs. 8 per day, B�s Rs. 6 per day and C�s Rs. 5 per day. A worked for 25 days;
B worked for 30 days; and C for 40 days. Distribute the amount of Rs. 725 among
the three workers. Solution Total wages on time basis : A B C 25 @ Rs. 8 30 @ Rs.
6 40 @ Rs. 5 Rs. 200 180 200 580 Payment for the task Bonus : (Rs. 725 � Rs. 580)
or, 25% of the time-basis wages. Earnings of each worker Worker A B C Illustration
Both direct and indirect labour of a department in a factory are entitled to
production bonus in accordance with a group incentive scheme, the outline of which
is as follows : (a) For any production in excess of the standard rate fixed at
16,800 tonnes per month (of 28 days) a general incentive of Rs. 15 per tonne is
paid in aggregate. The total amount payable to each separate group is determined
on the basis of an assumed percentage of such excess production being contributed
by it, namely @ 65% by direct labour, @ 15% Wages on time basis Rs. 200 180 200
580 Group task Bonus 25% Rs. 50 45 50 145 Total Rs. 250 225 250 725 145 725

3.56
Labour

by inspection staff, @ 12% by maintenance staff and @ 8% by supervisory staff. (b)


Moreover, if the excess production is more than 20% above the standard, direct
labour also get a special bonus @ Rs. 5 per tonne for all production in excess of
120% of standard. (c) Inspection staff are penalized @ Rs. 20 per tonne for
rejection by customer in excess of 2% of production. (d) Maintenance staff are
also penalized @ Rs. 20 per hour for breakdown. From the following particulars for
a month, work out production bonus earned by each group : (a) Actual working days
(b) Production (c) Rejection by customer (d) Machine breakdown Solution 1.
Standard output per day = = 2. : 25 : 21,000 tonnes : 500 tonnes : 40 hours

Standard output per month Budgeted number of days in a month 16,800 tonnes = 600
tonnes 28 days

Standard output for 25 days = 600 tonnes � 25 days = 15,000 tonnes (a) General
Incentive (i) (ii) Standard output Actual output : : : 15,000 tonnes 21,000 tonnes
21,000 � 15,000 = 6,000 tonnes 40%

(iii) Excess output over standard : (iv) Percentage of excess

production to standard output : (v) Aggregate general incentive (vi) Allocation of


general incentive Direct labour : : :

6,00 tonnes � 100 15,000 tonnes


= Excess output � Rs. 15 = 6,000 tonnes � Rs.15 = Rs. 90,000 65% of Rs. 90,000 Rs.
58,500

3.57
Cost Accounting

Inspection staff : Maintenance staff : Supervisory staff : Total (b) Special bonus
to direct workers (i) (ii) (i) (ii)

15% of Rs. 90,000 Rs. 13,500 12% of Rs. 90,000 Rs. 10,800 8% of Rs. 90,000 Rs.
7,200 Rs. 90,000

20% is the excess output over 120% of standard output or 3,000 tonnes (15,000
tonnes � 20%) Special incentive = 3,000 tonnes � Rs. 5 = Rs. 15,000 Normal
rejection = 2% of production = 2% of 21,000 tonnes = 420 tonnes Actual rejection =
500 tonnes

(c) Penalty imposed on inspection staff

(iii) Excess rejection over normal rejection = 500 � 420 = 80 tonnes (iv) Penalty
= 80 tonnes � Rs. 20 per tonne = Rs. 1,600 (d) Penalty imposed on maintenance
staff (i) (ii) Breakdown hours = 40 hours Penalty = 40 hours � Rs. 20 per hour =
Rs. 800 Statement of production bonus earned by each group Particulars Direct
labour (Rs.) 1 Aggregate general incentive Special bonus Penalty Production bonus
58,500 15,000 73,500 13,500 (1,600) 11,900 10,800 (800) 10,000 7,200 7,200 90,000
15,000 (2,400) 1,02,600 Inspection staff (Rs.) 2 Maintenance Supervisory staff
(Rs.) 3 staff (Rs.) 4 (Rs.) 5 Total

3.9.19 Profit-sharing and Co-partnership schemes - A profit-sharing scheme implies


that the net profit of business would be shared between the workers and the
shareholders or the partners in a pre-determined ratio. Co-partnership, on the
other hand, implies that the workers
3.58
Labour

shall own the business jointly with the shareholders. In this case, usually the
workers share of profits is given in the form of shares. Some employers in our
country originally introduced profit-sharing schemes with a view of stimulating
interest among workers for increasing production. But the schemes have not been
successful on account of unwillingness on the part of the management to consult
workers. Even a demand for copies of final accounts of the business to be shown to
them has been considered by some employers to be an unwarranted interference. The
question of bonus has thus been one of the major causes of industrial disputes in
recent years. (Payment of compulsory bonus is now governed by the payment of Bonus
Act.) Though profit sharing has become a normal feature of the industrial life in
this country, copartnership is comparatively unknown. Nevertheless it must be
pointed out that in England and other Western countries, a number of successful
concerns have been allotting shares to their workers in proportion to their shares
of bonus. Some of them have advanced loans to them to purchase shares. Both these
forms of benefit have been quite popular with labour. Advantages : (i) Employees
are made to feel that they too have a stake in the well-being of the undertaking
and have a contribution to make in earning of profits by improving production and
operations. Labour turnover may be reduced, particularly if a minimum period of
service is laid down as a condition for participating in such schemes. Profit may
fluctuate from year to year; there is thus an element of uncertainty in such
schemes. Profit depends upon many factors of which labour efficiency is only one.
Insufficiency of bonus may lead to dissatisfaction instead of promoting good
relations, if the good work done by labour is nullified by other factors.

(ii)

Disadvantages : (i) (ii)

(iii) The reward may be too remote to sustain continued interest in and zeal for
work. (iv) There may be doubt and suspicion about the profit disclosed. (v) Since
all are entitled to participate in such schemes, there is no recognition of
individual merit. (vi) The individual share of profit may be too meagre to be
appealing. (vii) Since in practice bonus has to be fought in India, so it has
become an important cause of labour disputes. Treatment in Costing : In foreign
countries bonus is an ex-gratia payment and hence it is

3.59
Cost Accounting

regarded as an appropriation of profit not to be included in costs. In fact trade


unions there do not look upon bonus with favour. In India however, payment of
bonus is compulsory under the Payment of Bonus Act under which 8.33% of wages
earned or Rs. 100 whichever is higher, is the minimum bonus payable, the maximum
being 20%. Hence bonus must be treated as part of costs in India. There can be two
methods of dealing with bonus. It may be treated as part of overheads; in any
case, this must be so for bonus paid to indirect workers. In the case of direct
workers the bonus payable may be estimated beforehand and wage rates for costing
purposes suitably inflated by including the bonus that would be paid. Suppose, a
worker gets Rs. 800 p.m. as wages and it is expected that he will be paid two
months�s wages as bonus. His total earning will be Rs. 11,200 (Rs. 9,600 + Rs.
1,600). If the worker works for 2,400 hours in a year the wage rate for costing
purposes will be : Rs. 4.67, i.e., Rs. 11,200/2,400 hours. Illustration A skilled
worker in XYZ Ltd. is paid a guaranteed wage rate of Rs. 30 per hour. The standard
time per unit for a particular product is 4 hours. P, a machineman, has been paid
wages under the Rowan Incentive Plan and he had earned an effective hourly rate of
Rs. 37.50 on the manufacture of that particular product. What could have been his
total earnings and effective hourly rate, had been put on Halsey Incentive Scheme
(50%) ? Solution Let T hours be the total time worked in hours by the skilled
worker (machineman P); Rs. 30/is the rate per hour; standard time is 4 hours per
unit and effective hourly earning rate is Rs. 37.50 then Earnings = Hours worked �
Rage per hour + (Under Rowan incentive plan) Rs. 37.5 T = T � Rs. 30 +

Time saved � Time taken � Rate per hour Time allowed

(4 - T ) � T � Rs. 30
4

Rs. 37.5 = Rs. 30 + (4 � T) � Rs. 7.5 Or Or T Rs. 7.5 T = Rs. 22.5 = 3 hours

Total earnings and effective hourly rate of skilled worker (machineman P) under
Halsey Incentive Scheme (50%)
3.60
Labour

Total earnings = Hours worked � Rate per hour + (Under 50% Halsey incentive
Scheme) = 3 hours � Rs. 30 +

1 Time saved � Rage per hour 2

1 � 1 hour � Rs. 30 = Rs. 105 2

Effective hourly rate Illustration

Time earnings Rs.105 = = Rs. 35 Time allowed 3 hours

During audit of account of G Company, your assistant found errors in the


calculation of the wages of factory workers and he wants you to verify his work.
He has extracted the following information : (i) The contract provides that the
minimum wage for a worker is his base rate. It is also paid for downtimes i.e.,
the machine is under repair or the worker is without work. The standard work week
is 40 hours. For overtime production, workers are paid 150 percent of base rates.
Straight Piece Work � The worker is paid at the rate of 20 paise per piece.

(ii)

(iii) Percentage Bonus Plan � Standard quantities of production per hour are
established by the engineering department. The workers� average hourly production,
determined from his total hours worked and his production, is divided by the
standard quantity of production to determine his efficiency ratio. The efficiency
ratio is then applied to his base rate to determine his hourly earnings for the
period. (iv) Emerson Efficiency Plan � A minimum wages is paid for production upto
66-2/3% of standard output or efficiency. When the workers production exceeds 66-
2/3% of the standard output, he is paid bonus as per the following table :
Efficiency Level Upto 66 Bonus Nil 10% 20% 45%

2 % 3 2 % to 79 % 3

Above 66

80% � 99% 100% � 125%

3.61
Cost Accounting

Your assistant has produced the following schedule pertaining to certain workers
of a weekly pay roll :
Workers Wage Incentive Plan Total Hours Down Time Hours Units Produced Standard
Units Base Rate Gross Wages as per Book Rs. 85 95 85 120 93 126

Rs. Rajesh Mohan* John Mahesh Anil (40 hours production) Straight piece work
Straight piece work Straight piece work Emerson Emerson 40 46 44 40 40 40 5 � � 4
� � 400 455 425 250 240 600 � � � 200 300 500 1.80 1.80 1.80 2.20 2.10 2.00

Harish Percentage bonus plan

* Total hours of Mohan include 6 overtime hours. Prepare a schedule showing


whether the above computation of workers� wages are correct or not. Give details.
Solution Schedule showing the correct figure of minimum wages, gross wages and
wages to be paid
Workers Wage Minimum Gross wages incentive wages computed as per plan Incentive
plan (Rs.) Rajesh Mohan John Harish Straight piece work Straight piece work
Straight piece work 72.00 88.20 82.80 (Refer to W. Note 1) 91.00 85.00 110.00 95
85 120 91.00 85.00 110.00 (Refer to W. Note 2) (Refer to W. Note 3) Percentage
bonus plan 88.00 (Refer to W. Note 4) (Rs.) 80.00 Gross wage Wages to be paid as
per book are Maximum of : minimum and gross computed wages (Rs.) (Rs.) 85 80.00

3.62
Labour Mahesh Anil Emerson Emerson 84.00 80.00 100.80 116.00 93 126 100.80 116.00

(Refer to W. Note 5) (Refer to W. Note 6)

Working notes : 1. Minimum wages Gross wages (computed) as per incentive plan 2.
Minimum wages hours � = Total normal hours � rate per hour = 40 hours � Rs. 1.80 =
Rs. 72 = No. of units � rate per unit = 400 units � Rs. 0.20 = Rs. 80 = Total
normal hours � Rate per hour + Overtime Overtime rate per hour = 40 hours � Rs.
1.80 + 6 hours � Rs. 2.70 = Rs. 72 + Rs. 16.20 = Rs. 88.20 Gross wages (computed)
as per incentive plan 3. Minimum wages Gross wages (computed) as per incentive
plan 4. Minimum wages Efficiency of worker = 40 hours � Re. 2.20 = Rs. 88 = =
Hourly rate Gross wages (computed) as percentage of bonus plan 5. Minimum wages =
40 hours � Rs. 2.75 = Rs. 110/= 40 hours � Rs. 2.10 = Rs. 84 = 455 units � Re.
0.20 = Rs. 91.00 = 40 hours � Rs. 1.80 + 4 hours � Rs. 2.70 = Rs. 72 + Rs. 10.80 =
Rs. 82.80 = 425 units � Rs. 0.20 = Rs. 85

Actual production per hour � 100 Standard productiion per hour (250 units/40
hours) � 100 = 125% (200 units/40 hours)

= Rate per hour � Efficiency of worker = Rs. 2.20 � 125% = Rs. 2.75

3.63
Cost Accounting

Efficiency of worker

(240 units/40 hours) � 100 = 80% (300 units/40 hours)

Bonus (as per Emerson�s plan) = Total minimum wages � Bonus percentage = Rs. 84 �
20% = Rs. 16.80 Gross wages (computed) as per Emerson�s Efficiency plan 6. Minimum
wages Efficiency of worker = Minimum wages + Bonus = Rs. 84 + Rs. 16.80 = Rs.
100.80 = 40 hours � Rs. 2 = Rs. 80 =

600 � 100 = 120% 500

Bonus (as per Emerson�s plan) = Rs. 80 � 45% = Rs. 36 Gross wages (computed) as
per Emerson�s Efficiency plan Illustration The present output details of a
manufacturing department are as follows : Average output per week Saleable value
of output Contribution made by output towards fixed expenses and profit Rs.
2,40,000 The Board of Directors plans to introduce more mechanisation into the
department at a capital cost of Rs. 1,60,000. The effect of this will be to reduce
the number of employees to 120, and increasing the output per individual employee
by 60%. To provide the necessary incentive to achieve the increased output, the
Board intends to offer a 1% increase on the piece work rate of Re. 1 per unit for
every 2% increase in average individual output achieved. To sell the increased
output, it will be necessary to decrease the selling price by 4%. Calculate the
extra weekly contribution resulting from the proposed change and evaluate for the
Board�s information, the desirability of introducing the change. 48,000 units from
160 employees Rs. 6,00,000 = Rs. 80 + Rs. 36 = Rs. 116

3.64
Labour

Solution 1. Present average output per employee and total future expected output
per week Present average output per employees per week = =

Total present output Total number of present employees 48,000 units = 300 units
160 employees

Total Future expected output per week= Total number of future employees(present
output + 60% of present output per employee) = 120 employees (300 units + 60% �
300 units) = 57,600 units 2. Present and proposed piece work rate Present piece
work rate Proposed piece work rate = Re 1.00 per unit = Present piece work rate +
30% � Re. 1 = Re. 1.00 + 0.30 P = Rs. 1.30 per unit 3. Present and proposed sale
price per unit Present sales price per unit (Rs. 6,00,000/48,000 units) Proposed
sale price per unit (Rs. 12.50 � 4% � Rs. 12.50) 4. Present marginal cost
(excluding wages) per unit : = = Rs. 12 = Rs. 12.50

Present sales value - Fixed expenses & profit - Contribution towards present wages
Present outputa (units)

Rs.6,00,000 - Rs.2,40,000 - Rs.48,000 = Rs. 6.50 per unit 48,000units

3.65
Cost Accounting

Statement of extra weekly contribution (Information resulting from the proposed


change of mechanisation meant for Board�s evaluation) Expected sales units (Refer
to Working note 1) Rs. Sales value : (A) (57,600 units � Rs. 12) (Refer to Working
note 3) Marginal costs (excluding wages) : (B) (57,600 units � Rs. 6.50) (Refer to
Working note 4) Wages : (C) (57,600 units � Rs. 1.30) (Refer to Working note 2)
Total marginal cost : (D) = {(B) � (C)} Marginal contribution : {(A) � (D)} Less :
Present contribution Increase in contribution (per week) 4,49,280 2,41,920
2,40,000 1,920 74,880 3,74,400 Rs. 6,91,200 57,600

Evaluation: Since the mechanisation has resulted in the increase of contribution


to the extent of Rs. 1,920 per week, therefore the proposed change should be
accepted. 3.10 ABSORPTION OF WAGES 3.10.1 Elements of wages: In common parlance,
the term �wages� represents monetary payment which an employee receives at regular
intervals for the services rendered. Strictly speaking, however, from the point of
view of the employer and the cost to the industry, wages should be taken to
include also non-monetary benefits which an employee receives by virtue of
employment. Such non-monetary benefits may include: (i) (ii) medical facilities;
educational and training facilities;

(iii) recreational and sports facilities; (iv) housing and social welfare; and (v)
cost of subsidised canteen and co-operative societies. Such benefits are generally
given in an industrial establishment. In some cases the provision of benefits is
compulsory. Therefore, while computing the wage cost per worker, the monetary
3.66
Labour

value of such non-monetary benefits should also be taken into account. The
monetary part of a worker�s remuneration includes the basic wages, dearness
allowance, overtime wages, other special allowance, if any, production bonus,
employer�s contribution to the provident fund, Employees State Insurance
Corporation premium, contribution to pension fund, leave pay, etc. The basic wage
is the payment for work done, measured in terms of hours attended or the units
produced, as the case may be. The basic wage rate is not normally altered unless
there is a fundamental change in the working conditions or methods of manufacture.
Dearness allowance is an allowance provided to cover the increase in cost of
living from one period to another. This allowance is calculated either as
percentage of the basic wage or as a fixed amount for the days worked. In either
case, the percentage or the fixed amount is subject to revision whenever the cost
of living index rises or falls by a certain figure as agreed to by the employer
with the labour union. When permanent rise in the cost of living index occurs, a
part of the dearness allowance is often absorbed in the basic wage. Overtime
allowance is an allowance paid for the extra hours worked at the rates laid down
in the Factories Act. In certain industries, where special allowance for the
working conditions, tool maintenance, etc., are paid they are also considered as
part of wages. Production bonus is an incentive payment made to workers for
efficiency that results in production above the standard. There are different
methods of computing incentives. Under the Payment of Bonus Act, a worker is
entitled to compulsory bonus of 8.33% wages earned in the relevant year or Rs. 100
(whichever is greater). The bonus may be upto 20% of wages depending upon the
quantum of profits calculated as per the Act. Contributions to provident fund and
to E.S.I. schemes are compulsory. Under the first mentioned scheme, the employer
contributes to the provident fund a sum equal to 8?% of the basic wages together
with the dearness allowance. Under the E.S.I. scheme, the employer contributes a
fixed amount per month per worker calculated on the amount of wages as prescribed
by the E.S.I. Act. The payments by the employer under these schemes are payable at
the time, wages are paid for the relevant period; these payments form a part of
the wages for the period. 3.10.2 Component of wages cost or wages for costing
purposes : In addition to wages (including allowances, such as D.A.) that are paid
to workers, a firm may have to spend on many other items (such as premium to the
Employees State Insurance Corporation or provident fund or bonus). Further, the
worker does not spend all the time for which he is paid on productive work. This
is because he is entitled to weekly holiday and various type of leave. There is
also a certain amount of unavoidable idle time. The question is : to what extent
such additional payment or cost in respect of labour can be charged directly to
unit of cost as part of direct labour cost? Of course, in the case of indirect
labour, all such payments as also the

3.67
Cost Accounting

wages paid to them, must be treated as part of overheads. But in the case of
direct workers, two alternatives are possible. The additional charges may be
treated as overheads. Alternatively, the wage rates being charged to job may be
computed by including such payments; automatically then, such payments will be
charged to the work done alongwith wages of the worker. (It should be remembered
that such wage rate will be only for costing purposes and not for payment to
workers). The total of wages and additional payment should be divided by effective
hours of work to get such wage rates for costing purposes. Illustration A worker
is paid Rs. 100 per month and a dearness allowance of Rs. 200 p.m. There is a
provident fund @ 8?% and the employer also contributes the same amount as the
employee. The Employees State Insurance Corporation premium is 1�% of wages of
which �% is paid by the employees. It is the firm�s practice to pay 2 months�
wages as bonus each year. The number of working days in a year are 300 of 8 hours
each. Out of these the worker is entitled to 15 days leave on full pay. Calculate
the wage rate per hour for costing purposes. Solution Wages paid to worker during
the year *Add Provident Fund @ 8.33% *E.S.I. Premium 1% Bonus at 2 months� wages
Total Effective hours per year: 285 � 8 = 2,280 Rs. 3,600 300 36 600 4,536

Wage-rate per hour (for costing purpose): Rs. 4,536/2,280 hours = Rs. 1.989
Illustration : Calculate the earnings of A and B from the following particulars
for a month and allocate the labour cost to each job X, Y and Z: A (i) (ii) Basic
wages Dearness allowance (on basic wages) Rs. 100 50% 8% 2% Hours 10 B 160 50% 8%
2%

(iii) Contribution to provident fund (on basic wages) (iv) Contribution to


employees� state insurance (on basic wages) (v) Overtime

3.68
Labour

The normal working hours for the month are 200. Overtime is paid at double the
total of normal wages and dearness allowance. Employer�s contribution to State
Insurance and Provident Fund are at equal rates and employees� contributions. The
two workers were employed on jobs X, Y and Z in the following proportions: Jobs X
Worker A Worker B Overtime was done on job Y. Solution Statement showing Earnings
of Workers A and B Workers Basic wages Dearness allowance (50% of basic wages)
Overtime wages (Refer to working note 1) Gross wages earned Less: - Provident
fund-8% of basic wages - ESI-2% of basic wage Net wages paid Statement of Labour
cost Gross wages (excluding overtime) Employer�s contribution to P.F. and E.S.I.
Ordinary wages Labour rate per hour Rs. 150 10 160 0.80 (Rs.160/200) Rs. 240 16
256 1.28 (Rs. 256/200) 10 155 16 224 A Rs. 100 50 15 165 B Rs. 160 80 240 40% 50%
Y 30% 20% Z 30% 30%

3.69
Cost Accounting

Statement showing allocation of wages to jobs Total Wages Rs. Worker A : Ordinary
wages: (4:3:3) Overtime Worker B : Ordinary wages: (5:2:3) Working Notes Normal
wages are considered as basic wages Overtime = 2 � 256 431 128 192 51.20 114.2
76.8 124.8 160 15 64 � 48 15 48 � X Rs. Jobs Y Rs. Z Rs.

(Basic + D.A.) � 10 hours 200


= Rs.15

= 2 � (Rs. 150/200) � 10 hours Illustration

In a factory working six days in a week and eight hours each day, a worker is paid
at the rate of Rs. 100 per day basic plus D.A. @ 120% of basic. He is allowed to
take 30 minutes off during his hours shift for meals-break and a 10 minutes recess
for rest. During a week, his card showed that his time was chargeable to : Job X
Job Y Job Z 15 hrs. 12 hrs. 13 hrs.

The time not booked was wasted while waiting for a job. In Cost Accounting, how
would you allocate the wages of the workers for the week?

3.70
Labour

Solution Working notes: (i) (ii) Total effective hours in a week : [(8 hrs. � (30
mts. + 10 mts.)] � 6 days Total wages for a week : (Rs. 100 + 120% of Rs. 100) � 6
days (iii) Wage rate per hour : (iv) Time wasted waiting for job (Abnormal idle
time): Allocation of wages in Cost Accounting Rs. Allocated to Job X Allocated to
Job Y Allocated to Job Z Charged to Costing Profit & Loss A/c Total : 15 hours �
Rs. 30 : 12 hours � Rs. 30 : 13 hours � Rs. 30 : 4 hours � Rs. 30 = = = = 450 360
390 120 1,320 = 44 hrs. � (15 hrs. + 12 hrs. + 13 hrs.) = 4 hrs. = Rs. 1,320 = Rs.
30 = 44 hours

3.10.3 Holiday and leave wages : One alternative to account for wages paid on
account of paid holiday and leave can be to include them as departmental
overheads. In such a case, it is necessary to record such wages separately from
�worked for wages�. Such a segregation can be made possible by providing a
separate column in the payroll for holiday and leave wages in the same way as
there are columns for dearness allowance, provident fund deductions, etc. If,
however, a separate or additional column cannot be provided for this purpose it
would be necessary to analyse the payroll periodically to ascertain how much of
the total payment pertains to �worked for wages� and how much is attributed to
leave and holiday wages. Another way could be to inflate the wage rate for costing
purposes to include holiday and leave wages. This can be done only in the case of
direct workers.

3.71
Cost Accounting

Illustration Calculate the labour hour rate of a worker X from the following data
: Basic pay D.A. Fringe benefits Rs. 1,000 p.m. Rs. 300 p.m. Rs. 100 p.m.

Number of working days in a year 300. 20 days are availed off as holidays on full
pay in a year. Assume a day of 8 hours. Solution (a) (i) Effective working
hour/days in a year Less : Leave days on full pay Effective working days Total
effective working hours (280 days � 8 hours) (ii) Total wages paid in a year Basic
pay D.A. Fringe benefits 300 20 280 days 2,240 Rs. 12,000 3,600 1,200 16,800 (iii)
Hourly rate : Rs. 16,800/2,240 hours Rs. 7.50

3.10.4 Night shift allowance: In some cases, workers get extra payment if they
work at night. Since the extra payment is not for any particular job, therefore
such a payment should be treated as part of overheads. 3.10.5 Principles of
remuneration: The term �remuneration� has been defined as the reward for labour
and service. It is the result of the agreement between the employer and the
employee, whereby for a specified work or service rendered by the employee the
employer agrees to pay a specified sum of money. Apart from this an employee by
virtue of the fact that he is an employee becomes entitled to certain non-monetary
benefits. The method of remuneration adopted varies from industry to industry and,
in certain cases, even among different units in the same industry. Whatever be the
variation, the method of fixing remuneration payable to the various categories of
employees has to be based on certain accepted principles. These are:

3.72
Labour

(i)

Wage-rates in an industry should be fixed in conformity with the general wage-


levels in the geographical area i.e. the rate should be more or less the same for
similar efforts and skill. The wage-level in an area would in turn depend upon
demand for labour, the availability of labour, the cost of living in the area, the
wage levels in neighbouring industrial area, and the capacity of the particular
industry to pay. Wage-rates should be related to the degree of skill, effort,
initiative and responsibility that the employee is expected to assume in respect
of the various jobs he may be called upon to perform. There should be generally
equal pay for equal work.

(ii)

(iii) Wage-rates should guarantee a minimum wage regardless of the existence of


factors listed under (ii) above, particularly when the working conditions are
difficult and dangerous. (iv) Wage-rates are considered satisfactory only if they
enable the workers to maintain a reasonable standard of living. (v) Separate wage
rates should be fixed for different classes of employees since each class expects
to maintain a different standard of living; also the education, physical and
mental efforts and responsibility required for performing different jobs are not
the same. (vi) It should be possible for worker to increase his earnings through
extra effort and by increasing output. If he alone is responsible, he should have
the full benefit of the increased productivity. Otherwise, if increased output has
resulted from co-operation between management and workers both should share the
benefit. It is important that these basic principles should be recognised in
fixing the wage rate of workers; otherwise, there will be dissatisfaction among
the employees and, consequently, there will be higher labour turnover.
Satisfactory employer-employee relationship is a primary necessity for industrial
development and this has to be ensured to a very great degree, by satisfactory
schemes of remunerating labour. The aim should be to keep labour cost per unit of
output (or service) as low as possible. It is not the same as keeping wages at low
levels. There is a definite correlation between wages and productivity; high wages
often lead to such an increase in productivity that wages per unit of output fall.
However, this rule is also subject to diminishing returns�a point is reached at
which any further increase in wage rates does not bring about a corresponding
increase in efficiency. But generally, higher wages result in lower cost per unit.
Wages affect the national economy through cost of goods produced. If an increase
in wages outpaces the corresponding increase in productivity, goods become
costlier and cannot compete with those of other countries in the world markets.
From the point of view of an expert it is necessary to keep wages in check like
other costs. The safe rule is to link up wages with productivity.

3.73
Cost Accounting

3.10.6 Absorption rates of labour cost: Labour cost as stated above include
monetary compensation and non-monetary benefits to workers. Monetary benefits
include, basic wages, D.A., overtime pay, incentive or production bonus
contribution to employee provident fund, House Rent Allowance, Holiday and
vacation pay etc. The non-monetary benefits include medical facilities, subsidized
canteen services, subsidized housing, education and training facilities.
Accounting of monetary and non-monetary benefits to indirect workers does not pose
any problems because the total of monetary and non-monetary benefits are treated
as overhead and absorbed on the basis of rate per direct labour hour, if overheads
are predominantly labour oriented. For direct workers, the ideal method is to
charge jobs or units produced by supplying per hour rate calculated as below :
Rate per hour =

Total of estimated monetary benefits and cost of non - monetary benefits Budgeted
direct labour hours - Normal idle time

Another alternative method is to treat the monetary benefits other than basic
wages and dearness allowance as well as cost of non-monetary benefits as
overheads. 3.11 EFFICIENCY RATING PROCEDURES Efficiency is usually related with
performance and may be computed by comparing the time taken with the standard time
allotted to perform the given job/task. If the time taken by a worker on a job
equals or less than the standard time, then he is rated efficient. In case he
takes more time then the standard time he is rated as inefficient. It may be
computed as follows : Efficiency in % =

Time allowed as per std. � 100 Time taken

For efficiency rating of employees the following procedures may be followed : 1.


Determining standard time/performance standards : The first step is to determine
the standard time taken by a worker for performing a particular job/task. The
standard time can be determined by using Time & Motion study or Work study
techniques. While determining the standard time for a job/task a heterogeneous
group of workers is taken and contingency allowances are added for determining
standard time. 2. Measuring Actual Performance of workers : For computing
efficiency rating it is necessary to develop a procedure for recording the actual
performance of workers. The system developed should record the output of each
worker along with the time taken by him. 3. Computation of efficiency rating : The
efficiency rating of each worker can be computed by using the above mentioned
Formula.
3.74
Labour

3.11.1 Need for efficiency rating : 1. As discussed earlier when a firm follows a
system of payment by results, the payment has a direct relationship with the
output given by a worker. The firm for making payment to worker is required to
ascertain his efficiency level. For instance, under Taylor's differential piece
work system the lower rate i.e. 83% of piece rate is given to a worker when his
efficiency rating is less than 100% and higher rate viz., 125% of piece rate is
offered at efficiency level of either 100% or more. Similarly under Emersion
efficiency plans bonus is paid at rising scale at various level of efficiency,
ranging from 66.67% to 150%. The efficiency rating also helps the management in
preparing labour requirement budget or for preparing manpower requirements. For
example, let P Ltd. manufactures two products by using one grade of labour. The
following estimates are available : Product A Budgeted production (units) Std.
hrs. allowed per product 3,480 5 Product B 4,000 4

2.

It is further worked out that the efficiency rating (efficiency ratio) for
productive hours worked by direct workers in actually manufacturing the production
is 80% then the exact standard labour requirement can be worked out as follows :
Product A Budgeted production (in units) Std. hours allowed for budgeted
production 3,480 17,400 (3,480 units � 5 hours) Product B 4,000 16,000 (4,000
units � 4 hours) 33,400 Total

Since efficiency ratio is given as 80% therefore Std. labour hours required for
100% efficiency 100 ? ? level are ? 33,400 hours � ? = 41,750 hours. 80 ? ? Labour
productivity : Productivity is generally determined by the input/output ratio. In
the case of labour it is calculated as below :

Standard time for doing actual amount of work Actual time taken to do work
Labour productivity is an important measure for measuring the efficiency of
individual workers. It is an index of efficiency and a sign of effectiveness in
the utilisation of resources-men, materials, capital, power and all kinds of
services and facilities. It is measured by the output in relation to input.
Productivity can be improved by reducing the input for a certain quantity or
3.75
Cost Accounting

value of output or by increasing the output from the same given quantity or value
of input. Factors for increasing labour productivity : The important factors which
must be taken into consideration for increasing labour productivity are as
follows: 1. 2. 3. 4. 5. Employing only those workers who possess the right type of
skill. Placing a right type of man on the right job. Training young and old
workers by providing them the right types of opportunities. Taking appropriate
measures to avoid the situation of excess or shortage of labour at the shop floor.
Carrying out work study for the fixation of wage rate, and for the simplification
and standardisation of work.

Self Examination Questions Multiple Choice Questions 1. The input-output ratio in


case of labour means the ratio of (a) the value of output to the wages paid. (b)
standard time of the production to the actual time paid for. (c) abnormal idle
time to normal idle time. (d) number of workers employed to the sanctioned
strength. 2. Job specification is (a) the list of operations to be performed for
completing the concerned job (b) the requirement in terms of goods to be produced
or work to be done. (c) the list of qualities and qualifications which the
employees concerned should have to do the job well. (d) the name of the employees
who will be assigned to a job. 3. Job specification is (a) the list of operations
to be performed for completing the concerned job (b) the requirement in terms of
goods to be produced or work to be done. (c) the list of qualities and
qualifications which the employees concerned should have to do the job well. (d)
the name of the employees who will be assigned to a job.

3.76
Labour

4.

Direct labour means (a) labour completing the work manually, (b) labour which is
recruited directly and not through contractors, (c) permanent labour in the
production department, (d) labour which can be conveniently associated with a
particular cost unit.

5.

Time and motion study is essential for (a) a rational promotion policy, (b)
completing a job on time, (c) determining the standard-time and correct method of
completing a task, (d) determining prices of products.

6.

For reducing the labour cost per unit, which of the following factors is the most
important? (a) low wage rates, (b) higher input-output ratio, (c) strict control
and supervision, (d) longer hours of work.

7.

Which of the following statements are true ? (a) Productivity of workers can be
improved only if they are supervised closely. (b) It is no use paying higher wages
to labour because they would spend their money on drinking and smoking. (c) A well
satisfied team of workers can raise productivity to a large extent. (d) None of
the above

8.

Labour turnover is measured by, (a) Replacement method. (b) Separation method. (c)
Flux method. (d) All of the above.

3.77
Cost Accounting

9.

Salary of a foreman should be classified as, (a) Fixed overhead. (b) Variable
overhead. (c) Semi fixed or semi variable overhead. (d) None of the above.

10. For reducing the labour cost per unit, which of the following factors is the
most important? (a) Low wage rates. (b) Higher input output ratio. (c) Strict
control. (d) Long hours of work. Answers To Multiple Choice Questions
1.(b);2.(c);3.(a) ;4.(d);5.(c);6.(b);7.(c);8.(d);9.(c);10 (b) Short Answer Type
Questions 1. Describe briefly the functions of the following departments in
relation to labour : (a) Personnel department. (b) Engineering department. (c)
Cost Accounting department. 2. Distinguish between : (a) Time keeping and Time
booking (b) Time study and motion study. 3. 4. 5. Discuss briefly the important
factors for the control of labour cost. Discuss briefly the objectives of time
keeping. Discuss briefly the various factors necessary for introducing an
incentive system.

Long Answer Type Questions 1. What is idle time? Explain the causes leading to
idle time and its treatment in Cost Accounts.

3.78
Labour

2. 3. 4. 5.

What do you mean by overtime premium? What are the causes of overtime? How would
you treat overtime premium in Cost Accounts ? What do you understand by labour
turnover? How is it measured? What are its causes? What steps should be taken to
check the increasing rate of labour turnover? Define job evaluation and
distinguish it from merit rating. Explain the methods and objectives of job
evaluation. Explain the factors to be considered in introducing an incentive
system.

Numerical Questions 1. Calculate the number of hours worked as overtime by the


following workers in a week: Ram Monday Tuesday Wednesday Thursday Friday Saturday
8 7 4.5 8 10 9 46.5 Shyam 8 9 8 7 9 9 50

2.

Three workers A, B and C are put on a common task for which the total remuneration
is Rs. 150. A works for 40 hours, B works for 60 hours and C works for 44 hours on
the job. The hourly rate is Re. 0.75 of A per hour, B gets Re. 0.80 per hour while
C�s remuneration is Re. 0.50 per hour. What should each man get ? A worker is paid
@ 50 paise per hour plus a dearness allowance of Rs. 60 per month. The provident
fund contribution both by the employee and the worker is 6�% each. The worker is
entitled to 15 days leave with full wages. His normal working per month is 25 days
of 8 hours each. (a) the wages per hour for costing purposes; and (b) the amount
to be paid to him for a week in which he puts in 52 hours of work.

3.

3.79
Cost Accounting

4.

The following particulars are available to you in respect of a worker: Job No.Time
Allowed 1844 1826 26 hours 30 hours Time Taken 20 hours 20 hours 8 hours 48 hours

Idle time (waiting)

The basic rate is Rs. 2 per day of 8 hours in addition there is a dearness
allowance of Rs. 12 per week of 48 hours. Calculate the wage of the worker on (1)
Time Basis (2) Piece Rate Basis (3) Halsey Plan Basis and (4) Rowan Plan Basis. 5.
A worker is paid 10% bonus on the hourly rate if he completes his work in the time
allotted for it and a further 1% on hourly rate for each 1% in excess of 100%
efficiency. His hourly rate is Rs. 5 per hour and he completed a job in 45 hours
whereas the time allowed for it was 50 hours. Ascertain the wages earned by this
worker. From the following data, calculate the labour turnover rate by applying :
(i) (ii) Separation method Accession method

6.

(iii) Flux method Number of workers on the payroll At the beginning of the month
At the end of the month 1,800 2,200

During the month 20 workers left, 80 workers were discharged and 500 workers were
recruited. Of these 50 workers were recruited in the vacancies of those separated,
while the rest were engaged due to expansion. 7. The company has a suggestion of
box scheme and an award equivalent to one and a half months saving in labour cost
is passed on to the employee whose suggestion is accepted. Suggestion of an
employee to use a Jig for a manufacturing operation of a component is accepted.
The cost of the Jig which has a life of one year is Rs. 1,000 and the use of the
Jig will reduce the standard time by 8 minutes. Compute from the following data
the amount of award payable to the employee who has given the suggestion. (i)
3.80

Number of pieces to be produced in the year :

15,000
Labour

(ii)

Standard time per piece before use of Jig

80 minutes

(iii) Average wage rate of workmen Rs. 160 per day of 8 hours. (iv) Average
efficiency of workmen : 80%. 8. The existing incentive system of a certain factory
is : Normal working week Rate of payment Additional bonus payable Average output
per operative for 54 hour week, i.e, normal working hours plus 3 hours late
sitting for 3 days : : : : 120 articles 5 days of 9 hours each plus Resorting to
overtime of 3 hours for 3 days. For day work - Rs. 20 per hour. For overtime Rs.
30 per hour. Rs. 25 per day if worker is not resorting to overtime. Rs. 40 per day
if worker resorts to overtime.

In order to increase output and eliminate overtime it was decided to switch on to


a system of payment by results. The factory considering the introduction of some
incentive scheme or to make payment on piece work basis. Assuming that 135
articles are produced in a 45 hour week and the additional bonus under the
existing system will be discontinued in the proposed incentive scheme. You are
required to calculate : (i) Weekly earnings; (ii) labour cost per article for an
operative under the following systems: (a) Existing time-rate system (b) Straight
piece-work system (c) Rowan system (d) Halsey system The following information is
obtained. Time rate (as usual) Basic time allowed Piece work rate Premium bonus :
: : : Rs. 20 per hour for 15 articles 5 hours Add 20% to price Add 50% to time

3.81
Cost Accounting

9.

The unit has a strength of 20 workmen worked for 300 working days of 8 hours each
with half an hour break based on the earlier years trend, it is forecast that
average absenteeism per workman would be 8 days, in addition to the eligibility of
30 days annual leave. The following details regarding actual working of the unit
are available for the year ending on 31st March, 1998. (i) (ii) The factory worked
2 extra days to meet the production targets, but one additional paid holiday had
to be declared. There was a severe breakdown of a major equipment leading to a
loss of 300 man hours.

(iii) Total overtime hours (in addition to 2 extra days worked) amounted to 650
hours. (iv) The actual average absenteeism per workman was 8 days. (v) Basic rate
is Rs. 10 per hour and overtime is paid at double rate. You are required to
calculate. (a) Actual working hours of the unit. (b) In Cost Accounting how would
you treat the wages of workmen for (ii) & (iii) above ? 10. A job can be executed
either through workman A or B. A takes 32 hours to complete the job while B
finishes it in 30 hours. The standard time to finish the job is 40 hours. The
hourly wage rate is same for both the workers. In addition workman A is entitled
to receive bonus according to Halsey plan (50%) sharing while B is paid bonus as
per Rowan plan. The works overheads are absorbed on the job at Rs. 7.50 per labour
hour worked. The factory cost of the job comes to Rs. 2,600 irrespective of the
workman engaged. Find out the hourly wage rate and cost of raw materials input.
Also show cost against each element of cost included in factory cost.

3.82
CHAPTER

OVERHEADS
Learning objectives When you have finished studying this chapter, you should be
able to ? Differentiate between direct costs and overheads. ? Understand the
meaning of allocation, apportionment and absorption of overheads. ? Identify,
whether overheads are under absorbed or over absorbed ? Understand the accounting
and control of administrative, selling and distribution overheads. 4.1
INTRODUCTION

Besides direct expenditure, i.e., expenditure which can be conveniently traced to


or identified with any particular unit of production, e.g., direct materials,
direct wages and direct expenses, every form of production involves expenses that
cannot be conveniently traced to or identified with the articles produced or
services provided. Such expenses are incurred for output generally and not for a
particular work order e.g., wages paid to watch and ward staff, heating and
lighting expenses of factory etc. Expenses of these nature are known as overhead
or indirect expenses. Often in a manufacturing concern, overheads exceed direct
wages or direct materials and at times even both put together. On this account, it
would be a grave mistake to ignore overheads either for the purpose of arriving at
the cost of a job or a product or for controlling total expenditure. Overheads
also represent expenses that have been incurred in providing certain ancillary
facilities or services which facilitate or make possible the carrying out of the
production process; by themselves these services are not of any use. For instance,
a boiler house produces steam so that machines may run and, without the generation
of steam, production would be seriously hampered. But if machines do not run or do
not require steam, the boiler house would be useless and the expenses incurred
would be a waste. Apart from the overheads incurred in the factory, overheads also
arise on account of administration, selling and distribution.
Cost Accounting

4.2

CLASSIFICATION OF OVERHEADS

4.2.1 Classification of overheads by function : Overheads principally are of four


types: (i) Factory or Manufacturing Overheads; (ii) Office and Administrative
Overheads; (iii) Selling and Distribution Overheads; (iv) Research and
Development. Whether an expense belongs to one class or another depends entirely
on the benefit derived from it. For instance, salaries of clerks will be (i)
factory or manufacturing expenses, when the clerks concerned work in the factory
office; (ii) office and administrative expense when the clerks work in the general
office; and (iii) selling and distribution expense when the clerks work in the
sales office. Small concerns may not distinguish between office and selling
expenses and still smaller concerns may treat all overhead, of whatever class,
together. Big concerns may have even a more detailed classification to be able to
exercise better control. A detailed classification of such expenses is given
below: (i) Factory or Manufacturing Expenses: (a) Stores overheads (expenses
connected with purchasing and handling of materials); (b) Labour overheads
(expenses connected with labour); and (c) Factory administration overheads
(expenses connected with administration of the factory). (ii) Office and
Administration Expenses: (a) Administrative expenses (expenses incurred on
managerial personnel - their salaries, costs of facilities provided to them and
salaries of their personal staff); and (b) Office expenses (expenses on the
routine office work). (iii) Selling and Distribution Expenses : (a) Selling
expenses (expenses incurred to persuade customers to purchase the firm�s products
and, or engage its services, that is to maintain and expand the market); and (b)
Distribution expenses are those which are incurred to execute orders. One should
note that many people use the two terms �selling� and �distribution� as
synonymous. Following are the definitions given by the Institute of Cost and
Management Accountants of England.

4.2
Overheads

Production Cost - The cost of the sequence of operations which begins with
supplying materials, labour and service and ends with the primary packing of the
product. Selling Cost - The cost of seeking to create and stimulate demand
sometimes termed (marketing) and of securing orders. Distribution Cost - The cost
of the �sequence� of operations which begins with making the packed product
available for despatch and ends with making the reconditioned returned empty
package, if any, available for re-use. As well as including expenditure incurred
in moving articles to central or local storage, distribution cost includes
expenditure incurred in moving articles to and from prospective customers as in
the case of goods on sale or return basis. In the gas, electricity and water
industries �Distribution� means pipes, mains and services which may be regarded as
equivalent to packing and transportation. Administration cost - The cost of
formulating the policy, directing the organisation and controlling the operations
of an undertaking which is not related directly to production, selling,
distribution, research or development activity or function. Research and
Development Expenses : The Terminology defines research expenses as �the expenses
of searching for new or improved products, new application of materials, or new or
improved methods.� Similarly, development expenses is defined as �the expenses of
the process which begins with the implementation of the decision to produce a new
or improved product.� If research is conducted in the methods of production, the
research expenses should be charged to the production overhead; while the
expenditure becomes a part of the administration overhead if research relates to
administration. Similarly, market research expenses are charged to the selling and
distribution overhead. Development costs incurred in connection with a particular
product should be charged directly to that product. Such expenses are usually
treated as �deferred revenue expenses,� and recovered as a cost per unit of the
product when production is fully established. General research expenses of a
routine nature incurred on new or improved methods of manufacture or the
improvement of the existing products should be charged to the general overhead.
Even in this case, if the amount involved is substantial it may be treated as a
deferred revenue expenditure, and spread over the period during which the benefit
would accrue. Expenses on fundamental research, not relating to any specific
product, are treated as a part of the administration overhead. Where research
proves a failure, the cost associated with it should be excluded from costs and
charged to the costing Profit and Loss Account. A list (not exhaustive) of various
items under three principal classes of overheads is presented on the next page.

4.3
Factory Expenses Buildings : Rent, repairs, depreciation and insurance of factory
premises, lighting of factory premises. : Depreciation, repairs and maintenance
and insurance of plant and equipment; power used for machines. Wages of indirect
workers; normal idle time (unless wage rates are inflated suitably); Employees�
State Insurance premium; Provident Fund contribution, leave pay, maternity pay;
etc. Salaries to foremen, departmental superintendents and Works Manager;
Technical Director�s fees. Purchasing and store keeping expenses, cost of
consumable stores and supplies, normal losses of materials unless prices are
suitably inflated, etc. Factory office telephone, stationery, factory office
clerks� salaries, etc.

Machinery

Office and Administration Expenses Administration : Fees to Directors� Salary to


General Manager, Managing Director, Finance Manager, Chief Accountant, Secretary,
their immediate personal staff and other expenses like air conditioning of office;

Labour

Supervision

Materials

Office : Salaries paid to other people working in the office; stationery, postage,
etc. lighting of office, rent, rates, and taxes on office premises. Depreciation,
power, insurance repairs and maintenance of office equipment, etc.

Selling and Distribution Expenses Selling: Fees to Director who looks after sale
and marketing salary of Sales Manager; salesmen and sales office clerks;
commission to agents, advertising catalogues, price lists. samples, show room
expenses; entertainment of customers; stationery, postage etc., used in the
office. Distribution: Finished goods godown expenses-salary, rent, insurance
lighting etc. packing; carriage outwards; insurance, in transit; delivery
expenses; expenses on receiving and reconditioning, returnable empties.

Misc

:
Overheads

Expenses that are not taken into account - The undermentioned expenses are usually
not included in overheads or, for that matter in cost : (a) Expenses or income of
purely financial nature like dividends received, rent received, cash discount
allowed, etc. (b) Expenses or profits of capital nature like profit or loss on
sale of investments, plant and equipment, etc. (c) Items not representing actual
costs but dependent on arbitrary decisions of the management, e.g., an
unreasonably high salary to the managing director, providing for depreciation at a
rate exceeding the economic rate. (d) Appropriation of profits for dividends,
payment of income tax and transfers to reserves. 4.2.2 Classification of overheads
by nature : On a change in the level of activity different expenses behave
differently. On this consideration, expenses are classified under the following
three categories: (i) Fixed or Constant : These are expenses that are not affected
by any variation in the volume of activity, e.g., managerial remuneration, rent,
that part of depreciation which is dependent purely on efflux of time, etc. Fixed
or constant expenses remain the same from one period to another except when they
are deliberately changed, e.g., on increments being granted to staff or additional
staff being engaged. (ii) Variable : Expenses that change in proportion to the
change in the volume of activity; when output goes up by 10% the variable expenses
also go up by 10%. Correspondingly, on a decline of the output, these expenses
also decline proportionately e.g., power consumed; consumable stores; repairs and
maintenance and depreciation are dependent on the use of assets. Variable expenses
are generally constant per unit of output or activity. Suppose variable expenses
amount to Rs. 10,000 for a production of 2,000 units i.e., Rs. 5 per unit. When
output goes upto 2,200 units, i.e., an increase of 10% the variable expenses
amount to Rs. 11,000. i.e., 10,000 plus 10%. The cost per unit will be the same as
before. (iii) Semi variable : The expenses that either (a) do not change when
there is a small change in the level of activity but change whenever there is a
slightly big change. In other words, they change by small steps; or (b) change in
the same direction as change in the level of activity but not in the same
proportion. An expense for example, may not change if output goes up or comes down
by 5% but may change by 3% when there is an increase in production between 5% and
10%. Similarly, another item of expense may change by 1% for every 2% change in
activity. Examples of such expenses are : delivery van expenses,

4.5
Cost Accounting

telephone charges, depreciation as a whole. Semi-variable expenses usually have


two parts�fixed and variable. For instance, the amount of depreciation usually
depends on two factors�one on time (fixed) and the other on wear and tear
(variable). The two together make depreciation (as a whole) semivariable. A
careful study can make it possible for all semi-variable expenses to split up into
two parts. Fundamentally, therefore, there are only two type of expenses�fixed and
variable. Graphically the three type of expenses may be shown as below:

One must note that fixed expenses remain unchanged upto the limit of the present
capacity. If output goes beyond the capacity limit, fixed expenses will record a
jump. Suppose a factory works one shift and produces 10,000 units in the shift.
For all levels of output of 10,000 units, fixed expenses will remain unchanged; if
the output goes beyond 10,000 units, a second shift will become necessary and this
will mean a big increase in fixed expenses such as salary for foremen, lighting
etc. Methods of segregating Semi-variable costs into fixed and variable costs �
For a detailed understanding please refer to chapter 1. Advantages of
Classification of Overheads into Fixed and Variable : The primary objective of
segregating semi-variable expenses into fixed and variable is to ascertain
marginal costs. Besides this, it has the following advantages also. (a)
Controlling expenses : The classification of expenses into fixed and variable
components helps in controlling expenses. Fixed costs are generally policy costs,
which cannot be easily reduced. They are incurred irrespective of the output and
hence are more or less non controllable. Variable expenses vary with the volume of
activity and the responsibility for incurring such an expenditure is determined in
relation to the output. The management can control these costs by giving proper
allowances in accordance with the

4.6
Overheads

output achieved. (b) Preparation of budget estimates : The segregation of


overheads into fixed and variable part helps in the preparation of flexible
budget. It enables a firm to estimate costs at different levels of activity and
make comparison with the actual expenses incurred. Suppose in October, 2005 the
output of a factory was 1,000 units and the expenses were: Rs. Fixed Variable
Semi-variable (40% fixed) 5,000 4,000 6,000

15,000 In November, 2005 the output was likely to increase to 1,200 units. In that
case the budget or estimate of expenses will be : Rs. Fixed Variable 5,000 4,800

? Rs.4,000 � 1,200 units ? ? ? 1,000 units ? ?


Semi-variable

Fixed, 40% of Rs. 6,000 ? Rs.3,600 � 1,200 units ? Variable : ? ? 4,320 1,000
units ? ?

2,400 6,720

16,520 It would be a mistake to think that with the output going up from 1,000
units to 1,200 units the expenses would increase proportionately to Rs. 18,000.
This would be wrong budgeting. (c) Decision making : The segregation of semi
variable cost between fixed and variable overhead also helps the management to
take many important decisions. For example, decisions regarding the price to be
charged during depression or recession or for export market. Likewise, decisions
on make or buy, shut down or continue, etc., are also taken after separating fixed
costs from variable costs. In fact, when any change is contemplated, say, increase
or decrease in production, change in the process of manufacture or distribution,
it is necessary to know the total effect on cost (or revenue) and that would be

4.7
Cost Accounting

impossible without a correct segregation of fixed and variable costs. The


technique of marginal costing, cost volume profit relationship and break-even
analysis are all based on such a segregation.
4.3 ACCOUNTING AND CONTROL OF MANUFACTURING OVERHEADS

We have already seen that overheads are by nature those costs which cannot be
directly related to a product or to any other cost unit. Yet for working out the
total cost of a product or a unit of service, the overheads must be included. Thus
we have to find out a way by which the overheads can be distributed over the
various units of production. One method of working out the distribution of
overheads over the various products could be to ascertain the amount of actual
overheads and distribute them over the products. This however, creates a problem
since the actual amount of overheads can be known only after the financial
accounts are closed. If we wait that long, the cost sheets lose their main
advantages and utility to the management. All the decisions for which cost sheets
are prepared are immediate decisions and cannot be postponed till the actual
overheads are known. Therefore, some method has to be found by which overheads can
be included in the cost of the products, as soon as prime cost, the cost of raw
materials, labour and other direct expenses, is ascertained. One method is to work
out pre-determined rates for absorbing overheads. These rates are worked out
before an accounting period begins by estimating the amount of overheads and the
level of activity in the ensuing period. Thus, as soon as the prime cost of a
product or a job is available, the various overheads are charged by these rates.
Of course, this implies that the overheads are charged on an estimated basis.
Later, when the actual overheads are known, the difference between the overheads
charged to the products and actual overheads is worked out and adjusted.
Manufacturing Overheads : Generally manufacturing overheads form a substantial
portion of the total overheads. It is important, that such overheads should be
properly absorbed over the cost of production. The following procedure may be
adopted in this regard. The steps given below shows how factory overhead rates are
estimated and overheads absorbed on that basis and the last one shows how actuals
are compared with the absorbed amount.
(Students should carefully note the distinction between the various terms used).

1. Estimation and collection of manufacturing overheads : The first stage is to


estimate the amount of overheads, keeping in view the past figures and adjusting
them for known future changes. There are four main sources available for the
collection of factory overheads viz., (a) Invoices; (b) Stores requisition ; (c)
Wage analysis book ; (d) Journal entries. 2. Cost allocation : The term
�allocation� implies relating overheads directly to the various departments. The
estimated amount of various items of manufacturing overheads
4.8
Overheads

should be allocated to various cost centres or departments. The salary of the


works manager cannot be directly allocated to any one department since he looks
after the whole factory. It is, therefore, obvious that many overhead items will
remain unallocated after this step. 3. Cost apportionment : At this stage, those
items of estimated overheads (like the salary of the works manager) which cannot
be directly allocated to the various departments and cost centres are apportioned.
Apportionment implies �the allotment of proportions of items of cost to cost
centres or departments�. It implies that the unallocable expenses are to be spread
over the various departments or cost centres on an equitable basis. After this
stage, all the overhead costs would have been either allocated to or apportioned
over the various departments. 4. Re-apportionment : The next stage is to re-
apportion the overhead costs of service departments over production departments.
Service departments are those departments which do not directly take part in the
production of goods. Such departments provide ancillary services. Examples of such
departments are boiler house, canteen, stores, time office, dispensary etc. The
overheads of these departments are to be re-apportioned over the production
departments since service departments operate primarily for the purpose of
providing services to production departments. At this stage, all the factory
overheads are collected under production departments. 5. Absorption : The
production department overheads are absorbed over production units. The overhead
expenses can be absorbed by estimating the overhead expenses and then working out
an absorption rate. When overheads are estimated, their absorption is carried out
by adopting a pre-determined overhead absorption rate. This rate can be calculated
by using any one method as discussed in this chapter at the end. As the actual
accounting period begins, each unit of production automatically absorbs a certain
amount of factory overheads through pre-determined rates. During the year a
certain amount will be absorbed over the various products. This is known as the
total amount of absorbed overheads. 6. Treatment of over and under absorption of
overheads : After the year end the total amount of actual factory overheads is
known. There is bound to be some difference between the actual amount of overheads
and the absorbed amount of overheads. The difference has to be adjusted keeping in
view of such differences and the reasons therefor. Students will thus see that the
whole discussion as above is meant to serve the following two purposes : (a) to
charge various products and services with an equitable portion of the total amount
of factory overheads ; and

4.9
Cost Accounting

(b) to charge factory overheads immediately as the product or the job is completed
without waiting for the figures of actual factory overheads.
4.4 STEPS FOR THE DISTRIBUTION OF OVERHEADS

The various steps for the distribution of overheads have been discussed in detail
as below:
4.4.1 Estimation and Collection of Manufacturing Overheads : The amount of factory
overheads is required to be estimated. The estimation is usually done with
reference to past data adjusted for known future changes. The overhead expenses
are usually collected through a system of standing orders. Standing Orders : In
every manufacturing business, expenses are incurred on direct materials and direct
labour in respect of several jobs or other units of production, manufacture of
which is undertaken. The incurring of these expenses is authorised by production
orders or work orders. The work order numbers are not ordinarily fixed or
permanent. They are generally allotted in a serial order according to the number
of manufacturing jobs undertaken by the business. In addition, indirect expenses
are incurred in connection with the rendering of services to the production
departments, or to the manufacturing process. The term �Standing Order� denotes
sanction for indirect expenses under various heads of expenditure.

In large factories, usually the classification of indirect expenditures is


combined with a system of Standing Orders (sometimes also referred as Service
�Orders�). It is a system under which a number is allotted to each item of expense
for the purpose of identification, and the same is continued from year to year.
All the indirect expenditure in such a case, is charged to one or the other of the
Standing Orders and periodical summaries, giving total of each Standing Order, are
prepared for comparison with budgets, as well as for apportioning them among the
various departments. The extent of such analysis and the nomenclature adopted are
settled by the management according to the needs of the industry.
4.4.2 Allocation of overheads over various Departments or Departmentalisation of
Overheads : Most of the manufacturing processes functionally are different and are
performed by different departments in the factory. Where such a division of
functions had been made, some of the departments should be engaged in actual
production of goods, and others in providing services ancillary thereto. At this
stage, the factory overheads which can be directly related to the various
production or service departments are allocated in this manner.

It may, sometime, become necessary to sub-divide a manufacturing organisation into


several cost centres, so that a closer distribution of expenses and a more
detailed control is practicable.
4.10
Overheads

It is thus obvious that the principal object of setting up cost centres is to


collect data, in respect of similar activities more conveniently. This avoids a
great deal of cost analysis. When costs are collected by setting up cost centres,
several items can be ascertained definitely and the element of estimation is
reduced considerably. For instance, the allowance of the normal idle time or the
amount to be spent on consumable stores, etc. There are two main type of cost
centres - machine or personal - depending on whether the process of manufacture is
carried on at a centre by man or machine. For the convenience of recording of
expenditure, cost centres are sometimes allotted a code number.
Advantages of Departmentalisation : The collection of overheads departmentwise
gives rise to the following advantages :

(a) Some expenses which relate to the departments will be estimated almost on an
exact basis and, to that extent, the accuracy of estimation of overheads will be
higher. (b) For the purpose of controlling expenses in a department, it is
obviously necessary that the figures in relation to each department should be
separately available. It is one of the main principles of control that one should
know for each activity how much should have been spent and how much is actually
spent. If information about expenses is available only for factory as a whole, it
will not be possible to know which department has been over spending. (c) From the
point of view of ascertaining the cost of each job, the expenses incurred in the
departments through which the job or the product has passed should be known. It is
only then that the cost of the job or the product can be charged with the
appropriate share of indirect expenses. It is not necessary that a job must pass
through all the departments or that the work required in each department should be
the same for all jobs. It is, therefore, necessary that only appropriate charge in
respect of the work done in the department is made. This can be done only if
overheads for each department are known separately. (d) A suitable method of
costing can be followed differently for each department e.g., batch costing when a
part is manufactured, but single or output costing when the product is assembled.
4.4.3 & 4.4.4 Apportioning overhead expenses over various departments and
reapportioning service department overheads over production department : After the
allocable overheads are related to the departments, expenses incurred for several
departments have to be apportioned over each department, e.g. rent, power,
lighting, insurance and depreciation. For distributing these overheads over
different departments benefiting thereby, it is necessary at first to determine
the proportion of benefit received by each department and then distribute the
total expenditure proportionately on that basis. But the same basis of
apportionment cannot be followed for different items of overheads

4.11
Cost Accounting

since the benefit of service to a department in each case has to be measured


differently. Some of the basis that are generally adopted for the apportionment of
expenses are stated below :
Basis Expense items

Area or cubic measurement of department Rent, rates, lighting Direct labour hours
or, where wage rates are maintenance Supervision more or less uniform, total
direct wages of department. Number of employees in departments Cost of material
used by departments Value of assets Horse power of machines Supervision Material
handling charges Depreciation and insurance Power

and

building

Other basis of apportioning overhead costs : We have considered already that the
benefit received by the department generally is the principal criterion on which
the costs of service departments or common expenses are apportioned. But other
criteria are equally valid. Three of them are mentioned below :

(a) Analysis or survey of existing conditions. (b) Ability to pay. (c) Efficiency
or incentive. A single concern may have only one criterion under consideration
predominantly or may use all (including the service or benefit criterion) for
different phases of its activity.
Analysis or Survey of existing conditions : At times it may not be possible to
determine the advantage of an item of expenses without undertaking an analysis of
expenditure. For example, lighting expenses can be distributed over departments
only on the basis of the number of light points fixed in each department. Ability
to pay : It is a principle of taxation which has been applied in cost accounting
as well for distributing the expenditure on the basis of income of the paying
department, on a proportionate basis. For example, if a company is selling three
different products in a territory, it may decide to distribute the expenses of the
sales organisation to the amount of sales of different articles in these
territories. This basis, though simple to apply, may be inequitable since the
expenditure charged to an article may have no relation to the actual effort
involved in selling it. Easy selling lines thus may have to bear the largest
proportion of expenses while, on the other hand, these should bear the lowest
charge. Efficiency or Incentives : Under this method, the distribution of
overheads is made on the
4.12
Overheads

basis of pre-determined levels of production or sales. When distribution of


overhead cost is made on this basis and if the level of production exceeds the
pre-determined level of production the incidence of overhead cost gets reduced and
the total cost per unit of production or of sales, lowered. The opposite is the
effect if the assumed levels are not reached. Thus the department whose sales are
increasing is able to show a greater profit and thereby is able to earn greater
goodwill and appreciation of the management than it would have if the distribution
of overheads was made otherwise.
Inter-departmental service costs : At first, expenses of all departments are
complied without making a distinction between production and service departments.
Then the expenses of the service departments are apportioned among the production
departments on a suitable basis. This is because ultimately the overheads are to
be absorbed over goods produced or jobs completed in the production departments.

The re-apportionment of service department expenses over the production


departments may be carried out by using any one of the following methods : (i)
Direct re-distribution method. (ii) Step method of secondary distribution or non-
reciprocal method. (iii) Reciprocal Service method.
Direct re-distribution method: Service department costs under this method are
apportioned over the production departments only, ignoring the services rendered
by one service department to the other. To understand the application of this
method go through the illustration which follows. Illustration XL Ltd., has three
production departments and four service departments. The expenses for these
departments as per Primary Distribution Summary are as follows :

Production Departments : A B C 30,000 26,000 24,000

Rs.

Rs;

80,000
Rs. Rs.

Service Departments :

Stores Time-keeping and Accounts Power Canteen

4,000 3,000 1,600 1,000

_____

9,600

4.13
Cost Accounting

The following information is also available in respect of the production


departments :
Dept. A Dept. B Dept. C

Horse power of Machine Number of workers

300 20

300 15

200 15 1,000

Value of stores requisition in (Rs.) 2,500 1,500 Apportion the costs of service
departments over the production departments.
Solution Secondary Overhead Distribution Statement Item of cost (as per primary
distribution summary) Rs. Basis of apportionment A Rs. B Rs. Total

Production Depts C Rs.

Cost as per primary distribution summary Stores Value of stores requisition :


(5:3:2) Time-keeping and Accounts (4:3:3) Power (3:3:2) Canteen (4:3:3) No. of
workers H.P. of M/c�s No. of workers

80,000

30,000

26,000

24,000

4,000

2,000

1,200

800

3,000

1,200

900

900

1,600

600

600

400

1,000
400

300

300

89,600 34,200 29,000 26,400 (ii) Step Method or Non-reciprocal method : This
method gives cognizance to the service rendered by service department to another
service department. Therefore, as

4.14
Overheads

compared to previous method, this method is more complicated because a sequence of


apportionments has to be selected here. The sequence here begins with the
department that renders service to the maximum number of other service
departments. In other words the cost of the service department which serves the
largest number of other service and production departments, is distributed first.
After this, the cost of service department serving the next largest number of
departments is apportioned. This process continues till the cost of last service
department is apportioned. The cost of last service department is apportioned
among production departments only.
Some authors are of the view that the cost of service department with largest
amount of cost should be distributed first. Refer to the illustration which
follows to understand this method. Illustration

Suppose the expenses of two production departments A and B and two service
departments X and Y are as under :
Amount Rs. Y A Apportionment Basis B

X Y A B
Solution

2,000 1,500 3,000 3,200

25% �

40% 40%

35% 60%

Summary of Overhead Distribution

Departments Amount as given above Expenses of X Dept. apportioned over Y,A and B
Dept. in the ratio (5:8:7) Expenses of Y Dept. apportioned over A

X
Rs.

Y
Rs.

A
Rs.

B
Rs.

2,000

1,500

3,000

3,200

�2,000

500
800

700

4.15
Cost Accounting

and B Dept. in the ratio (2:3) Total Nil

�2,000
Nil

800 4,600

1,200 5,100

(iii) Reciprocal Service Method : This method recognises the fact that where there
are two or more service departments they may render services to each other and,
therefore, these inter-departmental services are to be given due weight while re-
distributing the expenses of the service departments. The methods available for
dealing with reciprocal services are : (a) Simultaneous equation method ; (b)
Repeated distribution method ; (c) Trial and error method. (a) Simultaneous
equation method : According to this method firstly, the costs of service
departments are ascertained. These costs are then re-distributed to production
departments on the basis of given percentages. (Refer to the following
illustration to understand the method)
Illustration

Service departments expenses


Rs.

Boiler House Pump Room The allocation is :


Production Departments A B

3,000 600 3,600


Boiler House Pump Room

Boiler House Pump Room


Solution

60% 10%

35% 40%

� 50%

5% �

The total expenses of the two service departments will be determined as follows :

Let B stand for Boiler House expenses and P for Pump Room expenses.

4.16
Overheads

Then B = 3,000 + 1/2 P P = 600 + 1/20 B Substituting the value of B, P = 600 +


1/20 (3,000 + 1/2 P) = 600 + 150 + 1/40 P = 750 + 1/40 P 40 P = 30,000 + P 39 P =
30,000 P = Rs. 769 (approx.) The total of expenses of the Pump Room are Rs. 769
and that of the Boiler House is Rs. 3,385 i.e., Rs. 3,000 + 1/2 � Rs. 769. The
expenses will be allocated to the production departments as under :
Production departments : A Rs. B Rs.

Boiler House (60% and 35% of Rs. 3,385) Pump Room (10% and 40% of Rs. 769) Total
The total of expenses apportioned to A and B is Rs. 3,600.

2,031 77 2,108

1,185 307 1,492

(b) Repeated distribution method : Under this method, service departments costs
are distributed to other service and production departments on agreed percentages
and this process continues to be repeated, till the figures of service departments
are either exhausted or reduced to too small a figure. (Refer to the following
illustration to understand this method)
Illustration

PH Ltd., is a manufacturing company having three production departments, �A�, �B�


and �C� and two service departments �X� and �Y�. The following is the budget for
December 2005 :Total Rs. A Rs. B Rs. C Rs. X Rs. Y Rs.

Direct material Direct wages

1,000 5,000

2,000 2,000

4,000 8,000

2,000 1,000

1,000 2,000

4.17
Cost Accounting

Factory rent Power Depreciation Other overheads

4,000 2,500 1,000 9,000

Additional information : Area (Sq. ft.) 500 250 500 250 500 Capital value (Rs.
lacs) of assets 20 40 20 10 10 Machine hours 1,000 2,000 4,000 1,000 1,000 Horse
power of machines 50 40 20 15 25 A technical assessment of the apportionment of
expenses of service departments is as under : A B C X Y

% Service Dept. �X� Service Dept. �Y� Required : (i) 45 60

% 15 35

% 30 �

% � 5

% 10 �

A statement showing distribution of overheads to various departments.

(ii) A statement showing re-distribution of service departments expenses to


production departments. (iii) Machine hour rates of the production departments
�A�, �B� and �C�.
Solution (i) Overhead Distribution Summary Basis Total Rs. � � 4,000 A Rs. � �
1,000 B Rs. � � 500 C Rs. � � 1,000 X Rs. 2,000 1,000 500 Y Rs. 1,000 2,000 1,000

Direct materials Direct Direct Direct wages Area Factory rent Power H.P. � M/c
Hrs. Depreciation Cap., value Other overheads M/c hrs.

2,500 1,000 9,000

500 200 1,000 2,700

800 400 2,000 3,700

800 200 4,000 6,000

150 100 1,000 4,750

250 100 1,000 5,350

4.18
Overheads

(ii) Redistribution of Service Department�s expenses : A Rs. B Rs. C Rs. X Rs. Y


Rs.

Total overheads Dept. X overhead apportioned in the ratio (45:15:30:�:10) Dept. Y


overhead apportioned in the ratio (60: �35:�:5) Dept. X overhead apportioned in
the ratio (45:15:30:�:10) Dept. Y overhead apportioned in the ratio (60:35:�:5:�)
Dept. X overhead apportioned in the ratio (45:15:30:�:10)

2,700 2,138 3,495 131 17 1 8,482

3,700 712 2,039 44 10 - 6,505

6,000 1,425 - 87 - 1 7,513

4,750 �4,750 291 �291 2 �2 -

5,350 475 �5,825 29 �29 - -

(iii) Machine hour rate :

Machine hours Machine hour rate (Rs.)

1,000 8.48

2,000 3.25

4,000 1.88

(Rs. 8,482/1,000 hrs) (Rs. 6,505/2,000 hrs.) (Rs. 7,513/4,000 hrs.) (c) Trial and
error method - According to this method the cost of one service Cost Centre is
apportioned to another service Cost Centre. The cost of another service centre
plus the share received from the first Cost Centre is again apportioned to the
first cost centre. This process is repeated till the amount to be apportioned
becomes negligible. (Refer to the following illustration to understand this
method.)

4.19
Cost Accounting

Illustration

The ABC Company has the following account balances and distribution of direct
charges on 31st March, 2005.
Total Production Depts. Machine Shop Allocated Overheads : Rs. Rs. Rs. Packing
Service Depts. Gen. Plant Rs. Store & Maintenanace Rs.

Indirect labour Maintenance material Misc. supplies Superintendent�s salary Cost &
payroll salary Power Rent Fuel and heat Insurance Taxes Depreciation

14,650 5,020 1,750 4,000 � 10,000 8,000 12,000 6,000 1,000 2,000 1,00,000 1,64,420

4,000 1,800 400 � �

3,000 700 1,000 4,000 �

2,000 1,020 150 � 10,000

5,650 1,500 200 �

Overheads to be apportioned :

6,200

4,700

17,170

7,350

The following data were compiled by means of the factory survey made in the
previous year :
Floor Space Radiator Sections No. of Employees Investment Rs. H.P hours

Machine Shop Packing General Plant Store & Maint.

2,000 Sq. ft. 800 � � 400 � � 1,600 � � 4,800 � �

45 90 30 60 225

20 10 3 5 38

6,40,000 2,00,000 10,000 1,50,000 10,00,000

3,500 500 � 1,000 5,000

4.20
Overheads

Expenses charged to the stores and maintenance departments are to be distributed


to the other departments by the following percentages : Machine shop 50%; Packing
20%; General Plant 30% ; General Plant overheads is distributed on the basis of
number of employees : (a) Prepare an overhead distribution statement with
supporting schedules to show computations and basis of distribution including
distribution of the service department expenses to producing department. (b)
Determine the service department distribution by the method of continued
distribution. Carry through 3 cycles. Show all calculations to the nearest rupee.
Solution (a) Overhead Distribution Statement Production Departments Machine
Packing Shop Rs. Rs. Service Departments General Stores & Plant Maintenance Rs.
Rs.

Allocated Expenses:

Indirect labour Maintenance material Superintendent�s salary Misc. supplies Cost &
payroll salaries Total Apportioned expenses (See schedule below) Total
Item Basis

4,000 1,800 - 400 - 6,200 77,720

3,000 700 - 1,000 - 4,700 25,800

2,000 1,020 4,000 150 10,000 17,170 2,830

5,650 1,500 - 200 - 7,350 22,650 30,000


Stores & Maintenance Rs.

83,920 30,500 20,000 Schedule of Apportioned Expenses


Machine Shop Rs. Rs. Packing General Plant Rs.

Power Rent Fuel & Heat

Horse Power Hrs. 5,600 Floor Space Radiator Secs. 5,000 1,200

800 2,000 2,400

� 1,000 800

1,600 4,000 1,600

4.21
Cost Accounting

Insurance Taxes Depreciation Total (b)

Investment Investment Investment

640 1,280 64,000 77,720

200 400 20,000 25,800

10 20 1,000 2,830

150 300 15,000 22,650

Distribution of Service Department Expenses Production Departments Machine Rs.


Packing Rs. Service Departments General Plant Rs. Stores & Maintenance Rs.

Total Expense [as per (a)] Transfer from Stores & Maintenance Transfer from
General Plant Transfer from Stores & Maintenance Transfer from General Plant
Transfer from Stores & Maintenance Transfer from General Plant Total
Illustration

83,920 15,000 16,571 2,072 710 88 35 1,18,396

30,500 6,000 8,286 829 355 36 18 46,024

20,000 9,000 �29,000 1,242 �1,242 53 �53 �

30,000 �30,000 4,143 �4,143 177 �177 � �

Modern Manufactures Ltd. have three Production Departments P1, P2, P3 and two
Service Departments S1 and S2 details pertaining to which are as under :
P1 P2 P3 S1 S2

Direct wages (Rs.) Working hours Value of machines (Rs.) H.P. of machines Light
points Floor space (sq. ft.)
4.22

3,000 3,070 60,000 60 10 2,000

2,000 4,475 80,000 30 15 2,500

3,000 2,419 1,00,000 50 20 3,000

1,500 5,000 10 10 2,000

195 5,000 5 500


Overheads

The following figures extracted from the Accounting records are relevant:
Rs.

Rent and Rates General Lighting Indirect Wages Power Depreciation on Machines

5,000 600 1,939 1,500 10,000

Sundries 9,695 The expenses of the Service Departments are allocated as under :
P1 P2 P3 S1 S2

S1 S2

20% 40%

30% 20%

40% 30%

10%

10% -

Find out the total cost of product X which is processed for manufacture in
Departments P1, P2 and P3 for 4, 5 and 3 hours respectively, given that its Direct
Material Cost is Rs. 50 and Direct Labour Cost is Rs. 30.
Solution Statement Showing Distribution of Overheads of Modern Manufacturers Ltd.
Particulars Basis Total Rs. Production Depts P1 Rs. P2 Rs. P3 Rs. Service Depts.
S1 Rs. S2 Rs.

Rent & Rates General lighting Indirect wages Power Depreciation of machines
Sundries

Area Light points Direct wages H.P. Value of machines

5,000 600 1,939 1,500 10,000 ______ 28,734

1,000 100 600 600 2,400 3,000 _____ 7,700

1,250 150 400 300

1,500 200 600 500

1,000 100 300 100 200 1,500 _____ 3,200

250 50 39 - 200 195 ____ 734

3,2004,000 2,0003,000 _____ 7,300 _____ 9,800

Direct Wages 9,695

4.23
Cost Accounting

Redistribution of Service Department�s Expenses Over Production Departments


Production Depts. Total Rs. Total Overheads Dept. S1 Overheads apportioned in the
ratio (20:30:40:�:10) Dept. S2 Overheads apportioned in the ratio (40:20:30:10:�)
Dept. S1 Overheads apportioned in the ratio (20:30:40:�:10) Dept. S2 Overheads
apportioned in the ratio (40:20:30:10:�) Dept. S1 Overheads apportioned in the
ratio (20:30:40:�:10) Dept. S2 Overheads apportioned in the ratio (40:20:30:10:�)
Total Working hours Rate per hour 8,787.16 3,075.00 2.86 8,504.87 11,441.79
4,475.00 2,419.00 1.90 4.73 1.05 0.10 0.21 0.05 0.32 0.02 0.42 0.03 �1.0 � 50.10
-0.10 10.54 4.22 2.11 3.16 1.05 �10.54 105.40 21.08 31.62 42.16 �105.40 10.54
1,054.00 421.60 210.80 316.20 105.40 �1,054 28,734 3,200 P1 Rs. 7,700 640 P2 Rs.
7,300 960 P3 Rs. 9,800 1,280 Service Depts. S1 Rs. 3,200 �3,200 S2 Rs. 734 320

(See Working Note 1) Cost of the product �X� Rs.

Direct material cost Direct labour cost Overhead cost


Working Note :

50.00 30.00 35.13 115.13

1.

Working rate per hour for production department

P1 =

Rs.8,787.16 = Rs. 2.86 3,075

4.24
Overheads

Similarly production rate for production departments P2 and P3 are Rs. 1.90 and
Rs. 4.73. 2.
Overhead cost

Rs. 2.86 � 4 hrs. + Rs. 1.90 � 5 hrs. + Rs. 4.73 � 3 hrs. = Rs. 11.44 + Rs. 9.50 +
Rs. 14.19 = Rs. 35.13
Note The service departments have only indirect costs which are to be absorbed by
production departments. However if the direct wages appearing in the question are
assumed to be incurred on the service departments only, which have not been
accounted for, by any other activity carried on in the service departments, then
total expenses of the service departments including the aforesaid direct wages
should be charged in the respective production departments. If this assumption
holds good the alternative solution can appear as under. Alternative Solution
Statement Showing Distribution of Overheads of Modern Manufactures Ltd. Production
Departments Particulars Basis Total P1 P2 P3 Rs. Rs. Rs. Rs.

Service Departments S1 S2 Rs. Rs.

Direct wages Rent & rates General lighting Indirect wages Power Depreciation of
machines Sundries

Actual Area Light points Direct wages H.P. Value

1,695 5,000 600 1,939 1,500

1,000 100 600 600 2,400 3,000 7,700

1,250 150 400 300 3,200 2,000 7,300

1,500 200 600 500 4,000 3,000 9,800

1,500 1,000 100 300 100 200 1,500 4,700

195 250 50 39 - 200 195 929

of machines 10,000 Direct wages 9,695 30,429

4.25
Cost Accounting

Redistribution of Service Department�s Expenses over Production Departments


Total Rs. Total Overheads Dept. S1 Overheads apportioned in the ratio:
(20:30:40:�:10) Dept. S2 overheads apportioned in the ration :(40:20:30:10:�)
Dept. S1 overheads apportioned in the ratio (20:30:40:�:10) Dept. S2 overheads
apportioned in the ratio (40:20:30:10:�) Dept. S1 overheads apportioned in the
ratio (20:30:40:�:10) Dept. S2 overheads apportioned in the ratio (40:20:30:10:�)
Total Working hours Working rate per hour 9,233.52 3,070.00 3.00 9,035.02
12,160.47 4,475.00 2,419.00 2.02 5.03 0.14 0.06 0.03 0.05 �0.14 1.40 0.28 0.42
0.56 �1.40 0.14 13.99 5.60 2.80 4.20 1.40 �13.99 139.90 27.98 41.97 55.96 �139.90
13.99 1,399.00 559.60 279.80 419.70 139.90 �1,399.00 30,429 4,700 P1 Rs. 7,700 940
P2 Rs. 7,300 1,410 P3 Rs. 9,800 1,880 S1 Rs. 4,700 �4,700 S2 Rs. 929 470

Cost of the Product �X�

Direct material cost Direct labour cost Overhead cost (See working note)
Working note : Overhead cost :

Rs. P. 50.00 30.00 37.19

117.25

Rs. 3 � 4 hrs. + Rs. 2.02 � 5 hrs. + Rs. 5.03 � 3 hrs. = Rs. 12 + Rs. 10.10 + Rs.
15.09 = Rs. 37.19
Illustration :

Deccan Manufacturing Ltd., have three departments which are regarded as


4.26
Overheads

production departments. Service departments� costs are distributed to these


production departments using the �Step Ladder Method� of distribution. Estimates
of factory overhead costs to be incurred by each department in the forthcoming
year are as follows. Data required for distribution is also shown against each
department:
Department Factory overhead Rs. Direct labour hours No. of employees Area in sq.m.

Production: X Y Z Service: P Q R 45,000 75,000 1,05,000 1,000 5,000 6,000 10 50 40


500 1,500 1,000 1,93,000 64,000 83,000 4,000 3,000 4,000 100 125 85 3,000 1,500
1,500

S 30,000 3,000 50 1,000 The overhead costs of the four service departments are
distributed in the same order, viz., P,Q,R and S respectively on the following
basis.
Department Basis

P Q R S
You are required to :

Number of employees Direct labour hours Area in square metres Direct labour hours

(a) Prepare a schedule showing the distribution of overhead costs of the four
service departments to the three production departments; and (b) Calculate the
overhead recovery rate per direct labour hour for each of the three production
departments.

4.27
Cost Accounting

Solution

(a)

Deccan Manufacturing Limited Schedule Showing the Distribution of Overhead Costs


among Departments Service Production
P Rs. Overhead costs Distribution of overhead cost of Dept. �P� Distribution of
overhead costs of Dept. �Q� Distribution of overhead cost of Dept. �R�
Distribution of overhead costs of Dept. �S� Total (A) - (66,000) 24,000 18,000
24,000 3,00,000 1,35,000, 1,60,000 - (1,33,000) 19,000 57,000 28,500 28,500 -
(80,000) 24,000 12,000 16,000 12,000 16,000 (45,000) 5,000 4,000 5,000 10,000
12,500 8,500 Q Rs. R Rs. 1,05,000 S Rs. X Rs. Y Rs. 64,000 Z Rs. 83,000

45,000 75,000

30,000 1,93,000

(b) Direct labour hours (B) Overhead recovery rate per hour
Illustration

4,000

3,000 Rs. 45

4,000 Rs. 40

( A) (B)

Rs. 75

A Ltd., manufactures two products A and B. The manufacturing division consists of


two production departments P 1 and P 2 and two service departments S 1 and S 2 .
Budgeted overhead rates are used in the production departments to absorb factory
overheads to the products. The rate of Department P 1 is based on direct machine
hours, while the rate of Department P 2 is based on direct labour hours. In
applying overheads, the pre-determined rates are multiplied by actual hours. For
allocating the service department costs to production departments, the basis
adopted is as follows : (i) Cost of Department S 1 to Department P 1 and P 2
equally, and (ii) Cost of Department S 2 to Department P 1 and P 2 in the ratio of
2 : 1 respectively.
4.28
Overheads

The following budgeted and actual data are available:


Annual profit plan data :

Factory overheads budgeted for the year:


Rs. Rs.

Departments

P1 P2

25,50,000 21,75,000

S1 S2

6,00,000 4,50,000

Budgeted output in units :

Product A 50,000; B 30,000.


Budgeted raw-material cost per unit :

Product A Rs. 120; Product B Rs. 150.


Budgeted time required for production per unit :

Department P1 : Department P2 :

Product A : 1.5 machine hours Product B : 1.0 machine hour Product A : 2 Direct
labour hours Product B : 2.5 Direct labour hours

Average wage rates budgeted in Department P2 are :

Product A - Rs. 72 per hour and Product B � Rs. 75 per hour. All materials are
used in Department P1 only. Actual data : (for the month of July, 2005)
Units actually produced : Product A : 4,000 units

Product B : 3,000 units


Actual direct machine hours worked in Department P1 :

On product A 6,100 hours, Product B 4,150 hours.


Actual direct labour hours worked in Department P2 :

on product A 8,200 hours, Product B 7,400 hours.

4.29
Cost Accounting

Costs actually incurred:


Product A Rs. Product B Rs.

Raw materials Wages Overheads : Department

4,89,000 5,91,900
Rs.

4,56,000 5,52,000
Rs.

P1 2,31,000 P2 2,04,000

S1 60,000 S2 48,000

You are required to :

(i)

Compute the pre-determined overhead rate for each production department.

(ii) Prepare a performance report for July, 2005 that will reflect the budgeted
costs and actual costs.
Solution : (i) Computation of predetermined overhead rate for each production
department from budgeted data Production Deptts. Service Deptts. P1 P2 S1 S2

Budgeted factory overheads for the year in (Rs.) Allocation of service department
S1�s costs to production departments P1 and P2 equally in (Rs.) Allocation of
service department S2�s costs to production departments P1 and P2 in the ratio of
2:1 in (Rs.) Total (Rs.) Budgeted machine hours in department P1 (Refer to working
note 1)
4.30

25,50,000

21,75,000

6,00,000

4,50,000

3,00,000

3,00,000

�6,00,000

3,00,000 31,50,000 1,05,000

1,50,000 26,25,000


Nil
�4,50,000
Nil
Overheads

Budgeted labour hours in department P2 (Refer to working note 2) Budgeted machine


hour rate (Rs. 31,50,000/1,05,000) Budgeted labour hour rate (Rs.
26,25,000/1,75,000)
(ii)

1,75,000

Rs. 30 Rs. 15
Performance report for July, 2005

(When 4,000 and 3,000 units of products A and B respectively were actually
produced) Budgeted Rs. Raw materials used in department P1 Actual Rs.

A : 4,000 units � Rs. 120 B : 3,000 units � Rs. 150


Direct labour

4,80,000 4,50,000

4,89,000 4,56,000

Cost on the basis of labour hours worked in department P2 A : 4,000 units � 2 hrs.
� Rs. 72 B : 3,000 units � 2.5 hrs. � Rs. 75 A : 4,000 units � 1.5 hrs. � Rs. 30 B
: 3,000 units � 1 hr. � Rs. 30 A : 4,000 units � 2 hrs. � Rs. 15 B : 3,000 units �
2.5 hrs. � Rs. 15 * (Refer to working note 4) ** (Refer to working note 5)
5,76,000 5,62,500 1,80,000 90,000 1,20,000 1,12,500 25,71,000 5,91,900 5,52,000
1,74,400* 1,18,649 1,31,364 ** 1,18,548 26,31,861

Overhead absorbed on machine hour basis in department P1

Overhead absorbed on labour hour basis in department P2

4.31
Cost Accounting

Working notes : Product A Product B Total

1.

Budgeted output (in units) Budgeted machine hours in department P1 Budgeted labour
hours in department P2

50,000 75,000 (50,000�1.5 hrs.) 1,00,000 (50,000�2 hrs.) Product A

30,000 30,000 (40,000�1 hr.) 75,000 (30,000�2.5 hrs.) Product B 3,000 4,150 7,400
10,250 15,600 Total 1,75,000 1,05,000

2.

Actual output (in units) Actual machine hours utilised in department P1 Actual
labour hours utilised in department P2

4,000 6,100 8,200

3.

Computation of actual overhead rates for each production department from actual
data Production Deptts. P1 P2 Service Deptts. S1 S2

Actual factory overheads for the month of July, 2005 in (Rs.) Allocation of
service department S1�s costs in (Rs.) over production departments P1 and P2
equally Allocation of service department S2�s costs in (Rs.) over production
departments P1 and P2 in the ratio 2 : 1 Total (Rs.) 32,000 2,93,000 16,000
2,50,000 -
Nil

2,31,000

2,04,000

60,000

48,000

30,000

30,000

�60,000

�48,000
Nil

4.32
Overheads

Actual machine hours in department P1


(Refer to working note 2) Actual labour hours in department P2

10,250

15,600

- - -

- - -

(Refer to working note 2) Machine hour rate Rs. 28.59 - (Rs. 2,93,000/10,250)
Labour hour/rate - Rs. 16.02 (Rs. 2,50,000/15,600) 4. Actual overheads absorbed
(based on machine hours)

A : 6,100 hrs � Rs. 28.59 = B : 4,150 hrs � Rs. 28.59 = 5. A : 8,200 hrs � Rs.
16.02 = B : 7,400 hrs � Rs. 16.02 =

Rs. 1,74,400 (say) Rs. 1,18,649 (say) Rs. 1,31,364 Rs. 1,18,548

Actual overheads absorbed (based on labour hours)

4.4.5 Absorbing overheads over cost units, products, etc.: Collection of the
figure of overheads for the factory as a whole or for various departments is not
enough. It is clearly necessary to ascertain how much of the overheads is
debitable to the cost of the various jobs, products etc. This process is called
absorbing the overhead to cost units. We take up below the various implications of
this process. However, if only one uniform type of work is done, the task is easy
and under such a situation the overhead expenses to be absorbed may be calculated
by dividing actual overheads by the number of units of work done or estimated
overheads by the estimated output. Normal and pre-determined overhead rates :
Various items of overhead expenses generally are not incurred uniformly throughout
the accounting period e.g., insurance premium are paid annually, rates and taxes
quarterly, etc. The monthly total of overhead expenses for each department or cost
centre, therefore, may fluctuate from month to month. As such, monthly totals
cannot be regarded as satisfactory. It being necessary to absorb the overhead
costs in the cost of production of each lot or batch of production, as soon as its
manufacture is complete an estimate of the total amounts of annual overhead
expenses must be made in advance. Likewise, an advance estimate of the annual
volume of production (in terms of the amount of direct wages, number of direct
labour hours, etc.) must be made and, on that basis, a pre-determined overhead
rate is calculated.

4.33
Cost Accounting

The overhead rate of expenses for absorbing them to production may be estimated on
the following three basis. (1) The figure of the previous year or period may be
adopted as the overhead rate to be charged to production in the current year. The
assumption is that the value of production as well as overheads will remain
constant or that the two will change, proportionately. (2) The overhead rate for
the year may be determined on the basis of estimated expenses and anticipated
volume of production activity. For instance, if expenses are estimated at Rs.
10,000 and output at 4,000 units, the overhead rate will be Rs. 250 per unit. (3)
The overhead rate for a year may be fixed on the basis of the normal volume of the
business. If, in the example given above, the normal capacity is 5,000 units, the
overhead rate will be Rs. 2. The first method is rather crude and is not likely to
yield satisfactory results unless the undertaking is small and output and expenses
are fairly constant over the period. In large concerns, conditions are rarely
static and, hence, expenses fluctuate from one period to another. Therefore, so
far as a large enterprise is concerned, the overhead rates of the past periods may
not have much relevance for the purpose of arriving at the cost of production in
the current period. The second method is based on the assumption that all expenses
shall have to be recovered irrespective of the volume of output. From such an
assumption, it follows that, if in any period there is a large idle or unused
capacity, the entire amount of the overhead expenses shall have to be absorbed
over the reduced volume of the output. A sizeable portion of the overhead expenses
is made up of the fixed charges; if those fixed charges have to be included in the
costs of the reduced output the incidence of overheads per unit of production
would necessarily be high. Similarly, if the volume of output in any period
exceeds the normal level, the incidence of fixed overheads per unit will be
comparatively lower. The effect of this method is that during periods of falling
production (and perhaps of falling prices), the cost of production would be higher
and, correspondingly, during a period of rising production and perhaps rising
prices), the cost of production would be low. This would be illogical idleness or
a level of activity above the normal capacity is abnormal and should not affect
costs; the Costing Profit and Loss Account is the place where the effect of such
abnormal factors should be shown. It is also possible that when output falls and
total cost per unit goes up (if expenses are absorbed over actual output), the
firm may demand a price above the market price and may find itself without
customers. On this consideration, the most appropriate basis for the computation
of
4.34
Overheads

predetermined rate of overhead cost is the normal capacity of production.


Estimation on this basis as suggested in the third method, has many advantages
over the other two methods. It enables not only the computation of correct costs
but also true cost to be recovered. In a competitive economy, the concept of costs
is essentially that of normal costs. Normal costs should be the yard stick against
which the efficiency or otherwise, of the competing unit should be measured. A
cost that is correct need not necessarily be true. True costs are in a sense
notional cost and the term �normal� is derived from the concept of normal costs.
Hence, the use of normal overhead rates has a two fold advantage. It enables the
true as well as correct costs to be calculated; at the same time by highlighting
differences between the amount of overhead expenses actually incurred and those
absorbed in production it provides useful guidance to the management in taking
decisions as regards production and sales.
Blanket and departmental overhead rates: Blanket overhead rate refers to the
computation of one single overhead rate for the whole factory. It is to be
distinguished from the departmental overhead rate which refers to a separate rate
for each individual cost centre or department. The use of blanket rate may be
proper in certain factories producing only one major product in a continuous
process (e.g., chemical factories) or where the work performed in every department
is fairly uniform or standardised. Where, however, the product lines are varied or
machinery is used to a varying degree in the different departments, that is, where
conditions throughout the factory are not uniform, the use of departmental rates
is to be preferred. The working condition in the last mentioned case would be such
that varying amount of expenses would be continually incurred by the several
service departments and hence, the incidence of overhead cost of each department
would be different. Since not all products would ordinarily undergo the same type
or of different type of operations in different departments, the charging of a
single overhead rate in such a case would give misleading results.

It may therefore, be concluded that a blanket rate should be applied. (1) where
only one major product is being produced. (2) where several products are produced,
but (a) all products pass through all departments; and (b) all products are
processed for the same length of time in each department. Where these conditions
do not exist, departmental rates should be used.
4.5 METHODS OF ABSORBING OVERHEADS TO VARIOUS PRODUCTS OR JOBS

Before we describe various methods, it would be better to know how to judge


whether a method will give good results or not. The method selected for charging
overheads to

4.35
Cost Accounting

products or jobs should be such as will ensure : (i) that the total amount charged
(or recovered) in a period does not differ materially from the actual expenses
incurred in the period. In other words, there should not be any significant over
or under recovery of overhead; and that the amount charged to individual jobs or
products is equitable. In case of factory overhead, this means : (a) that the time
spent on completion of each job should be taken into consideration; (b) that a
distinction should be made between jobs done by skilled workers and those done by
unskilled workers. Usually, the latter class of workers needs more supervision,
causes greater wear and tear of machines and tools and waste a larger quantity of
materials. Hence jobs done by such workers should bear a correspondingly higher
burden for overheads; and (c) that jobs done by manual labour and those done by
machines should be distinguished. It stands to reason that no machine expenses
should be charged to jobs done by manual labour. In addition, the methods should
(i) (ii) be capable of being used conveniently; and yield uniform result from
period to period as far as possible; any change that is apparent should reflect a
change in the underlying situation such as substitution of human labour by
machines.

(ii)

Several methods are commonly employed either individually or jointly for computing
the appropriate overhead rate to be employed. The more common of these are : (1)
Percentage of direct materials. (2) Percentage of prime cost. (3) Percentage of
direct labour cost. (4) Labour hour rate. (5) Machine hour rate.
4.5.1 & 4.5.2 Percentage of direct material and prime cost

Suppose for a given period, actual figures are estimated as follows :


Rs.

Direct materials

2,00,000

4.36
Overheads

Direct labour Factory overheads

1,00,000 90,000

The percentage of factory overheads to direct materials will be 45%, to prime cost
30% and to direct labour 90%. If, on a job, material cost is Rs. 10,000 and direct
labour is Rs. 7,000 the cost, after absorbing factory overhead, will be as follows
: (i) (ii) Rs. 17,000 + 45% Rs. 10,000 or Rs. 21,500, Rs. 17,000 + 30% Rs. 17,000
or Rs. 22,100, and

(iii) Rs. 17,000 + 90% Rs. 7,000 or Rs. 23,300. One can see how, with a different
method, the works cost comes out to be different. Of these methods, the first and
second are generally considered to be unsuitable on account of the following
reasons : (i) Manufacturing overhead expenses are mostly a function of time i.e.,
time is the determining factor for the incurrence and application of manufacturing
overhead expenses. That they are so would be clear if we recall that overhead
expenses, specially manufacturing expenses, can in the ultimate analysis be
regarded as expenditure incurred in providing the necessary facilities and service
to workers employed in the productive process. The question of facilities and
service made available to workers naturally is dependent on the length of time
during which workers make use of the facilities. It may, therefore, be said that
the job or product on which more time has been spent would entail larger
manufacturing expenses than the job requiring less time. The factor is ignored
altogether by the first method and largely by the second method. (ii) Overheads
are neither related to the prime cost nor to direct material cost except to a very
small extent. Thus, if the percentage of material cost is used when there are two
jobs requiring the same operational time but using material having varying prices,
their manufacturing overhead cost would be different whereas this should not
normally be so. The method of absorbing overhead costs on the basis of prime cost
also does not take into consideration the time factor. The fact that the amount
includes labour cost in addition to material cost does not render the prime cost
to be more suitable; infact, the results are liable to be more misleading because
of the cumulative error of using both the labour and material cost as the basis of
allocation of overhead expenses, on neither of which they are already dependent.
(iii) Since material prices are prone to frequent and wide fluctuations, the
manufacturing overheads, if based on material cost or prime cost, also would
fluctuate violently from period to period. (iv) The skill of the workers involved
and whether machines were used or not, are

4.37
Cost Accounting

ignored when these methods are used. Percentage of materials cost may, however, be
used for the limited purpose of absorbing material handling and store overheads.
4.5.3 Percentage of direct labour cost : This method also fails to give full
recognition to the element of the time which is of prime importance in the
accounting for and treatment of manufacturing overhead expenses except in so far
as the amount of wages is a product of the rate factor multiplied by the time
factor. Thus, the time factor is taken into consideration only indirectly or
partially in the computation of the overhead percentage rate. This method
therefore, cannot be depended upon to produce very accurate results where the same
type of work is performed in the same time by different type of workers, skilled
and unskilled, with varying rates of pay. Also no distinction is made between jobs
done by manual labour and those done by machines.

Inspite of the inaccuracies which may arise under this method it is widely used in
actual practice because it is simple and does not involve much calculation. If on
the other hand, a more scientific method is employed e.g., the direct labour hour
rate or the machine hour rate, more complexities in the overhead accounting
procedure would be introduced, though the selected method will give proper
allowance to time element. Thus, the advantage of elimination of a small error in
practice may involve a heavy price on account of introduction of complexities.
Advantages:

(i)

The method is simple and economical to apply.

(ii) The time factor is given recognition even if indirectly. (iii) Total expenses
recovered will not differ much from the estimated figure since total wages paid
are not likely to fluctuate much.
Disadvantages:

(i)

It gives rise to certain inaccuracies due to the time factor not being given full
importance.

(ii) Where machinery is used to some extent in the process of manufacture, an


allowance for such a factor is not made. (iii) It does not provide for varying
skills of workers.
4.5.4 Labour hour rate: This method is an improvement on the percentage of direct
wage basis, as it fully recognises the significance of the element of time in the
incurring and absorption of manufacturing overhead expenses. This method is
admirably suited to operations which do not involve any large use of machinery. To
calculate labour hour rate, the amount of factory overheads is divided by the
total number of direct labour hours.
4.38
Overheads

Suppose factory overheads are estimated at Rs. 90,000 and labour hours at
1,50,000. The overhead absorption rate will be Re. 0.60. If 795 direct labour
hours are spent on a job, Rs. 477 will be absorbed as overhead. It can be
calculated for each category of workers.
4.5.5 Machine hour rate : By the machine hour rate method, manufacturing overhead
expenses are charged to production on the basis of number of hours machines are
used on jobs or work orders. There is a basic similarity between the machine hour
and the direct labour hour rate method insofar as both are based on the time
factor. The choice of one or the other method is conditioned by the actual
circumstance of the individual case. In respect of departments or operations in
which machines predominate and the operators perform relatively a passive part,
the machine hour rate is more appropriate. This is generally the case for
operations or processes performed by costly machines, which are automatic or semi-
automatic and where operators are essential merely for feeding them rather than
for regulating the quantity of the output. In such case, the machine hour rate
method alone can be depended on to correctly absorb the manufacturing overhead
expenses to different items of production. What is needed for computing the
machine hour rate is to divide overhead expenses for a specific machine or group
of machines for a period by the operating hours of the machine or the group of
machines for the period.

Usually, the computation is made on the basis of the estimated expenses or the
normal expenses for the coming period. Thus the machine hour rate usually is a
predetermined rate. It is desirable to work out a rate for each individual
machine; where a number of similar machines are working in a group, there may be
single rate for the whole group. There are two methods of computing the machine
hour rate. According to the first method, only the expenses directly or
immediately connected with the operation of the machine are taken into account
e.g., power, depreciation, repairs and maintenance, insurance, etc. The rate is
calculated by dividing the estimated total of these expenses for a period by the
estimated number of operational hours of the machines during the period. It will
be obvious, however, that in addition to the expenses stated above there may still
be other manufacturing expenses such as supervision charges, shop cleaning and
lighting, consumable stores and shop supplies, shop general labour, rent and
rates, etc. incurred for the department as a whole and, hence, not charged to any
particular machine or group of machines. In order to see that such expenses are
not left out of production costs, one should include a portion of such expenses to
compute the machine hour rate. Alternatively, the overheads not directly related
to machines may be absorbed on the basis of Productive Labour Hour Rate Method or
any other suitable method.

4.39
Cost Accounting

Illustration

A machine costing Rs. 10,000 is expected to run for 10 years. At the end of this
period its scrap value is likely to be Rs. 900. Repairs during the whole life of
the machine are expected to be Rs. 18,000 and the machine is expected to run 4,380
hours per year on the average. Its electricity consumption is 15 units per hour,
the rate per unit being 5 paise. The machine occupies one-fourth of the area of
the department and has two points out of a total of ten for lighting. The foreman
has to devote about one sixth of his time to the machine. The monthly rent of the
department is Rs. 300 and the lighting charges amount to Rs. 80 per month. The
foreman is paid a monthly salary of Rs. 960. Find out the machine hour rate,
assuming insurance is @ 1% p.a. and the expenses on oil, etc., are Rs. 9 per
month.
Solution Fixed expenses per month Rs.

Rent (one fourth of the total) Lighting (one fifth of the total) Foreman�s salary
(one sixth of the total) Sundry expenses�oil, waste etc. Insurance (1% on the
value of the machine per year) Total constant expenses per month Total number of
hours per annum Total number of hours per month 4,380 365
Rs.

75.00 16.00 160.00 9.00 8.33 268.33

Rs.

Fixed expenses per hour


Variable expenses per hour : Depreciation : Cost of the machine Less: Scrap value

Rs. 268.33 365 hrs.

0.735

Depreciation per annum Depreciation per hour:

10,000 900 9,100 910 0.208

910 4,380 hrs.

4.40
Overheads

Repairs for the whole life for one hour

18,000 0.411 0.750 2.104

Rs.18,000 4,380 � 10 years

Electricity for one hour : 15 units @ 0.05 P Machine hour rate :


Illustration

Gemini Enterprises undertakes three different jobs A, B and C. All of them require
the use of a special machine and also the use of a computer. The computer is hired
and the hire charges work out to Rs. 4,20,000 per annum. The expenses regarding
the machine are estimated as follows : Rent for the quarter Depreciation per annum
Rs. 17,500

2,00,000

Indirect charges per annum 1,50,000 During the first month of operation the
following details were taken from the job register:
Job A B C

Number of hours the machine was used : (a) Without the use of the computer (b)
With the use of the computer
You are required to compute the machine hour rate :

600 400

900 600

� 1,000

(a) For the firm as a whole for the month when the computer was used and when the
computer was not used. (b) For the individual jobs A, B and C.
Solution Working notes:

(i) (ii)

Total machine hours used (600 + 900 + 400 + 600 + 1,000) Total machine hours
without the use of computers (600 + 900)

3,500 1,500

4.41
Cost Accounting

(iii)

Total machine hours with the use of computer (400 + 600 + 1,000)

2,000
Rs.

(iv) Total overheads of the machine per month Rent (Rs. 17,500 � 3 months)
Depreciation (Rs. 2,00,000 � 12 months) Indirect Charges (Rs. 1,50,000 � 12
months) Total (v) Computer hire charges for a month = Rs. 35,000 (Rs. 4,20,000 �
12 months) (vi) Overheads for using machines without computer =

5,833.33 16,666.67 12,500.00 35,000.00

Rs. 35,000 � 1,500 hrs. = Rs. 15,000 3,500 hrs.

(vii) Overheads for using machines with computer =

Rs. 35,000 2,000 hrs. + Rs. 35,000 = Rs. 55,000 3,500 hrs. Rs. 55,000 = Rs. 27.50
per hour 2,000 hours Rs. 35,000 = Rs. 10 per hour 3,500 hrs.
Rate per hr. A Hrs. Rs. Job B Hrs. Rs. Rate per hr. C Hrs. Rs.

(a) Machine hour rate of Gemini Enterprises for the firm as a whole for a month.
(1) When the Computer was used :

(2) When the computer was note used :

(b) Machine hour rate for individual job

Rs. Overheads Without Computer With computer

10.00 27.50

600 400 1,000

Machine hour rate


4.42

6,000 11,000 17,000 Rs. 17

900 600 1,500

9,000 16,500 25,500 Rs. 17

- - 1,000 27,500 1,000 27,500 Rs. 27.50


Overheads

Illustration

A machine shop has 8 identical Drilling machines manned by 6 operators. The


machine cannot be worked without an operator wholly engaged on it. The original
cost of all these machines works out to Rs. 8 lakhs. These particulars are
furnished for a 6 months period: Normal available hours per month Absenteeism
(without pay) hours Leave (with pay) hours Normal idle time unavoidable-hours
Average rate of wages per worker for 8 hours a day. Production bonus estimated
Value of power consumed Supervision and indirect labour Lighting and electricity
208 18 20 10 Rs. 20 Rs. 8,050 Rs. 3,300 Rs. 1,200 15% on wages

These particulars are for a year Repairs and maintenance including consumables 3%
of value of machines. Insurance Rs. 40,000 Depreciation 10% of original cost.
Other sundry works expenses Rs. 12,000 General management expenses allocated Rs.
54,530. You are required to work out a comprehensive machine hour rate for the
machine shop.
Solution Computation of comprehensive machine hour rate of machine shop Rs.

Operator�s wages (Refer to working note 2) Production bonus (15% on wages) Power
consumed Supervision and indirect labour Lighting and electricity

17,100 2,565 8,050 3,300 1,200

4.43
Cost Accounting

Repairs and maintenance Insurance Depreciation Sundry works expenses General


management expenses

12,000 20,000 40,000 6,000 27,265 1,37,480

Machine hour rate

Total overheads of machine shop Hours of machines operation


Rs.1,37,480 (Refer to working note 1) 5.760 hours

= Rs. 23.87
Working notes.

1.

Computation of hours, for which 6 operators are available for 6 months.

Normal available hours p.m. per operator.


Less: Absenteeism hours Less: Leave hours Less: Idle time hours

208 18 20 10 48

Utilisable hours p.m. per operator 160 Total utilisable hours for 6 operators and
for 6 months are = 160 � 6 � 6 = 5,760 hours As machines cannot be worked without
an operator wholly engaged on them therefore, hours for which 6 operators are
available for 6 months are the hours for which machines can be used. Hence 5,760
hours represent total machine hours. 2. Computation of operator�s wages Average
rate of wages :

Rs. 20 = Rs. 2.50 per hour 8 hours

4.44
Overheads

Hours per month for which wages are paid to a worker (208 hours � 18 hours) = 190
hours. Total wages paid to 6 operators for 6 months = 190 hours � 6 � 6 � Rs. 2.50
= Rs. 17,100
Advantages of Machine Hour Rate : (1) Where machinery is the main factor of
production, it is usually the best method of charging machine operating expenses
to production. (2) The under-absorption of machine overheads would indicate the
extent to which the machines have been idle. (3) It is particularly advantageous
where one operator attends to several machines (e.g. automatic screw manufacturing
machine), or where several operators are engaged on the machine e.g. the belt
press used in making conveyer belts. Disadvantages : (1) Additional data
concerning the operation time of machines, not otherwise necessary, must be
recorded and maintained. (2) As general department rates for all the machines in a
department may be suitable, the computation of a separate machine hour rate for
each machine or group of machines would mean further additional work. Note: Some
people even prefer to add the wages paid to the machine operator in order to get a
comprehensive rate of working a machine for one hour. If all expenses are not
allocated to machines, it will be necessary to calculate another rate for charging
the general department expenses to production. This second rate can be calculated
on the basis of direct labour hours. In effect therefore, both the machine hour
and the direct labour hour rate will be applied, though separately. Illustration
Job No. 198 was commenced on October 10, 2005 and completed on November 1, 2005.
Materials used were Rs. 600 and labour charged directly to the job was Rs. 400.
Other information is as follows:

Machine No. 215 used for 40 hours, the machine hour rate being Rs. 3.50. Machine
No. 160 used for 30 hours, the machine hour rate being Rs. 4.00. 6 welders worked
on the job for five days of 8 hours each : the Direct labour hour per welder is
20P. Expenses not included for calculating the machine hour or direct labour hour
rate totalled

4.45
Cost Accounting

Rs. 2,000,total direct wages for the period being Rs. 20,000. Ascertain the works
costs of job No. 198.
Solution Rs.

Materials Direct labour Factory overheads : Machine No. 215 : 40 hours @ Rs. 3.50
Machine No. 160 : 30 hours @ Rs. 4.00 2401 hours of welders @ 20 P. per hr.
General2 10% of wages Works cost 1. 6�5�8=240 2.
Rs.

600.00 400.00 1,000.00 140.00 120.00 48.00 40.00 348.00 1,348.00

Unapportioned expenses Rs. 2,000 which works out at 10% of direct wages.

4.6 TREATMENT OF UNDER-ABSORBED AND OVER-ABSORBED OVERHEADS IN COST ACCOUNTING :

Overhead expenses are usually applied to production on the basis of pre-determined


rates. Production overheads are to be determined in advance as follows for fixing
selling price, quote tender price and to formulate budgets etc. Pre-determined
overhead rate =

Estimated/Normal overheads for the period Budgeted Number of units during the
period

The actual overhead rate will rarely coincide with the pre-determined overhead
rate, due to variation in pre-determined overhead rate and actual overhead rate.
Such a variation may arise due to any one of the following situations: (i)
Estimated overheads for the period under consideration may remain the same or they
coincide with actual overheads but the number of units produced during the period
is either more or less in comparison with budgeted figure. In the former case
actual overhead rate will be less and in the latter case, actual overhead rate
will be more than the pre-determined overhead rate, hence over-absorption and
under-absorption will occur respectively.

(ii) Similarly, if the number of units actually produced during the period remains
the
4.46
Overheads

same as budgeted figure but the actual overheads incurred are more or less than
the estimated overheads for the period, then a situation of under-absorption or
overabsorption will arise respectively. (iii) If changes occur in different
proportion both in the actual overheads and in the number of units produced during
the period, then a situation of under or overabsorption (depending upon the
situation) will arise. (iv) If the changes in the numerator (i.e. in actual
overheads) and denominator (i.e. in number of units produced) occur uniformly
(without changing the proportion between the two) then a situation of neither
under nor of over-absorption will arise. Such over or under-absorption as arrived
at under different situations may also be termed as overhead variance. The amount
of over-absorption being represented by a credit balance in the account and
conversely, the amount of under-absorption being a debit balance. As regards the
treatment of such debit or credit balances, the general view is that if the
balances are small they should be transferred to the Costing Profit and Loss
Account and the cost of individual products should not be increased or reduced as
these would be representing normal cost. Where, however the difference is large
and due to wrong estimation, it would be desirable to adjust the cost of products
manufactured, as otherwise the cost figures would convey a misleading impression.
Such adjustments usually take the form of supplementary rates where there is a
debit balance in the overhead account and a credit in the other case. Now, the
production of any period can be identified in three forms, goods finished and
sold, goods finished but held in stock (not yet sold) and semi-finished goods
(work in progress). So far as the first category of goods is concerned, it is
arguable that the postmortem of the costs of individual products long after they
have been sold may have some academic utility but it is frequently devoid of any
practical significance. Therefore, it is suggested that the total variance
concerning goods finished and sold should be adjusted by transferring the amount
to the Cost of Sale Account, the costs of the individual items of such goods not
being affected. As regards the variance pertaining to goods finished and held in
stock (i.e. not yet sold), it would be necessary to adjust the value of the stock;
similarly the value of work-in-progress should be adjusted. However, over or under
recovery of overheads due to abnormal reasons (such as abnormal over or under
capacity utilisation) should be transferred to the Costing Profit and Loss
Account.

4.47
Cost Accounting

Illustration

A light engineering factory fabricates machine parts to customers. The factory


commenced fabrication of 12 Nos. machine parts to customers� specifications and
the expenditure incurred on the job for the week ending 21st August, 2005 is given
below:
Rs. Rs.

Direct materials (all items) Direct labour (manual) 20 hours @ Rs. 1.50 per hour
Machine facilities : Machine No. I : 4 hours @ Rs. 4.50 Machine No. II : 6 hours @
Rs. 6.50 Total Overheads @ Rs. 0.80 per hour on 20 manual hours Total cost 18.00
39.00

78.00 30.00

57.00 165.00 16.00 181.00

The overhead rate of Re. 0.80 per hour is based on 3,000 man hours per week;
similarly, the machine hour rates are based on the normal working of Machine Nos.
I and II for 40 hours out of 45 hours per week. After the close of each week, the
factory levies a supplementary rate for the recovery of full overhead expenses on
the basis of actual hours worked during the week. During the week ending 21st
August, 2005, the total labour hours worked was 2,400 and Machine Nos. I and II
had worked for 30 hours and 32� hours respectively. Prepare a Cost Sheet for the
job for the fabrication of 12 Nos. machine parts duly levying the supplementary
rates.
Solution Fabrication of 12 Nos. machine parts (job No......) Date of commencement
: 16 August, 2005 Date of Completion. Cost sheet for the week ending, August 21,
2005 : Rs. Materials 78.00 Labour 20 hours @ Rs. 1.50 30.00 Machine facilities :
Rs. Machine No. I : 4 hours @ Rs. 4.50 18.00 Machine No. II : 6 hours @ Rs. 6.50
39.00 57.00
4.48
Overheads

Overheads 20 hours @ 0.80 P. per hour


Supplementary Rates

16.00 181.00 4.00 6.00 9.00 19.00 200.00

Overheads 20 hours @ 20. P. per hour Machine facilities : Machine No. I - 4 hours
@ Re. 1.50 Machine No. II - 6 hours @ Re. 1.50 Cost Rs.
Working notes:

Overheads budgeted : 3,000 hours @ 80 P. or Rs. 2,400


Actual hours : 2,400 Actual rate per hour Rs. 2,400/2,400 hours = Re. 1

Supplementary charge Re. 1 less 80 P, or 20 P per hour


Machine facilities :

Machine No. I Budgeted Actual number of hours Actual rate per hour Supplementary
rate per hour
Illustration

Machine No. II (40 � Rs. 6.50) = Rs. 260 32� Rs. 8 (Rs. 8.00 � Rs. 6.50)

(40 � Rs. 4.50) = Rs. 180 30 Rs. 6 (Rs. 6.00 � Rs. 4.50)

In a factory, overheads of a particular department are recovered on the basis of


Rs. 5 per machine hour. The total expenses incurred and the actual machine hours
for the department for the month of August were Rs. 80,000 and 10,000 hours
respectively. Of the amount of Rs. 80,000, Rs. 15,000 became payable due to an
award of the Labour Court and Rs. 5,000 was in respect of expenses of the previous
year booked in the current month (August). Actual production was 40,000 units, of
which 30,000 units were sold. On analysing the reasons, it was found that 60% of
the under-absorbed overhead was due to defective planning and the rest was
attributed to normal cost increase. How would you treat the under-absorbed
overhead in the cost accounts?

4.49
Cost Accounting

Solution Under-absorbed overhead expenses during the month of August Rs. 80,000

Total expenses incurred in the month of August :


Less: The amount paid according to labour

court award (Assumed to be non-recurring) Expenses of previous year Net overhead


expenses incurred for the month Overhead recovered for 10,000 hours @ Rs. 5 per
hour Under-absorbed overheads
Treatment of under-absorbed overhead in the Cost Accounts

Rs. 15,000 Rs. 5,000 20,000 60,000 50,000 10,000

It is given in the question that 40,000 units were produced out of which 30,000
units were sold. It is also given that 60% of the under-absorbed overhead was due
to defective planning and the rest was attributed to normal cost increase. Rs. 1.
60 percent of under-absorbed overhead is due to defective planning. This being
abnormal, should be debited to Profit and Loss A/c (60% of Rs. 10,000) 2. Balance
40 percent of under-absorbed overhead should be distributed over, Finished Goods
and Cost of Sales by supplementary rate (40% of Rs. 10,000) Rs. 4,000 may be
distributed over Finished Goods and Cost of Sales as follows : Finished Goods
*Working notes

6,000

4,000 10,000

*Rs. 1,000

Cost of Sales *Rs. 3,000 Under-absorbed overhead Units produced : : Rs. 4,000
40,000 Re. 0.10 per unit

Rate of under-absorbed overhead recover


4.50
Overheads

Amount of under-absorbed overheads charged to finished goods (10,000 � 0.10P)


Amount of under-absorbed overheads charged to cost of sales : (30,000 � 0.10P)
Illustration

Rs. 1,000 Rs. 3,000

In a manufacturing unit, factory overhead was recovered at a pre-determined rate


of Rs. 25 per man-day. The total factory overhead expenses incurred and the man-
days actually worked were Rs. 41.50 lakhs and 1.5 lakh man-days respectively. Out
of the 40,000 units produced during a period, 30,000 were sold. On analysing the
reasons, it was found that 60% of the unabsorbed overheads were due to defective
planning and the rest were attributable to increase in overhead costs. How would
unabsorbed overheads be treated in Cost Accounts ?
Solution Computation of unabsorbed overheads

Man-days worked Overhead actually incurred


Less: Overhead absorbed @ Rs. 25 per man-day

1,50,000
Rs.

41,50,000 37,50,000 ________ 4,00,000 2,40,000 1,60,000

(Rs. 25 � 1,50,000) Unabsorbed overheads Unabsorbed overheads due to defective


planning (i.e. 60% of Rs. 4,00,000) Balance of unabsorbed overhead
Treatment of unabsorbed overheads in Cost Accounts

(i)

The unabsorbed overheads of Rs. 2,40,000 due to defective planning to be treated


as abnormal and therefore be charged to Costing Profit and Loss Account. (ii) The
balance unabsorbed overheads of Rs. 1,60,000 be charged to production i.e., 40,000
units at the supplementary overhead absorption rate i.e., Rs. 4 per unit (Refer to
Working Note)

4.51
Cost Accounting

Charge to Costing Profit and Loss Account as part of the cost of unit sold (30,000
units @ Rs. 4 p.u.)
Add: To closing stock of finished goods

Rs.

1,20,000 40,000 _______ 1,60,000

(10,000 units @ Rs. 4 p.u.) Total


Working note :

Supplementary overhead absorption rate =


Illustration

Rs.1,60,000 = Rs. 4 p.u. 40,000 units

A factory has three production departments. The policy of the factory is to


recover the production overheads of the entire factory by adopting a single
blanket rate based on the percentage of total factory overheads to total factory
wages. The relevant data for a month are given below :
Department Direct materials Rs. Budget Direct wages Rs. Factory overheads Rs.
Direct labour Hour Machine hours

Machining Assembly Packing


Actuals

6,50,000 1,70,000 1,00,000

80,000 5,50,000 70,000

3,60,000 1,40,000 1,25,000

20,000 1,00,000 50,000

80,000 10,000 -

Machining Assembly Packing

7,80,000 1,36,000 1,20,000

96,000 2,70,000 90,000

3,90,000 84,000 1,35,000

24,000 90,000 60,000

96,000 11,000 -

4.52
Overheads

The details of one of the representative jobs produced during the month are as
under: Job No. CW 7083 : Department Direct Direct Direct Machine materials wages
labour hours Rs. Rs. hours Machining 1,200 240 60 180 Assembly 600 360 120 30
Packing 300 60 40 - The factory adds 30% on the factory cost to cover
administration and selling overheads and profit. Required : (i) Calculate the
overhead absorption rate as per the current policy of the company and determine
the selling price of the Job No. CW 7083.

(ii) Suggest any suitable alternative method(s) of absorption of the factory


overheads and calculate the overhead recovery rates based on the method(s) so
recommended by you. (iii) Determine the selling price of Job CW 7083 based on the
overhead application rates calculated in (ii) above. (iv) Calculate the
departmentwise and total under or over recovery of overheads based on the
company�s current policy and the method(s) recommended by you.
Solution (i) Department Computation of overhead absorption rate (as per the
current policy of the company) Budgeted factory overheads Rs. Budgeted direct
wages Rs.

Machinery Assembly Packing Total Overhead absorption rate =

3,60,000 1,40,000 1,25,000 6,25,000

80,000 3,50,000 70,000 5,00,000

Budgeted factory overheads � 100 Budgeted direct wages

4.53
Cost Accounting

= =

Rs. 6,25,000 � 100 Rs. 5,00,000


125% of Direct wages

Selling Price of the Job No. CW-7083


Rs.

Direct materials (Rs. 1,200 + Rs. 600 + Rs. 300) Direct wages (Rs. 240 + Rs. 360 +
Rs. 60) Overheads (125% � Rs. 660) Total factory cost
Add: Mark-up (30% � Rs. 3,585)

2,100.00 660.00 825.00 3,585.00 1,075.50 4,660.50

Selling price

(ii) Methods available for absorbing factory overheads and their overhead recovery
rates in different departments

1.

Machining Department

In the machining department, the use of machine time is the predominant factor of
production. Hence machine hour rate should be used to recover overheads in this
department. The overhead recovery rate based on machine hours has been calculated
as under:�
Machine hour rate =

Budgeted factory overheads Budgeted machine hours


=

Rs. 3,60,000 = Rs. 4.50 per hour 80,000 hours

2.

Assembly Department

In this department direct labour hours is the main factor of production. Hence
direct labour hour rate method should be used to recover overheads in this
department. The overheads recovery rate in this case is: Direct labour hour rate =

Budgeted factory overheads Budgeted direct labour hours

4.54
Overheads

= 3.
Packing Department :

Rs. 1,40,000 = Rs. 1.40 per hour 1,00,000 hours

Labour is the most important factor of production in this department. Hence direct
labour hour rate method should be used to recover overheads in this department.
The overhead recovery rate in this case comes to: Budgeted factory overhead Direct
labour hour rate =

Budgeted factory overheads Direct labour hours

=
(iii)

Rs. 1,25,000 = Rs. 2.50 per hour 50,000 hours

Selling Price of Job CW-7083 [based on the overhead application rates calculated
in (ii) above] (Rs.)

Direct materials Direct wages Overheads (Refer to Working note) Factory cost
Add: Mark up (30% of Rs. 3,838)

2,100.00 660.00 1,078.00 3,838.00 1,151.40 4,989.40

Selling price
Working note : Overhead Summary Statement Dept. Basis Hours

Rate Rs.

Overheads Rs.

Machining Assembly Packing

Machine hour Direct labour hour Direct labour hour

180 120 40

4.50 1.40 2.50 Total

810 168 100 1,078

4.55
Cost Accounting

(iv) Departmentwise statement of total under or over recovery of overheads

(a) Under current policy


Departments Machining Assembly Rs. Rs. Packing Rs. Total Rs.

Direct wages (Actual) Overheads recovered @ 125% of Direct wages: (A) Actual
overheads: (B) (Under)/Over recovery of overheads : (A�B) (b) As per methods
suggested

96,000 1,20,000 3,90,000 (2,70,000)

2,70,000 3,37,500 84,000 2,53,500

90,000 1,12,500 1,35,000 (22,500) 5,70,000 6,09,000 (39,000)

Basis of overhead recovery Machine Direct labour Direct labour hours hours hours
Total Rs.

Hours worked Rate/hour (Rs.) Actual overheads (Rs.) : (B) (Under)/Over recovery:
(A-B)
Illustration

96,000 4.50 3,90,000 42,000

90,000 1.40 1,26,000 84,000 42,000

60,000 2.50 1,50,000 1,35,000 15,000 7,08,000 6,09,000 99,000

Overhead recovered (Rs.): (A) 4,32,000

The total overhead expenses of a factory are Rs. 4,46,380. Taking into account the
normal working of the factory, overhead was recovered in production at Rs. 1.25
per hour. The actual hours worked were 2,93,104. How would you proceed to close
the books of accounts, assuming that besides 7,800 units produced of which 7,000
were sold, there were 200 equivalent units in work-in-progress ? On investigation,
it was found that 50% of the unabsorbed overhead was on account of increase in the
cost of indirect materials and indirect labour and the remaining 50% was due to
factory inefficiency. Also give the profit implication of the method suggested.

4.56
Overheads

Solution

Actual factory overhead expenses incurred Less : Overheads recovered from


production (2,93,104 hours � Rs. 1.25) Unabsorbed overheads
Reasons for unabsorbed overheads (i) 50% of the unabsorbed overhead was on account
of increased in the cost of indirect materials and indirect labour

Rs. 4,46,380 3,66,380

80,000 40,000

(ii)

50% of the unabsorbed overhead was due to factory inefficiency.

40,000

Treatment of unabsorbed overheads in Cost Accounting

1.

Unabsorbed overhead amounting to Rs. 40,000, which were due to increase in the
cost of indirect material and labour should be charged to units produced by using
a supplementary rate. Supplementary rate =

Rs. 40,000 = Rs. 5 per unit (7,800 + 200) units

The sum of Rs. 40,000 (unabsorbed overhead) should be distributed by using a


supplementary rate among cost of sales, finished goods and work-in progress as
below : Cost of sales (7,000 units � Rs. 5) Finished goods (800 units � Rs. 5)
Work-in progress (200 units � Rs. 5)
Rs. 35,000

4,000 1,000 40,000

The use of cost of sales figure, would reduce the profit for the period by Rs.
35,000 and will increase the value of stock of finished goods and work-in-progress
by Rs. 4,000 and Rs. 1,000 respectively.

4.57
Cost Accounting

2.

The balance amount of unabsorbed overheads viz. of Rs. 40,000 due to factory
inefficiency should be charged to Costing Profit & Loss Account, as this is an
abnormal loss.

Illustration

ABC Ltd. manufactures a single product and absorbs the production overheads at a
predetermined rate of Rs. 10 per machine hour. At the end of financial year 2005-
06, it has been found that actual production overheads incurred were Rs. 6,00,000.
It included Rs. 45,000 on account of �written off� obsolete stores and Rs. 30,000
being the wages paid for the strike period under an award. The production and
sales data for the year 2005-06 is as under :
Production :

Finished goods Work-in-progress (50% complete in all respects)


Sales :

20,000 units 8,000 units

Finished goods 18,000 units The actual machine hours worked during the period were
48,000. It has been found that one-third of the under-absorption of production
overheads was due to lack of production planning and the rest was attributable to
normal increase in costs. (i) Calculate the amount of under-absorption of
production overheads during the year 2005-06; and

(ii) Show the accounting treatment of under-absorption of production overheads.


Solution (i) Amount of under-absorption of production overheads during the year
2005-06 Rs.

Total production overheads actually incurred during the year 2005-06


Less : �Written off� obsolete stores

6,00,000 Rs. 45,000 Rs. 30,000 75,000 5,25,000

Wages paid for strike period Net production overheads actually incurred : (A)
Production overheads absorbed by 48,000 machine
4.58
Overheads

hours @ Rs. 10 per hour : (B) Amount of under � absorption of production overheads
: [(A) � (B)]

4,80,000 45,000

(ii) Accounting treatment of under absorption of production overheads It is given


in the statement of the question that 20,000 units were completely finished and
8,000 units were 50% complete, one third of the under-absorbed overheads were due
to lack of production planning and the rest were attributable to normal increase
in costs.

1.

(33 � 1/3% of Rs. 45,000) i.e., Rs. 15,000 of under-absorbed overheads were due to
lack of production planning. This being abnormal, should be debited to the Profit
and Loss A/c. Balance (66�2/3% of Rs. 45,000) i.e., Rs. 30,000 of under-absorbed
overheads should be distributed over work-in-progress, finished goods and cost of
sales by using supplementary rate. Total under-absorbed overheads

Rs.

15,000

2.

30,000 45,000

Apportionment of unabsorbed overheads of Rs. 30,000 over, work-in progress,


finished goods and cost of sales Equivalent Rs. Completed Units

Work-in-Progress (4,000 units � Rs. 1.25) (Refer to working note) Finished goods
(2,000 units � Rs. 1.25) Cost of sales (18,000 units � Rs. 1.25)
Working Note:

4,000 2,000 18,000 24,000


Rs.30,000 = Rs. 1.25 24,000

5,000 2,500 22,500 30,000

Supplementary rate per unit =


4.7

ACCOUNTING AND CONTROL OF ADMINISTRATIVE OVERHEADS

Definition - According to I.C.M.A. Terminology, Administrative overhead is defined


as �The sum of those costs of general management and of secretarial accounting and

4.59
Cost Accounting

administrative services, which cannot be directly related to the production,


marketing, research or development functions of the enterprise.� According to this
definition, administrative overhead constitutes the expenses incurred in
connection with the formulation of policy directing the organisation and
controlling the operations of an undertaking. These overheads are also collected
and classified in the same way as the factory overheads.
4.8.1 Accounting of Administrative overheads : There are three distinct methods of
accounting of administrative overheads, which are briefly discussed below :

(a) Apportioning Administrative Overheads between Production and Sales Departments


: According to this method administrative overheads are apportioned over
production and sales departments. The reason for the apportionment of overhead
expenses over these departments, recognises the fact that administrative overheads
are incurred for the benefit of both of these departments. Therefore each
department should be charged with the proportionate share of the same. When this
method is adopted, administrative overheads lose their identity and get merged
with production and selling and distribution overheads.
Disadvantages :

(1) It is difficult to find suitable bases of administrative overhead


apportionment over production and sales departments. (2) Lot of clerical work is
involved in apportioning overheads. (3) It is not justified to apportion total
administrative overheads only over production and sales departments when other
equally important department like finance is also there. (b) Charging to Profit
and Loss Account - According to this method administrative overheads are charged
to Costing Profit & Loss Account. The reason for charging to Costing Profit & Loss
are firstly, the administrative overheads are concerned with the formulation of
policies and thus are not directly concerned with either the production or the
selling and distribution functions. Secondly, it is difficult to determine a
suitable basis for apportioning administrative overheads over production and sales
departments. Lastly, these overheads are the fixed costs. In view of these
arguments, administrative overheads should be charged to Profit and Loss Account.
Disadvantages :

(1) Cost of products are understated as administrative overheads are not charged
to costs. (2) The exclusion of administrative overheads from cost of products is
against sound accounting principle.
4.60
Overheads

(c) Treating Administrative Overheads as a separate addition to Cost of


Production/Sales : This method considers administration as a separate function
like production and sales and, as such costs relating to formulating the policy,
directing the organisation and controlling the operations are taken as a separate
charge to the cost of the jobs or a product, sold along with the cost of other
functions. The basis which are generally used for apportionment are : (i) Works
cost (ii) Sales value or quantity (iii) Gross profit on sales (iv) Quantity
produced (v) Conversion cost, etc. 4.8.2 Control of Administrative Overheads -
Mostly administrative overheads are of fixed nature, and they arise as a result of
management policies. These fixed overheads are generally non-controllable. But at
the same time these overheads should not be allowed to grow disproportionately.
Some degree of control has to be exercised over them. The methods usually adopted
for controlling administrative overheads are as follows : (i) Classification and
analysis of overheads by administrative departments according to their functions,
and a comparison with the accomplished results: According to this method the
expenses incurred by each administrative department are collected under standing
order numbers for each class of expenditure. These are compared with similar
figures of the previous period in relation to accomplishment. Such a comparison
will reveal efficiency or inefficiency of the concerned department. However, this
method provides only a limited degree of control and comparison does not give
useful results if the level of activity is not constant during the periods under
comparison. To overcome this difficulty, overhead absorption rates may also be
compared from period to period; the extent of over or under absorption will reveal
the efficiency or otherwise of the department. It may be possible to compare the
cost of a service department with that of similar services obtainable from outside
and a decision may be taken whether it is economical to continue the department or
entrust the work to outsiders. (ii) Control through Budgets - According to this
method, administration budgets (monthly or annually) are prepared for each
department. The budgeted figures are compared with actual ones to determine
variances. The variances are analysed and responsibility assigned to the concerned
department to control these variances. (iii) Control through Standard - Under this
method, standards of performance are fixed for each administrative activity, and
the actual performance is compared with the standards set. In this way, standards
serve not only as yardstick of performance but also facilitate control of costs.

4.61
Cost Accounting

Illustration

In an engineering company, the factory overheads are recovered on a fixed


percentage basis on direct wages and the administrative overheads are absorbed on
a fixed percentage basis on factory cost. The company has furnished the following
data relating to two jobs undertaken by it in a period:
Job 101 Rs. Job 102 Rs.

Direct materials Direct wages Selling price Profit percentage on Total Cost
Required :

54,000 42,000 1,66,650 10%

37,500 30,000 1,28,250 20%

Computation of percentage recovery rates of factory overheads and administrative


overheads. (ii) Calculation of the amount of factory overheads, administrative
overheads and profit for each of the two jobs. (iii) Using the above recovery
rates fix the selling price of job 103. The additional data being: Direct
materials Direct wages Profit percentage on selling price
Solution (i)

(i)

Rs. 24,000 Rs. 20,000 12-�%

Let factory overhead recovery rate, as percentage of direct wages be F and


administrative overheads recovery rate, as percentage of factory cost be A.
Factory Cost of Jobs :

Job 101 =

Rs. 96,000 + Rs. 42,000F

Job 102 = Rs. 67,500 + Rs. 30,000F Total Cost of Production of Jobs : Job 101 =
(Rs. 96,000 + Rs. 42,000F) + (Rs. 96,000+ Rs. 42,000F)A = Rs. 1,51,500 Job-102 =
Rs. 67,500 + Rs. 30,000F) + (Rs. 67,500+ Rs. 30,000F)A = Rs. 1,06,875
(Refer to working note)
4.62
Overheads

On solving above relations: F = 0.60 and A = 0.25 Hence, percentage recovery rates
of factory overheads and administrative overheads are 60% and 25% respectively.
Working note: Job 101 Job 102

Total cost of production (Rs.)

1,51,500 (Rs. 1,66,650/110%)

1,06,875 (Rs. 1,28,250/120%)

Selling price (100% + Percentage of profit)


(ii)

Statement of jobs, showing amount of factory overheads, administrative overheads


and profit Job 101 Rs. Job 102 Rs.

Direct materials Direct wages Prime cost


Factory overheads

54,000 42,000 96,000 25,200 1,21,200 30,300 1,51,500 15,150 1,66,650


Selling price of Job 103

37,500 30,000 67,500 18,000 85,200 21,375 1,06,875 21,375 1,28,250


Rs. 24,000

60% of direct wages Factory cost


Administrative overheads

25% of factory cost Total cost


Profit (balancing figure)

Selling price
(iii)

Direct materials Direct wages Prime cost Factory overheads (60% of Direct Wages)

20,000 44,000 12,000

4.63
Cost Accounting

Factory cost Administrative overheads (25% of factory cost) Total cost Profit
margin (balancing figure) Selling price
4.8

56,000 14,000 70,000 10,000 80,000

Total Cost 87.5%

ACCOUNTING AND CONTROL OF SELLING AND DISTRIBUTION OVERHEADS

Selling cost or overhead expenses are the expenses incurred for the purpose of
promoting the marketing and sales of different products. Distribution expenses, on
the other hand, are expenses relating to delivery and despatch of goods sold.
Examples of selling and distribution expenses have been considered earlier in this
booklet. From the definitions it is clear that the two type of expenses represent
two distinct type of functions. Some concerns group together these two type of
overhead expenses into one composite class, namely, selling and distribution
overhead, for the purpose of Cost Accounting.
4.8.1 Accounting of selling and distribution overheads: The collection and
accumulation of each expense is made by means of appropriate standing order
numbers in the usual way. Where it is decided to apportion a part of the
administrative overhead to the selling division the same should also be collected
through appropriate standing order numbers.

As in the case of administrative overheads, it is not easy to determine an


entirely satisfactory basis for computing the overhead rate for absorbing selling
overheads. The bases usually adopted are : (a) Sales value of goods; (b) Cost of
goods sold; (c) Gross Profit on sales; and (d) Number of orders or units sold. It
is considered that the sale value is ordinarily the most logical basis, there
being some connection between the amount of sales and the amount of expenses
incurred to achieve them. The cost of production, however, is not so satisfactory
on basis as it is difficult to conceive of any relationship even remote, between
the cost of production of any article and its selling cost. Articles having a high
cost of production may require little effort in their sale and vice versa. The
basis of gross profit on sales results in a larger share of the selling overhead
being applied to goods yielding a large margin of profit and vice versa. The basis
therefore follows the principle of �ability to pay�, it may not reflect costs or
incurred efforts.
An estimated amount per unit - The best method for absorbing selling and
distributing expenses over various products is to separate fixed expenses from
variable expenses. Apportion the fixed expenses according to the benefit derived
by each product and thus
4.64
Overheads

ascertaining the fixed expenses per unit. We give below some of the fixed expenses
and the basis of apportionment :
Expenses Basis

Salaries in the Sales Department and of the sales men. Advertisement

Estimated time devoted to the sale of various products. Actual amount incurred for
each product since these days it is usual to advertise each product separately;
common expenses, such as in an exhibition, should be apportioned on the basis of
advertisement expenditure on each product. Average space occupied by each product.
Average quantities delivered during a period.

Show Room expenses Rent of finished goods godowns and Expenses on own delivery
vans

If a suitable basis for apportioning expenses does not exist it may be apportioned
in the proportion of sales of various products. The total of fixed expenses
apportioned in this manner, divided by the number of units sold or likely to be
sold, will give the fixed expenses per unit. To this should be added the variable
expenses which will be different for each product. These expenses are, packaging,
freight outwards, insurance in transit, commission payable to salesmen, rebate
allowed to customers, etc. All these items will be worked out per unit for each
product separately. These items added to fixed expenses per unit will give an
estimated amount of the selling and distribution expenses per unit.
4.8.2 Control of Selling & Distribution Overheads - Control of selling and
distribution expenses is a difficult task. The reasons for this are as follows :

1.

The incidence of selling and distribution overheads depends mainly on external


factors, such as distance of market, extent and nature of competition, terms of
sales, etc. which are beyond the control of management. These overheads are
dependent upon the customers, behaviour, their liking and disliking, tastes etc.
Therefore, as such control over the overheads may result in loss of customers.
These expenses being of the nature of policy costs, are not amenable to control.
Comparison with past performance - According to this method, selling and

2.

3. (a)

In spite of the above difficulties, the following methods may be used for
controlling them.

4.65
Cost Accounting

distribution overheads are compared with the figures of the previous period.
Alternatively, the expenses may be expressed as a percentage of sales, and the
percentages may be compared with those of the past period. This method is suitable
for small concerns. (b) Budgetary Control - A budget is set up for selling and
distribution expenses. The expenses are classified into fixed and variable. If
necessary, a flexible budget may be prepared indicating the expenses at different
levels of sales. The actual expenses are compared with the budgeted figures and in
the case of variances suitable actions are taken. (c) Standard Costing - Under
this method standards are set up in relation to the standard sales volume.
Standards may be set up for salesmen, territories, products etc. Once the
standards are set up, comparison is made between the actuals and standards :
variances are enquired into and suitable action taken.
Illustration

A company which sells four products, some of them unprofitable, proposes


discontinuing the sale of one of them. The following information is available
regarding income, costs and activity for the year ended 31st March, 2006.
Products A B C D

Sales (Rs.) Cost of sales (Rs.) Area of storage (Sq.ft.) Number of parcels sent

3,00,000 2,00,000 50,000 1,00,000

5,00,000 4,50,000 40,000 1,50,000

2,50,000 2,10,000 80,000 75,000 60,000

4,50,000 2,25,000 30,000 1,75,000 1,20,000

Number of invoices sent 80,000 1,40,000 Selling and Distribution overheads and the
basis of allocation are :
Fixed Costs

Basis of allocation

Rs. to products

Rent & Insurance Depreciation Salesmen�s salaries & expenses Administrative wages
and salaries
Variable Costs :

30,000 10,000 50,000 50,000

Sq.Ft. Parcel Sales Volume No. of invoices

Packing wages & materials

20 paise per parcel

4.66
Overheads

Commission Stationery

4% of sales 10 P. per invoice

You are required to prepare Profit & Loss Statement, showing the percentage of
profit or loss to sales for each product.
Solution Statement of Profit or Loss on Various Products during the year ended
March 31, 2006. Products Total Rs. A Rs. B Rs. C Rs. D Rs.

Sales
Variable costs

15,00,000 10,85,000 60,000 1,00,000 40,000 12,85,000 2,15,000 30,000 10,000 60,000
50,000 1,50,000 65,000 4.3

3,00,000 2,00,000 12,000 20,000 8,000 2,40,000 60,000 7,500 2,000 12,000 10,000
31,500 28,500 9.5

5,00,000 2,50,000 4,50,000 2,10,000 20,000 30,000 14,000 10,000 15,000 6,000

4,50,000 2,25,000 18,000 35,000 12,000 2,90,000 1,60,000 4,500 3,500 18,000 15,000
41,000 1,19,000 26.4

Cost of sales Commissions 4% of sales Packing wages & materials @ 20 P. per parcel
Stationery @ 10 P. per invoice Total variable costs Contribution (Sales� variable
cost)
Fixed Costs

5,14,000 2,41,000 �14,000 6,000 3,000 20,000 17,500 46,500 �60,500 �12.1 9,000
12,000 1,500 10,000 7,500 31,000 �22,000 �8.8

Rent & Insurance Depreciation Salesmen�s salaries & expenses Administrative wages
& salaries Total Fixed costs Profit or Loss (Contribution�fixed Costs) Percentage
of profit or Loss on sales

4.67
Cost Accounting

4.9

CONCEPTS RELATED TO CAPACITY Rated capacity : It refers to the capacity of a


machine or a plant as indicated by its manufacturer. In fact this capacity is the
maximum possible productive capacity of a plant. It is also known as installed
capacity of a plant. Due to the loss of operating time of a plant it is difficult
to achieve this rated capacity. In other words, it is only a theoretical capacity
and is therefore, seldom achieved.

(i)

(ii) Practical capacity : It is defined as actually utilised capacity of a plant.


It is also known as operating capacity. This capacity takes into account loss of
time due to repairs, maintenance, minor breakdown, idle time, set up time, normal
delays, Sundays and holidays, stock taking etc. Generally, practical capacity is
taken between 80 to 90% of the rated capacity. It is also used as a base for
determining overhead rates. Practical capacity is also called net capacity or
available capacity. (iii) Normal capacity : It is the capacity of a plant which is
expected to be utilised over a long period based on sales expectancy. The
determination of this capacity considers the average utilisation of plant capacity
during one full business cycle which may extend over 2 to 3 years. It is also
known as average capacity and is used to compute overhead recovery rate. (iv)
Capacity based on sales expectancy : It is the capacity of a plant utilised based
on sales expectancy. (v) Actual capacity : It is the capacity actually achieved
during a given period. This capacity may lie between practical capacity and
capacity based on sales expectancy. (vi) Idle capacity : It is that part of the
capacity of a plant, machine or equipment which cannot be effectively utilised in
production. In other words, it is the difference between the practical or normal
capacity and capacity utilisation based on expected sales. For example, if the
practical capacity of production of a machine is to the tune of 10,000 units in a
month, but is used only to produce 8,000 units, because of market demand of the
product, then in such a case, 2,000 units will be treated as idle capacity of the
machine. The idle capacity may arise due to lack of product demand, non-
availability of raw material, shortage of skilled labour, absenteeism, shortage of
power fuel or supplies, seasonal nature of product etc.
Idle capacity cost : Costs associated with idle capacity are mostly fixed in
nature. These include depreciation, repairs and maintenance charges, insurance
premium, rent, rates, management and supervisory costs. These costs remain
unabsorbed or unrecovered due

4.68
Overheads

to under-utilisation of plant and service capacity. Idle capacity cost can be


calculated as follows : Idle capacity cost =

Aggregate overhead related to plant Normal plant capacity

Treatment of Idle capacity costs : Idle capacity costs can be treated in product
costing, in the following ways :

(a) If the idle capacity cost is due to unavoidable reasons such as repairs,
maintenance, change over of job etc. a supplementary overhead rate may be used to
recover the idle capacity cost. In this case, the costs are charged to the
production capacity utilised. (b) If the idle capacity cost is due to avoidable
reasons such as faulty planning, power failure etc.; the cost should be charged to
profit and loss account. (c) If the idle capacity cost is due to seasonal factors,
then, the cost should be charged to the cost of production by inflating overhead
rates. (vii) Idle facility : The term �facility� has a wider connotation which may
also include prediction capacity. Facilities may be provided by fixed assets such
as building space, plants equipment capacity, etc. or by various service functions
such as material services, production services, personal services etc. If a firm
fails to make full use of the facilities of its disposal, the firm may be said to
have idle facilities. Thus idle facilities refer to that part of total facilities
which remains unutilised due to any reason such as non-availability of raw
material, power, lack of demand etc. In Cost Accounting idle facilities are
treated in the same way as those of idle capacity.
4.10 TREATMENT OF CERTAIN ITEMS IN COSTING 4.10.1 Treatment of interest and
financial charges : There is controversy whether financial charges, specially
interest, should be included in the costs or not. The following arguments are
generally advanced in favour of interest to be included in overhead expenses.

(1) Computation of total cost is impossible unless interest is taken into account.
Interest is an element of cost and therefore, should be included in cost. This is
specially true in business where raw materials in different stages can be used.
Thus a timber merchant, if he buys standing trees and seasons the timber himself,
would incur a large amount of costs as interest. Another merchant who buys his
timber already seasoned would automatically have to pay a higher price; obviously,
this price includes interest.

4.69
Cost Accounting

(2) Interest is the cost to be paid for the use of capital; capital is also a
factor of production just as labour. Thus, if wages are included in cost of
production, why not interest ? (3) If interest is not included in cost
calculation, a number of managerial decisions may be taken wrongly. Thus, where a
decision involves replacement of labour with expensive machinery, the question of
interest assumes importance, since, if interest is not included, the cost
accountant may conclude that machinery is cheaper. (4) Inclusion of interest also
allows comparison of profit on different jobs. Thus, if a job takes 3 months and
another takes 6 months the cost of the jobs must include a charge by way of
interest before profit can be compared. (5) In inventory control, interest is an
important item to be considered. Where large stocks are kept, the advantage of one
time purchase is offset by increase in interest charges. (6) While submitting
tenders for cost plus contracts, etc., interest must be taken into account.
However, many cost accountants argue that interest should not be included in cost
accounts since it is not an item of cost and would vary with different methods of
financing. Some of the arguments are listed below : (1) Payment of interest
depends entirely on the financing policies and financing pattern. A firm working
with proprietor�s capital only will have no interest to pay whereas a firm working
with borrowed capital will have to pay a large amount of interest. In reality,
whether a firm raises a certain sum of money from the proprietor or borrows from
the outside does not make any difference as far as production efficiencies are
concerned. If we compare the two firms and include interest as an item of cost,
the firm, which works on the proprietor�s capital will here is only an
insignificant risk of failure of the company to repay the amount at maturity).
Cash flows exclude movements between items that constitute cash or cash equivalent
because these components are part of the cash management of an enterprise rather
than part of its operating, investing and financing activities. Cash management
includes the investment of excess cash in cash equivalents.
2.7 PRESENTATION OF CASH FLOW STATEMENT

The cash flow statement should report cash flows during the period classified by
operating, investing and financing activities. An enterprise presents its cash
flows from operating, investment and financing activities in a manner which is
most appropriate to its business. Classification by activity provides information
that allows users to assess the impact of those activities on the financial
position of the enterprise and the amount of its cash and cash equivalents. This
information may also be used to evaluate the relationships among those activities.
A single transaction may include cash flows that are classified differently. For
example, when the instalment paid in respect of a fixed asset acquired on deferred
payment basis includes both interest and loan, the interest element is classified
under financing activities and the loan element is classified under investing
activities.
2.7.1 Operating Activities

The amount of cash flows arising from operating activities is a key indicator of
the extent to which the operations of the enterprise have generated sufficient
cash flows to maintain the operating capability of the enterprise, pay dividends,
repay loans and make new investments without recourse to external sources of
financing. Information about the specific components of historical operating cash
flows is useful, in conjunction with other information, in forecasting future
operating cash flows. Cash flows from operating activities are primarily derived
from the principal revenue-producing activities of the enterprise. Therefore, they
generally result from the transactions and other events that enter into the
determination of net profit or loss. Examples of cash flows from operating
activities are: (a) Cash receipts from the sale of goods and the rendering of
services;
3.45
Financial Management

(b) Cash receipts from royalties, fees, commissions and other revenue; (c) Cash
payments to suppliers for goods and services; (d) Cash payments to and on behalf
of employees; (e) Cash receipts and cash payments of an insurance enterprise for
premiums and claims, annuities and other policy benefits; (f) Cash payments or
refunds of income taxes unless they can be specifically identified with financing
and investing activities; and

(g) Cash receipts and payments relating to futures contracts, forward contracts,
option contracts and swap contracts when the contracts are held for dealing or
trading purposes. Some transactions, such as the sale of an item of plant, may
give rise to a gain or loss which is included in the determination of net profit
or loss. However, the cash flows relating to such transactions are cash flows from
investing activities. An enterprise may hold securities and loans for dealing or
trading purposes, in which case they are similar to inventory acquired
specifically for resale. Therefore, cash flows arising from the purchase and sale
of dealing or trading securities are classified as operating activities.
Similarly, cash advances and loans made by financial enterprises are usually
classified as operating activities since they relate to the main revenue-producing
activity of that enterprise.
2.7.2 Investing Activities

The separate disclosure of cash flows arising from investing activities is


important because the cash flows represent the extent to which expenditures have
been made for resources intended to generate future income and cash flows.
Examples of cash flows arising from investing activities are: (a) Cash payments to
acquire fixed assets (including intangibles). These payments include those
relating to capitalized research and development costs and self-constructed fixed
assets; (b) Cash receipts from disposal of fixed assets (including intangibles);
(c) Cash payments to acquire shares, warrants or debt instruments of other
enterprises and interests in joint ventures (other than payments for those
instruments considered to be cash equivalents and those held for dealing or
trading purposes); (d) Cash receipts from disposal of shares, warrants or debt
instruments of other enterprises and interests in joint ventures (other than
receipts from those instruments considered to be cash equivalents and those held
for dealing or trading purposes);

3.46
Financial Analysis and Planning

(e) Cash advances and loans made to third parties (other than advances and loans
made by a financial enterprise); (f) Cash receipts from the repayment of advances
and loans made to third parties (other than advances and loans of a financial
enterprise); (g) Cash payments for futures contracts, forward contracts, option
contracts and swap contracts except when the contracts are held for dealing or
trading purposes, or the payments are classified as financing activities; and (h)
Cash receipts from futures contracts, forward contracts, option contacts and swap
contracts except when the contracts are held for dealing or trading purposes, or
the receipts are classified as financing activities. When a contract is accounted
for as a hedge of an identifiable position, the cash flows of the contract are
classified in the same manner as the cash flows of the position being hedged.
2.7.3 Financing Activities

The separate disclosure of cash flows arising from financing activities is


important because it is useful in predicting claims on future cash flows by
providers of funds (both capital and borrowings) to the enterprise. Examples of
cash flows arising from financing activities are: (a) Cash proceeds from issuing
shares or other similar instruments; (b) Cash proceeds from issuing debentures,
loans, notes, bonds and other short or long-term borrowings; and (c) Cash
repayments of amounts borrowed. In addition to the general classification of three
types of cash flows, AS-3 (Revised) provides for the treatment of the cash flows
of certain special items as under:
Foreign Currency Cash Flows

Cash flows arising from transactions in a foreign currency should be recorded in


an enterprises reporting currency. The reporting should be done by applying the
exchange rate at the date of cash flow statement. A rate which approximates the
actual rate may also be used. For example, weighted average exchange rate for a
period may be used for recording foreign currency transactions. The effect of
changes in exchange rates on cash and cash equivalents held in foreign currency
should be reported as a separate part in the form of reconciliation in order to
reconcile cash and cash equivalents at the beginning and end of the period.

3.47
Financial Management

Unrealised gains and losses arising from changes in foreign exchange rates are not
cash flows. The difference of amount raised due to changes in exchange rate should
not be included in operating investing and financing activities. This shall be
shown separately in the reconciliation statement.
2.7.4 Extraordinary Items

Any cash flows relating to extraordinary items should as far as possible classify
them into operating, investing or financing activities and those items should be
separately disclosed in the cash flow statement. Some of the examples for
extraordinary items is bad debts recovered, claims from insurance companies,
winning of a law suit or lottery etc. The above disclosure is in addition to
disclosure mentioned in AS-5, ‘Net Profit or Loss for the period, prior period
items and changes in accounting policies.’
2.7.5 Interest and Dividends

Cash flows from interest and dividends received and paid should each be disclosed
separately. The treatment of interest and dividends, received and paid, depends
upon the nature of the enterprise i.e., financial enterprises and other
enterprises. In case of financial enterprises, cash flows arising from interest
paid and interest & Dividends received, should be classified as cash flows from
operating activities.
In case of other enterprises

Cash outflows arising from interest paid on terms loans and debentures should be
classified as cash outflows from financing activities. Cash outflows arising from
interest paid on working capital loans should be classified as cash outflow from
operating activities. Interest and dividends received should be classified as cash
inflow from investing activities. Interest and dividends received should be
classified as cash inflow from investing activities. Dividend paid on equity and
preference share capital should be classified as cash outflow from financing
activities.
Taxes on Income

Cash flows arising from taxes on income should be separately disclosed. It should
be classified as cash flows from operating activities unless they can be
specifically identified with financing and investing activities.

3.48
Financial Analysis and Planning

When tax cash flows are allocated over more than one class of activity, the total
amount of taxes paid is disclosed.
2.7.6 Investments in Subsidiaries, Associates and Joint Ventures

Any such investments should be reported in the cash flow statement as investing
activity. Any dividends received should also be reported as cash flow from
investing activity.
2.7.7 Non-Cash Transactions

Investing and financing transactions that do not require the use of cash or cash
equivalents should be excluded from a cash flow statement. Such transactions
should be disclosed elsewhere in the financial statements in a way that provides
all the relevant information about these investing and financing activities. The
exclusion of non-cash transactions from the cash flow statement is consistent with
the objective of a cash flow statement as these do not involve cash flows in the
current period. Examples of non-cash transactions: (a) The acquisition of assets
by assuming directly related liabilities. (b) The acquisition of an enterprise by
means of issue of shares. (c) Conversion of debt into equity.
2.8 PROCEDURE IN PREPARATION OF CASH FLOW STATEMENT

The procedure used for the preparation of cash flow statement is as follows:
Calculation of net increase or decrease in cash and cash equivalents accounts: The
difference between cash and cash equivalents for the period may be computed by
comparing these accounts given in the comparative balance sheets. The results will
be cash receipts and payments during the period responsible for the increase or
decrease in cash and cash equivalent items. Calculation of the net cash provided
or used by operating activities: It is by the analysis of Profit and Loss Account,
Comparative Balance Sheet and selected additional information. Calculation of the
net cash provided or used by investing and financing activities: All other changes
in the Balance sheet items must be analysed taking into account the additional
information and effect on cash may be grouped under the investing and financing
activities. Preparation of a Cash Flow Statement: It may be prepared by
classifying all cash inflows and outflows in terms of operating, investing and
financing activities. The net cash flow provided or used in each of these three
activities may be highlighted.

Ensure that the aggregate of net cash flows from operating, investing and
financing activities is equal to net increase or decrease in cash and cash
equivalents.

3.49
Financial Management

Report any significant investing financing transactions that did not involve cash
or cash equivalents in a separate schedule to the Cash Flow Statement.
2.8.1 Reporting of Cash Flow from Operating Activities

The purpose for determining the net cash from operating activities is to
understand why net profit/loss as reported in the Profit and Loss account must be
converted. The financial statements are generally prepared on accrual basis of
accounting under which the net income will not indicate the net cash provided by
or net loss will not indicate the net cash used in operating activities. In order
to calculate the net cash flows in operating activities, it is necessary to
replace revenues and expenses with actual receipts and payments in cash. This is
done by eliminating the non-cash revenues and/non-cash expenses from the given
earned revenues and incurred expenses. There are two methods of converting net
profit into net cash flows from operating activities(i) (ii) Direct method, and
Indirect method.

(i) Direct Method: Under direct method, cash receipts from operating revenues and
cash payments for operating expenses are arranged and presented in the cash flow
statement. The difference between cash receipts and cash payments is the net cash
flow from operating activities. It is in effect a cash basis Profit and Loss
account. In this case, each cash transaction is analysed separately and the total
cash receipts and payments for the period is determined. The summarized data for
revenue and expenses can be obtained from the financial statements and additional
information. We may convert accrual basis of revenue and expenses to equivalent
cash receipts and payments. Make sure that a uniform procedure is adopted for
converting accrual base items to cash base items. Under direct method, items like
depreciation, amortisation of intangible assets, preliminary expenses, debenture
discount, etc. are ignored from cash flow statement since the direct method
includes only cash transactions and non-cash items are omitted. Likewise, no
adjustment is made for loss or gain on the sale of fixed assets and investments.
(ii) Indirect Method: In this method the net profit (loss) is used as the base and
converts it to net cash provided or used in operating activities. The indirect
method adjusts net profit for items that affected net profit but did not affect
cash. Non-cash and non-operating charges in the Profit and Loss account are added
back to the net profit while non-cash and non-operating credits are deducted to
calculate operating profit before working capital changes. It is a partial
conversion of accrual basis profit to cash basis profit. Necessary adjustments are
made for increase or decrease in current assets and current liabilities to obtain
net cash from operating activities.

3.50
Financial Analysis and Planning

2.8.2

Other Disclosure Requirements

If any significant cash and cash equivalent balances held by the enterprise are
not available for use by it, it should be disclosed in the cash flow statement.
For example cash held by the overseas branch which is not available for use by the
enterprise due to exchange control regulations or due to other legal restrictions.
Any additional information to understand the financial position and liquidity
position of an enterprise should be disclosed. For example: The amount of undrawn
borrowing facilities that may be available for future operating activities and to
settlement of capital commitments, indicating any restrictions on the use of these
facilities; and The aggregate amount of cash flows that represent increases in
operating capacity separately from those cash flows that are required to maintain
operating capacity. A reconciliation of cash and cash equivalents given in its
cash flow statement with equivalent items reported in the Balance Sheet. An
enterprise should disclose the policy which it adopts in determining the
composition of cash and cash equivalent. The effect of any change in the policy
for determining components of cash and cash equivalents should be reported in
accordance with AS-5, ‘Net Profit or Loss for the period, Prior period items, and
Changes in accounting policies’.
2.8.3 Format of Cash Flow Statement

AS 3 (Revised) has not provided any specific format for the preparation of cash
flow statements, but a general idea can be had from the illustration given in the
appendix to the Accounting Standard. There seems to be flexibility in the
presentation of cash flow statements. However, a widely accepted format under
direct method and indirect method is given below:
Cash Flow Statement (Direct Method) Rs.

Cash Flow from Operating Activities Cash receipts from customers Cash paid to
suppliers and employees Cash generated from operations Income tax paid Cash flow
before extraordinary items Proceeds from earthquake disaster settlement etc xxx
(xxx) xxx (xxx) xxx xxx

3.51
Financial Management

Net cash from Operating Activities Cash Flows from Investing Activities Purchase
of fixed assets Proceeds from sale of equipment Interest received Dividend
received Net cash from investing Activities Cash Flows from Financing Activities
Proceeds from issuance of share capital Proceeds from long-term borrowings
Repayments of long-term borrowings Interest paid Dividend paid Net cash from
Financing Activities Net increase (decrease) in Cash and Cash Equivalent Cash and
Cash Equivalents at beginning of period Cash and Cash Equivalent at end of period
Cash Flow Statement (Indirect Method) Cash Flow from Operating Activities Net
profit before tax and extraordinary items Adjustments for: - Depreciation -
Foreign exchange - Investments - Gain or loss on sale of fixed assets -
Interest/dividend Operating profit before working capital changes Adjustments for:
- Trade and other receivables

(a) (xxx) xxx xxx xxx (b) xxx xxx (xxx) (xxx) (xxx) (c) (a+b+c)

xxx

xxx
Rs.

xxx xxx xxx xxx


(Rs.)

xxx xxx xxx xxx (xxx) xxx xxx xxx

3.52
Financial Analysis and Planning

- Inventories - Trade payable Cash generation from operations - Interest paid -


Direct Taxes Cash before extraordinary items Deferred revenue Net cash from
Operating Activities Cash Flow from Investing Activities Purchase of fixed assets
Sale of fixed assets Purchase of investments Interest received Dividend received
Loans to subsidiaries Net cash from Investing Activities Cash Flow from Financing
Activities Proceeds from issue of share capital Proceeds from long term borrowings
Repayment to finance/lease liabilities Dividend paid Net cash from Financing
Activities Net increase (decrease) in Cash and Cash Equivalents Cash and Cash
Equivalents at the beginning of the year Cash and Cash Equivalents at the end of
the year (c) (a+b+c) (b) (a)

(xxx) xxx xxx (xxx) (xxx) xxx xxx xxx (xxx) xxx xxx (xxx) xxx xxx xxx xxx xxx
(xxx) (xxx) xxx xxx xxx xxx

3.53
Financial Management

Cash from Operations

(Rs.)

Funds from Operations


Add:

xxx (excluding Bank Overdraft) (excluding cash & bank balance) (excluding cash &
bank balance) (excluding bank overdraft) xxx xxx xxx xxx xxx xxx xxx

Increase in Current Liabilities Decrease in Current Assets

Less:

Increase in Current Assets Decrease Liabilities in Current

Cash from Operations


2.9 FUNDS FLOW STATEMENT VERSUS CASH FLOW STATEMENT

Both funds flow and cash flow statements are used in analysis of past transactions
of a business firm. The difference between these two statements is given below:
Funds flow statement is based on the accrual accounting system. In case of
preparation of cash flow statements all transactions effecting the cash or cash
equivalents only is taken into consideration. Funds flow statement analyses the
sources and application of funds of long-term nature and the net increase or
decrease in long-term funds will be reflected on the working capital of the firm.
The cash flow statement will only consider the increase or decrease in current
assets and current liabilities in calculating the cash flow of funds from
operations. Funds Flow analysis is more useful for long range financial planning.
Cash flow analysis is more useful for identifying and correcting the current
liquidity problems of the firm. Funds flow statement tallies the funds generated
from various sources with various uses to which they are put. Cash flow statement
starts with the opening balance of cash and reach to the closing balance of cash
by proceeding through sources and uses. The concept and technique of preparing a
Cash Flow Statement will be clear with the help of illustrations given in the
following pages.
Illustration 1: From the following information prepare a Cash Flow Statement
according to (a) Direct Method (b) Indirect Method as per AS-3 (Revised). Working
notes would from part of your answer

3.54
Financial Analysis and Planning

(1)

BALANCE SHEET AS ON 31.12. 2005 (Rs. in ‘000) 2005 2004 25 135 1,200 -1,950 2,500
1,910 (1,060) 730 6,800 2005 850 6,660 2004

Assets

Cash on hand and balances with banks Short-term investments Sundry debtors
Interest receivable Inventories Long-term investments Fixed assets at cost
Less: Accumulated depreciation 2,180 (1,450)

200 670 1,700 100 900 2,500

Fixed assets (net)


Total Assets

Liabilities

Sundry creditors Interest payable Income taxes payable Long-term debt


Total liabilities

150 230 400 1,110 1,890 1,500 3,410 4,910 6,800

1,890 100 1,000 1,040 4,030 1,250 1,380 2,630 6,660

Shareholders’ funds Share capital Reserves Total shareholders’ funds


Total Liabilities and Shareholders’ funds

3.55
Financial Management

(2)

STATEMENT OF PROFIT AND LOSS for the period ended 31.12. 2005 (Rs. in ‘000)

Sales Cost of sales Gross profit Depreciation Administrative and selling expenses
Interest expense Interest income Dividend income Foreign exchange loss Net profit
before taxation and extraordinary item Extraordinary itemInsurance proceeds from
earthquake disaster settlement Net profit after extraordinary item Income tax Net
Profit Additional Information: (Figures in Rs. ‘000).

30,650 (26,000) 4,650 (450) (910) (400) 300 200 (40) 3,350 180 3,530 (300) 3,230

(a) An amount of 250 was raised from the issue of share capital and a further 250
was raised from long-term borrowings. (b) Interest expense was 400 of which 170
was paid during the period. 100 relating to interest expense of the prior period
was also paid during the period. (c) Dividends paid were 1,200. (d) Tax deducted
at source on dividends received (included in the tax expense of 300 for the year)
amounted to 40. (e) During the period, the enterprise acquired fixed assets for
350. The payment was made in cash. (f) Plant with original cost of 80 and
accumulated depreciation of 60 was sold for 20. (g) Foreign exchange loss of 40
represents the reduction in the carrying amount of a short-

3.56
Financial Analysis and Planning

term investment in foreign currency designated bonds arising out of a change in


exchange rate between the date of acquisition of the investment and the balance
sheet date. (h) Sundry debtors and sundry creditors include amounts relating to
credit sales and credit purchases only.
Solution CASH FLOW STATEMENT (Direct Method) (Rs. in ‘000) 2005 Cash flows from
operating activities Cash receipts from customers Cash paid to suppliers and
employees Cash generated from operations Income taxes paid Cash flow before
extraordinary item Proceeds from earthquake disaster settlement Net cash from
operating activities Cash flows from investing activities Purchase of fixed assets
Proceeds from sale of equipment Interest received Dividend received Net cash from
investing activities Cash Flows from financing activities Proceeds from issuance
of share capital Proceeds from long-term borrowings Repayments of long-term
borrowings Interest paid Dividend paid Net cash used in financing activities Net
increase in cash and cash equivalents Cash and cash equivalents at beginning of
period (See Note 1) Cash and cash equivalents at end of period (See Note 1) Notes
to the Cash Flow Statement (Direct & Indirect Method)

30,150 (27,600) 2,550 (860) 1,690 180 1,870 (350) 20 200 160 30 250 250 (180)
(270) (1,200) (1,150) 750 160 910

3.57
Financial Management

1. Cash and cash equivalents: Cash and cash equivalents consist of cash on hand
and balances with banks, and investments in money-market instruments. Cash and
cash equivalents included in the cash flow statement comprise the following
balance sheet amounts. 2005 2004

Cash on hand and balances with banks Short-term investments Cash and cash
equivalents Effects of exchange rate changes Cash and cash equivalents as restated

200 670 870 40 910

25 135 160 -160

Cash and cash equivalents at the end of the period include deposits with banks of
100 held by a branch which are not freely permissible to the company because of
currency exchange restrictions. The company has undrawn borrowing facilities of
2,000 of which 700 may be used only for future expansion. 2. Total tax paid during
the year (including tax deducted at source on dividends received) amounted to 900.
CASH FLOW STATEMENT (Indirect Method) (Rs. in ‘000) 2005 Cash flows from operating
activities

Net profit before taxation, and extraordinary item Adjustments for: Depreciation
Foreign exchange loss Interest income Dividend income Interest expense 450 40
(300) (200) 400

3,350

3.58
Financial Analysis and Planning

Operating profit before working capital changes Increase in sundry debtors


Decrease in inventories Decrease in sundry creditors Cash generated from
operations Income taxes paid Cash flows before extraordinary item Proceeds from
earthquake disaster settlement
Net cash from operating activities Cash flows from investing activities

3,740 (500) 1,050 (1,740) 2,550 (860) 1,690 180 1,870 (350) 20 200 160 30 250 250
(180) (270) (1,200) (1,150) 750 160 910

Purchase of fixed assets Proceeds from sale of equipment Interest received


Dividends received
Net cash from investing activities Cash flows from financing activities

Proceeds from issuance of share capital Proceeds from long-term borrowings


Repayment of long-term borrowings Interest paid Dividends paid
Net cash used in financing activities

Net increase in cash and cash equivalents


Cash and cash equivalents at beginning of period (See Note 1) Cash and cash
equivalents at end of period (See Note 1)

3.59
Financial Management

Alternative Presentation (Indirect Method)

As an alternative, in an indirect method cash flow statement, operating profit


before working capital changes is sometimes presented as follows: Revenues
excluding investment income Operating expenses excluding depreciation Operating
profit before working capital changes
Working Notes:

30,650 (26,910) 3,740

The working notes given below do not form part of the cash flow statement. The
purpose of these working notes is merely to assist in understanding the manner in
which various figures in the cash flow statement have been derived. (Figures are
in Rs.’000). 1.
Cash receipts from customers

Sales
Add: Sundry debtors at the beginning of the year Less: Sundry debtors at the end
of the year

30,650 1,200 31,850 1,700 30,150

2.

Cash paid to suppliers and employees

Cost of sales Administrative & selling expenses

26,000 910 26,910

Add: Sundry creditors at the beginning of the year

1,890 900 150 1,950 2,100 27,600 2,790 29,700

Inventories at the end of the year


Less: Sundry creditors at the end of the year

Inventories at the beginning of the year

3.60
Financial Analysis and Planning

3.

Income taxes paid (including tax deducted at source from dividends received)

Income tax expense for the year (including tax deducted at source from dividends
received
Add: Income tax liability at the beginning of the year Less: Income tax liability
at the end of the year

300

1,000 1,300 400 900

Out of 900, tax deducted at source on dividends received (amounting to 40), is


included in cash flows from investing activities and the balance of 860 is
included in cash flows from operating activities. 4.
Repayment of long-term borrowings

Long-term debt at the beginning of the year


Add: Long-term borrowings made during the year Less: Long-term borrowings at the
end of the year

1,040 250 1,290 1,110 180

5.

Interest paid

Interest expense for the year


Add: Interest payable at the beginning of the year Less: Interest payable at the
end of the year

400 100 500 230 270

3.61
Financial Management

Illustration 2: Swastik Oils Ltd. has furnished the following information for the
year ended 31st March, 2006: (Rs. in lakhs)

Net profit Dividend (including interim dividend paid) Provision for income-tax
Income-tax paid during the year Loss on sale of assets (net) Book value of assets
sold Depreciation charged to P&L Account Profit on sale of investments Interest
income on investments Value of investments sold Interest expenses Interest paid
during the year Increase in working capital (excluding cash and bank balance)
Purchase of fixed assets Investments on joint venture Expenditure on construction
work-in-progress Proceeds from long-term borrowings Proceeds from short-term
borrowings Opening cash and bank balances Closing cash and bank balances

37,500.00 12,000.00 7,500.00 6,372.00 60.00 277.50 30,000.00 150.00 41,647.50


3,759.00 15,000.00 15,780.00 84,112.50 21,840.00 5,775.00 69,480.00 38,970.00
30,862.50 11,032.50 2,569.50

You are required to prepare the cash flow statement in accordance with AS-3 for
the year ended 31st March, 2006. (Make assumptions wherever necessary).

3.62
Financial Analysis and Planning

Solution SWASTIK OILS LIMITED Cash Flow Statement for the Year Ended 31st March,
2006 Cash Flows from Operating Activities (Rs. in lakhs)

Net profit before taxation (37,500 + 7,500) Adjustment for: Depreciation charged
to P&L A/c Loss on sale of assets (net) Profit on sale of investments Interest
income on investments Interest expenses Operating profit before working capital
changes Increase (change) in working capital (excluding cash balance) Cash
generated from operations Income tax paid Net cash used in operating activities
(A)
(b) Cash Flow from investing Activities

45,000.00 30,000.00 60.00 (150.00) (3,759.00) 15,000.00 86,151.00 and bank


(84,112.50) 2,038.50 (6,372.00) (4,333.50) 217.50 41,797.50 3,759.00 (21,840.00)
(5,775.00) (69,480.00) (51,321.00) 38,970.00 30,862.50 (15,780.00) (12,000.00)

Sale of Assets (277.50-60.00) Sale of Investments (41,647.50+150) Interest Income


on investments (assumed) Purchase of fixed assets Investments in Joint Venture
Expenditure on construction work-in-progress Net Cash used in investing activities
(B)
(c) Cash Flow from Financing Activities

Proceeds from long-term borrowings Proceeds from short-term borrowings Interest


paid Dividends (including interim dividend paid)

3.63
Financial Management

Net cash from financing activities (C) Net increase in cash and cash equivalents
(A) + (B) + (C) Cash and cash equivalents at the beginning of the year Cash and
cash equivalents at the end of the year
Self Examination Questions A. Objective Type Questions

42,052.50 (13,602.00) 11,032.50 2,569.50

1.

The term cash includes (a) Cash and Bank Balances (b) All the Current Assets (c)
All the Current Liabilities (d) None of the above.

2.

“Cash flow statement reveals the effects of transactions involving movement of


cash”. This statement is (a) Correct (b) Incorrect (c) Partially Correct (d)
Irrelevant.

3.

The Preparation of Cash flow statement is governed by AS-3 (Revised). This


statement is (a) False (b) True (c) Partially true (d) Cannot say.

4.

A cash flow statement is like an income statement (a) I agree (b) I disagree

3.64
Financial Analysis and Planning

(c) I cannot say (d) The statement is ambiguous. 5. Funds flow statement and cash
flow statement are one and the same (a) True (b) False (c) I cannot say (d) The
statement is irrelevant. 6. Increase in the amount of bills payable results in (a)
Increase in cash (b) Decrease in cash (c) No change in cash (d) I cannot say. 7.
Cash from operations is equal to (a) Net profit plus increase in outstanding
expenses (b) Net profit plus increase in debtors (c) Net profit plus increase in
stock (d) None of the above. 8. Cash equivalents are short term highly liquid
investments that are readily convertible into known amounts of cash and which are
subject to an insignificant risk of changes in value. (a) I agree (b) I do not
agree (c) I cannot say (d) Irrelevant. 9. For an investment to qualify as a cash
equivalent, it must be readily convertible to a known amount of (a) Gold (b) Cash

3.65
Financial Management

(c) Investment (d) Real estate. 10. Non-cash transactions (a) Form part of cash
flow statement (b) Do not form part of cash flow statement (c) May or may not form
part of cash flow statement (d) I cannot say whether they are part of cash flow
statement.
Answers to Objective Type Questions 1. (a); 2. (a); 3. (b) 4. (b); 5. (b); 6. (a);
7. (a); 8. (a); 9. (b); 10. (a);

B. Long Answer Type Questions

1. 2. 3. 4. 5.

What is a cash flow statement? Discuss briefly its major classification.


Distinguish between Funds flow statement and Cash flow statement. Outline the
Importance of cash flow statement in managing cash flows of a business
organization. Explain the technique of preparing a cash flow statement with
imaginary figures. Explain the difference between direct and indirect methods of
reporting cash flows from operating activities as per AS-3 (Revised).

C. Practical Problems 1.

From the following Summary Cash Account of X Ltd. prepare Cash Flow Statement for
the year ended 31st March, 2006 in accordance with AS-3 (Revised) using the direct
method. The Company does not have any cash equivalents.
Summary Cash Account for the year ended 31.3.2006

Balance on 1-4-2005 Issue of Equity Share Receipts from Customers Sale of Fixed
Assets

50 Payment of Suppliers 300 Purchase of Fixed Assets 2,800 Overhead expense 100
Wags and Salaries Taxation

2,000 200 200 100 250

3.66
Financial Analysis and Planning

Dividend Repayment of Bank Loan Balance on 31-3-2006 3,250


2.

50 300 150 3,250

Ms. Jyoti of Star Oils Limited has collected the following information for the
preparation of cash flow statement for the year 2005:
(Rs. in lakhs)

Net Profit Dividend (including dividend tax) paid Provision for Income-tax Income-
tax paid during the year Loss on sale of assets (net) Book value of the assets
sold Depreciation charged to Profit & Loss Account Amortisation of Capital grant
Profit on sale of Investments Carrying amount of Investment sold Interest income
on investments Interest expenses Interest paid during the year Increase in Working
Capital (excluding Cash & Bank balance) Purchase of fixed assets Investment in
joint venture Expenditure on construction work-in-progress Proceeds from calls in
arrear Receipt of grant for capital projects

25,000 8,535 5,000 4,248 40 185 20,000 6 100 27,765 2,506 10,000 10,520 56,075
14,560 3,850 34,740 2 12

3.67
Financial Management

Proceeds from long-term borrowings Proceeds from short-term borrowings Opening


cash and Bank balance Closing Cash and Bank balance

25,980 20,575 5,003 6,988

Required: Prepare the Cash Flow Statement for the year 2005 in accordance with AS-
3, Cash Flow Statements issued by the Institute of Chartered Accountants of India.
(Make necessary assumption). 3.

The summarized Balance Sheets of XYZ Ltd. as at 31st December, 2004 and 2005 are
given below:
(Rs.) Particulars Liabilities 2004 2005

Share capital General Reserve Profit and Loss account Creditors Provision for tax
Mortgage loan

4,50,000 3,00,000 56,000 1,68,000 75,000 --10,49,000

4,50,000 3,10,000 68,000 1,34,000 10,000 2,70,000 12,42,000

Assets

Fixed assets Investments Stock Debtors Bank

4,00,000 50,000 2,40,000 2,10,000 1,49,000 10,49,000

3,20,000 60,000 2,10,000 4,55,000 1,97,000 12,42,000

3.68
Financial Analysis and Planning

Additional information:

(a) Investments costing Rs.8,000 were sold during the year 2005 for Rs.8,500. (b)
Provision for tax made during the year was Rs.9,000. (c) During the year, part of
the fixed assets costing Rs.10,000 was sold for Rs.12,000 and the profit was
included in profit and loss account. (d) Dividend paid during the year amounted to
Rs.40,000. You are required to prepare a Statement of Sources and Uses of cash. 4.
The following are the changes in the account balances taken from the Balance
Sheets of P Q Ltd. as at the beginning and end of the year:
Changes in Rupees in debit or credit

Equity share capital 30,000 shares of Rs.10 each issued and fully paid Capital
reserve 8% debentures Debenture discount Freehold property at cost/revaluation
Plant and machinery at cost Depreciation on plant and machinery Debtors Stock and
work-in-progress Creditors Net profit for the year Dividend paid in respect of
earlier year Provision for doubtful debts Trade investments at cost Bank You are
informed that:

0 (49,200) (50,000) 1,000 43,000 60,000 (14,400) 50,000 38,500 (11,800) (76,500)
30,000 (3,300) 47,000 (64,300) 0

(a) Capital reserve as at the end of the year represented realized profits on sale
of one freehold property together with surplus arising on the revaluation of
balance of freehold properties.

3.69
Financial Management

(b) During the year plant costing Rs.18,000 against which depreciation provision
of Rs.13,500 was lying was sold for Rs.7,000. (c) During the middle of the year
Rs.50,000 debentures were issued for cash at a discount of Rs.1,000. (d) The net
profit for the year was after crediting the profit on sale of plant and charging
debenture interest. You are required to prepare a cash flow statement which will
explain, why bank borrowing has increased by Rs.64,300 during the year end. Ignore
taxation.
5.

Given below are the condensed Balance Sheets of M.M. Kusha Ltd. for two years and
the statement of Profit and Loss for one year:
(Rs. lakhs)

As at 31st March Share Capital In Equity shares of Rs.100 each 10% Redeemable
Preference Shares of Rs.100 each Capital Redemption Reserve General Reserve Profit
and Loss Account balance 8% Debentures with Convertible Option Other Term Loans
Fixed assets less Depreciation Long-Term Investments Working Capital

2005 150 10 10 15 30 20 15 250 130 40 80 250

2004 110 40 -10 20 40 30 250 100 50 100 250

Statement of Profit and Loss for the year ended 31st March, 2005 (Rs. lakhs)

Sales Less: Cost of sale Establishment charges 30

600 400 200

3.70
Financial Analysis and Planning

Selling and distribution expenses Interest expenses Loss on sale of equipment


(Book value Rs.40 lakhs) Interest income Dividend income Foreign exchange gain
Damages received for loss of reputation Depreciation Taxation Dividends Net Profit
carried to Balance Sheet

60 5 15 4 2 10 14

110 90

30 120 50 70 30 40 15 25

You are informed by the accountant that ledgers relating to debtors, creditors and
stock for both the years were seized by the income-tax authorities and it would
take at least two months to obtain copies of the same. However, he is able to
furnish the following data:
(Rs. lakhs)

Particulars Dividend receivable Interest receivable Cash on hand and with bank
Investments maturing within two months Interest payable Taxes payable Current
ratio Acid test ratio

2005 2 3 7 3 15 4 6 10 1.5 1.1

2004 4 2 10 2 18 5 3 8 1.4 0.8

3.71
Financial Management

It is also gathered that debenture-holders owning 50% of the debentures


outstanding as on 31-3-2004 exercised the option for conversion into equity shares
during the financial year and the same was put through. You are required to
prepare a direct method cash flow statement for the financial year, 2005 in
accordance with Accounting Standard (AS 3) revised.

3.72
CHAPTER

FINANCING DECISIONS
UNIT – I : COST OF CAPITAL Learning Objectives After studying this unit you will
be able to learn ♦ ♦ ♦ ♦ 1.1 What is cost of capital? How to measure the cost of
each component of capital? What is weighted average cost of capital (WACC) and
marginal cost of capital? and; How cost of capital is important in financial
management? INTRODUCTION

The financing decision relates to the composition of relative proportion of


various sources of finance. The financial management weighs the merits and
demerits of different sources of finance while taking the financing decision. A
business can be financed from either the shareholders funds or borrowings from
outside agencies. The shareholders funds include equity share capital, preference
share capital and the accumulated profits whereas borrowings from outsiders
include borrowed funds like debentures and loans from financial institutions. The
borrowed funds have to be paid back with interest and some amount of risk is
involved if the principal and interest is not paid. Equity has no fixed commitment
regarding payment of dividends or principal amount and therefore, no risk is
involved. It is the decision of the business to decide the ratio of borrowed funds
and owned funds. However, most of the companies use a combination of both the
shareholders funds and borrowed funds. Whether the companies choose shareholders
funds or borrowed funds, each type of fund carries a cost. Borrowed funds involve
interest payment whereas equities, as such do not have any fixed obligation but
definitely they involve a cost. The cost of equity is the minimum return the
shareholders would have received if they had invested elsewhere. Both types of
funds incur cost and this is the cost of capital to the company. This means, cost
of capital is the minimum return expected by the company. The financing decision
is an important managerial decision. It influences the shareholder’s return and
risk. As a result, the market value of the share may be affected by the financing
decision. Subsequently, whenever funds are to be raised to finance investments,
capital
Financial Management

structure decision is involved. The process of financing or capital structure


decision is depicted in the figure below. A demand for raising funds generates a
new capital structure since a decision has to be made as to the quantity and forms
of financing. This decision will involve an analysis of the existing capital
structure and the factors governing the decision. The company’s policy to retain
or distribute earnings affects the shareholders claims. Retention of earnings
strengthens the shareholders position. The debt-equity mix of the company is also
affected by the new financing decision of the company. The financing mix (debt-
equity mix) has implications for the shareholders’ earnings and risk, which in
turn, will affect the cost of capital and the market value of the firm.

Financing Decision Process

4.2
Financing Decisions

1.2

DEFINITION OF COST OF CAPITAL

Cost of capital maybe defined as the cut off rate for determining estimated future
cash proceeds of a project and eventually deciding whether the project is worth
undertaking or not. It is also the minimum rate of return that a firm must earn on
its investment which will maintain the market value of share at its current level.
It can also be stated as the opportunity cost of an investment, i.e. the rate of
return that a company would otherwise be able to earn at the same risk level as
the investment that has been selected. For example, when an investor purchases
stock in a company, he/she expects to see a return on that investment. Since the
individual expects to get back more than his/her initial investment, the cost of
capital is equal to this return that the investor receives, or the money that the
company misses out on by selling its stock. It can also be said as the required
return necessary to make a capital budgeting project - such as building a new
factory - worthwhile. Cost of capital includes the cost of debt and the cost of
equity. The cost of capital determines how a company can raise money maybe through
a stock issue, borrowing, or a mix of the two. This is the rate of return that a
firm would receive if it invested its money someplace else with similar risk.
Another way to think of the cost of capital is as the opportunity cost of funds,
since this represents the opportunity cost for investing in assets with the same
risk as the firm. When investors are shopping for places in which to invest their
funds, they have an opportunity cost. The firm, given its riskiness, must strive
to earn the investor’s opportunity cost. If the firm does not achieve the return
investors expect (i.e. the investor’s opportunity cost), investors will not invest
in the firm’s debt and equity. As a result, the firm’s value (both their debt and
equity) will decline. The total capital for a firm is the value of its equity plus
the cost of its debt (the cost of debt should be continually updated as the cost
of debt changes as a result of interest rate changes). The cost of capital is
given as: Kc= (1-δ)Ke+ δKd Where, Kc δ Ke = = = Weighted cost of capital for the
firm Debt to capital ratio, D / (D + E) Cost of equity

4.3
Financial Management

Kd D E 1.3

= = =

After tax cost of debt Market value of the firm's debt, including bank loans and
leases Market value of all equity (including warrants, options, and the equity
portion of convertible securities)

MEASUREMENT OF COST OF CAPITAL

The cost of capital is useful in determining a financial plan. A company has to


employ a combination of creditor’s and owner’s funds. As more than one type of
capital is used in a company, the composite cost of capital can be determined
after the cost of each type of funds has been obtained. The first step is,
therefore, the calculation of specific cost which is the minimum financial
obligation required to secure the use of capital for a particular source. In order
to calculate the specific cost of each type of capital, recognition should be
given to the explicit and the implicit cost. The explicit cost of any source of
capital may be defined as the discount rate that equals that present value of the
cash inflows that are incremental to the taking of financing opportunity with the
present value of its incremental cash outflows. Implicit cost is the rate of
return associated with the best investment opportunity for the firm and its
shareholders that will be foregone if the project presently under consideration by
the firm was accepted. The explicit cost arises when funds are raised and when
funds are used, implicit cost arises. For capital budgeting decisions, cost of
capital is nothing but the explicit cost of capital. Now the different components
of cost of capital i.e. each source of finance have been discussed in detail.
1.3.1 COST OF DEBT A bond is a long term debt instrument or security. Bonds issued
by the government do not have any risk of default. The government honour
obligations on its bonds. Bonds of the public sector companies in India are
generally secured, but they are not free from the risk of default. The private
sector companies also issue bonds, which are also called debentures in India. A
company in India can issue secured or unsecured debentures. In the case of a bond
or debenture, the rate of interest is generally fixed and known to investors. The
principal of a redeemable bond or bond with a maturity is payable after a
specified period, called maturity period. The chief characteristics of a bond or
debenture are as follows: Face value: Face value is called par value. A bond or
debenture is generally issued at a par value of Rs. 100 or Rs. 1,000, and interest
is paid on face value.

4.4
Financing Decisions

Interest rate: Interest rate is fixed and known to bondholders or debenture


holders. Interest paid on a bond or debenture is tax deductible. The interest rate
is also called coupon rate. Coupons are detachable certificates of interest.
Maturity: A bond or debenture is generally issued for a specified period of time.
It is repaid on maturity. Redemption value: The value that a bondholder or
debenture holder will get on maturity is called redemption or maturity value. A
bond or debenture may be redeemed at par or at premium (more than par value) or at
discount (less than par value). Market value: A bond or debenture may be traded in
a stock exchange. The price at which it is currently sold or bought is called the
market value of the bond or debenture. Market value may be different from par
value or redemption value. 1.3.1.1 Cost of Debentures: The cost of debentures and
long term loans is the contractual interest rate adjusted further for the tax
liability of the company. When the firm employs debt, it must ensure that common
shareholder’s earnings are not diluted. To keep the earnings unchanged, the firm
must earn a return equal to the interest rate of debt. If the firm earns less than
the interest rate, market share price would be adversely affected. In calculating
weighted (average) cost of capital, cost of debt (after tax) should be used. For a
company, the higher the interest charges, the lower the amount of tax payable by
the company. An illustration will help you in understanding this point.
Illustration 1: Consider two companies X and Y: Company X Earnings before interest
and taxes (EBIT) Interest (I) Profit before tax (PBT) Tax (T) 1 Profit after tax
(PAT) Assume an effective rate of tax of 35 percent Solution A comparison of the
two companies shows that an interest payment of 40 in company Y results in a tax
shield of 14 - that is 40 multiplied by 0.35, the corporate tax rate. The
important point to remember, while calculating the average cost of capital, is
that the posttax cost of debt must be used and not the pre-tax cost of debt.
4.5

Company Y 100 40 60 21 39

100 100 35 65
Financial Management

1.3.1.1.1 Cost of Irredeemable Debentures: Cost of debentures not redeemable


during the life time of the company.

Kd =
Where, Kd I t = = =

1 (1 − t ) NP

Cost of debt after tax Annual interest rate Net proceeds of debentures Tax rate

NP =

Suppose a company issues 1,000, 15% debentures of the face value of Rs. 100 each
at a discount of Rs. 5. Suppose further, that the under-writing and other costs
are Rs. 5,000/- for the total issue. Thus Rs. 90,000 is actually realised, i.e.,
Rs. 1,00,000 minus Rs. 5,000 as discount and Rs. 5,000 as under-writing expenses.
The interest per annum of Rs. 15,000 is therefore the cost of Rs. 90,000, actually
received by the company. This is because interest is a charge on profit and every
year the company will save Rs. 7,500 as tax, assuming that the income tax rate is
50%. Hence the after tax cost of Rs. 90,000 is Rs. 7,500 which comes to 8.33%.
1.3.1.1.2 Cost of Redeemable Debentures: If the debentures are redeemable after
the expiry of a fixed period, the cost of debentures would be:
Kd = I(I − t ) + (RV − NP) / N RV + NP 2

Where,

I NP t N

= = = =

Annual interest payment Net proceeds of debentures Redemption value of debentures


Tax rate Life of debentures.

RV =

Illustration 2: A company issued 10,000, 10% debentures of Rs. 100 each on


1.4.2006 to be matured on 1.4.2011. If the market price of the debentures is Rs.
80. Compute the cost of debt assuming 35% tax rate.

4.6
Financing Decisions

Solution

Kd =

I (1 − t) +

RV − NP N RV + NP 2

⎛ 100 − 80 ⎞ 10 (1 − .35) + ⎜ ⎟ 5⎠ ⎝ Kd = 100 + 80 2

6.5 + 4 90

= 0.1166 = 0.12 1.3.1.2 Value of Bonds: It is comparatively easy to find out the
present value of a bond since its cash flows and the discount rate can be
determined easily. If there is no risk of default, then there is no difficulty in
calculating the cash flows associated with a bond. The expected cash flows consist
of annual interest payments plus repayment of principal. The appropriate
capitalisation or discount rate would depend upon the risk of the bond. The risk
in holding a government bond is less than the risk associated with a debenture
issued by a company. Therefore, a lower discount rate would be applied to the cash
flows of the government bond and a higher rate to the cash flows of the company
debenture. 1.3.1.2.1 Amortisation of Bond: A bond may be amortised every year i.e.
principal is repaid every year rather than at maturity. In such a situation, the
principal will go down with annual payments and interest will be computed on the
outstanding amount. The cash flows of the bonds will be uneven. The formula for
determining the value of a bond or debenture that is amortised every year is as
follows:

VB =

Cn C2 C1 + + ......... + 2 1 (1 + k d ) n (1 + k d ) (1 + k d )
n t =1

VB = ∑

Ct (1 + k d ) t

Illustration 3: Reserve Bank of India is proposing to sell a 5-year bond of Rs.


5,000 at 8 per cent rate of interest per annum. The bond amount will be amortised
equally over its life. What
4.7
Financial Management

is the bond’s present value for an investor if he expects a minimum rate of return
of 6 per cent? Solution The amount of interest will go on declining as the
outstanding amount of bond will be reducing due to amortisation. The amount of
interest for five years will be: First year: Second year: Third year: Fourth year:
Fifth year: Rs. 5,000 × 0.08 = Rs. 400; (Rs. 5,000 – Rs. 1,000) × 0.08 = Rs. 320;
(Rs. 5,000 – Rs. 1,000) × 0.08 = Rs. 240; (Rs. 5,000 – Rs. 1,000) × 0.08 = Rs.
160; and (Rs. 5,000– Rs.1,000) × 0.08 = Rs. 108.

The outstanding amount of bond will be zero at the end of fifth year. Since
Reserve Bank of India will have to return Rs. 1,000 every year, the outflows every
year will consist of interest payment and repayment of principal: First year:
Third year: Fourth year: Fifth year: Rs. 1000 + Rs. 400 = Rs. 1,400; Rs. 1000 +
Rs. 240 = Rs. 1,240; Rs. 1000 + Rs. 160 = Rs. 1,160; and Rs. 1000 + Rs. 108 = Rs.
1,108. Second year: Rs. 1000 + Rs. 320 = Rs. 1,320;

Referring to the present value table at the end of the study material, the value
of the bond is calculated as follows:
VB = 1,400 1,320 1,240 1,160 1,080 + + + + 1 2 3 4 (1.06) (1.06) (1.06) (1.06)
(1.06) 5

= 1,400 × 0.943 + 1,320 × 0.890 + 1,240 × 0.840 + 1,160 × 0.792 + 1,080 × 0.747 =
1,320.20 + 1,174.80 + 1,041.60 + 918.72 + 806.76 = Rs. 5,262.08 1.3.2 COST OF
PREFERENCE SHARES The cost of preference share capital is the dividend expected by
its holders. Though payment of dividend is not mandatory, non-payment may result
in exercise of voting rights by them. The payment of preference dividend is not
adjusted for taxes as they are paid after taxes and

4.8
Financing Decisions

is not deductible. The cost of preference share capital is calculated by dividing


the fixed dividend per share by the price per preference share. Illustration 4: If
Reliance Energy is issuing preferred stock at Rs.100 per share, with a stated
dividend of Rs.12, and a floatation cost of 3% then, what is the cost of
preference share? Solution
Kp = Pr eferred stock dividend Market price of preferred stock (1 − floatation cos
t )

Rs.12 = 12.4% Rs.100(1 − 0.03)

1.3.2.1 Cost of Irredeemable Preference Shares: Cost of irredeemable preference


shares PD = PO Where, PD = Annual preference dividend PO = Net proceeds in issue
of preference shares. Illustration 5: XYZ & Co. issues 2,000 10% preference shares
of Rs. 100 each at Rs. 95 each. Calculate the cost of preference shares. Solution

Kp =
Kp =

PD PO

(10 × 2,000 ) (95 × 2,000 )


10 95

= 0.1053 1.3.2.2 Cost of Redeemable Preference Shares: If the preference shares


are redeemable after the expiry of a fixed period the cost of preference shares
would be:

4.9
Financial Management

Kp =

PD + (RV − NP) / N RV + NP 2

Where, PD = RV = NP = N = Annual preference dividend Redemption value of


preference shares Net proceeds on issue of preference shares Life of preference
shares.

However, since dividend of preference shares is not allowed as deduction from


income for income tax purposes, there is no question of tax advantage in the case
of cost of preference shares. It would, thus, be seen that both in the case of
debt as well as preference shares, cost of capital is calculated by reference to
the obligations incurred and proceeds received. The net proceeds received must be
taken into account in working out the cost of capital. Illustration 6: Referring
to the earlier question but taking into consideration that if the company proposes
to redeem the preference shares at the end of 10th year from the date of issue.
Calculate the cost of preference share? Solution
Kp = PD + (RV − NP) / N RV + NP 2

⎛ 100 − 95 ⎞ 10 + ⎜ ⎟ ⎝ 10 ⎠ Kp = ⎛ 100 + 95 ⎞ ⎜ ⎟ 2⎠ ⎝
1.3.3 COST OF EQUITY

= .107 (approx.)

It may prima facie appear that equity capital does not carry any cost. But this is
not true. The market share price is a function of return that equity shareholders
expect and get. If the company does not meet their requirements, it will have an
adverse effect on the market share price. Also, it is relatively the highest cost
of capital. Since expectations of equity holders are high, higher cost is
associated with it.

4.10
Financing Decisions

Cost of equity capital is the rate of return which equates the present value of
expected dividends with the market share price. The calculation of equity capital
cost raises a lot of problems. Its purpose is to enable the corporate manager, to
make decisions in the best interest of equity holders. In theory the management
strives to maximize the position of equity holders and the effort involves many
decisions. Different methods are employed to compute the cost of equity capital.
(a) Dividend Price Approach: Here, cost of equity capital is computed by dividing
the current dividend by average market price per share. This dividend price ratio
expresses the cost of equity capital in relation to what yield the company should
pay to attract investors. However, this method cannot be used to calculate cost of
equity of units suffering losses.

Ke =
Where,

D1 Po

Ke = Cost of equity D1 = Annual dividend Po = Market value of equity (ex dividend)


This model assumes that dividends are paid at a constant rate to perpetuity. It
ignores taxation. (b) Earning/ Price Approach: The advocates of this approach co-
relate the earnings of the company with the market price of its share.
Accordingly, the cost of ordinary share capital would be based upon the expected
rate of earnings of a company. The argument is that each investor expects a
certain amount of earnings, whether distributed or not from the company in whose
shares he invests.

Thus, if an investor expects that the company in which he is going to subscribe


for shares should have at least a 20% rate of earning, the cost of ordinary share
capital can be construed on this basis. Suppose the company is expected to earn
30% the investor will be ⎞ ⎛ 30 prepared to pay Rs. 150 ⎜ Rs. ×100 ⎟ for each
share of Rs. 100. This approach is similar to ⎠ ⎝ 20 the dividend price approach;
only it seeks to nullify the effect of changes in the dividend policy. This
approach also does not seem to be a complete answer to the problem of determining
the cost of ordinary share since it ignores the factor of capital appreciation or
depreciation in the market value of shares.
(c) Dividend Price + Growth Approach: Earnings and dividends do not remain
constant and the price of equity shares is also directly influenced by the growth
rate in dividends.
4.11
Financial Management

Where earnings, dividends and equity share price all grow at the same rate, the
cost of equity capital may be computed as follows: Ke = (D/P) + G Where, D =
Current dividend per share P = Market price per share G = Annual growth rate of
earnings of dividend. Illustration 7: A company has paid dividend of Rs. 1 per
share (of face value of Rs. 10 each) last year and it is expected to grow @ 10%
next year. Calculate the cost of equity if the market price of share is Rs. 55.
Solution

Ke = =

D +G P 1 (1 + .10) + .10 55

= .1202 (approx.)
(d) Earnings Price + Growth Approach: This approach is an improvement over the
earlier methods. But even this method assumes that dividend will increase at the
same rate as earnings, and the equity share price is the regulator of this growth
as deemed by the investor. However, in actual practice, rate of dividend is
recommended by the Board of Directors and shareholders cannot change it. Thus,
rate of growth of dividend subsequently depends on director’s attitude. The
dividend method should, therefore, be modified by substituting earnings for
dividends. So, cost of equity will be given by: Ke = (E/P) + G Where, E = Current
earnings per share P = Market share price G = Annual growth rate of earnings. The
calculation of ‘G’ (the growth rate) is an important factor in calculating cost of
equity capital. The past trend in earnings and dividends may be used as an
approximation to predict the future growth rate if the growth rate of dividend is
fairly stable in the past.

4.12
Financing Decisions

G = 1.0 (1+G)n where n is the number of years (e) Realized Yield Approach:
According to this approach, the average rate of return realized in the past few
years is historically regarded as ‘expected return’ in the future. The yield of
equity for the year is:

Yt =
Where, Yt Dt Pt = = =

D t + Pt −1 Pt −1
Yield for the year t Dividend for share for end of the year t Price per share at
the end of the year t Price per share at the beginning and at the end of the year
t

Pt – 1 =

Though, this approach provides a single mechanism of calculating cost of equity,


it has unrealistic assumptions. If the earnings do not remain stable, this method
is not practical. (f) Capital Asset Pricing Model Approach (CAPM): This model
describes the linear relationship between risk and return for securities. The risk
a security is exposed to are diversifiable and non-diversifiable. The
diversifiable risk can be eliminated through a portfolio consisting of large
number of well diversified securities. The non-diversifiable risk is assessed in
terms of beta coefficient (b or β) through fitting regression equation between
return of a security and the return on a market portfolio.

Cost of Equity under CAPM

4.13
Financial Management

Thus, the cost of equity capital can be calculated under this approach as:

Ke = Rf + b (Rm − Rf)
Where,

Ke Rf
b

= = =

Cost of equity capital Rate of return on security Beta coefficient Rate of return
on market portfolio
Equity Shares

Rm =

10 Pref.Shares 9 8 7 6 5 4 3 2 1 0 Govt. Bonds Corp. Debts

Risk Return relationship of various securities Therefore, required rate of return


= risk free rate + risk premium

The idea behind CAPM is that investors need to be compensated in two ways- time
value of money and risk. The time value of money is represented by the risk-free
rate in the formula and compensates the investors for placing money in any
investment over a period of time. The other half of the formula represents risk
and calculates the amount of compensation the investor needs for taking on
additional risk. This is calculated by taking a risk measure (beta) which compares
the returns of the asset to the market over a period of time and compares it to
the market premium. The CAPM says that the expected return of a security or a
portfolio equals the rate on a risk-

4.14
Financing Decisions

free security plus a risk premium. If this expected return does not meet or beat
the required return, then the investment should not be undertaken.
The shortcomings of this approach are: (a) Estimation of betas with historical
data is unrealistic; and (b) Market imperfections may lead investors to
unsystematic risk. Despite these shortcomings, the capital asset pricing approach
is useful in calculating cost of equity, even when the firm is suffering losses.
The basic factor behind determining the cost of ordinary share capital is to
measure the expectation of investors from the ordinary shares of that particular
company. Therefore, the whole question of determining the cost of ordinary shares
hinges upon the factors which go into the expectations of particular group of
investors in a company of a particular risk class. Illustration 8: Calculate the
cost of equity capital of H Ltd., whose risk free rate of return equals 10%. The
firm’s beta equals 1.75 and the return on the market portfolio equals to 15%.
Solution

Ke = Rf + b (Rm − Rf) Ke = .10 + 1.75 (.15 − .10) = .10 + 1.75 (.05) = .1875
1.3.4 COST OF RETAINED EARNINGS Like another source of fund, retained earnings
involve cost. It is the opportunity cost of dividends foregone by shareholders.
The given figure depicts how a company can either keep or reinvest cash or return
it to the shareholders as dividends. (Arrows represent possible cash flows or
transfers.) If the cash is reinvested, the opportunity cost is the expected rate
of return that shareholders could have obtained by investing in financial assets.

4.15
Financial Management

Cost of Retained Earnings There are two approaches to measure this opportunity
cost. One approach is by using discounted cash flow (DCF) method and the second
approach is by using capital asset pricing model. (a) By DCF : Where, D1 P0 G
Where, = = = Dividend Current market price Growth rate. Ks =

D1 +G P0

(b) By CAPM :

Ks = Rf + b (Rm − Rf)
Cost of equity capital Rate of return on security Beta coefficient Rate of return
on market portfolio

Ks Rf
b

= = =

Rm = D0 = Rs. 4.19

Illustration 9: ABC Company provides the following details:

P0

Rs. 50

G = 5%

Calculate the cost of retained earnings based on DCF method.

4.16
Financing Decisions

Solution

Ks

= = = = =

D1 +G P0 D 0 (1 + G) +G P0

Rs. 4.19 (1.05) + 0.05 Rs.50


0.088 + 0.05 13.8% b = 1.20 RM - Rf = 6%

Illustration 10: ABC Company provides the following details:

Rf = 7%
Solution

Calculate the cost of retained earnings based on CAPM method. Ks = = = Ks = Rf + b


(RM – Rf) 7% + 1.20 (6%) 7% + 7.20 14.2%

1.3.5 COST OF DEPRECIATION

Depreciation provisions may be considered in a similar manner to retained earnings


- they have an opportunity cost and represent an increased stake in the firm by
its shareholders. However, a distribution of depreciation provisions would produce
a capital reduction, probably requiring outstanding debts to be repaid due to the
depletion of the capital base, the security against which the debt was obtained.
This indicates a proportional combination between the cost of debt repaid and the
cost of retained earnings to calculate the cost of capital in the form of
depreciation provisions.
1.4 WEIGHTED AVERAGE COST OF CAPITAL (WACC)

As you know the capital funding of a company is made up of two components: debt
and equity. Lenders and equity holders each expect a certain return on the funds
or capital they have provided. The cost of capital is the expected return to
equity owners (or shareholders) and to debt holders, so weighted average cost of
capital tells the return that both stakeholders -

4.17
Financial Management

equity owners and lenders - can expect. WACC, in other words, represents the
investors' opportunity cost of taking on the risk of putting money into a company.
Since every company has a capital structure i.e. what percentage of debt comes
from retained earnings, equity shares, preference shares, and bonds, so by taking
a weighted average, it can be seen how much interest the company has to pay for
every rupee it borrows. This is the weighted average cost of capital. The weighted
average cost of capital for a firm is of use in two major areas: in consideration
of the firm's position and in evaluation of proposed changes necessitating a
change in the firm's capital. Thus, a weighted average technique may be used in a
quasi-marginal way to evaluate a proposed investment project, such as the
construction of a new building. Thus, weighted average cost of capital is the
weighted average after tax costs of the individual components of firm’s capital
structure. That is, the after tax cost of each debt and equity is calculated
separately and added together to a single overall cost of capital. K0 = % D(mkt)
(Ki) (1 – t) + (% Psmkt) Kp + (Cs mkt) Ke Where, K0 Ki 1–t Kp Ke = = = = = Overall
cost of capital Before tax cost of debt 1 – Corporate tax rate Cost of preference
capital Cost of equity % of debt in capital structure % of preference share in
capital structure % of equity share in capital structure.

% Dmkt = %Psmkt = % Cs =

The cost of weighted average method is preferred because the proportions of


various sources of funds in the capital structure are different. To be
representative, therefore, cost of capital should take into account the relative
proportions of different sources of finance. Securities analysts employ WACC all
the time when valuing and selecting investments. In discounted cash flow analysis,
WACC is used as the discount rate applied to future cash flows for deriving a
business's net present value. WACC can be used as a hurdle rate against which to
assess return on investment capital performance. It also plays a key role in
economic value added (EVA) calculations. Investors use WACC as a tool to decide
whether or not to invest. The WACC represents the minimum rate of return at which
a company produces value for its investors. Let's say a
4.18
Financing Decisions

company produces a return of 20% and has a WACC of 11%. By contrast, if the
company's return is less than WACC, the company is shedding value, which indicates
that investors should put their money elsewhere. Therefore, WACC serves as a
useful reality check for investors.
1.4.1 CALCULATION OF WACC Capital Component Retained Earnings Common Stocks
Preferred Stocks Bonds Total Cost 10% 11% 9% 6% Times X X X X % of capital
structure 25% 10% 15% 50% Total 2.50% 1.10% 1.35% 3.00% 7.95%

So the WACC of this company is 7.95%. But there are problems in determination of
weighted average cost of capital. These mainly relate to computation of equity
capital and the assignment of weights to the cost of specific source of financing.
Assignment of weights can be possible either on the basis of marginal weighting or
historical weighting. The most serious limitation of marginal weighting is that it
does not consider the long run implications of firm’s current financing. The
validity of the assumption of historical weighting is that choosing between the
book value weights and market value weights. While the book value weights may be
operationally convenient, the market value basis is theoretically more consistent,
sound and a better indicator of firm’s capital structure. The desirable practice
is to employ market weights to compute the firm’s cost of capital. This rationale
rests on the fact that the cost of capital measures the cost of issuing securities
– stocks as well as bonds – to finance projects, and that these securities are
issued at market value, not at book value.
Illustration 11: Calculate the WACC using the following data by using:

(a) Book value weights (b) Market value weights

4.19
Financial Management

The capital structure of the company is as under:


Rs.

Debentures (Rs. 100 per debenture) Preference shares (Rs. 100 per share) Equity
shares (Rs. 10 per share) The market prices of these securities are: Debenture
Preference Equity Rs. 105 per debenture Rs. 110 per preference share Rs. 24 each.

5,00,000 5,00,000 10,00,000 20,00,000

Additional information:

(1) Rs. 100 per debenture redeemable at par, 10% coupon rate, 4% floatation costs,
10 year maturity. (2) Rs. 100 per preference share redeemable at par, 5% coupon
rate, 2% floatation cost and 10 year maturity. (3) Equity shares has Rs. 4
floatation cost and market price Rs. 24 per share. The next year expected dividend
is Rs. 10 with annual growth of 5%. The firm has practice of paying all earnings
in the form of dividend. Corporate tax rate is 50%.
Solution

Cost of equity

Cost of equity = K e =

10 + .05 20

= .05 + .05 = .10 Cost of debt = K d = 10(1 − .5) + (100 − 96) 10 (100 + 96) 2

4.20
Financing Decisions

⎛ 5 + .4 ⎞ =⎜ ⎟ × 2 = .055 (approx.) ⎝ 196 ⎠


2⎞ ⎛ ⎜ 5+ ⎟ 10 ⎟ Cost of preference shares = K p = ⎜ ⎜ 198 ⎟ ⎟ ⎜ ⎝2⎠
⎛ 5.2 ⎞ =⎜ ⎟ ⎜ q ⎟ = .053 (approx.) ⎠ ⎝

Calculation of WACC using book value weights Source of capital Book Value Specific
cost (K%) Total cost

10% Debentures 5% Preference shares Equity shares

5,00,000 5,00,000 10,00,000 20,00,000


K0 = Rs. 1,54,000 = 0.077 (approx.) Rs. 20,00,000

.055 .053 .10

27,500 26,500 1,00,000 1,54,000

Calculation of WACC using market value weights Source of capital Book Value
Specific cost (K%) Total cost

10% Debentures 5% Preference shares Equity shares

5,25,000 5,50,000 24,00,000 34,75,000


K0 = Rs. 2,98,025 = 0.08576 (approx.) Rs.34,75,000

.055 .053 .10

28,875 29,150 2,40,000 2,98,025

1.5

MARGINAL COST OF CAPITAL

The marginal cost of capital may be defined as the cost of raising an additional
rupee of capital. Since the capital is raised in substantial amount in practice
marginal cost is referred to

4.21
Financial Management

as the cost incurred in raising new funds. Marginal cost of capital is derived,
when the average cost of capital is calculated using the marginal weights. The
marginal weights represent the proportion of funds the firm intends to employ.
Thus, the problem of choosing between the book value weights and the market value
weights does not arise in the case of marginal cost of capital computation. To
calculate the marginal cost of capital, the intended financing proportion should
be applied as weights to marginal component costs. The marginal cost of capital
should, therefore, be calculated in the composite sense. When a firm raises funds
in proportional manner and the component’s cost remains unchanged, there will be
no difference between average cost of capital (of the total funds) and the
marginal cost of capital. The component costs may remain constant upto certain
level of funds raised and then start increasing with amount of funds raised. For
example, the cost of debt may remain 7% (after tax) till Rs. 10 lakhs of debt is
raised, between Rs. 10 lakhs and Rs. 15 lakhs, the cost may be 8% and so on.
Similarly, if the firm has to use the external equity when the retained profits
are not sufficient, the cost of equity will be higher because of the floatation
costs. When the components cost start rising, the average cost of capital will
rise and the marginal cost of capital will however, rise at a faster rate.
Illustration 12: ABC Ltd. has the following capital structure which is considered
to be optimum as on 31st March, 2006. Rs.

14% debentures 11% Preference shares Equity (10,000 shares)

30,000 10,000 1,60,000 2,00,000

The company share has a market price of Rs. 23.60. Next year dividend per share is
50% of year 2006 EPS. The following is the trend of EPS for the preceding 10 years
which is expected to continue in future.
Year EPS (Rs.) Year EPS Rs.)

1997 1998 1999 2000 2001

1.00 1.10 1.21 1.33 1.46

2002 2003 2004 2005 2006

1.61 1.77 1.95 2.15 2.36

4.22
Financing Decisions

The company issued new debentures carrying 16% rate of interest and the current
market price of debenture is Rs. 96. Preference share Rs. 9.20 (with annual
dividend of Rs. 1.1 per share) were also issued. The company is in 50% tax
bracket. (A) Calculate after tax: (i) (ii) Cost of new debt Cost of new preference
shares

(iii) New equity share (consuming new equity from retained earnings) (B) Calculate
marginal cost of capital when no new shares are issued. (C) How much needs to be
spent for capital investment before issuing new shares? 50% of the 2006 earnings
are available as retained earnings for the purpose of capital investment. (D) What
will the marginal cost of capital when the funds exceeds the amount calculated in
(C), assuming new equity is issued at Rs. 20 per share?
Solution (A) (i) Cost of new debt

Kd = =
(ii)

I (1 − t) N

16 (1 − .5) = .0833 96 P O

Cost of new preference shares

Kp = =

1.1 = .12 9.2


D1 +G P0

(iii) Cost of new equity shares

Ke =

1.18 + 0.10 = 10.10 = 0.15 23.60


4.23
Financial Management

Calculation of D1

D1 = 50% of 2006 EPS = 50% of 2.36 = Rs. 1.18


(B) Type of Capital (1) Proportion (2) Specific Cost (3) Product (2) × (3) = (4)

Debt Preference Equity

0.15 0.05 0.80 Marginal cost of capital

0.0833 0.12 0.15

0.0125 0.0060 0.1200 0.1385

(C) The company can spend the following amount:

Retained earnings = (.50) (2.36 × 10,000) = Rs. 11,800 The ordinary equity is 80%
of total capital

Capital investment =
(D)

Rs. 11,800 = Rs. 14,750 .80

If the company require fund in excess of Rs. 14,750 it will have to issue new
shares. The cost of new issue will be

Ke =

Rs. 1.18 + .10 = .159 20


Proportion (2) Specific Cost (3) Product (2) × (3) = (4)

The marginal cost of capital will be


Type of Capital (1)

Debt Preference Equity (New)

0.15 0.05 0.80

0.0833 0.1200 0.1590

0.0125 0.0060 0.1272 0.1457

1.6

CONCLUSION

The determination of cost of capital is thus beset with a number of problems in


dynamic world
4.24
Financing Decisions

of today. Conditions which are present now may not remain static in future.
Therefore, howsoever cost of capital is determined now, it is dependent on certain
conditions or situations which are subject to change. Firstly, the firms’ internal
structure and character change. For instance, as the firm grows and matures, its
business risk may decline resulting in new structure and cost of capital.
Secondly, capital market conditions may change, making either debt or equity more
favourable than the other. Thirdly, supply and demand for funds may vary from time
to time leading to change in cost of different components of capital. Fourthly,
the company may experience subtle change in capital structure because of retained
earnings unless its growth rate is sufficient to call for employment of debt on a
continuous basis. Because of these reasons the firm should periodically re-examine
its cost of capital before determining annual capital budget.

4.25
Financial Management

UNIT – II : CAPITAL STRUCTURE DECISIONS Learning Objectives

After studying this unit you will be able to learn ♦ ♦ ♦


2.1

What is capital structure? What are optimal capital structure, and the theories
relating to the value of firm; and Understand EBIT-EPS break even or indifference
analysis and construct and interpret an EBIT-EPS chart.
MEANING OF CAPITAL STRUCTURE

Capital structure refers to the mix of a firm’s capitalisation and includes long
term sources of funds such as debentures, preference share capital, equity share
capital and retained earnings. According to Gerstenberg capital structure is “the
make-up of a firm’s capitalisation”. The decisions regarding the forms of
financing, their requirements and their relative proportions in total
capitalisation are known as capital structure decisions. In arriving at and
accomplishing this goal, the finance manager must take extreme care and prudence
keeping in mind factors under which the company has to operate as also certain
guiding principles of financing. Accordingly, he should choose a pattern of
capital which minimises cost of capital and maximises the owners’ return.
2.2 CHOICE OF CAPITAL STRUCTURE

A firm has the choice to raise funds for financing its investment proposals from
different sources in different proportions. It can: (a) Exclusively use debt, or
(b) Exclusively use equity capital, or (c) Exclusively use preference capital, or
(d) Use a combination of debt and equity in different proportions, or (e) Use a
combination of debt, equity and preference capital in different proportions, or
(f) Use a combination of debt and preference capital in different proportions. The
choice of the combination of these sources is called capital structure mix. But
the question is which of the pattern should the firm choose? Well, while choosing
a suitable financing pattern, certain fundamental principles should be kept in
mind, which are discussed below:

4.26
Financing Decisions

(a) Cost Principle: According to this principle an ideal pattern or capital


structure is one that minimises cost of capital structure and maximises earnings
per share (EPS). Debt capital is cheaper than equity capital from the point of its
cost and interest being deductible for income tax purpose, where no such deduction
is allowed for dividends. Consequently effective rate of interest which the
company has to bear would be less than the nominal rate at which debentures are
issued. This requires the mix of debt finance with equity finance so as to reduce
the aggregate cost of capital. (b) Risk Principle: According to this principle,
reliance is placed more on common equity for financing capital requirements than
excessive use of debt. Use of more and more debt and preference capital affects
share values and in unfavourable situation share prices may consequently drop.
There are two risks associated with this principle:

(i) Business risk: It is an unavoidable risk because of the environment in which


the firm has to operate and business risk is represented by the variability of
earnings before interest and tax (EBIT). The variability in turn is influenced by
revenues and expenses. Revenues and expenses are affected by demand of firm
products, variations in prices and proportion of fixed cost in total cost. (ii)
Financial risk: It is a risk associated with the availability of earnings per
share caused by use of financial leverage. It is also unavoidable if firm does not
use debt in its capital structure. Generally, a firm should neither be exposed to
high degree of business risk and low degree of financial risk or vice-versa, so
that shareholders do not bear a higher risk.
(c) Control Principle: While designing a capital structure, the finance manager
may also keep in mind that existing management control and ownership remains
undisturbed. Issue of new equity will dilute existing control pattern and also it
involves higher cost. Issue of more debt causes no dilution in control, but causes
a higher degree of financial risk. This concern over dilution of control is mostly
felt in closely-held companies. (d) Flexibility Principle: By flexibility it means
that the management chooses such a combination of sources of financing which it
finds easier to adjust according to changes in need of funds in future too. In
attaining flexibility cost considerations should be kept in mind. If the company
is loaded with a debt of 18% and funds are available at 15%, it can return old
debt with new debt, at a lesser interest rate.

Besides these principles, other factors such as nature of industry, timing of


issue and competition in the industry are also being considered. Timing of raising
capital should take into account the state of economy and capital market.
Industries facing severe competition also resort to more equity than debt. Thus a
finance manager in designing a suitable pattern of capital structure must bring
about

4.27
Financial Management

satisfactory compromise between these militant principles. The compromise can be


reached by assigning weights to these principles in terms of various
characteristics of the company.
2.3 SIGNIFICANCE OF CAPITAL STRUCTURE

The capital structure decisions are so significant in financial management, as


they influence debt – equity mix which ultimately affects shareholders return and
risk. Since cost of debt is cheaper, firm prefers to borrow rather than to raise
from equity. The value of equity depends on earnings per share. So long as return
on investment is more than the cost of borrowing, extra borrowing increases the
earnings per share. However, beyond a limit, it increases the risk and share price
may fall because shareholders may assume that their investment is associated with
more risk. But the effect of fall in share price due to heavy load of debt is
difficult to measure. Market factors are so highly psychological and complex as
they hardly follow these theoretical considerations. However, an appropriate debt
-equity mix can be determined empirically within the company taking into
consideration the following factors:
2.3.1 Leverages

There are two leverages associated with the study of capital structure, namely
operating leverage and financial leverage. Operating leverage exists when a firm
has a fixed cost that must be defrayed regardless of volume of business. The
contrast to the operating leverage, financial leverage refers to mix of debt and
equity in the capitalisation of a firm. In order to decide proper financial
policy, operating leverage may also be taken into consideration, as the financial
leverage is a superstructure built on the operating leverage. The operating
profits otherwise known as earnings before interest and tax (EBIT), serves as a
fulcrum in defining these two leverages. Financial leverage represents the
relationship between firms earnings before interest and tax and earnings available
for equity holders. When there is an increase in EBIT, there is a corresponding
increase in market price of equity share. However, increased use of debt in the
capital structure which proportionately increases EBIT has certain limitations. If
debt is employed in greater proportions, marginal cost of debt will also increase
and share price may fall down as investors feel it is risky. On the other, in
spite of increased risk, market share price may increase because investors
speculate future profits. Thus before using financial leverage, its impact on EPS
must be weighed. The degree of financial leverage can be found out as:
Percentage change in Earnings per share (EPS) Percentage change in Earnings before
interest and tax (EBIT)

A company having higher operating leverage should be accompanied by a low


financial leverage and vice versa, otherwise it will face problems of insolvency
and inadequate liquidity.
4.28
Financing Decisions

Thus a combination of both the leverages is a challenging task.


2.3.2 Trading on Equity

The term ‘trading on equity’ is derived from the fact that debts are contracted
and loans are raised mainly on the basis of equity capital. Those who provide debt
have a limited share in the firm’s earnings and hence want to be protected in
terms of earnings and values represented by equity capital. Since fixed charges do
not vary with the firms earnings before interest and tax, a magnified effect is
produced on earnings per share. Whether the leverage is favourable in the sense
increase in earnings per share more proportionately to the increased earnings
before interest and tax depends on the profitability of investment proposals. If
the rate of return on investment exceeds their explicit cost financial leverage is
said to be positive. However, the determination of optimal level of debt is a
formidable task and is a major policy decision. Determination of optimal level of
debt involves equalising between return and risk. Though, there are number of
approaches to determine the level of debt, they cannot be considered as
satisfactory and as such can serve only as a guideline. Whatever approaches may be
followed for determining the optimal level of debt, the objective of maximising
share price should be borne in mind. EBIT-EPS analysis is a widely used tool to
determine level of debt in a firm. Through this analysis, a comparison can be
drawn for various methods of financing by obtaining indifference point. It is a
point to the EBIT level at which EPS remain unchanged irrespective of debt level
equity mix. For example indifference point for the capital mix (equity share
capital and debt) can be determined as follows: (EBIT − I1 )(1 − T ) (EBIT − I2 )
(1 − T ) = E1 E2 Where, EBIT = E1 E2 I1 12 T = = = = = Indifference point Number
of equity shares in Alternative 1 Number of equity shares in Alternative 2
Interest charges in Alternative 1 Interest charges in Alternative 2 Tax-rate

Alternative 1= All equity finance Alternative 2= Debt-equity finance.

4.29
Financial Management

The chief deficiency of this method is that it does not take into account the
implicit cost associated with debt. The concepts of leverages and EBIT-EPS
analysis would be dealt in detail separately for better understanding.
2.3.3 Coverage Ratio

The ability of the firm to use debt in the capital structure can also be judged in
terms of coverage ratio namely EBIT/Interest. Higher the ratio, greater is the
certainty of meeting interest payments.
2.3.4 Cash flow analysis

It is a good supporting tool for EBIT-EPS analysis in framing a suitable capital


structure. To determine the debt capacity, cash flow under adverse conditions
should be examined. A high debt equity ratio is not risky if the company has the
ability to generate cash flows. It would, therefore be possible to increase the
debt until cash flows equal the risk set out by debt. The main drawback of this
approach is that it fails to take into account uncertainty due to technological
developments or changes in political climate. These approaches as discussed above
do not provide solution to the problem of determining an appropriate level of
debt. However, with the information available a range can be determined for an
optimum level of debt in the capital structure.
2.4 OPTIMAL CAPITAL STRUCTURE

The theory of optimal capital structure deals with the issue of the right mix of
debt and equity in the long term capital structure of a firm. This theory states
that if a company takes on debt, the value of the firm increases up to a point.
Beyond that point if debt continues to increase then the value of the firm will
start to decrease. Similarly if the company is unable to repay the debt within the
specified period then it will affect the goodwill of the company in the market and
may create problems for collecting further debt. Therefore, the company should
select its appropriate capital structure with due consideration to the factors
mentioned above.
2.5 EBIT-EPS ANALYSIS

The basic objective of financial management is to design an appropriate capital


structure which can provide the highest earnings per share (EPS) over the firm’s
expected range of earnings before interest and taxes (EBIT). EPS measures a firm’s
performance for the investors. The level of EBIT varies from year to year and
represents the success of a firm’s operations. EBIT-EPS analysis is a vital tool
for designing the optimal capital structure of a firm. The objective of this
analysis is to find the EBIT level that will equate EPS regardless of the
financing plan chosen.

4.30
Financing Decisions

2.5.1

Financial Break-even and Indifference Analysis

Financial break-even point is the minimum level of EBIT needed to satisfy all the
fixed financial charges i.e. interest and preference dividends. It denotes the
level of EBIT for which the firm’s EPS equals zero. If the EBIT is less than the
financial breakeven point, then the EPS will be negative but if the expected level
of EBIT is more than the breakeven point, then more fixed costs financing
instruments can be taken in the capital structure, otherwise, equity would be
preferred. EBIT-EPS breakeven analysis is used for determining the appropriate
amount of debt a firm might carry. Another method of considering the impact of
various financing alternatives on earnings per share is to prepare the EBIT chart
or the range of Earnings Chart. This chart shows the likely EPS at various
probable EBIT levels. Thus, under one particular alternative, EPS may be Rs. 2 at
a given EBIT level. However, the EPS may go down if another alternative of
financing is chosen even though the EBIT remains at the same level. At a given
EBIT, earnings per share under various alternatives of financing may be plotted. A
straight line representing the EPS at various levels of EBIT under the alternative
may be drawn. Wherever this line intersects, it is known as break-even point. This
point is a useful guide in formulating the capital structure. This is known as EPS
equivalency point or indifference point since this shows that, between the two
given alternatives of financing (i.e., regardless of leverage in the financial
plans), EPS would be the same at the given level of EBIT. The equivalency or
indifference point can also be calculated algebraically in the following manner.
(EBIT − I1 )(1 − T ) (EBIT − I2 )(1 − T ) = E1 E2 Where, EBIT = E1 E2 I1 12 T = =
= = = Indifference point Number of equity shares in Alternative 1 Number of equity
shares in Alternative 2 Interest charges in Alternative 1 Interest charges in
Alternative 2 Tax-rate

Alternative 1= All equity finance Alternative 2= Debt-equity finance.

4.31
Financial Management

The indifference point can also be depicted graphically as:

Debt-Equity Indifference Point Illustration 1: Best of Luck Ltd., a profit making


company, has a paid-up capital of Rs. 100 lakhs consisting of 10 lakhs ordinary
shares of Rs. 10 each. Currently, it is earning an annual pre-tax profit of Rs. 60
lakhs. The company's shares are listed and are quoted in the range of Rs. 50 to
Rs. 80. The management wants to diversify production and has approved a project
which will cost Rs. 50 lakhs and which is expected to yield a pre-tax income of
Rs. 40 lakhs per annum. To raise this additional capital, the following options
are under consideration of the management:

(a) To issue equity capital for the entire additional amount. It is expected that
the new shares (face value of Rs. 10) can be sold at a premium of Rs. 15. (b) To
issue 16% non-convertible debentures of Rs. 100 each for the entire amount. (c) To
issue equity capital for Rs. 25 lakhs (face value of Rs. 10) and 16% non-
convertible debentures for the balance amount. In this case, the company can issue
shares at a premium of Rs. 40 each. You are required to advise the management as
to how the additional capital can be raised, keeping in mind that the management
wants to maximise the earnings per share to maintain its goodwill. The company is
paying income tax at 50%.

4.32
Financing Decisions

Solution Calculation of Earnings per share under the three options: Particulars
Option I (Issue of equity only) (Rs. in lakhs) Option II (Issue of debentures
only) (Rs in lakhs) Option III (Issue of equity and debentures equally) (Rs in
lakhs)

Number (lakhs): Existing

of

Equity

Shares 10 2 12 Nil 10 10 Rs. 50 lakhs 10.00 0.50 10.50 Rs. 25 lakhs

Now issued Total 16% debentures Estimated total income: From current operations
From new projects
Less: Interest debentures

60 40 100

60 40 100 8 92 46 46 Rs. 4.60

60 40 100 4 96 48 48 Rs. 4.57

on

16%

100 50 50 Rs. 4.17

Profit before tax Tax at 50% Profit after tax EPS


Advise: share. 2.6

Option II i.e. issue of 16% debentures is most suitable to maximize the earnings
per
COST OF CAPITAL, CAPITAL STRUCTURE AND MARKET PRICE OF SHARE

The financial leverage has a magnifying effect on earnings per share, such that
for a given level of financial percentage increases with EBIT beyond the point of
financial indifference, there will be more than proportionate change in the same
direction in the earnings per share. The financing decision of the firm is one of
the basic conditions oriented to the achievement of
4.33
Financial Management

maximisation for the shareholders wealth. The capital structure should be examined
from their view point of its impact on the value of the firm. If the capital
structure affects the total value of the firm, a firm should select such a
financing mix (a combination of debt and equity) which will maximise the market
value of the firm. Such an optimum leverage not only maximises the value of the
company and wealth of its owners, but also minimises the cost of capital. As a
result, the company is able to increase its economic rate of investment and
growth. In theory, capital structure can affect the value of the firm by affecting
either its expected earnings or cost of capital or both. While financing mix
cannot affect the total earnings, it can affect the share of earnings belonging to
the share holders. But financial leverage can largely influence the value of the
firm through the cost of capital.
2.7 CAPITAL STRUCTURE THEORIES

The following approaches explain the relationship between cost of capital, capital
structure and value of the firm: (a) Net income approach (b) Net operating income
approach (c) Modigliani-Miller approach (d) Traditional approach. However, the
following assumptions are made to understand this relationship. ♦ ♦ ♦ ♦ ♦ ♦ There
are only two kinds of funds used by a firm i.e. debt and equity. Taxes are not
considered. The payout ratio is 100% The firm’s total financing remains constant
Business risk is constant over time The firm has perpetual life.

(a) Net Income Approach (NI): According to this approach, capital structure
decision is relevant to the value of the firm. An increase in financial leverage
will lead to decline in the weighted average cost of capital, while the value of
the firm as well as market price of ordinary share will increase. Conversely a
decrease in the leverage will cause an increase in the overall cost of capital and
a consequent decline in the value as well as market price of equity shares.

4.34
Financing Decisions

From the above diagram, ke and kd are assumed not to change with leverage. As debt
increases, it causes weighted average cost of capital to decrease. The value of
the firm on the basis of Net Income Approach can be ascertained as follows: V=S+D
Where, V = Value of the firm S = Market value of equity D = Market value of debt
Market value of equity (S) = Where, NI = Earnings available for equity
shareholders Ke = Equity Capitalisation rate Under, NI approach, the value of the
firm will be maximum at a point where weighted average cost of capital is minimum.
Thus, the theory suggests total or maximum possible debt financing for minimising
the cost of capital. The overall cost of capital under this approach is :

NI Ke

4.35
Financial Management

Overall cos t of capital =

EBIT Value of the firm

Thus according to this approach, the firm can increase its total value by
decreasing its overall cost of capital through increasing the degree of leverage.
The significant conclusion of this approach is that it pleads for the firm to
employ as much debt as possible to maximise its value.
Illustration 2: Rupa Company’s EBIT is Rs. 5,00,000. The company has 10%, 20 lakh
debentures. The equity capitalization rate i.e. Ke is 16%.

You are required to calculate: (i) (ii) (i) Net operating income/EBIT
Less: Interest on debentures (10% of Rs. 20,00,000)

Market value of equity and value of firm Overall cost of capital.


Statement showing value of firm Rs.

Solution

5,00,000 2,00,000 3,00,000 16% 18,75,000 20,00,000 38,75,000

Earnings available for equity holders i.e. NI Equity capitalisation rate (Ke)
Market value of equity (S) = Market value of debt (D) Total value of firm V = S +
D NI ⎛ 3,00,000 ⎞ =⎜ × 100 ⎟ K e ⎝ 16.00 ⎠

(ii)

Overall cost of capital =

EBIT 5,00,000 = = 12.90% Value of firm 38,75,000

(b) Net Operating Income Approach (NOI): NOI means earnings before interest and
tax. According to this approach, capital structure decisions of the firm are
irrelevant. Any change in the leverage will not lead to any change in the total
value of the firm and the market price of shares, as the overall cost of capital
is independent of the degree of leverage. As a result, the division between debt
and equity is irrelevant. An increase in the use of debt which is apparently
cheaper is offset by an increase in the equity capitalisation rate. This happens
because equity investors seek higher compensation as they are opposed to greater
risk due to
4.36
Financing Decisions

the existence of fixed return securities in the capital structure.

The above diagram shows that Ko (Overall capitalisation rate) and (debt –
capitalisation rate) are constant and Ke (Cost of equity) increases with leverage.
Illustration 3: Amita Ltd’s. operating income is Rs. 5,00,000. The firms cost of
debt is 10% and currently firm employs Rs. 15,00,000 of debt. The overall cost of
capital of the firm is 15%.

You are required to determine: (i) (ii) (i) Total value of the firm. Cost of
equity.
Statement showing value of the firm Rs.

Solution

Net operating income/EBIT


Less: Interest on debentures (10% of Rs. 15,00,000)

5,00,000 1,50,000 3,50,000 15%

Earnings available for equityholders Total cost of capital (K0) (given)

4.37
Financial Management

Value of the firm V = (ii)

EBIT 5,00,000 = k0 0.15

33,33,333

Calculation of cost of equity

Ke =

Earnings available for equity holders Value of equity(s)


Rs.

Market value of debt (D) Market value of equity (s) S = V − D = 33,33,333 –


15,00,000
Cost of equity (K e ) = Earnings availabe for equityholders Market value of equity
=
= =

15,00,000 18,33,333

Or,

EBIT − Interest paid on debt Market value of equity


5,00,000 − 1,50,000 18,33,333 Rs. 3,50,000 = 19.09% 18,33,333

⎛S⎞ ⎛D⎞ Ko = Ke ⎜ ⎟ + Kd ⎜ ⎟ ⎝V⎠ ⎝V⎠ ⎛D⎞ ⎛V⎞ Ke = Ko ⎜ ⎟ − Kd ⎜ ⎟ ⎝S⎠ ⎝S⎠


⎛ 33,33,333 ⎞ ⎛ 15,00,000 ⎞ = 0.15 ⎜ ⎟ − 0.10 ⎜ ⎟ ⎝ 18,33,333 ⎠ ⎝ 18,33,333 ⎠
= 1 [(0.15 × 33,33,333) − (0.10 × 15,00,000)] 18,33,333

1 [5,00,000 − 1,50,000] = 19.09% 18,33,333

4.38
Financing Decisions

(c) Modigliani-Miller Approach (MM): The NOI approach is definitional or


conceptual and lacks behavioural significance. It does not provide operational
justification for irrelevance of capital structure. However, Modigliani-Miller
approach provides behavioural justification for constant overall cost of capital
and, therefore, total value of the firm.

The approach is based on further additional assumptions like: ♦ ♦ ♦ ♦ (i) (ii)


Capital markets are perfect. All information is freely available and there are no
transaction costs. All investors are rational. Firms can be grouped into
‘Equivalent risk classes’ on the basis of their business risk. Non-existence of
corporate taxes. Total market value of a firm is equal to its expected net
operating income dividend by the discount rate appropriate to its risk class
decided by the market. The expected yield on equity is equal to the risk free rate
plus a premium determined as per the following equation: Kc = Ko + (Ko– Kd) B/S

Based on the above assumptions, Modigliani-Miller derived the following three


propositions.

(iii) Average cost of capital is not affected by financial decision.

4.39
Financial Management

It is evident from the above diagram that the average cost of the capital (Ko) is
a constant and not affected by leverage. The operational justification of
Modigliani-Miller hypothesis is explained through the functioning of the arbitrage
process and substitution of corporate leverage by personal leverage. Arbitrage
refers to buying asset or security at lower price in one market and selling it at
higher price in another market. As a result equilibrium is attained in different
markets. This is illustrated by taking two identical firms of which one has debt
in the capital structure while the other does not. Investors of the firm whose
value is higher will sell their shares and instead buy the shares of the firm
whose value is lower. They will be able to earn the same return at lower outlay
with the same perceived risk or lower risk. They would, therefore, be better off.
The value of the levered firm can either be neither greater nor lower than that of
an unlevered firm according this approach. The two must be equal. There is neither
advantage nor disadvantage in using debt in the firm’s capital structure. Simply
stated, the Modigliani Miller approach is based on the thought that no matter how
the capital structure of a firm is divided among debt, equity and other claims,
there is a conservation of investment value. Since the total investment value of a
corporation depends upon its underlying profitability and risk, it is invariant
with respect to relative changes in the firm’s financial capitalisation. The
approach considers capital structure of a firm as a whole pie divided into equity,
debt and other securities. According to MM, since the sum of the parts must equal
the whole, therefore, regardless of the financing mix, the total value of the firm
stays the same as shown in the figures below:

The shortcoming of this approach is that the arbitrage process as suggested by


Modigliani-Miller will fail to work because of imperfections in capital market,
existence of transaction cost and
4.40
Financing Decisions

presence of corporate income taxes. However in their 1963 article, they recognised
that the value of the firm will increase or cost of capital will decrease where
corporate taxes exist. As a result there will be some difference in the earnings
of equity and debt-holders in an levered and unlevered firm and value of levered
firm will be greater than the value of unlevered firm by an amount equal to amount
of debt multiplied by corporate tax rate.
Illustration 4: When value of levered firm is more than the value of unlevered
firm

There are two firms N and M, having same earnings before interest and taxes i.e.
EBIT of Rs. 20,000. Firm M is levered company having a debt of Rs. 1,00,000 @ 7%
rate of interest. The cost of equity of N company is 10% and of M company is
11.50%. Find out how arbitrage process will be carried on?
Solution Firms N M

NOI/EBIT Debt Ke Kd Value of equity (S) =

Rs. 20,000 − 10% −

Rs. 20,000 Rs. 1,00,000 11.50% 7%

NOI − Interest Cost of equity

SN =

20,000 10%

= Rs. 2,00,000

SM =

20,000 − 7,000 = Rs. 1,13,043 11.50%

VN = Rs. 2,00,000 VM = 1,13,043 + 1,00,000 {V = S + D} = Rs. 2,13,043 Assume you


have 10% share of levered company. i.e. M. Therefore, investment in 10% of equity
of levered company = 10% × 1,13,043 = Rs. 11,304.3 Return will be 10% of (20,000 –
7,000) = Rs. 1,400.

4.41
Financial Management

Alternate Strategy will be:

Sell your 10% share of levered firm for Rs. 11,304.3 and borrow 10% of levered
firms debt i.e. 10% of Rs. 1,00,000 and invest the money i.e. 10% in unlevered
firms stock: Total resources /Money we have = 11,304.3 + 10,000 = 21,304.3 and you
invest 10% of 2,00,000 = Rs. 20,000 Surplus cash available with you is = 21,304.3
– 20,000 = Rs. 1,304.3 Your return = 10% EBIT of unlevered firm – Interest to be
paid on borrowed funds i.e. = 10% of Rs. 20,000 – 7% of Rs. 10,000 = 2,000 – 700 =
Rs. 1,400 i.e. your return is same i.e. Rs. 1,400 which you are getting from ‘N’
company before investing in ‘M’ company. But still you have Rs. 1,304.3 excess
money available with you. Hence, you are better off by doing arbitrage.
Illustration 5: When value of unlevered firm is more than the value of levered
firm

There are two firms U and L having same NOI of Rs. 20,000 except that the firm L
is a levered firm having a debt of Rs. 1,00,000 @ 7% and cost of equity of U & L
are 10% and 18% respectively. Show how arbitrage process will work.
Solution Firms U L

NOI/EBIT Debt capital Kd Ke

Rs. 20,000 − − 10%

Rs. 20,000 Rs. 1,00,000 7% 18%

⎛ EBIT − Interest ⎞ ⎟ Value of equity capital (s) = ⎜ ⎜ ⎟ Ke ⎝ ⎠


=

20,000 0.10

20,000 − 7,000 0.18


Rs. 72,222 Rs. 72,222 + 1,00,000 = Rs. 1,72,222

= Rs. 2,00,000 Total value of the firm V=S+D Rs. 2,00,000

4.42
Financing Decisions

Assume you have 10% share of unlevered firm i.e. investment of 10% of Rs. 2,00,000
= Rs. 20,000 and Return @ 10% on Rs. 20,000. Investment will be 10% of earnings
available for equity i.e. 10% × 20,000 = Rs. 2,000.
Alternative strategy:

Sell your share in unlevered firm for Rs. 20,000 and buy 10% share of levered
firm’s equity plus debt i.e. 10% equity of levered firm 10% debt of levered firm
Total investment Your resources are Rs. 20,000 Surplus cash available = Surplus –
Investment = 20,000 – 17,222 = Rs. 2,778 Your return on investment is: 7% on debt
of Rs. 10,000 10% on equity i.e. 10% of earnings available for equity holders i.e.
(10% × 13,000) Total return 700 1,300 2,000 = 7,222 = 10,000 = 17,222

i.e. in both the cases the return received is Rs. 2,000 and still you have excess
cash of Rs. 2,778. Hence, you are better off i.e you will start selling unlevered
company shares and buy levered company’s shares thereby pushing down the value of
shares of unlevered firm and increasing the value of levered firm till equilibrium
is reached.
(d) Traditional Approach: This approach favours that as a result of financial
leverage up to some point, cost of capital comes down and value of firm increases.
However, beyond that point, reverse trends emerge.

The principle implication of this approach is that the cost of capital is


dependent on the capital structure and there is an optimal capital structure which
minimises cost of capital. At the optimal capital structure, the real marginal
cost of debt and equity is the same. Before the optimal point, the real marginal
cost of debt is less than real marginal cost of equity and beyond this point the
real marginal cost of debt is more than real marginal cost of equity.

4.43
Financial Management

The above diagram suggests that cost of capital is a function of leverage. It


declines with Kd (debt) and starts rising. This means that there is a range of
capital structure in which cost of capital is minimised. The net income approach
argues that leverage always affects overall cost of capital and value of the firm.
Optimum capital structure occurs at the point where value of the firm is highest
and the cost of capital at the lowest. According to net operating income approach
capital structure decisions are totally irrelevant. Modigliani-Miller supports the
net operating income approach but provides behavioural justification. The
traditional approach strikes a balance between these extremes. According to this
approach the firm should strive to reach the optimal capital structure and its
total valuation through a judicious use of the both debt and equity in capital
structure. At the optimal capital structure the overall cost of capital will be
minimum and the value of the firm is maximum. It further states that the value of
the firm increases with financial leverage upto a certain point. Beyond this point
the increase in financial leverage will increase its overall cost of capital and
hence the value of firm will decline. This is because the benefits of use of debt
may be so large that even after off setting the effect of increase in cost of
equity, the overall cost of capital may still go down. However, if financial
leverage increases beyond a acceptable limit the risk of debt investor may also
increase, consequently cost of debt also starts increasing. The increasing cost of
equity owing to increased financial risk and increasing cost of debt makes the
overall cost of capital to increase.
Illustration 6: Indra company has EBIT of Rs. 1,00,000. The company makes use of
debt and equity capital. The firm has 10% debentures of Rs. 5,00,000 and the
firm’s equity

4.44
Financing Decisions

capitalization rate is 15%. You are required to compute: (i) (ii) Current value of
the firm Overall cost of capital.

Solution

(i) EBIT

Calculation of total value of the firm Rs.

1,00,000 50,000 50,000 15%

Less: Interest (@10% on Rs. 5,00,000)

Earnings available for equity holders Equity capitalization rate i.e. Ke Value of
equity holders = = 50,000 = Rs. 3,33,333 0.15 Earnings available for equity
holders Ke

Value of Debt (given) D Total value of the firm V = D + S {5,00,000 + 3,33,333)

5,00,000 8,33,333

(ii)

⎛S⎞ ⎛D⎞ Overall cost of capital = K o = K e ⎜ ⎟ + K d ⎜ ⎟ ⎝V⎠ ⎝V⎠


⎛ 5,00,000 ⎞ ⎛ 3,33,333 ⎞ = 0.15 ⎜ ⎟ ⎟ + 0.10 ⎜ ⎝ 8,33,333 ⎠ ⎝ 8,33,333 ⎠
= 1 [50,000 + 50,000] 8,33,333

= 12.00%

4.45
Financial Management

2.8

CAPITAL STRUCTURE AND TAXATION

The leverage irrelevance theorem of MM is valid if the perfect market assumptions


underlying their analysis are satisfied. However, in the face of imperfections
characterising the real world capital markets, the capital structure of a firm may
affect its valuation. Presence of taxes is a major imperfection in the real world.
This section examines the implications of corporate and personal taxes for the
capital structure.
2.8.1 Corporate Taxes

When taxes are applicable to corporate income, debt financing is advantageous.


This is because dividends and retained earnings are not deductible for tax
purposes; interest on debt is a tax-deductible expense. As a result, the total
income available for both stockholders and debt-holders is greater when debt
capital is used.
Illustration 7: There are two firms Company A and B having net operating income of
Rs. 15,00,000 each. Company B is a levered company whereas Company A is all equity
company. Debt employed by Company B is of Rs. 7,00,000 @ 11%. The tax rate
applicable to both the companies is 25%. Calculate earnings available for equity
and debt for both the firms. Solution Statement of calculation of earnings
available to equity holders and debt holders Company A 15,00,000 − 15,00,000
3,75,000 11,25,000 11,25,000 B 15,00,000 77,000 14,23,000 3,55,750 10,67,250
10,67,250 + 77,000 =11,44,250

Net operating income Less: Interest on Debt (11% of Rs. 7,00,000) Profit before
taxes Less: Tax @ 25% Profit after tax/Earnings available in equity holders Total
earnings available to equity holders + Debt holders

As we can see that the earnings in case of Company B is more than the earnings of
Company A because of tax shield available to shareholders of Company B due to the
presence of debt structure in Company B. The interest is deducted from EBIT
without tax deduction at the corporate level; equity holders also get their income
after tax deduction due to which income of both the investors increase to the
extent of tax saving on the interest paid i.e. tax shield i.e. 25% × 77,000 =
19,250 i.e. difference in the income of two companies’ earnings i.e. 11,44,250 –
11,25,000 = Rs. 19,250.
4.46
Financing Decisions

UNIT – III : BUSINESS RISK AND FINANCIAL RISK Learning Objectives

After studying this unit, you will be able to ♦ ♦ ♦ ♦ ♦


3.1

Define, discuss, and quantify “business risk” and “financial risk”; Define
operating and financial leverage and identify causes of both; Define, calculate,
and interpret a firm’s degree of operating, financial, and total leverage;
Calculate a firm’s operating break-even (quantity) point and break-even (sales)
point; and Understand what is involved in determining the appropriate amount of
financial leverage for a firm?
INTRODUCTION

A firm can finance its operations through common and preference shares, with
retained earnings, or with debt. Usually a firm uses a combination of these
financing instruments. The proportion of short and long-term debt is considered
when analyzing capital structure. Capital structure refers to a firm's debt-to-
equity ratio, which provides insight into how risky a company is. Usually a
company more heavily financed by debt poses greater risk. Firms can obtain their
long-term financing from either debt or equity or some combination of debt and
equity. Capital structure decisions by firms will have an effect on the expected
profitability of the firm, the risks facing debt holders and shareholders, the
probability of failure, the cost of capital and the market value of the firm. Risk
facing the common shareholders is a function of i.e. is affected by two types of
risks, namely business risk and financial risk. Risk Facing Common Shareholders =
f {Business Risk, Financial Risk}
3.1.1 BUSINESS RISK AND FINANCIAL RISK

Business risk refers to the risk associated with the firm's operations. It is the
uncertainty about the future operating income (EBIT), i.e. how well can the
operating income be predicted? Business risk can be measured by the standard
deviation of the Basic Earning Power ratio.

4.47
Financial Management

Financial risk refers to the additional risk placed on the firm's shareholders as
a result of debt use i.e. the additional risk a shareholder bears when a company
uses debt in addition to equity financing. Companies that issue more debt
instruments would have higher financial risk than companies financed mostly or
entirely by equity. Financial risk can be measured by ratios such as the firm's
financial leverage multiplier, total debt to assets ratio or degree of financial
leverage. A company's risk is composed of financial risk, which is linked to debt,
and risk, which is often linked to economic climate. If a company is entirely
financed by equity, it would pose almost no financial risk, but, it would be
susceptible to business risk or changes in the overall economic climate. Leverage
refers to the ability of a firm in employing long term funds having a fixed cost,
to enhance returns to the owners. In other words, leverage is the amount of debt
that a firm uses to finance its assets. The use of various financial instruments
or borrowed capital, to increase the potential return of an investment. A firm
with a lot of debt in its capital structure is said to be highly levered. A firm
with no debt is said to be unlevered.
3.2 DEBT VERSUS EQUITY FINANCING

Financing a business through borrowing is cheaper than using equity. This is


because: ♦ Lenders require a lower rate of return than ordinary shareholders. Debt
financial securities present a lower risk than shares for the finance providers
because they have prior claims on annual income and liquidation. A profitable
business effectively pays less for debt capital than equity for another reason:
the debt interest can be offset against pre-tax profits before the calculation of
the

4.48
Financing Decisions

corporate tax, thus reducing the tax paid. ♦ Issuing and transaction costs
associated with raising and servicing debt are generally less than for ordinary
shares.

These are some benefits from financing a firm with debt. Still firms tend to avoid
very high gearing levels. One reason is financial distress risk. This could be
induced by the requirement to pay interest regardless of the cash flow of the
business. If the firm goes through a rough period in its business activities it
may have trouble paying its bondholders, bankers and other creditors their
entitlement. The relationship between Expected return (Earnings per share) and the
level of gearing can be represented as:

Relationship between leverage and risk

Leverage can occur in either the operating or financing portions of the income
statement. The effect of leverage is to magnify the effects of changes in sales
volume on earnings. Leverage serves to increase both expected return and risk to
the firm's stockholders. Leverage helps both the investor and the firm to invest
or operate. However, it comes with greater risk. If an investor uses leverage to
make an investment and the investment moves against the investor, his or her loss
is much greater than it would have been if the investment had not been leveraged -
leverage magnifies both gains and losses. In the business world, a company can use
leverage to try to generate shareholder wealth, but if it fails to do so, the
interest expense and credit risk of default destroys shareholder value. Most
companies use debt to finance operations. By doing so, a company increases its
leverage because it can invest in business operations without increasing its
equity. For example, if a company formed with an investment of Rs. 50 lakhs from
investors, the equity in

4.49
Financial Management

the company is Rs. 50 lakhs - this is the money the company uses to operate. If
the company uses debt financing by borrowing Rs. 200 lakhs, the company now has
Rs. 250 lakhs to invest in business operations and more opportunity to increase
value for shareholders.
3.3 TYPES OF LEVERAGE

The term Leverage in general refers to a relationship between two interrelated


variables. In financial analysis it represents the influence of one financial
variable over some other related financial variable. These financial variables may
be costs, output, sales revenue, Earnings Before Interest and Tax (EBIT), Earning
per share (EPS) etc. There are three commonly used measures of leverage in
financial analysis. These are: (i) (ii) Operating Leverage Financial Leverage

(iii) Combined Leverage


3.3.1 OPERATING LEVERAGE

Operating leverage (OL) maybe defined as the employment of an asset with a fixed
cost in the hope that sufficient revenue will be generated to cover all the fixed
and variable costs. The use of assets for which a company pays a fixed cost is
called operating leverage. With fixed costs the percentage change in profits
accompanying a change in volume is greater than the percentage change in volume.
The higher the turnover of operating assets, the greater will be the revenue in
relation to the fixed charge on those assets.

Operating leverage is a function of three factors: (i) Rupee amount of fixed cost,

4.50
Financing Decisions

(ii)

Variable contribution margin, and

(iii) Volume of sales. Operating leverage is the ratio of net operating income
before fixed charges to net operating income after fixed charges. Degree of
operating leverage is equal to the percentage increase in the net operating income
to the percentage increase in the output.
OL = N(P − V ) N(P − V ) − F

Where, OL N P V F = = = = = Operating leverage Number of units sold Selling price


per unit Variable cost per unit Fixed cost

Degree of operating leverage =

Percentage increase in net operating income Percentage increase in output

Operating leverage is directly proportional to business risk. More operating


leverage leads to more business risk, for then a small sales decline causes a big
profit. This can be illustrated graphically as:

4.51
Financial Management

Illustration 1: A Company produces and sells 10,000 shirts. The selling price per
shirt is Rs. 500. Variable cost is Rs. 200 per shirt and fixed operating cost is
Rs. 25,00,000.

(a) Calculate operating leverage. (b) If sales are up by 10%, then what is the
impact on EBIT?
Solution

(a)

Statement of Profitability Rs.

Sales Revenue (10,000 × 500)


Less: Variable Cost (10,000 × 200)

50,00,000 20,00,000 30,00,000 25,00,000 5,00,000

Contribution
Less: Fixed Cost

EBIT

Operating Leverage =
(b)

Contribution 30 lakhs = = 6 times 5 lakhs EBIT

OL =
6=

% ∆ in EBIT % ∆ in sales

x / 5,00,000 5,00,000 50,00,000

x = 30,000 ∴ ∆EBIT = 30,000/5,00,000 = 6%


3.3.2 FINANCIAL LEVERAGE

Financial leverage (FL) maybe defined as ‘the use of funds with a fixed cost in
order to increase earnings per share.’ In other words, it is the use of company
funds on which it pays a limited return. Financial leverage involves the use of
funds obtained at a fixed cost in the hope of increasing the return to common
stockholders. Degree of financial leverage is the ratio of the percentage increase
in earning per share (EPS) to the percentage increase in earnings before interest
and taxes (EBIT).

Degree of financial leverage =

Percentage increase in earning per share (EPS) Percentage increase in earnings


before interest and tax (EBIT)

4.52
Financing Decisions

FL =
Or, Where,

Y Y −I EBIT EBIT − Interest

FL =

Y = EBIT at a point for which the degree of financial leverage is being calculated
I = Amount of interest charges
Illustration 2: Suppose there are two firms with the same operating leverage,
business risk, and probability distribution of EBIT and only differ with respect
to their use of debt (capital structure). Firm U Firm L

No debt 40% tax rate


Firm U: Unleveraged

Rs. 10,000 of 12% debt 40% tax rate


Economy Bad Avg. Good

Rs. 20,000 in assets Rs. 20,000 in assets

Probability EBIT Interest EBIT Taxes (40%) NI


Firm L: Leveraged

0.25 Rs. 2,000 0 Rs. 2000 800 Rs. 1,200

0.50 Rs. 3,000 0 Rs. 3,000 1,200 Rs. 1,800

0.25 Rs. 4,000 0 Rs. 4,000 1,600 Rs. 2,400

Probability EBIT Interest EBIT

Bad 0.25 Rs. 2,000 1,200 Rs. 800

Economy Avg. 0.50 Rs. 3,000 1,200 Rs. 1,800

Good 0.25 Rs. 4,000 1,200 Rs. 2,800

4.53
Financial Management

Taxes (40%) NI *Same as for Firm U.

320 Rs. 480

720 Rs.1080

1,120 Rs. 1,680

Ratio comparison between leveraged and unleveraged firms FIRM U Bad Avg. Good

BEP(=EBIT/TOTAL ASSETS) ROE(=PAT/NETWORTH) TIE(INTEREST COVERAGE RATIO


(=EBIT/INTEREST)

10.0% 6.0% ∞

15.0% 9.0% ∞

20.0% 12.0% ∞

FIRM L

Bad

Avg.

Good

BEP ROE TIE

10.0% 4.8% 1.67%

15.0% 10.8% 2.50%

20.0% 16.8% 3.30%

Risk and return for leveraged and unleveraged firms Expected Values:

Firm U E(BEP) E(ROE) E(TIE)


Risk Measures:

Firm L 15.0% 10.8% 2.5x Firm L 4.24% 0.39

15.0% 9.0% ∞ Firm U

σROE CVROE

2.12% 0.24

4.54
Financing Decisions

Thus, the effect of leverage on profitability and debt coverage can be seen from
the above example. For leverage to raise expected ROE, BEP must be greater than kd
i.e. BEP > kd because if kd > BEP, then the interest expense will be higher than
the operating income produced by debt-financed assets, so leverage will depress
income. As debt increases, TIE decreases because EBIT is unaffected by debt, and
interest expense increases (Int Exp = kdD). Thus, it can be concluded that the
basic earning power (BEP) is unaffected by financial leverage. Firm L has higher
expected ROE because BEP > kd and it has much wider ROE (and EPS) swings because
of fixed interest charges. Its higher expected return is accompanied by higher
risk.
3.3.3 DEGREE OF COMBINED LEVERAGE

Combined leverage maybe defined as the potential use of fixed costs, both
operating and financial, which magnifies the effect of sales volume change on the
earning per share of the firm. Degree of combined leverage (DCL) is the ratio of
percentage change in earning per share to the percentage change in sales. It
indicates the effect the sales changes will have on EPS.

Degree of combined leverage = Degree of operating leverage × Degree of financial


leverage
DCL = DOL × DFL
Where, DCL = Degree of combined leverage DOL = Degree of operating leverage DFL =
Degree of financial leverage

Degree of combined leverage =


Illustration 3: A firm’s details are as under:

Percentage change in EPS Percentage change in sales

Sales (@100 per unit) Variable Cost Fixed Cost

Rs. 24,00,000 50% Rs. 10,00,000

It has borrowed Rs. 10,00,000 @ 10% p.a. and its equity share capital is Rs.
10,00,000 (Rs. 100 each)

4.55
Financial Management

Calculate: (a) Operating Leverage (b) Financial Leverage (c) Combined Leverage (d)
Return on Investment (e) If the sales increases by Rs. 6,00,000; what will the new
EBIT?
Solution Rs.

Sales
Less: Variable cost

24,00,000 12,00,000 12,00,000 10,00,000 2,00,000 1,00,000 1,00,000 50,000 50,000


10,000 5
12,00,000 = 6 times 2,00,000

Contribution
Less: Fixed cost

EBIT
Less: Interest

EBT
Less: Tax (50%)

EAT No. of equity shares EPS (a) Operating Leverage = (b) Financial Leverage =

2,00,000 = 2 times 1,00,000

(c) Combined Leverage = OL × FL = 6 × 2 = 12 times. (d) R.O.I =


50,000 × 100 = 5% 10,00,000

(e) Operating Leverage = 6

6=

∆ EBIT .25

4.56
Financing Decisions

∆ EBIT =

6 ×1 = 1.5 4

Increase in EBIT = Rs. 2,00,000 × 1.5 = Rs. 3,00,000 New EBIT = 5,00,000
Illustration 4: Betatronics Ltd. has the following balance sheet and income
statement information: Balance Sheet as on March 31st Liabilities (Rs.)

Assets

(Rs.)

Equity capital (Rs. 10 per share) 10% Debt Retained earnings Current liabilities

8,00,000 Net fixed assets 6,00,000 Current assets 3,50,000 1,50,000 19,00,000

10,00,000 9,00,000

19,00,000

Income Statement for the year ending March 31 (Rs.)

Sales Operating expenses depreciation) EBIT


Less: Interest

3,40,000 (including Rs. 60,000 1,20,000 2,20,000 60,000 1,60,000 56,000 1,04,000

Earnings before tax


Less: Taxes

Net Earnings (EAT)

(a) Determine the degree of operating, financial and combined leverages at the
current sales level, if all operating expenses, other than depreciation, are
variable costs. (b) If total assets remain at the same level, but sales (i)
increase by 20 percent and (ii) decrease by 20 percent, what will be the earnings
per share at the new sales level?

4.57
Financial Management

Solution (a) Calculation of Degree of Operating (DOL), Financial (DFL) and


Combined leverages (DCL).
DOL = Rs.3,40,000 − Rs.60,000 Rs.2,20,000 Rs.2,20,000 Rs.1,60,000

= 1.27
DFL =

= 1.37 DCL = DOL×DFL = 1.27×1.37 = 1.75


(b) Earnings per share at the new sales level Increase by 20% (Rs.) Decrease by
20% (Rs.)

Sales level
Less: Variable expenses Less: Fixed cost

4,08,000 72,000 60,000 2,76,000 60,000 2,16,000 75,600 1,40,400 80,000 1.75

2,72,000 48,000 60,000 1,64,000 60,000 1,04,000 36,400 67,600 80,000 0.84

Earnings before interest and taxes


Less: Interest

Earnings before taxes


Less: Taxes

Earnings after taxes (EAT) Number of equity shares EPS


Working Notes:

(i) (ii)

Variable Costs = Rs. 60,000 (total cost − depreciation) Variable Costs at:

(a) Sales level, Rs. 4,08,000 = Rs. 72,000 (b) Sales level, Rs. 2,72,000 = Rs.
48,000
4.58
Financing Decisions

Illustration 5: Calculate the operating leverage, financial leverage and combined


leverage from the following data under Situation I and II and Financial Plan A and
B:

Installed Capacity Actual Production and Sales Selling Price Variable Cost Fixed
Cost: Under Situation I Under Situation-II
Capital Structure:

4,000 units 75% of the Capacity Rs. 30 Per Unit Rs. 15 Per Unit Rs. 15,000
Rs.20,000

Financial Plan A Rs. B Rs.

Equity Debt (Rate of Interest at 20%)

10,000 10,000 20,000

15,000 5,000 20,000

Solution Operating Leverage: Situation-I Rs. 90,000 Situation-II Rs. 90,000

Sales (s) 3000 units @ Rs. 30/- per unit Less: Variable Cost (VC) @ Rs. 15 per
unit Contribution (C) Less: Fixed Cost (FC) Operating Profit (OP) (EBIT)

45,000 45,000 15,000 30,000

45,000 45,000 20,000 25,000

4.59
Financial Management

(i)

Operating Leverage

C = OP
= (ii)

Rs.

45,000 30,000

Rs. 1.8

45,000 25,000

1.5

Financial Leverages A (Rs.) Situation 1 B (Rs.)

Operating Profit (EBIT)


Less: Interest on debt

30,000 2,000 28,000

30,000 1,000 29,000

PBT

Financial Leverage =

OP 30,000 = Rs. PBT 28,000

= 1 . 07

Rs.

30,000 24,000

= 1 . 04

A (Rs.) Situation-II Operating Profit (OP) (EBIT) Less: Interest on debt PBT

B (Rs.)

25,000 2,000 23,000

25,000 1,000 24,000

Financial Leverage = (iii) Combined Leverages

OP 25,000 = Rs. = 1.09 PBT 23,000

Rs.

25,000 = 1.04 24,000


(a) (b)

Situation I Situation II

A (Rs.) 1.5 x 1.07 =1.6 1.8 x 1.09 =1.96

B (Rs.) 1.5 x 1.04 = 1.56 1.8 x 1.04 =1.87

4.60
Financing Decisions

Self Examination Questions A. Objective Type Questions

1.

A firm's cost of capital is the: (a) Cost of borrowing money (b) Cost of issuing
stock (c) Cost of bonds (d) Overall cost of financing to the firm.

2.

The cost of debt financing is generally __________ the cost of preferred or common
equity financing. (a) Less than (b) More than (c) Equal to (d) Not enough
information to tell.

3.

The cost of issuing new stock is called: (a) (b) The cost of equity Flotation
costs

(c) Marginal cost of capital (d) None of the above. 4. The cost of each component
of a firm's capital structure multiplied by its weight in the capital structure is
called the: (a) Marginal cost of capital (b) The cost of debt (c) 5. Weighted
average cost of capital (d) None of the above. When establishing their optimal
capital structure, firms should strive to: (a) Minimize the weighted average cost
of capital (b) Minimize the amount of debt financing used (c) Maximize the
marginal cost of capital (d) None of the above.

4.61
Financial Management

6.

The ____________________ is the percentage change in operating income that results


from a percentage change in sales. (a) Degree of financial leverage (b) Breakeven
point (c) Degree of operating leverage (d) Degree of combined leverage.

7.

If interest expenses for a firm rise, we know that firm has taken on more
______________. (a) Financial leverage (b) Operating leverage (c) Fixed assets (d)
None of the above.

8.

The ________________ is the percentage change in earnings per share that results
from a percentage change in operating income. (a) Degree of combined leverage (b)
Degree of financial leverage (c) Breakeven point (d) Degree of operating leverage.

9.

Combined leverage is the percentage change in relationship between sales and


____________. (a) Operating income (b) Operating leverage (c) Earnings per share
(d) Breakeven point.

10. A highly leveraged firm is __________ risky than its peers. (a) Less (b) More
(c) The same (d) None of the above.

4.62
Financing Decisions

11. An advantage of debt financing is: (a) Interest payments are tax deductible
(b) The use of debt, up to a point, lowers the firm's cost of capital (c) Does not
dilute owner's earnings (d) All of the above. 12. The cost of equity capital is
all of the following EXCEPT: (a) The minimum rate that a firm should earn on the
equity-financed part of an investment (b) A return on the equity-financed portion
of an investment that, at worst, leaves the market price of the tock unchanged (c)
By far the most difficult component cost to estimate (d) Generally lower than the
before-tax cost of debt. 13. In calculating the costs of the individual components
of a firm’s financing, the corporate tax rate is important to which of the
following component cost formulae? (a) Common stock (b) Debt (c) Preferred stock
(d) None of the above. 14. Market values are often used in computing the weighted
average cost of capital because (a) This is the simplest way to do the calculation
(b) This is consistent with the goal of maximizing shareholder value (c) This is
required in India by the SEBI (d) This is a very common mistake. 15. An EBIT-EPS
indifference analysis chart is used for_______ (a) Evaluating the effects of
business risk on EPS (b) Examining EPS results for alternative financing plans at
varying EBIT levels (c) Determining the impact of a change in sales on EBIT (d)
Showing the changes in EPS quality over time.
Answers to Objective Type Questions 1. (d); 2. (a); 3. (b); 4. (c); 5. (a); 6.
(c); 7. (a); 8. (b); 9. (c); 10. (b); 11. (d); 12. (d); 13. (b); 14. (b); 15. (b)
4.63
Financial Management

B. Short Answer Type Questions

1.

Write short notes on: (a) Weighted average cost of capital (b) Marginal cost of
capital (c) Indifference point.

2.

Differentiate between the following: (a) Business risk and Financial risk (b)
Operating leverage and Financial leverage (c) Net Income approach and Net
Operating Income approach of Capital Structure.

C. Long Answer Type Questions

1. 2. 3. 4. 5. 6. 7. 8. 9.

Explain briefly major considerations in capital structure planning. Explain


briefly the four approaches for determining the cost of equity shares. Explain
briefly, Modigliani and Miller approach on cost of capital. Explain briefly the
traditional theory of capital structure. Define Operating Leverage and Financial
Leverage. How the two leverages can be measured? Explain the impact of Financial
Leverage on earning per share. What is Combined Leverage? Explain its significance
in financial planning of a firm. Explain the advantages of equity financing. What
are the advantages of debt financing from the point of company and investors?

D. Practical Problems

1.

(a) A company issues Rs. 10,00,000 16% debentures of Rs. 100 each. The company is
in 35% tax bracket. You are required to calculate the cost of debt after tax. If
debentures are issued at (i) Par, (ii) 10% discount and (iii) 10% premium. (b) If
brokerage is paid at 2% what will be cost of debentures if issue is at par? A
company's share is quoted in market at Rs. 40 currently. A company pays a dividend
of Rs. 2 per share and investors expect a growth rate of 10% per year, compute:
(a) The company’s cost of equity capital. (b) If anticipated growth rate is 11%
p.a. calculate the indicated market price per share.

2.

4.64
Financing Decisions

(c) If the company’s cost of capital is 16% and anticipated growth rate is 10%
p.a., calculate the market price if dividend of Rs. 2 per share is to be
maintained. 3. Three companies A, B & C are in the same type of business and hence
have similar operating risks. However, the capital structure of each of them is
different and the following are the details:
A B C

Equity Share capital [Face value Rs. 10 per share] Market value per share Dividend
per share Debentures [Face value per debenture Rs. 100] Market value per debenture
Interest rate

Rs. Rs. Rs. Rs. Rs.

4,00,000 15 2.70 Nil — —

2,50,000 20 4 1,00,000 125 10%

5,00,000 12 2.88 2,50,000 80 8%

Assume that the current levels of dividends are generally expected to continue
indefinitely and the income tax rate at 50%. You are required to compute the
weighted average cost of capital of each company. 4. ZED Limited is presently
financed entirely by equity shares. The current market value is Rs. 6,00,000. A
dividend Rs. 1,20,000 has just been paid. This level of dividends is expected to
be paid indefinitely. The company is thinking of investing in a new project
involving a outlay of Rs. 5,00,000 now and is expected to generate net cash
receipts of Rs. 1,05,000 per annum indefinitely. The project would be financed by
issuing Rs. 5,00,000 debentures at the market interest rate of 18%. Ignoring tax
consideration: (1) Calculate the value of equity shares and the gain made by the
shareholders if the cost of equity rises to 21.6%. (2) Prove that weighted average
cost of capital is not affected by gearing. 5. The following figures are made
available to you: Net profits for the year
Less: Interest on secured debentures at 15% p.a.

18,00,000

4.65
Financial Management

(Debentures were issued 3 months after the commencement of the year) Profit before
tax
Less: Income-tax at 35% and dividend distribution

1,12,500 16,87,500 8,43,750 8,43,750 1,00,000 Rs. 109.70

Tax Profit after tax Number of equity shares (Rs. 10 each) Market quotation of
equity share

The company has accumulated revenue reserves of Rs. 12 lakhs. The company is
examining a project calling for an investment obligation of Rs. 10 lakhs. This
investment is expected to earn the same rate as funds already employed. You are
informed that a debt equity ratio (Debt divided by debt plus equity) higher than
60% will cause the price earning ratio to come down by 25% and the interest rate
on additional borrowals will cost company 300 basis points more than on their
current borrowal in secured. You are required to advise the company on the
probable price of the equity share, if (a) The additional investment were to be
raised by way loans; or (b) The additional investment were to be raised by way of
equity. 6. The Modern Chemicals Ltd. requires Rs. 25,00,000 for a new plant. This
plant is expected to yield earnings before interest and taxes of Rs. 5,00,000.
While deciding about the financial plan, the company considers the objective of
2aximising earnings per share. It has three alternatives to finance the project—by
raising debt of Rs. 2,50,000 or Rs. 10,00,000 or Rs. 15,00,000 and the balance, in
each case, by issuing equity shares. The company's share is currently selling at
Rs. 150, but is expected to decline to Rs. 125 in case the funds are borrowed in
excess of Rs. 10,00,000. The funds can be borrowed at the rate of 10% upto Rs.
2,50,000, at 15% over Rs. 2,50,000 and upto Rs. 10,00,000 and at 20% over Rs.
10,00,000. The tax rate applicable to the company is 50%. Which form of financing
should the company choose ? A company provides the following figures:
Rs.

7.

Profit
Less: Interest on debentures @ 12%

26,00,000 6,00,000 20,00,000

Profit before tax


4.66
Financing Decisions

Less: Income-tax @ 50%

10,00,000 10,00,000 4,00,000 2.50 25 10

Profit after tax Number of equity shares (of Rs. 10 each) Earning per share (EPS)
Ruling price in market P/E (Price/Earning) Ratio The company has undistributed
reserves of Rs. 60,00,000.

The company needs Rs. 20,00,000 for expansion; this amount will earn the same rate
as funds already employed. You are informed that a debt equity ratio higher than
35% pulls the PE ratio down to 8 and raises the interest rate on additional amount
borrowed at 14%. You are required to ascertain the probable price of the share
if(i) (ii) 8. The additional funds are raised as a loans; or The amount is raised
by issuing equity shares.

EXE Limited is considering three financing plans. The key information is as


follows: (a) Total investment to be raised is Rs. 2,00,000 (b) Plans of Financing
Proportion
Plan Equity Debt Preference Shares —

A B C

100% 50% 50%

— 50 -

— 50%

(c) Cost of debt 8%; Cost of preference 8% (d) Tax rate 50% (Assume no dividend
tax) (e) Equity shares of face value of Rs. 10 each will be issued at a premium of
Rs. 10 per share. (g) (i) Expected EBIT is Rs. 80,000 Earning per share
4.67

You are required to determine for each plan:


Financial Management

(ii) (iii) 9.

The Financial break-even point. Compute the EBIT range among the plans of
indifference. Sales Debt/Equity Interest rate Operating profit Rs. 50,000 3:1 12%
Rs. 20,00,000

Calculate the Operating Leverage from the following data:

10. Calculate the Degree of Operating Leverage, Degree of Financial Leverage and
the Degree of Combined Leverage for the following firms and interpret the results:
Output (unit) Fixed Costs (Rs.) Unit Variable costs (Rs) Interest expenses (Rs.)
Unit Selling price (Rs.)
P 3,00,000 Q 75,000 R 5,00,000

3,50,000 1.00 25,000 3.00

7,00,000 7.50 40,000 25.00

5,00,000 0.10 – 0.50

11. XYZ Ltd. sells 2000 units @ Rs. 10 per unit. The variable cost of production
is Rs. 7 and Fixed cost is Rs. 1,000. The company raised the required funds by
issue of 100, 10% deben- tures @ Rs. 100 each and 2000 equity shares @ Rs. 10 per
share. The sales of XYZ Ltd. are expected to increase by 20%. Assume tax rate of
company is 50%. You are required to calculate the impact of increase in sales on
earning per share. 12. The following figures relate to two companies:
P Ltd. 500 200 300 150 150 50 100 (Rs. in lakhs) Q Ltd. 1,000 300 700 400 300 100
200

Sales Less : Variable costs Contribution Less : Fixed costs EBIT Less : Interest
Profit before tax (PBT)

4.68
Financing Decisions

You are required to : (i) (ii) Calculate the operating, financial and combined
leverages for the two companies; and Comment on the relative risk position of
them.

13. The capital structure of ABC Ltd. consist of an ordinary share capital of Rs.
5,00,000 (equity shares of Rs. 100 each at par value) and Rs. 5,00,000 (10%
debenture of Rs. 100 each). Sales increased from 50,000 units to 60,000 units, the
selling price is Rs. 12 per unit, variable cost amounts to Rs. 8 per unit and
fixed expenses amount to Rs. 1,00,000. The income tax rate is assumed to be 50%.
You are required to calculate the following: (a) The percentage increase in
earning per share; (b) The degree of financial leverage at 50,000 units and 60,000
units; (c) The degree of operating leverage at 50,000 units and 60,000 units; (d)
Comment on the behaviour E.P.S., operating and financial leverage in relation to
increases in sales from 50,000 units to 60,000 units. 14. A firm has sales of Rs.
75,00,000, variable cost of Rs. 42,00,000 and fixed cost of Rs. 6,00,000. It has a
debt of Rs. 45,00,000 at 9% and equity of Rs. 55,00,000. (i) (ii) What is the
firm’s R.O.I.? Does it have favourable financial leverage?

(iii) What are the operating, financial and combined leverage of the firm ? (iv)
If the sales drop to Rs. 50,00,000, what will be the new E.B.I.T. ? 15. Calculate
the operating leverage, financial leverage and combined leverage from the
following data under Situations I and II and Financial Plan A and B : Installed
Capacity Actual Production and Sales Selling Price Variable Cost Fixed Cost: Under
Situation I Under Situation II Rs. 15,000 Rs. 20,000 4,000 units 75% of the
Capacity Rs. 30 Per Unit Rs. 15 Per Unit

4.69
Financial Management

Financial Plan A Rs. B Rs.

Equity Debt (Rate of Interest at 20%)

10,000 10,000 20,000

15,000 5,000 20,000

16. From the following prepare Income statement of Company A, B and C. Company
Financial Leverage Interest Operating Leverage Variable cost as a percentage to
sales sales Income tax rate A 3:1 Rs. 200 4:1 66 2/3% 45% B 4:1 Rs. 300 5:1 75%
45% C 2:1 Rs. 1,000 3:1 50% 45%

4.70
CHAPTER

TYPES OF FINANCING
Learning Objectives After studying this chapter, you will be able to ♦ Understand
the different sources of finance available to a business; ♦ Differentiate between
the various long term, medium term and short term sources of finance; ♦ Understand
the meaning and purpose of Venture Capital financing; ♦ Understand the meaning and
purpose of securitisation and debt securitization; ♦ Understand the concept of
lease financing; ♦ Understand the financing of export trade by banks; and ♦
Understand the various financial instruments dealt with in the International
market. 1. INTRODUCTION

One of the most important consideration for an entrepreneur–company in


implementing a new project or undertaking expansion, diversification,
modernisation and rehabilitation scheme is ascertaining the cost of project and
the means of finance. There are several sources of finance/funds available to any
company. An effective appraisal mechanism of various sources of funds available to
a company must be instituted in the company to achieve its main objectives. Such a
mechanism is required to evaluate risk, tenure and cost of each and every source
of fund. The selection of the fund source is dependent on the financial strategy
pursued by the company, the leverage planned by the company, the financial
conditions prevalent in the economy and the risk profile of both the company as
well as the industry in which the company operates. Each and every source of fund
has some advantages as well as disadvantages. 2. FINANCIAL NEEDS AND SOURCES OF
FINANCE OF A BUSINESS

2.1 Financial Needs of a Business: Business enterprises need funds to meet their
different
Financial Management types of requirements. All the financial needs of a business
may be grouped into the following three categories: (i) Long term financial needs:
Such needs generally refer to those requirements of funds which are for a period
exceeding 5-10 years. All investments in plant, machinery, land, buildings, etc.,
are considered as long term financial needs. Funds required to finance permanent
or hard core working capital should also be procured from long term sources. (ii)
Medium term financial needs: Such requirements refer to those funds which are
required for a period exceeding one year but not exceeding 5 years. For example,
if a company resorts to extensive publicity and advertisement campaign then such
type of expenses may be written off over a period of 3 to 5 years. These are
called deferred revenue expenses and funds required for them are classified in the
category of medium term financial needs. Sometimes long term requirements, for
which long term funds cannot be arranged immediately may be met from medium term
sources and thus the demand of medium term financial needs, are generated. As and
when the desired long term funds are made available, medium term loans taken
earlier may be paid off. (iii) Short term financial needs: Such type of financial
needs arise to finance in current assets such as stock, debtors, cash, etc.
Investment in these assets is known as meeting of working capital requirements of
the concern. Firms require working capital to employ fixed assets gainfully. The
requirement of working capital depends upon a number of factors which may differ
from industry to industry and from company to company in the same industry. The
main characteristic of short term financial needs is that they arise for a short
period of time not exceeding the accounting period. i.e., one year. The basic
principle for meeting the short term financial needs of a concern is that such
needs should be met from short term sources, and for medium term financial needs
from medium term sources and long term financial needs from long term sources.
Accordingly, the method of raising funds is to be decided with reference to the
period for which funds are required. Basically, there are two sources of raising
funds for any business enterprise. viz., owner’s capital and borrowed capital. The
owner’s capital is used for meeting long term financial needs and it primarily
comes from share capital and retained earnings. Borrowed capital for all the other
types of requirement can be raised from different sources such as debentures,
public deposits, loans from financial institutions and commercial banks, etc. The
following section shows at a glance the different sources from where the three
aforesaid types of finance can be raised in India.

5.2
Types of Financing

2.2 Sources of Finance of a Business (i) 1. 2. 3. 4. 5. 6. 7. 8. 9. (ii) 1. 2. 3.


4. 5. 6. 7. 8. 9. Long-term Share capital or Equity share Preference shares
Retained earnings Debentures/Bonds of different types Loans from financial
institutions Loans from State Financial Corporation Loans from commercial banks
Venture capital funding Asset securitisation Medium-term Preference shares
Debentures/Bonds Public deposits/fixed deposits for duration of three years
Commercial banks Financial institutions State financial corporations Lease
financing/Hire-Purchase financing External commercial borrowings Euro-issues

10. International financing like Euro-issues, Foreign currency loans

10. Foreign Currency bonds (iii) Short-term 1. 2. Trade credit Accrued expenses
and deferred income
5.3
Financial Management 3. 4. 5. 6. Commercial banks Fixed deposits for a period of 1
year or less Advances received from customers Various short-term provisions

It is evident from the above section that funds can be raised from the same source
for meeting different types of financial requirements. 2.3 Financial sources of a
business can also be classified as follows by using different basis : 1. (i) (ii)
2. (i) 3. (i) (ii) According to period: Long term sources Medium term sources
According to ownership: Owners capital or equity capital, retained earnings etc.
According to source of generation: Internal sources e.g. retained earnings and
depreciation funds etc. External sources e.g. debentures, loans etc.

(iii) Short term sources

(ii) Borrowed capital such as debentures, public deposits, loans etc.

However for the sake of convenience, the different sources of funds can also be
classified into following categories. (i) (ii) Security financing - financing
through shares and debentures. Internal financing - financing through retained
earning, depreciation.

(iii) Loans financing - this includes both short term and long term loans. (iv)
International financing. (v) Other sources. 3. LONG TERM SOURCES OF FINANCE There
are different sources of funds available to meet long term financial needs of the
5.4
Types of Financing

business. These sources may be broadly classified into share capital (both equity
and preference) and debt (including debentures, long term borrowings or other debt
instruments). In recent times in India, many companies have raised long term
finance by offering various instruments to public like deep discount bonds, fully
convertible debentures etc. These new instruments have characteristics of both
equity and debt and it is difficult to categorised these either as debt or equity.
The different sources of long term finance can now be discussed: 3.1 Owners
Capital or Equity Capital : A public limited company may raise funds from
promoters or from the investing public by way of owners capital or equity capital
by issuing ordinary equity shares. Ordinary equity shares are a source of
permanent capital. Ordinary shareholders are owners of the company and they
undertake the risks of business. They are entitled to dividends after the income
claims of other stakeholders are satisfied. Similarly, in the event of winding up,
ordinary shareholders can exercise their claim on assets after the claims of the
other suppliers of capital have been met. They elect the directors to run the
company and have the optimum control over the management of the company. Since
equity shares can be paid off only in the event of liquidation, this source has
the least risk involved. This is more so due to the fact that equity shareholders
can be paid dividends only when there are distributable profits. However, the cost
of ordinary shares is usually the highest. This is due to the fact that such
shareholders expect a higher rate of return on their investment as compared to
other suppliers of long-term funds. Such behaviour is directly related to the risk
undertaken by ordinary shareholders when compared to the providers of other forms
of capital e.g. debt. Whereas, an ordinary shareholder shall take responsibility
of losses incurred by the company by foregoing dividend or accepting a lesser
amount, a debt holder shall be statutorily entitled to get regular payments as per
the contract. Hence, when compared to those who have provided loan capital to the
company, ordinary shareholders carry a higher amount of risk and so expect a
higher return. Further, the dividend payable on shares is an appropriation of
profits and not a charge against profits. This means that unlike debt, ordinary
equity shares do not provide any tax shield to the company, thereby resulting in a
higher cost. Ordinary share capital also provides a security to other suppliers of
funds. Thus, a company having substantial ordinary share capital may find it
easier to raise further funds, in view of the fact that share capital provides a
security to other suppliers of funds. The Companies Act, 1956 and SEBI Guidelines
for disclosure and investors' protections and the clarifications thereto lay down
a number of provisions regarding the issue and management of equity shares
capital.

5.5
Financial Management Advantages and disadvantages of raising funds by issue of
equity shares are : (i) (ii) It is a permanent source of finance. Since such
shares are not redeemable, the company has no liability for cash outflows
associated with its redemption. Equity capital increases the company’s financial
base and thus helps further the borrowing powers of the company.

(iii) The company is not obliged legally to pay dividends. Hence in times of
uncertainties or when the company is not performing well, dividend payments can be
reduced or even suspended. (iv) The company can make further issue of share
capital by making a right issue. Apart from the above mentioned advantages, equity
capital has some disadvantages to the company when compared with other sources of
finance. These are as follows: (i) (ii) The cost of ordinary shares is higher
because dividends are not tax deductible and also the floatation costs of such
issues are higher. Investors find ordinary shares riskier because of uncertain
dividend payments and capital gains.

(iii) The issue of new equity shares reduces the earning per share of the existing
shareholders until and unless the profits are proportionately increased. (iv) The
issue of new equity shares can also reduce the ownership and control of the
existing shareholders. 3.2 Preference Share Capital: These are a special kind of
shares; the holders of such shares enjoy priority, both as regards to the payment
of a fixed amount of dividend and repayment of capital on winding up of the
company. Long-term funds from preference shares can be raised through a public
issue of shares. Such shares are normally cumulative, i.e., the dividend payable
in a year of loss gets carried over to the next year till there are adequate
profits to pay the cumulative dividends. The rate of dividend on preference shares
is normally higher than the rate of interest on debentures, loans etc. Most of
preference shares these days carry a stipulation of period and the funds have to
be repaid at the end of a stipulated period. Preference share capital is a hybrid
form of financing which imbibes within itself some characteristics of equity
capital and some attributes of debt capital. It is similar to equity because
preference dividend, like equity dividend is not a tax deductible payment. It

5.6
Types of Financing

resembles debt capital because the rate of preference dividend is fixed.


Typically, when preference dividend is skipped it is payable in future because of
the cumulative feature associated with most of preference shares. Cumulative
Convertible Preference Shares (CCPs) may also be offered, under which the shares
would carry a cumulative dividend of specified limit for a period of say three
years after which the shares are converted into equity shares. These shares are
attractive for projects with a long gestation period. Preference share capital may
be redeemed at a pre decided future date or at an earlier stage inter alia out of
the profits of the company. This enables the promoters to withdraw their capital
from the company which is now self-sufficient, and the withdrawn capital may be
reinvested in other profitable ventures. It may be mentioned that irredeemable
preference shares cannot be issued by any company. Preference shares have gained
importance after the Finance bill 1997 as dividends became tax exempted in the
hands of the individual investor and are taxable in the hands of the company as
tax is imposed on distributed profits at a flat rate. At present, a domestic
company paying dividend will have to pay dividend distribution tax @ 12.5% plus
surcharge of 10% plus an education cess equalling 2% (total 14.025%). Advantages
and disadvantages of raising funds by issue of preference shares are: (i) (ii) No
dilution in EPS on enlarged capital base - If equity is issued it reduces EPS,
thus affecting the market perception about the company. There is leveraging
advantage as it bears a fixed charge. Non payment of preference dividends does not
force company into liquidity.

(iii) There is no risk of takeover as the preference shareholders do not have


voting rights except in case where dividend arrears exist. (iv) The preference
dividends are fixed and pre decided. Hence Preference shareholders do not
participate in surplus profits as the ordinary shareholders. (v) Preference
capital can be redeemed after a specified period. The following are the
disadvantages of the preference shares: (i) One of the major disadvantages of
preference shares is that preference dividend is not tax deductible and so does
not provide a tax shield to the company. Hence a preference share is costlier to
the company than debt e.g. debenture.

5.7
Financial Management (ii) Preference dividends are cumulative in nature. This
means that although these dividends may be omitted, they shall need to be paid
later. Also, if these dividends are not paid, no dividend can be paid to ordinary
shareholders. The non payment of dividend to ordinary shareholders could seriously
impair the reputation of the company concerned. 3.3 Retained Earnings: Long-term
funds may also be provided by accumulating the profits of the company and by
ploughing them back into business. Such funds belong to the ordinary shareholders
and increase the net worth of the company. A public limited company must plough
back a reasonable amount of profit every year keeping in view the legal
requirements in this regard and its own expansion plans. Such funds also entail
almost no risk. Further, control of present owners is also not diluted by
retaining profits. 3.4 Debentures or Bonds: Loans can be raised from public by
issuing debentures or bonds by public limited companies. Debentures are normally
issued in different denominations ranging from Rs. 100 to Rs. 1,000 and carry
different rates of interest. By issuing debentures, a company can raise long term
loans from public. Normally, debentures are issued on the basis of a debenture
trust deed which lists the terms and conditions on which the debentures are
floated. Debentures are either secured or unsecured. As compared with preference
shares, debentures provide a more convenient mode of longterm funds. The cost of
capital raised through debentures is quite low since the interest payable on
debentures can be charged as an expense before tax. From the investors' point of
view, debentures offer a more attractive prospect than the preference shares since
interest on debentures is payable whether or not the company makes profits.
Debentures are thus instruments for raising long-term debt capital. Secured
debentures are protected by a charge on the assets of the company. While the
secured debentures of a wellestablished company may be attractive to investors,
secured debentures of a new company do not normally evoke same interest in the
investing public. Debentures can be straight debentures or convertible debentures.
A convertible debenture is the type which can be converted, either fully or
partly, into shares after a specified period of time. Debentures can be divided
into the following three categories: (i) (ii) Non convertible debentures – These
types of debentures do not have any feature of conversion and are repayable on
maturity. Fully convertible debentures – Such debentures are converted into equity
shares as per the terms of issue in relation to price and the time of conversion.
Interest rates on such debentures are generally less than the non convertible
debentures because of their carrying the attractive feature of getting themselves
converted into shares.

5.8
Types of Financing

(iii) Partly convertible debentures – Those debentures which carry features of a


convertible and a non convertible debenture belong to this category. The investor
has the advantage of having both the features in one debenture. Advantages of
raising finance by issue of debentures are: (i) The cost of debentures is much
lower than the cost of preference or equity capital as the interest is tax-
deductible. Also, investors consider debenture investment safer than equity or
preferred investment and, hence, may require a lower return on debenture
investment. Debenture financing does not result in dilution of control.

(ii)

(iii) In a period of rising prices, debenture issue is advantageous. The fixed


monetary outgo decreases in real terms as the price level increases. The
disadvantages of debenture financing are: (i) (ii) Debenture interest and capital
repayment are obligatory payments. The protective covenants associated with a
debenture issue may be restrictive.

(iii) Debenture financing enhances the financial risk associated with the firm.
(iv) Since debentures need to be paid during maturity, a large amount of cash
outflow is needed at that time. These days many companies are issuing convertible
debentures or bonds with a number of schemes/incentives like warrants/options etc.
These bonds or debentures are exchangeable at the option of the holder for
ordinary shares under specified terms and conditions. Thus for the first few years
these securities remain as debentures and later they can be converted into equity
shares at a pre-determined conversion price. The issue of convertible debentures
has distinct advantages from the point of view of the issuing company. Firstly,
such an issue enables the management to raise equity capital indirectly without
diluting the equity holding, until the capital raised has started earning an added
return to support the additional shares. Secondly, such securities can be issued
even when the equity market is not very good. Thirdly, convertible bonds are
normally unsecured and, therefore, their issuance may ordinarily not impair the
borrowing capacity. These debentures/bonds are issued subject to the SEBI
guidelines notified from time to time. Public issue of debentures and private
placement to mutual funds now require that the issue be rated by a credit rating
agency like CRISIL (Credit Rating and Information Services of India Ltd.). The
credit rating is given after evaluating factors like track record of the company,
profitability, debt servicing capacity, credit worthiness and the perceived risk
of lending.
5.9
Financial Management 3.5 Loans from Financial Institutions: In India specialised
institutions provide long- term financial assistance to industry. Thus, the
Industrial Finance Corporation of India, the State Financial Corporations, the
Life Insurance Corporation of India, the National Small Industries Corporation
Limited, the Industrial Credit and Investment Corporation, the Industrial
Development Bank of India, and the Industrial Reconstruction Corporation of India
provide term loans to companies. Before a term loan is sanctioned, a company has
to satisfy the concerned financial institution regarding the technical,
commercial, economic, financial and managerial viability of the project for which
the loan is required. Such loans are available at different rates of interest
under different schemes of financial institutions and are to be repaid according
to a stipulated repayment schedule. The loans in many cases stipulate a number of
conditions regarding the management and certain other financial policies of the
company. Term loans represent secured borrowings and at present it is the most
important source of finance for new projects. They generally carry a rate of
interest inclusive of interest tax, depending on the credit rating of the
borrower, the perceived risk of lending and the cost of funds. These loans are
generally repayable over a period of 6 to 10 years in annual, semiannual or
quarterly instalments. Term loans are also provided by banks, State
financial/development institutions and all- India term lending financial
institutions. Banks and State Financial Corporations normally provide term loans
to projects in the small scale sector while for the medium and large industries
term loans are provided by State developmental institutions alone or in consortium
with banks and State financial corporations. For large scale projects All India
financial institutions provide the bulk of term finance either singly or in
consortium with other All India financial institutions, State level institutions
and/or banks. After Independence, the institutional set up in India for the
provision of medium and long term credit for industry has been broadened. The
assistance sanctioned and disbursed by these specialised institutions has
increased impressively during the years.. A number of such specialised
institutions have been established all over the country. 3.6 Loans from Commercial
Banks: The primary role of the commercial banks is to cater to the short term
requirements of industry. Of late, however, banks have started taking an interest
in term financing of industries in several ways, though the formal term lending
is, so far, small and is confined to major banks only. Term lending by banks has
become a controversial issue these days. It has been argued that term loans do not
satisfy the canon of liquidity which is a major consideration in all bank

5.10
Types of Financing

operations. According to the traditional values, banks should provide loans only
for short periods and for operations which result in the automatic liquidation of
such credits over short periods. On the other hand, it is contended that the
traditional concept of liquidity requires to be modified. The proceeds of the term
loan are generally used for what are broadly known as fixed assets or for
expansion in plant capacity. Their repayment is usually scheduled over a long
period of time. The liquidity of such loans is said to depend on the anticipated
income of the borrowers. As a matter of fact, a working capital loan is more
permanent and long term than a term loan. The reason for making this statement is
that a term loan is always repayable on a fixed date and ultimately, a day will
come when the account will be totally adjusted. However, in the case of working
capital finance, though it is payable on demand, yet in actual practice it is
noticed that the account is never adjusted as such; and, if at all the payment is
asked back, it is with a clear purpose and intention of refinance being provided
at the beginning of the next year or half year. To illustrate this point let us
presume that two loans are granted on January 1, 2006 (a) to A; term loan of Rs.
60,000/- for 3 years to be paid back in equal half yearly instalments, and (b) to
B : cash-credit limit against hypothecation, etc. of Rs. 60,000. If we make two
separate graphs for the two loans, they may appear to be like the figure shown
below.

06

07

08

09

06

07

08

09

Note : It has been presumed that both the concerns are good. Payment of interest
has been ignored. It has been presumed that cash credit limit is being enhanced
gradually. The above graphs clearly indicate that at the end of 2009 the term loan
would be fully settled whereas the cash credit limit may have been enhanced to
over a lakh of rupees. It really amounts to providing finances for long term. This
technique of providing long term finance can be technically called as “rolled over
for periods exceeding more than one year”. Therefore, instead of indulging in term
financing by the rolled over method, banks can and should extend credit term after
a proper appraisal of
5.11
Financial Management applications for terms loans. In fact, as stated above, the
degree of liquidity in the provision for regular amortisation of term loans is
more than in some of these so called demand loans which are renewed from year to
year. Actually, term financing disciplines both the banker and the borrower as
long term planning is required to ensure that cash inflows would be adequate to
meet the instruments of repayments and allow an active turnover of bank loans. The
adoption of the formal term loan lending by commercial banks will not in any way
hamper the criteria of liquidity and as a matter of fact, it will introduce
flexibility in the operations of the banking system. The real limitation to the
scope of bank activities in this field is that all banks are not well equipped to
make appraisal of such loan proposals. Term loan proposals involve an element of
risk because of changes in the conditions affecting the borrower. The bank making
such a loan, therefore, has to assess the situation to make a proper appraisal.
The decision in such cases would depend on various factors affecting the
conditions of the industry concerned and the earning potential of the borrower.
Bridge Finance: Bridge finance refers to loans taken by a company normally from
commercial banks for a short period, pending disbursement of loans sanctioned by
financial institutions. Normally, it takes time for financial institutions to
disburse loans to companies. However, once the loans are approved by the term
lending institutions, companies, in order not to lose further time in starting
their projects, arrange short term loans from commercial banks. Bridge loans are
also provided by financial institutions pending the signing of regular term loan
agreement, which may be delayed due to non-compliance of conditions stipulated by
the institutions while sanctioning the loan. The bridge loans are repaid/ adjusted
out of the term loans as and when disbursed by the concerned institutions. Bridge
loans are normally secured by hypothecating movable assets, personal guarantees
and demand promissory notes. Generally, the rate of interest on bridge finance is
higher as com- pared with that on term loans. 4. VENTURE CAPITAL FINANCING

The venture capital financing refers to financing of new high risky venture
promoted by qualified entrepreneurs who lack experience and funds to give shape to
their ideas. In broad sense, under venture capital financing venture capitalist
make investment to purchase equity or debt securities from inexperienced
entrepreneurs who undertake highly risky ventures with a potential of success. 4.1
Methods of Venture Capital Financing: In India , Venture Capital financing was
first the responsibility of developmental financial institutions such as the
Industrial Development Bank of India (IDBI) , the Technical Development and
Information Corporation of India(now known
5.12
Types of Financing

as ICICI) and the State Finance Corporations(SFCs). In the year 1988, the
Government of India took a policy initiative and announced guidelines for Venture
Capital Funds (VCFs). In the same year, a Technology Development Fund (TDF)
financed by the levy on all payments for technology imports was established This
fund was meant to facilitate the financing of innovative and high risk technology
programmes through the IDBI. The guidelines mentioned above restricted the setting
up of Venture Capital Funds by banks and financial institutions only. Subsequently
guidelines were issued in the month of September 1995, for overseas investment in
Venture Capital in India. A major development in venture capital financing in
India was in the year 1996 when the Securities and Exchange Board of India (SEBI)
issued guidelines for venture capital funds to follow. These guidelines described
a venture capital fund as a fund established in the form of a company or trust,
which raises money through loans, donations, issue of securities or units and
makes or proposes to make investments in accordance with the regulations. This
move was instrumental in the entry of various foreign venture capital funds to
enter India.. The guidelines were further amended in April 2000 with the objective
of fuelling the growth of Venture Capital activities in India. A few venture
capital companies operate as both investment and fund management companies; others
set up funds and function as asset management companies. It is hoped that the
changes in the guidelines for the implementation of venture capital schemes in the
country would encourage more funds to be set up to give the required momentum for
venture capital investment in India. Some common methods of venture capital
financing are as follows: (i) Equity financing : The venture capital undertakings
generally requires funds for a longer period but may not be able to provide
returns to the investors during the initial stages. Therefore, the venture capital
finance is generally provided by way of equity share capital. The equity
contribution of venture capital firm does not exceed 49% of the total equity
capital of venture capital undertakings so that the effective control and
ownership remains with the entrepreneur. (ii) Conditional loan: A conditional loan
is repayable in the form of a royalty after the venture is able to generate sales.
No interest is paid on such loans. In India venture capital financiers charge
royalty ranging between 2 and 15 per cent; actual rate depends on other factors of
the venture such as gestation period, cash flow patterns, risk and other factors
of the enterprise. Some Venture capital financiers give a choice to the enterprise
of paying a high rate of interest (which could be well above 20 per cent) instead
of royalty on sales once it becomes
5.13
Financial Management commercially sounds. (iii) Income note: It is a hybrid
security which combines the features of both conventional loan and conditional
loan. The entrepreneur has to pay both interest and royalty on sales but at
substantially low rates. IDBI's VCF provides funding equal to 80 – 87.50% of the
projects cost for commercial application of indigenous technology. (iv)
Participating debenture: Such security carries charges in three phases — in the
start up phase no interest is charged, next stage a low rate of interest is
charged up to a particular level of operation, after that, a high rate of interest
is required to be paid. 5. DEBT SECURITISATION

Securitisation is a financial transaction in which assets are pooled and


securities representing interests in the pool are issued. The following example
illustrates the process in a conceptual manner: A finance company has issued a
large number of car loans. It desires to raise further cash so as to be in a
position to issue more loans. One way to achieve this goal is by selling all the
existing loans, however, in the absence of a liquid secondary market for
individual car loans, this may not be feasible. Instead, the company pools a large
number of these loans and sells interest in the pool to investors. This process
helps the company to raise finances and get the loans off its Balance Sheet.
.These finances shall help the company disburse further loans. Similarly, the
process is beneficial to the investors as it creates a liquid investment in a
diversified pool of auto loans, which may be an attractive option to other fixed
income instruments. The whole process is carried out in such a way, that the
ultimate debtors- the car owners – may not be aware of the transaction. They shall
continue making payments the way they were doing before, however, these payments
shall reach the new investors instead of the company they (the car owners) had
financed their car from. The example provided above illustrates the general
concept of securitisation as understood in common spoken English. Securitisation
can take the form of ‘debt securitisation’ in which the underlying pool of assets
(debt) is sold to a company or a trust for an immediate cash payment. The company
which buys these pool of assets issues securities and utilises the regular cash
flows arising out of the underlying pool of assets for servicing such issued
securities. Thus securitisation follows a two way process, (1) the sale of an
asset or a pool of assets to a company for immediate cash payment and (2) the
repackaging and selling the security interests representing claims on incoming
cash flows from the asset or pool of assets to third party investors by issuance
of tradable securities.

5.14
Types of Financing

The company to which the underlying pool of assets or asset is sold is known as a
‘Special Purpose Vehicle’ (SPV) and the company which sells the underlying pool of
assets or asset is known as the originator. The process of securitisation is
generally without recourse i.e. the investor bears the credit risk or risk of
default and the issuer is under an obligation to pay to investors only if the cash
flows are received by him from the collateral. The issuer however, has a right to
legal recourse in the event of default. The risk run by the investor can be
further reduced through credit enhancement facilities like insurance, letters of
credit and guarantees. In a simple pass through structure, the investor owns a
proportionate share of the asset pool and cash flows when generated are passed on
directly to the investor. This is done by issuing pass through certificates. In
mortgage or asset backed bonds, the investor has a lien on the underlying asset
pool. The SPV accumulates payments from the original borrowers from time to time
and makes payments to investors at regular predetermined intervals. The SPV can
invest the funds received in short term instruments and improve yield when there
is time lag between receipt and payment. In India, the Reserve Bank of India had
issued draft guidelines on securitisation of standard assets in April 2005. These
guidelines were applicable to banks, financial institutions and non banking
financial companies. The guidelines were suitably modified and brought into effect
from February 2006. 5.1 Benefits to the Originator (i) The assets are shifted off
the balance sheet, thus giving the originator recourse to off balance sheet
funding. (ii) It converts illiquid assets to liquid portfolio. (iii) It
facilitates better balance sheet management as assets are transferred off balance
sheet facilitating satisfaction of capital adequacy norms. (iv) The originator's
credit rating enhances. For the investor securitisation opens up new investment
avenues. Though the investor bears the credit risk, the securities are tied up to
definite assets. As compared to factoring or bill discounting which largely solve
the problems of short term trade financing, securitisation helps to convert a
stream of cash receivables into a source of long term finance.

5.15
Financial Management 6. LEASE FINANCING

Leasing is a general contract between the owner and user of the asset over a
specified period of time. The asset is purchased initially by the lessor (leasing
company) and thereafter leased to the user (lessee company) which pays a specified
rent at periodical intervals. Thus, leasing is an alternative to the purchase of
an asset out of own or borrowed funds. Moreover, lease finance can be arranged
much faster as compared to term loans from financial institutions. In recent
years, leasing has become a popular source of financing in India. From the
lessee's point of view, leasing has the attraction of eliminating immediate cash
outflow, and the lease rentals can be deducted for computing the total income
under the Income tax Act. As against this, buying has the advantages of
depreciation allowance (including additional depreciation) and interest on
borrowed capital being tax-deductible. Thus, an evaluation of the two alternatives
is to be made in order to take a decision. Practical problems for lease financing
are covered at Final level in paper of Strategic Financial Management. 7. SHORT
TERM SOURCES OF FINANCE

There are various sources available to meet short term needs of finance. The
different sources are discussed below: 7.1 Trade Credit: It represents credit
granted by suppliers of goods, etc., as an incident of sale. The usual duration of
such credit is 15 to 90 days. It generates automatically in the course of business
and is common to almost all business operations. It can be in the form of an 'open
account' or 'bills payable'. Trade credit is preferred as a source of finance
because it is without any explicit cost and till a business is a going concern it
keeps on rotating. Another very important characteristic of trade credit is that
it enhances automatically with the increase in the volume of business. 7.2.
Accrued Expenses and Deferred Income: Accrued expenses represent liabilities which
a company has to pay for the services which it has already received. Such expenses
arise out of the day to day activities of the company and hence represent a
spontaneous source of finance. Deferred income, on the other hand, reflects the
amount of funds received by a company in lieu of goods and services to be provided
in the future. Since these receipts increase a company’s liquidity, they are also
considered to be an important source of spontaneous finance.

5.16
Types of Financing

7.3 Advances from Customers: Manufacturers and contractors engaged in producing or


constructing costly goods involving considerable length of manufacturing or
construction time usually demand advance money from their customers at the time of
accepting their orders for executing their contracts or supplying the goods. This
is a cost free source of finance and really useful. 7.4. Commercial Paper: A
Commercial Paper is an unsecured money market instrument issued in the form of a
promissory note. The Reserve Bank of India introduced the commercial paper scheme
in the year 1989 with a view to enabling highly rated corporate borrowers to
diversify their sources of short term borrowings and to provide an additional
instrument to investors. Subsequently, in addition to the Corporate, Primary
Dealers and All India Financial Institutions have also been allowed to issue
Commercial Papers. Commercial Papers can be issued for maturities between 15 days
and a maximum up to one year from the date of issue. These can be issued in
denominations of Rs 5 lakh or multiples thereof. All eligible issuers are required
to get the credit rating from Credit Rating Information Services of India
Ltd,(CRISIL), or the Investment Information and Credit Rating Agency of India Ltd
(ICRA) or the Credit Analysis and Research Ltd (CARE) or the FITCH Ratings India
Pvt Ltd or any such other credit rating agency as is specified by the Reserve Bank
of India .Individuals, banking companies, corporate bodies incorporated in India,
Non Resident Indians, Foreign Institutional Investors etc are allowed to invest in
Commercial Paper, the minimum amount of such investment being Rs 5 lakhs. 7.5 Bank
Advances: Banks receive deposits from public for different periods at varying
rates of interest. These funds are invested and lent in such a manner that when
required, they may be called back. Lending results in gross revenues out of which
costs, such as interest on deposits, administrative costs, etc., are met and a
reasonable profit is made. A bank's lending policy is not merely profit motivated
but has to also keep in mind the socio- economic development of the country. Bank
advances are in the form of loan, overdraft, cash credit and bills
purchased/discounted etc. Banks do not sanction advances on a long term basis
beyond a small proportion of their demand and time liabilities. Advances are
granted against tangible securities such as goods, shares, government promissory
notes, Bills etc. In very rare cases, clean advances may also be allowed. (i)
Loans : In a loan account, the entire advance is disbursed at one time either in
cash or by transfer to the current account of the borrower. It is a single
advance. Except by way of interest and other charges no further adjustments are
made in this account. Loan accounts are not running accounts like overdraft and
cash credit accounts, repayment under the loan
5.17
Financial Management account may be the full amounts or by way of schedule of
repayments agreed upon as in case of term loans. The securities may be shares,
government securities, life insurance policies and fixed deposit receipts, etc.
(ii) Overdraft: Under this facility, customers are allowed to withdraw in excess
of credit balance standing in their Current Deposit Account. A fixed limit is
therefore granted to the borrower within which the borrower is allowed to overdraw
his account. Opening of an overdraft account requires that a current account will
have to be formally opened. Though overdrafts are repayable on demand, they
generally continue for long periods by annual renewals of the limits. This is a
convenient arrangement for the borrower as he is in a position to avail of the
limit sanctioned, according to his requirements. Interest is charged on daily
balances. Since these accounts are operative like cash credit and current
accounts, cheque books are provided. As in the case of a loan account the security
in an overdraft account may be shares, debentures and Government securities. In
special cases, life insurance policies and fixed deposit receipts are also
accepted. (iii) Clean Overdrafts: Request for clean advances are entertained only
from parties which are financially sound and reputed for their integrity. The bank
has to rely upon the personal security of the borrowers. Therefore, while
entertaining proposals for clean advances; banks exercise a good deal of restraint
since they have no backing of any tangible security. If the parties are already
enjoying secured advance facilities, this may be a point in favour and may be
taken into account while screening such proposals. The turnover in the account,
satisfactory dealings for considerable period and reputation in the market are
some of the factors which the bank will normally see. As a safeguard, banks take
guarantees from other persons who are credit worthy before granting this facility.
A clean advance is generally granted for a short period and must not be continued
for long. (iv) Cash Credits: Cash Credit is an arrangement under which a customer
is allowed an advance up to certain limit against credit granted by bank. Under
this arrangement, a customer need not borrow the entire amount of advance at one
time; he can only draw to the extent of his requirements and deposit his surplus
funds in his account. Interest is not charged on the full amount of the advance
but on the amount actually availed of by him. Generally cash credit limits are
sanctioned against the security of goods by way of pledge or hypothecation. The
borrower can also provide alternative security of goods by way of pledge or
hypothecation. Though these accounts are repayable on demand, banks usually do not
recall such advances, unless they are compelled to do so by adverse factors.
Hypothecation is an equitable charge on movable goods for an amount of debt where
neither possession nor ownership is passed on to the creditor. In case of pledge,
the borrower delivers the goods to

5.18
Types of Financing

the creditor as security for repayment of debt. Since the banker, as creditor, is
in possession of the goods, he is fully secured and in case of emergency he can
fall back on the goods for realisation of his advance under proper notice to the
borrower. (v) Advances against goods : Advances against goods occupy an important
place in total bank credit. Goods are security have certain distinct advantages.
They provide a reliable source of repayment. Advances against them are safe and
liquid. Also, there is a quick turnover in goods, as they are in constant demand.
So a banker accepts them as security. Generally goods are charged to the bank
either by way of pledge or by way of hypothecation. The term 'goods' includes all
forms of movables which are offered to the bank as security. They may be
agricultural commodities or industrial raw materials or partly finished goods. For
the purpose of calculation of the drawing limits, valuation of the goods is made
from time to time. In case of hypothecation advance, an undertaking is obtained
from the borrower that the goods are not charged to some other bank. The bank also
takes periodical statements of stocks regarding quantity valuation etc. The
Reserve Bank of India issues directives from time to time imposing restrictions on
advances against certain commodities. It is obligatory on banks to follow these
directives in letter and spirit. The directives also sometimes stipulate changes
in the margin. (vi) Bills Purchased/Discounted : These advances are allowed
against the security of bills which may be clean or documentary. Bills are
sometimes purchased from approved customers in whose favour limits are sanctioned.
Before granting a limit the banker satisfies himself as to the credit worthiness
of the drawer. Although the term 'bills purchased' gives the impression that the
bank becomes the owner or purchaser of such bills, in actual practice the bank
holds the bills only as security for the advance. The bank, in addition to the
rights against the parties liable on the bills, can also exercise a pledge’s
rights over the goods covered by the documents. Usance bills maturing at a future
date or sight are discounted by the banks for approved parties. When a bill is
discounted, the borrower is paid the present worth. The bankers, however, collect
the full amounts on maturity. The difference between these two amounts represents
earnings of the bankers for the period. This item of income is called 'discount'.
Sometimes, overdraft or cash credit limits are allowed against the security of
bills. A suitable margin is usually maintained. Here the bill is not a primary
security but only a collateral security. The banker in the case, does not become a
party to the bill, but merely collects it as an agent for its customer.

5.19
Financial Management When a banker purchases or discounts a bill, he advances
against the bill; he has therefore to be very cautious and grant such facilities
only to those customers who are creditworthy and have established a steady
relationship with the bank. Credit reports are also compiled on the drawees. (vii)
Advance against documents of title to goods: A document becomes a document of
title to goods when its possession is recognised by law or business custom as
possession of the goods. These documents include a bill of lading, dock warehouse
keeper's certificate, railway receipt, etc. A person in possession of a document
to goods can by endorsement or delivery (or both) of document, enable another
person to take delivery of the goods in his right. An advance against the pledge
of such documents is equivalent to an advance against the pledge of goods
themselves. (viii) Advance against supply of bills: Advances against bills for
supply of goods to government or semi-government departments against firm orders
after acceptance of tender fall under this category. The other type of bills which
also come under this category are bills from contractors for work executed either
wholly or partially under firm contracts entered into with the above mentioned
Government agencies. These bills are clean bills without being accompanied by any
document of title of goods. But they evidence supply of goods directly to
Governmental agencies. Sometimes these bills may be accompanied by inspection
notes from representatives of government agencies for having inspected the goods
before they are despatched. If bills are without the inspection report, banks like
to examine them with the accepted tender or contract for verifying that the goods
supplied under the bills strictly conform to the terms and conditions in the
acceptance tender. These supply bills represent debt in favour of
suppliers/contractors, for the goods supplied to the government bodies or work
executed under contract from the Government bodies. It is this debt that is
assigned to the bank by endorsement of supply bills and executing irrevocable
power of attorney in favour of the banks for receiving the amount of supply bills
from the Government departments. The power of attorney has got to be registered
with the Government department concerned. The banks also take separate letter from
the suppliers / contractors instructing the Government body to pay the amount of
bills direct to the bank. Supply bills do not enjoy the legal status of negotiable
instruments because they are not bills of exchange. The security available to a
banker is by way of assignment of debts represented by the supply bills.

5.20
Types of Financing

(ix) Term Loans by banks: Term loans are an instalment credit repayable over a
period of time in monthly/quarterly/half-yearly or yearly instalment. Banks grant
term loans for small projects falling under priority sector, small scale sector
and big units. Banks have now been permitted to sanction term loan for projects as
well without association of financial institutions. The banks grant loans for
periods which normally range from 3 to 7 years and some- times even more. These
loans are granted on the security of fixed assets. 7.6 Financing of Export Trade
by Banks: Exports play an important role in accelerating the economic growth of
developing countries like India. Of the several factors influencing export growth,
credit is a very important factor which enables exporters in efficiently executing
their export orders. The commercial banks provide short term export finance mainly
by way of pre and post-shipment credit. Export finance is granted in Rupees as
well as in foreign currency. In view of the importance of export credit in
maintaining the pace of export growth, RBI has initiated several measures in the
recent years to ensure timely and hassle free flow of credit to the export sector.
These measures, inter alia, include rationalization and liberalization of export
credit interest rates, flexibility in repayment/prepayment of pre-shipment credit,
special financial package for large value exporters, export finance for
agricultural exports, Gold Card Scheme for exporters etc. Further, banks have been
granted freedom by RBI to source funds from abroad without any limit for
exclusively for the purpose of granting export credit in foreign currency, which
has enabled banks to increase their lending’s under export credit in foreign
currency substantially during the last few years. The advances by commercial banks
for export financing are in the form of: (i) (ii) Pre-shipment finance i.e.,
before shipment of goods. Post-shipment finance i.e., after shipment of goods.

7.6.1 Pre-Shipment Finance: This generally takes the form of packing credit
facility; packing credit is an advance extended by banks to an exporter for the
purpose of buying, manufacturing, processing, packing, shipping goods to overseas
buyers. Any exporter, having at hand a firm export order placed with him by his
foreign buyer or an irrevocable letter of credit opened in his favour, can
approach a bank for availing of packing credit. An advance so taken by an exporter
is required to be liquidated within 180 days from the date of its commencement by
negotiation of export bills or receipt of export proceeds in an approved manner.
Thus packing credit is essentially a short term advance. Normally, banks insist
upon their customers to lodge with them irrevocable letters of credit opened in
favour of the customers by the overseas buyers. The letter of credit and firm sale
contracts not only serve as evidence of a definite arrangement for realisation of
the export
5.21
Financial Management proceeds but also indicate the amount of finance required by
the exporter. Packing credit, in the case of customers of long standing, may also
be granted against firm contracts entered into by them with overseas buyers.
7.6.1.1 Types of Packing Credit (a) Clean packing credit : This is an advance made
available to an exporter only on production of a firm export order or a letter of
credit without exercising any charge or control over raw material or finished
goods. It is a clean type of export advance. Each proposal is weighed according to
particular requirements of the trade and credit worthiness of the exporter. A
suitable margin has to be maintained. Also, Export Credit Guarantee Corporation
(ECGC) cover should be obtained by the bank. (b) Packing credit against
hypothecation of goods: Export finance is made available on certain terms and
conditions where the exporter has pledge able interest and the goods are
hypothecated to the bank as security with stipulated margin. At the time of
utilising the advance, the exporter is required to submit, along with the firm
export order or letter of credit relative stock statements and thereafter continue
submitting them every fortnight and/or whenever there is any movement in stocks.
(c) Packing credit against pledge of goods: Export finance is made available on
certain terms and conditions where the exportable finished goods are pledged to
the banks with approved clearing agents who will ship the same from time to time
as required by the exporter. The possession of the goods so pledged lies with the
bank and is kept under its lock and key. (d) E.C.G.C. guarantee: Any loan given to
an exporter for the manufacture, processing, purchasing, or packing of goods meant
for export against a firm order qualifies for the packing. credit guarantee issued
by Export Credit Guarantee Corporation (ECGC). (e) Forward exchange contract:
Another requirement of packing credit facility is that if the export bill is to be
drawn in a foreign currency, the exporter should enter into a forward exchange
contact with the bank, thereby avoiding risk involved in a possible change in the
rate of exchange. Documents required : In case of partnership firms, banks usually
require the following documents: (i) Joint and several demand promote signed on
behalf of the firm as well as by the partners individually. (ii) Letter of
continuity (signed on behalf of the firm and partners individually).

5.22
Types of Financing

(iii) Letter of Pledge to secure demand cash credit against goods (in case of
pledge) OR Agreement of Hypothecation to secure demand cash credit (in case of
hypothecation). (iv) Letter of Authority to operate the account. (v) Declaration
of Partnership. (In case of sole traders, sole proprietorship declaration). (vi)
Agreement to utilise the monies drawn in terms of contract. (vii) Letter of
Hypothecation (for bills). In case of limited companies banks usually require the
following documents : (i) (ii) Demand Pro-note Letter of continuity.

(iii) Agreement of hypothecation or Letter of pledge signed on behalf of the


company. (iv) General guarantee of the directors of the company in their joint and
several personal capacities. (v) Certified copy of the board of directors'
resolution. (vi) Agreement to utilise the monies drawn in terms of contract should
bear the seal of the company. (vii) Letter of Hypothecation (for bills). 7.6.2
Post-shipment Finance: It takes the following forms: (a) Purchase/discounting of
documentary export bills : Finance is provided to exporters by purchasing export
bills drawn payable at sight or by discounting usance export bills covering
confirmed sales and backed by documents including documents of the title of goods
such as bill of lading, post parcel receipts, or air consignment notes. Documents
to be obtained: (i) (ii) Letter of hypothecation covering the goods; and General
guarantee of directors or partners of the firm as the case may be.

(b) E.C.G.C. Guarantee: Post-shipment finance, given to an exporter by a bank


through purchase, negotiation or discount of an export bill against an order,
qualifies for post-shipment export credit guarantee. It is necessary, however,
that exporters should obtain a shipment or

5.23
Financial Management contracts risk policy of E.C.G.C. Banks insist on the
exporters to take a contracts shipments (comprehensive risks) policy covering both
political and commercial risks. The Corporation, on acceptance of the policy, will
fix credit limits for individual exporters and the Corporation’s liability will be
limited to the extent of the limit so fixed for the exporter concerned
irrespective of the amount of the policy. (c) Advance against export bills sent
for collection : Finance is provided by banks to exporters by way of advance
against export bills forwarded through them for collection, taking into account
the creditworthiness of the party, nature of goods exported, usance, standing of
drawee, etc. appropriate margin is kept. Documents to be obtained : (i) (ii)
Demand promissory note. Letter of continuity.

(iii) Letter of hypothecation covering bills. (iv) General Guarantee of directors


or partners of the firm (as the case may be). (d) Advance against duty draw backs,
cash subsidy, etc.: To finance export losses sustained by exporters, bank advance
against duty draw-back, cash subsidy, etc., receivable by them against export
performance. Such advances are of clean nature; hence necessary precaution should
be exercised. Conditions : Bank providing finance in this manner see that the
relative export bills are either negotiated or forwarded for collection through it
so that it is in a position to verify the exporter's claims for duty draw-backs,
cash subsidy, etc. 'An advance so availed of by an exporter is required to be
liquidated within 180 days from the date of shipment of relative goods. Documents
to be obtained: (i) (ii) Demand promissory note. Letter of continuity.

(iii) General Guarantee of directors of partners of the firm as the case may be.
(iv) Undertaking from the borrowers that they will deposit the cheques/payments
received from the appropriate authorities immediately with the bank and will not
utilise such amounts in any other way.

5.24
Types of Financing

Other facilities extended to exporters: (i) (ii) On behalf of approved exporters,


banks establish letters of credit on their overseas or up country suppliers.
Guarantees for waiver of excise duty, etc. due performance of contracts, bond in
lieu of cash security deposit, guarantees for advance payments etc., are also
issued by banks to approved clients.

(iii) To approved clients undertaking exports on deferred payment terms, banks


also provide finance. (iv) Banks also endeavour to secure for their exporter-
customers status reports of their buyers and trade information on various
commodities through their correspondents. (v) Economic intelligence on various
countries is also provided by banks to their ex- porter clients. 7.7 Inter
Corporate Deposits: The companies can borrow funds for a short period say 6 months
from other companies which have surplus liquidity. The rate of interest on inter
corporate deposits varies depending upon the amount involved and time period. 7.8
Certificate of Deposit (CD): The certificate of deposit is a document of title
similar to a time deposit receipt issued by a bank except that there is no
prescribed interest rate on such funds. The main advantage of CD is that banker is
not required to encash the deposit before maturity period and the investor is
assured of liquidity because he can sell the CD in secondary market. 7.9 Public
Deposits: Public deposits are very important source of short-term and medium term
finances particularly due to credit squeeze by the Reserve Bank of India. A
company can accept public deposits subject to the stipulations of Reserve Bank of
India from time to time maximum up to 35 per cent of its paid up capital and
reserves, from the public and shareholders. These deposits may be accepted for a
period of six months to three years. Public deposits are unsecured loans; they
should not be used for acquiring fixed assets since they are to be repaid within a
period of 3 years. These are mainly used to finance working capital requirements.
8. OTHER SOURCES OF FINANCING

8.1 Seed Capital Assistance: The Seed capital assistance scheme is designed by
IDBI for

5.25
Financial Management professionally or technically qualified entrepreneurs and/or
persons possessing relevant experience, skills and entrepreneurial traits. All the
projects eligible for financial assistance from IDBI, directly or indirectly
through refinance are eligible under the scheme. The project cost should not
exceed Rs. 2 crores and the maximum assistance under the project will be
restricted to 50% of the required promoter's contribution or Rs. 15 lacs whichever
is lower. The Seed Capital Assistance is interest free but carries a service
charge of one per cent per annum for the first five years and at increasing rate
thereafter. However, IDBI will have the option to charge interest at such rate as
may be determined by IDBI on the loan if the financial position and profitability
of the company so permits during the currency of the loan. The repayment schedule
is fixed depending upon the repaying capacity of the unit with an initial
moratorium upto five years. For projects with a project cost exceeding Rs. 200
lacs, seed capital may be obtained from the Risk Capital and Technology
Corporation Ltd. (RCTC) For small projects costing upto Rs. 5 lacs, assistance
under the National Equity Fund of the SIDBI may be availed. 8.2 Risk Capital
Foundation Scheme: The Risk Capital Foundation Scheme is an autonomous foundation
set up and funded by IFCI. It assists promoters of projects costing between Rs 2
crores and Rs 15 crore. The ceiling on the assistance varies between Rs 15 lakhs
and Rs 40 lakhs depending on the number of applicant promoters. 8.3 Internal Cash
Accruals: Existing profit making companies which undertake an expansion/
diversification programme may be permitted to invest a part of their accumulated
reserves or cash profits for creation of capital assets. In such cases, past
performance of the company permits the capital expenditure from within the company
by way of disinvestment of working/invested funds. In other words, the surplus
generated from operations, after meeting all the contractual, statutory and
working requirement of funds, is available for further capital expenditure. 8.4
Unsecured Loans: Unsecured loans are typically provided by promoters to meet the
promoters' contribution norm. These loans are subordinate to institutional loans.
The rate of interest chargeable on these loans should be less than or equal to the
rate of interest on institutional loans and interest can be paid only after
payment of institutional dues. These loans cannot be repaid without the prior
approval of financial institutions. Unsecured loans are considered as part of the
equity for the purpose of calculating of debt equity ratio. 8.5 Deferred Payment
Guarantee: Many a time suppliers of machinery provide deferred credit facility
under which payment for the purchase of machinery can be made over a period of
time. The entire cost of the machinery is financed and the company is not required
5.26
Types of Financing

to contribute any amount initially towards acquisition of the machinery. Normally,


the supplier of machinery insists that bank guarantee should be furnished by the
buyer. Such a facility does not have a moratorium period for repayment. Hence, it
is advisable only for an existing profit making company. 8.6 Capital Incentives:
The backward area development incentives available often determine the location of
a new industrial unit. These incentives usually consist of a lump sum subsidy and
exemption from or deferment of sales tax and octroi duty. The quantum of
incentives is determined by the degree of backwardness of the location. The
special capital incentive in the form of a lump sum subsidy is a quantum
sanctioned by the implementing agency as a percentage of the fixed capital
investment subject to an overall ceiling. This amount forms a part of the long-
term means of finance for the project. However, it may be mentioned that the
viability of the project must not be dependent on the quantum and availability of
incentives. Institutions, while appraising the project, assess the viability of
the project per se, without considering the impact of incentives on the cash flows
and profitability of the project. Special capital incentives are sanctioned and
released to the units only after they have complied with the requirements of the
relevant scheme. The requirements may be classified into initial effective steps
and final effective steps. The initial effective steps include formation of the
firm/company, acquisition of land in the backward area and registration for
manufacture of the products. The final effective steps include obtaining
clearances under FEMA, capital goods clearance/import licence, conversion of
Letter of Intent to Industrial License, tie up of the means of finance, all
clearances required for the setting up of the unit, aggregate expenditure incurred
for the project should exceed 25% of the project cost and at least 10% of the
fixed assets should have been created/acquired site. The release of special
capital incentives by the concerned State Government generally takes one to two
years. The promoters therefore find it convenient to avail bridge finance against
the capital incentives. Provision for the same should be made in the pre-operative
expenses considered in the project cost. Further, as the bridge finance may be
available to the extent of 85%, the balance 15% may have to be brought in by the
promoters from their own resources. 9. NEW INSTRUMENTS

The new instruments that have been introduced since early 90’s as a source of
finance is staggering in their nature and diversity. These new instruments are as
follows: 9.1 Deep Discount Bonds: Deep Discount Bonds is a form of zero-interest
bonds. These
5.27
Financial Management bonds are sold at a discounted value and on maturity face
value is paid to the investors. In such bonds, there is no interest payout during
lock in period. IDBI was the first to issue a deep discount bond in India in
January, 1992. The bond of a face value of Rs. 1 lakh was sold for Rs. 2,700 with
a maturity period of 25 years. The investor could hold the bond for 25 years or
seek redemption at the end of every five years with a specified maturity value as
shown below. Holding Period (years) Maturity value (Rs.) Annual rate of interest
(%) 5 5,700 16.12 10 12,000 16.09 15 25,000 15.99 20 25 50,000 1,00,000 15.71
15.54

The investor can sell the bonds in stock market and realise the difference between
face value (Rs. 2,700) and market price as capital gain. 9.2 Secured Premium
Notes: Secured Premium Notes is issued along with a detachable warrant and is
redeemable after a notified period of say 4 to 7 years. The conversion of
detachable warrant into equity shares will have to be done within time period
notified by the company. 9.3 Zero interest fully convertible debentures: These are
fully convertible debentures which do not carry any interest. The debentures are
compulsorily and automatically converted after a specified period of time and
holders thereof are entitled to new equity shares of the company at predetermined
price. From the point of view of company this kind of instrument is beneficial in
the sense that no interest is to be paid on it, if the share price of the company
in the market is very high than the investors tends to get equity shares of the
company at the lower rate. 9.4 Zero Coupon Bonds: A Zero Coupon Bonds does not
carry any interest but it is sold by the issuing company at a discount. The
difference between the discounted value and maturing or face value represents the
interest to be earned by the investor on such bonds. 9.5 Double Option Bonds:
These have also been recently issued by the IDBI. The face value of each bond is
Rs. 5,000. The bond carries interest at 15% per annum compounded half yearly from
the date of allotment. The bond has maturity period of 10 years. Each bond has two
parts in the form of two separate certificates, one for principal of Rs. 5,000 and
other for interest (including redemption premium) of Rs. 16,500. Both these
certificates are listed on all major stock exchanges. The investor has the
facility of selling either one or both parts anytime he likes.
5.28
Types of Financing

9.6 Option Bonds: These are cumulative and non-cumulative bonds where interest is
payable on maturity or periodically. Redemption premium is also offered to attract
investors. These were recently issued by IDBI, ICICI etc. 9.7 Inflation Bonds:
Inflation Bonds are the bonds in which interest rate is adjusted for inflation.
Thus, the investor gets interest which is free from the effects of inflation. For
example, if the interest rate is 11 per cent and the inflation is 5 per cent, the
investor will earn 16 per cent meaning thereby that the investor is protected
against inflation. 9.8 Floating Rate Bonds: This as the name suggests is bond
where the interest rate is not fixed and is allowed to float depending upon the
market conditions. This is an ideal instrument which can be resorted to by the
issuer to hedge themselves against the volatility in the interest rates. This has
become more popular as a money market instrument and has been successfully issued
by financial institutions like IDBI, ICICI etc. 10. INTERNATIONAL FINANCING

The essence of financial management is to raise and utilise the funds raised
effectively. There are various avenues for organisations to raise funds either
through internal or external sources. The sources of external sources include:
10.1 Commercial Banks: Like domestic loans, commercial banks all over the world
extend Foreign Currency (FC) loans also for international operations. These banks
also provide to overdraw over and above the loan amount. 10.2 Development Banks:
Development banks offer long & medium term loans including FC loans. Many agencies
at the national level offer a number of concessions to foreign companies to invest
within their country and to finance exports from their countries. E.g. EXIM Bank
of USA. 10.3 Discounting of Trade Bills: This is used as a short term financing
method. It is used widely in Europe and Asian countries to finance both domestic
and international business. 10.4 International Agencies: A number of international
agencies have emerged over the years to finance international trade & business.
The more notable among them include The Inter- national Finance Corporation (IFC),
The International Bank for Reconstruction and Development (IBRD), The Asian
Development Bank (ADB), The International Monetary Fund (IMF), etc. 10.5
International Capital Markets: Today, modern organisations including MNC's depend
upon sizeable borrowings in Rupees as well as Foreign Currency. In order to cater
to the
5.29
Financial Management needs of such organisations, international capital markets
have sprung all over the globe such as in London. In international capital market,
the availability of FC is assured under the four main systems viz: * * * * Euro-
currency market Export credit facilities Bonds issues Financial Institutions.

The origin of the Euro-currency market was with the dollar denominated bank
deposits & loans in Europe particularly in London. Euro-dollar deposits are dollar
denominated time deposits available at foreign branches of US banks & at some
foreign banks. Banks based in Europe accept dollar denominated deposits & make
dollar denominated deposits to the clients. This forms the backbone of the Euro-
currency market all over the globe. In this market, funds are made available as
loans through syndicated Euro-credit of instruments such as FRN's. FR certificates
of deposits. 10.6 Financial Instruments: Some of the various financial instruments
dealt with in the international market are briefly described below: (a) External
Commercial Borrowings(ECB) : ECBs refer to commercial loans (in the form of bank
loans , buyers credit, suppliers credit, securitised instruments ( e.g. floating
rate notes and fixed rate bonds) availed from non resident lenders with minimum
average maturity of 3 years. Borrowers can raise ECBs through internationally
recognised sources like (i) international banks, (ii) international capital
markets, (iii) multilateral financial institutions such as the IFC, ADB etc, (iv)
export credit agencies, (v) suppliers of equipment, (vi) foreign collaborators and
(vii) foreign equity holders. External Commercial Borrowings can be accessed under
two routes viz (i) Automatic route and (ii) Approval route. Under the Automatic
route there is no need to take the RBI/Government approval whereas such approval
is necessary under the Approval route. Company’s registered under the Companies
Act and NGOs engaged in micro finance activities are eligible for the Automatic
Route where as Financial Institutions and Banks dealing exclusively in
infrastructure or export finance and the ones which had participated in the
textile and steel sector restructuring packages as approved by the government are
required to take the Approval Route.

5.30
Types of Financing

(b) Euro Bonds: Euro bonds are debt instruments which are not denominated in the
currency of the country in which they are issued. E.g. a Yen note floated in
Germany. Such bonds are generally issued in a bearer form rather than as
registered bonds and in such cases they do not contain the investor’s names or the
country of their origin. These bonds are an attractive proposition to investors
seeking privacy. (c) Foreign Bonds: These are debt instruments issued by foreign
corporations or foreign governments. Such bonds are exposed to default risk,
especially the corporate bonds. These bonds are denominated in the currency of the
country where they are issued, however, in case these bonds are issued in a
currency other than the investors home currency, they are exposed to exchange rate
risks. An example of a foreign bond ‘A British firm placing Dollar denominated
bonds in USA’. (d) Fully Hedged Bonds: As mentioned above, in foreign bonds, the
risk of currency fluctuations exists. Fully hedged bonds eliminate the risk by
selling in forward markets the entire stream of principal and interest payments.
(e) Medium Term Notes: Certain issuers need frequent financing through the Bond
route including that of the Euro bond. However it may be costly and ineffective to
go in for frequent issues. Instead, investors can follow the MTN programme. Under
this programme, several lots of bonds can be issued, all having different features
e.g. different coupon rates, different currencies etc. The timing of each lot can
be decided keeping in mind the future market opportunities. The entire
documentation and various regulatory approvals can be taken at one point of time
(f) Floating Rate Notes: These are issued up to seven years maturity. Interest
rates are adjusted to reflect the prevailing exchange rates. They provide cheaper
money than foreign loans. (g) Euro Commercial Papers (ECP): ECPs are short term
money market instruments. They are for maturities less than one year. They are
usually designated in US Dollars. (h) Foreign Currency Option: A FC Option is the
right to buy or sell, spot, future or forward, a specified foreign currency. It
provides a hedge against financial and economic risks. (i) Foreign Currency
Futures: FC Futures are obligations to buy or sell a specified currency in the
present for settlement at a future date. 10.7 Euro Issues by Indian Companies :
Indian companies are permitted to raise foreign currency resources through issue
of ordinary equity shares through Global Depository Receipts(GDRs)/ American
Depository Receipts (ADRs) and / or issue of Foreign Currency

5.31
Financial Management Convertible Bonds (FCCB) to foreign investors i.e.
institutional investors or individuals (including NRIs) residing abroad . Such
investment is treated as Foreign Direct Investment. The government guidelines on
these issues are covered under the Foreign Currency Convertible Bonds and Ordinary
Shares (Through depositary receipt mechanism) Scheme, 1993 and notifications
issued after the implementation of the said scheme. (a) American Depository
Deposits (ADR) : These are securities offered by non-US companies who want to list
on any of the US exchange. Each ADR represents a certain number of a company's
regular shares. ADRs allow US investors to buy shares of these companies without
the costs of investing directly in a foreign stock exchange. ADRs are issued by an
approved New York bank or trust company against the deposit of the original
shares. These are deposited in a custodial account in the US. Such receipts have
to be issued in accordance with the provisions stipulated by the SEC. USA which
are very stringent. ADRs can be traded either by trading existing ADRs or
purchasing the shares in the issuer's home market and having new ADRs created,
based upon availability and market conditions. When trading in existing ADRs, the
trade is executed on the secondary market on the New York Stock Exchange (NYSE)
through Depository Trust Company (DTC) without involvement from foreign brokers or
custodians. The process of buying new, issued ADRs goes through US brokers,
Helsinki Exchanges and DTC as well as Deutsche Bank. When transactions are made,
the ADRs change hands, not the certificates. This eliminates the actual transfer
of stock certificates between the US and foreign countries. In a bid to bypass the
stringent disclosure norms mandated by the SEC for equity shares, the Indian
companies have however, chosen the indirect route to tap the vast American
financial market through private debt placement of GDRs listed in London and
Luxemberg Stock Exchanges. The Indian companies have preferred the GDRs to ADRs
because the US market exposes them to a higher level or responsibility than a
European listing in the areas of disclosure, costs, liabilities and timing. The
SECs regulations set up to protect the retail investor base are some what more
stringent and onerous, even for companies already listed and held by retail
investors in their home country. The most onerous aspect of a US listing for the
companies is to provide full, half yearly and quarterly accounts in accordance
with, or at least reconciled with US GAAPs. (b) Global Depository Receipt (GDRs):
These are negotiable certificate held in the bank of one country representing a
specific number of shares of a stock traded on the exchange of another country.
These financial instruments are used by companies to raise capital in either
dollars or Euros. These are mainly traded in European countries and particularly
in London. ADRs/GDRs and the Indian Scenario : Indian companies are shedding their
reluctance to tap the US markets. Infosys Technologies was the first Indian
company to be listed on Nasdaq in 1999. However, the first Indian firm to issue
sponsored GDR or ADR was Reliance industries
5.32
Types of Financing

Limited. Beside, these two companies there are several other Indian firms are also
listed in the overseas bourses. These are Satyam Computer, Wipro, MTNL, VSNL,
State Bank of India, Tata Motors, Dr Reddy's Lab, Ranbaxy, Larsen & Toubro, ITC,
ICICI Bank, Hindalco, HDFC Bank and Bajaj Auto. (c) Indian Depository Receipts
(IDRs): The concept of the depository receipt mechanism which is used to raise
funds in foreign currency has been applied in the Indian Capital Market through
the issue of Indian Depository Receipts (IDRs). IDRs are similar to ADRs/GDRs in
the sense that foreign companies can issue IDRs to raise funds from the Indian
Capital Market in the same lines as an Indian company uses ADRs/GDRs to raise
foreign capital. The IDRs are listed and traded in India in the same way as other
Indian securities are traded. 10.8 Other Types of International Issues (a) Foreign
Euro Bonds: In domestic capital markets of various countries the Bonds issues
referred to above are known by different names such as Yankee Bonds in the US,
Swiss Frances in Switzerland, Samurai Bonds in Tokyo and Bulldogs in UK. (b) Euro
Convertible Bonds: A convertible bond is a debt instrument which gives the holders
of the bond an option to convert the bonds into a pre-determined number of equity
shares of the company. Usually the price of the equity shares at the time of
conversion will have a premium element. These bonds carry a fixed rate of interest
and if the issuer company so desires may also include a Call Option (where the
issuer company has the option of calling/ buying the bonds for redemption prior to
the maturity date) or a Put Option (which gives the holder the option to put/sell
his bonds to the issuer company at a pre-determined date and price). (c) Euro
Bonds: Plain Euro Bonds are nothing but debt Instruments. These are not very
atattractive for an investor who desires to have valuable additions to his
investments. (d) Euro Convertible Zero Bonds: These bonds are structured as a
convertible bond. No interest is payable on the bonds. But conversion of bonds
takes place on maturity at a predetermined price. Usually there is a five years
maturity period and they are treated as a deferred equity issue. (e) Euro Bonds
with Equity Warrants: These bonds carry a coupon rate determined by market rates.
The warrants are detachable. Pure bonds are traded at a discount. Fixed Income
Funds Management may like to invest for the purposes of regular income.

5.33
Financial Management Self Examination Questions A. Objective Type Questions 1. The
largest provider of short-term credit for a business is: (a) Banks (b) Suppliers
to the firm (c) Commercial paper (d) None of the above. 2. The number of days
until the firm is past due to a supplier is called the: (a) Discount period (b)
Term to credit (c) 3. Payment period (d) None of the above. The principal value of
a bond is called the: (a) The coupon rate (b) The par value (c) The maturity value
(d) None of the above. 4. An advantage of debt financing is: (a) Interest payments
are tax deductible (b) The use of debt, up to a point, lowers the firm's cost of
capital (c) Does not dilute owner's earnings (d) All of the above. 5. Zero coupon
bonds: (a) Are sold at par (b) Pay no interest payment (c) Are sold at a deep
discount (d) b and c above.
5.34
Types of Financing

6.

Commercial paper can be issued by: (a) Corporate (b) Primary Dealers (c) All India
Financial Institutions (d) All of the above.

7.

Commercial paper can be issued in the following denominations: (a) Rs 5 Lakhs (b)
Rs 10 Lakhs (c) Rs 5 Lakhs and multiples there of (d) Rs 1 Lakh and multiples
there of.

8.

Commercial paper is a: (a) Short term source of Finance (b) Medium term source of
Finance (c) Long term source of Finance (d) None of the above.

9.

Which amongst the following are short term sources of finance? (a) Accrued
expenses (b) Deferred income (c) Equity shares (d) Debentures.

10. In a tax environment, the cost of debenture to the issuing company: (a) Lower
than the cost of equity shares (b) Higher than the cost of equity shares (c)
Higher than the cost of preference shares (d) None of the above.

5.35
Financial Management Answers to Objective Type Questions 1. (b); 2. (c); 3. (b);
4. (d); 5. (d); 6. (d); 7. (c); 8. (a); 9. (a) & (b); 10. (a) B. Short Answer Type
Questions 1. 2. Briefly discuss the different sources of long term finance. Write
short notes on: (a) Zero interest fully convertible debentures (b) Deep discount
bonds (c) Inflation bonds (d) Debt securitization. 3. 4. 5. C. 1. 2. 3. 4. 5. Why
are debentures considered cheaper than equity as a source of finance? Discuss
briefly. What do you understand by the term ‘ploughing back of profits’? Discuss
briefly the concept of ‘Seed Capital Assistance’. Long Answer Type Questions
Explain the advantages of equity financing. What are the advantages of debt
financing from the point of company and investors? What do you mean by venture
capital financing and what are the methods of this type of financing? What are the
advantages of lease financing? Discuss the various ways in which Indian companies
can raise foreign currency.

5.36
CHAPTER

INVESTMENT DECISIONS
Learning objectives After studying this chapter, you will be able to ♦ ♦ ♦ ♦
Describe capital budgeting decisions; Understand the purpose and process of
Capital Budgeting; Appreciate the importance of cash flows and understand the
basic principles for measuring the same; Evaluate projects using various capital
budgeting techniques like PB (Pay Back ), NPV (Net Present Value), PI
(Profitability Index) , IRR (Internal Rate of Return), MIRR (Modified Internal
Rate of Return) and ARR (Accounting Rate of Return); and Understand the advantages
and disadvantages of the above mentioned techniques. INTRODUCTION

♦ 1.

Financing and investment of funds are two crucial financial functions. The
investment of funds also termed as capital budgeting requires a number of
decisions to be taken in a situation in which funds are invested and benefits are
expected over a long period. The term capital budgeting means planning for capital
assets. It involves proper project planning and commercial evaluation of projects
to know in advance technical feasibility and financial viability of the project.
The capital budgeting decision means a decision as to whether or not money should
be invested in long-term projects such as the setting up of a factory or
installing a machinery or creating additional capacities to manufacture a part
which at present may be purchased from outside. It includes a financial analysis
of the various proposals regarding capital expenditure to evaluate their impact on
the financial condition of the company and to choose the best out of the various
alternatives. In any business the commitment of funds in land, buildings,
equipment, stock and other types of assets must be carefully made. Once the
decision to acquire a fixed asset is taken, it becomes very difficult to reverse
that decision. The expenditure on plant and machinery and other long term assets
affects operations over a period of years. It becomes a commitment that influences
long term prospects and the future earning capacity of the firm.
Financial Management

However, Capital Budgeting excludes certain investment decisions, wherein, the


benefits of investment proposals cannot be directly quantified. For example,
management may be considering a proposal to build a recreation room for employees.
The decision in this case will be based on qualitative factors, such as management
− employee relations, with less consideration on direct financial returns.
However, most investment proposals considered by management will require
quantitative estimates of the benefits to be derived from accepting the project. A
bad decision can be detrimental to the value of the organisation over a long
period of time. 2. PURPOSE OF CAPITAL BUDGETING

The capital budgeting decisions are important, crucial and critical business
decisions due to following reasons: (i) Substantial expenditure: Capital budgeting
decisions involves the investment of substantial amount of funds. It is therefore
necessary for a firm to make such decisions after a thoughtful consideration so as
to result in the profitable use of its scarce resources. The hasty and incorrect
decisions would not only result into huge losses but may also account for the
failure of the firm. (ii) Long time period: The capital budgeting decision has its
effect over a long period of time. These decisions not only affect the future
benefits and costs of the firm but also influence the rate and direction of growth
of the firm. (iii) Irreversibility: Most of the investment decisions are
irreversible. Once they are taken, the firm may not be in a position to reverse
them back. This is because, as it is difficult to find a buyer for the second-hand
capital items. (iv) Complex decision: The capital investment decision involves an
assessment of future events, which in fact is difficult to predict. Further it is
quite difficult to estimate in quantitative terms all the benefits or the costs
relating to a particular investment decision. 3. CAPITAL BUDGETING PROCESS

The extent to which the capital budgeting process needs to be formalised and
systematic procedures established depends on the size of the organisation; number
of projects to be considered; direct financial benefit of each project considered
by itself; the composition of the firm's existing assets and management's desire
to change that composition; timing of expenditures associated with the projects
that are finally accepted. (i) Planning: The capital budgeting process begins with
the identification of potential investment opportunities. The opportunity then
enters the planning phase when the potential effect on the firm's fortunes is
assessed and the ability of the management of the firm to exploit the opportunity
is determined. Opportunities having little merit are rejected and

6.2
Investment Decisions

promising opportunities are advanced in the form of a proposal to enter the


evaluation phase. (ii) Evaluation: This phase involves the determination of
proposal and its investments, inflows and outflows. Investment appraisal
techniques, ranging from the simple payback method and accounting rate of return
to the more sophisticated discounted cash flow techniques, are used to appraise
the proposals. The technique selected should be the one that enables the manager
to make the best decision in the light of prevailing circumstances. (iii)
Selection: Considering the returns and risks associated with the individual
projects as well as the cost of capital to the organisation, the organisation will
choose among projects so as to maximise shareholders’ wealth. (iv) Implementation:
When the final selection has been made, the firm must acquire the necessary funds,
purchase the assets, and begin the implementation of the project. (v) Control: The
progress of the project is monitored with the aid of feedback reports. These
reports will include capital expenditure progress reports, performance reports
comparing actual performance against plans set and post completion audits. (vi)
Review: When a project terminates, or even before, the organisation should review
the entire project to explain its success or failure. This phase may have
implication for firms planning and evaluation procedures. Further, the review may
produce ideas for new proposals to be undertaken in the future. 4. TYPES OF
CAPITAL INVESTMENT DECISIONS

There are many ways to classify the capital budgeting decision. Generally capital
investment decisions are classified in two ways. One way is to classify them on
the basis of firm’s existence. Another way is to classify them on the basis of
decision situation. 4.1 On the basis of firm’s existence: The capital budgeting
decisions are taken by both newly incorporated firms as well as by existing firms.
The new firms may be required to take decision in respect of selection of a plant
to be installed. The existing firm may be required to take decisions to meet the
requirement of new environment or to face the challenges of competition. These
decisions may be classified as follows: (i) Replacement and Modernisation
decisions: The replacement and modernisation decisions aim at to improve operating
efficiency and to reduce cost. Generally all types of plant and machinery require
replacement either because of the economic life of the plant or machinery is over
or because it has become technologically outdated. The former decision is known as
replacement decisions and later one is known as modernisation decisions. Both
replacement and modernisation decisions are called cost reduction decisions. (ii)
Expansion decisions: Existing successful firms may experience growth in demand of
their product line. If such firms experience shortage or delay in the delivery of
their products

6.3
Financial Management

due to inadequate production facilities, they may consider proposal to add


capacity to existing product line. (iii) Diversification decisions: These
decisions require evaluation of proposals to diversify into new product lines, new
markets etc. for reducing the risk of failure by dealing in different products or
by operating in several markets. Both expansion and diversification decisions are
called revenue expansion decisions. 4.2 On the basis of decision situation: The
capital budgeting decisions on the basis of decision situation are classified as
follows: (i) Mutually exclusive decisions: The decisions are said to be mutually
exclusive if two or more alternative proposals are such that the acceptance of one
proposal will exclude the acceptance of the other alternative proposals. For
instance, a firm may be considering proposal to install a semi-automatic or highly
automatic machine. If the firm install a semiautomatic machine it exclude the
acceptance of proposal to install highly automatic machine. (ii) Accept-reject
decisions: The accept-reject decisions occur when proposals are independent and do
not compete with each other. The firm may accept or reject a proposal on the basis
of a minimum return on the required investment. All those proposals which give a
higher return than certain desired rate of return are accepted and the rest are
rejected. (iii) Contingent decisions: The contingent decisions are dependable
proposals. The investment in one proposal requires investment in one or more other
proposals. For example if a company accepts a proposal to set up a factory in
remote area it may have to invest in infrastructure also e.g. building of roads,
houses for employees etc. 5. PROJECT CASH FLOWS

Project cash flows are defined as the financial costs and benefits associated with
a project. The estimation of costs and benefits are made with the help of inputs
provided by marketing, production, engineering, costing, purchase, taxation, and
other departments. The project cash flow stream consists of cash outflows and cash
inflows. The costs are denoted as cash outflows whereas the benefits are denoted
as cash inflows. The future costs and benefits associated with each project are as
follows: (i) (ii) Capital costs Operating costs

(iii) Revenue (iv) Depreciation (v) Residual value An investment decision implies
the choice of an objective, an appraisal technique and the

6.4
Investment Decisions

project’s life. The objective and technique must be related to definite period of
time. The life of the project may be determined by taking into consideration the
following factors: (i) (ii) Technological obsolescence Physical deterioration

(iii) A decline in demand for the output of the project. No matter how good a
company's maintenance policy, its technological forecasting ability or its demand
forecasting ability, uncertainty will always be present because of the difficulty
in predicting the duration of a project life. To allow realistic appraisal, the
value of cash payment or receipt must be related to the time when the transfer
takes place. In particular, it must be recognised that Re. 1 received today is
worth more than Re. 1 received at some future date because Re. 1 received today
could be earning interest in the intervening period. This is the concept of 'Time
Value of Money' (for a detailed understanding of the Time Value of Money please
refer to Chapter 2). The process of converting future sums into their present
equivalent is known as discounting, which is used to determine the present value
of future cash flows. 6. BASIC PRINCIPLES FOR MEASURING PROJECT CASH FLOWS

For developing the project cash flows the following principles must be kept in
mind: 1. Incremental Principle: The cash flows of a project must be measured in
incremental terms. To ascertain a project’s incremental cash flows, one has to
look at what happens to the cash flows of the firm 'with the project and without
the project', and not before the project and after the project as is sometimes
done. The difference between the two reflects the incremental cash flows
attributable to the project. Project cash flows for year t = Cash flow for the
firm with the project for year t − Cash flow for the firm without the project for
year t. 2. Long Term Funds Principle: A project may be evaluated from various
points of view: total funds point of view, long-term funds point of view, and
equity point of view. The measurement of cash flows as well as the determination
of the discount rate for evaluating the cash flows depends on the point of view
adopted. It is generally recommended that a project may be evaluated from the
point of view of long-term funds (which are provided by equity stockholders,
preference stock holders, debenture holders, and term lending institutions)
because the principal focus of such evaluation is normally on the profitability of
long-term funds. 3. Exclusion of Financing Costs Principle: When cash flows
relating to long-term funds are being defined, financing costs of long-term funds
(interest on long-term debt and equity

6.5
Financial Management

dividend) should be excluded from the analysis. The question arises why? The
weighted average cost of capital used for evaluating the cash flows takes into
account the cost of longterm funds. Put differently, the interest and dividend
payments are reflected in the weighted average cost of capital. Hence, if interest
on long-term debt and dividend on equity capital are deducted in defining the cash
flows, the cost of long-term funds will be counted twice. The exclusion of
financing costs principle means that: (i) (ii) The interest on long-term debt (or
interest) is ignored while computing profits and taxes and; The expected dividends
are deemed irrelevant in cash flow analysis.

While dividends pose no difficulty as they come only from profit after taxes,
interest needs to be handled properly. Since interest is usually deducted in the
process of arriving at profit after tax, an amount equal to interest (1 − tax
rate) should be added back to the figure of profit after tax. That is, Profit
before interest and tax (1 − tax rate) = (Profit before tax + interest) (1 − tax
rate) = (Profit before tax) (1 − tax rate) + (interest) (1 − tax rate) = Profit
after tax + interest (1 − tax rate) Thus, whether the tax rate is applied directly
to the profit before interest and tax figure or whether the tax − adjusted
interest, which is simply interest (1 − tax rate), is added to profit after tax,
we get the same result. 4. Post−tax Principle: Tax payments like other payments
must be properly deducted in deriving the cash flows. That is, cash flows must be
defined in post-tax terms. Illustration 1 ABC Ltd is evaluating the purchase of a
new project with a depreciable base of Rs. 1,00,000; expected economic life of 4
years and change in earnings before taxes and depreciation of Rs. 45,000 in year
1, Rs. 30,000 in year 2, Rs. 25,000 in year 3 and Rs. 35,000 in year 4. Assume
straight-line depreciation and a 20% tax rate. You are required to compute
relevant cash flows.

6.6
Investment Decisions

Solution Rs. Years 1 Earnings before depreciation Less: Depreciation Earnings


before tax Less: Tax [@20%] Add: Depreciation Net Cash flow Working Note:
Depreciation = Rs. 1, 00,000÷4 = Rs. 25,000 Illustration 2 XYZ Ltd is considering
a new investment project about which the following information is available. (i)
The total outlay on the project will be Rs. 100 lacks. This consists of Rs.60
lacks on plant and equipment and Rs.40 lacks on gross working capital. The entire
outlay will be incurred at the beginning of the project. The project will be
financed with Rs.40 lacks of equity capital; Rs.30 lacks of long term debt (in the
form of debentures); Rs. 20 lacks of short-term bank borrowings, and Rs. 10 lacks
of trade credit. This means that Rs.70 lacks of long term finds (equity + long
term debt) will be applied towards plant and equipment (Rs. 60 lacks) and working
capital margin (Rs.10 lacks) – working capital margin is defined as the
contribution of long term funds towards working capital. The interest rate on
debentures will be 15 percent and the interest rate on short-term borrowings will
be 18 percent. tax and 45,000 25,000 20,000 4,000 16,000 25,000 41,000 2 30,000
25,000 5,000 1,000 4,000 25,000 29,000 3 25,000 25,000 0 0 0 25,000 25,000 4
35,000 25,000 10,000 2,000 8,000 25,000 33,000

(ii)

(iii) The life of the project is expected to be 5 years and the plant and
equipment would fetch a salvage value of Rs. 20 lacks. The liquidation value of
working capital will be equal to Rs.10 lacks. (iv) The project will increase the
revenues of the firm by Rs. 80 lacks per year. The increase in operating expenses
on account of the project will be Rs.35.0 lacks per year. (This

6.7
Financial Management

includes all items of expenses other than depreciation, interest, and taxes). The
effective tax rate will be 50 percent. (v) Plant and equipment will be depreciated
at the rate of 331/3 percent per year as per the written down value method. So,
the depreciation charges will be : Rs. (in lacs) First year 20.0 Second year 13.3
Third year 8.9 Fourth year 5.9 Fifth year 4.0 Given the above details, you are
required to work out the post-tax, incremental cash flows relating to long-term
funds. Solution Cash Flows for the New Project
Rs. (in lacs) Years 0 (a) (b) (c) (d) (e) (f) (g) (h) (i) (j) (k) (l) (m) Plant
and equipment Working capital margin Revenues Operating Costs Depreciation
Interest on short-term bank borrowings Interest on debentures Profit before tax
Tax Profit after tax Net salvage value of plant and equipment Net recovery working
capital margin Initial Investment [(a) + (b)] (70.0) (60.0) (10.0) 80.0 35.0 20.0
3.6 4.5 16.90 8.45 8.45 80.0 35.0 13.33 3.6 4.5 23.57 11.79 11.78 80.0 35.0 8.89
3.6 4.5 28.01 14.01 14.00 80.0 35.0 5.93 3.6 4.5 30.97 15.49 15.48 80.0 35.0 3.95
3.6 4.5 32.95 16.48 16.47 20.0 10.0 1 2 3 4 5

6.8
Investment Decisions
(n) Operating cash inflows [(j) +(e) + (g) (1–T)] (o) Terminal cash flow [(k) +
(l) ] (p) Net cash flow. [(m) + (n) + (o) ] (70.0) 30.70 27.36 25.14 23.66 52.67
30.00 30.70 27.36 25.14 23.66 22.67

Working Notes (with explanations): (i) The initial investment, occurring at the
end of year 0, is Rs. 70 lacs. This represents the commitment of long-term funds
to the project. The operating cash inflow, relating to longterm funds, at the end
of year 1 is Rs. 30.7 lacs. That is, Profit after tax + Depreciation + Interest on
debentures (1 – tax ) Rs. 8.45 lacs + Rs.20 lacs + Rs. 4.5 lacs (1 – 0.50) The
operating cash inflows for the subsequent years have been calculated similarly.
(ii) The terminal cash flow relating to long-term funds is equal to : Net Salvage
value of plant and equipment + Net recovery of working capital margin When the
project is terminated, its liquidation value will be equal to: Net Salvage value
of plant and equipment + Net recovery of working capital The first component
belongs to the suppliers of long-term funds. The second component is applied to
repay the current liabilities and recover the working capital margin. 7. CAPITAL
BUDGETING TECHNIQUES

In order to maximise the return to the shareholders of a company, it is important


that the best or most profitable investment projects are selected. Because the
results for making a bad long-term investment decision can be both financially and
strategically devastating, particular care needs to be taken with investment
project selection and evaluation. There are a number of techniques available for
appraisal of investment proposals and can be classified as presented below:

6.9
Financial Management

CAPITAL BUDGETING TECHNIQUES

Traditional or Non-Discounting

Time-adjusted or Discounted Cash Flows Net Present Value Profitability Index

Payback Period

Accounting Rate of Return

Internal Rate of Return Modified Internal Rate of Return

Discounted Payback

Organizations may use any or more of capital investment evaluation techniques;


some organizations use different methods for different types of projects while
others may use multiple methods for evaluating each project. These techniques have
been discussed below – net present value, profitability index, internal rate of
return, modified internal rate of return, payback period, and accounting (book)
rate of return. Payback Period: The payback period of an investment is the length
of time required for the cumulative total net cash flows from the investment to
equal the total initial cash outlays. At that point in time, the investor has
recovered the money invested in the project. As with other methods discussed, the
first steps in calculating the payback period are determining the total initial
capital investment and the annual expected after-tax net cash flows over the
useful life of the investment. When the net cash flows are uniform over the useful
life of the project, the number of years in the payback period can be calculated
using the following

6.10
Investment Decisions

equation:
Payback period = Total initial capital investment Annual expected after - tax net
cash flow

When the annual expected after-tax net cash flows are not uniform, the cumulative
cash inflow from operations must be calculated for each year by subtracting cash
outlays for operations and taxes from cash inflows and summing the results until
the total is equal to the initial capital investment. Advantages: A major
advantage of the payback period technique is that it is easy to compute and to
understand as it provides a quick estimate of the time needed for the organization
to recoup the cash invested. The length of the payback period can also serve as an
estimate of a project’s risk; the longer the payback period, the riskier the
project as long-term predictions are less reliable. The payback period technique
focuses on quick payoffs. In some industries with high obsolescence risk or in
situations where an organization is short on cash, short payback periods often
become the determining factor for investments. Limitations: The major limitation
of the payback period technique is that it ignores the time value of money. As
long as the payback periods for two projects are the same, the payback period
technique considers them equal as investments, even if one project generates most
of its net cash inflows in the early years of the project while the other project
generates most of its net cash inflows in the latter years of the payback period.
A second limitation of this technique is its failure to consider an investment’s
total profitability; it only considers cash flows from the initiation of the
project until its payback period and ignores cash flows after the payback period.
Lastly, use of the payback period technique may cause organizations to place too
much emphasis on short payback periods thereby ignoring the need to invest in
long-term projects that would enhance its competitive position. Illustration 3
Suppose a project costs Rs. 20,00,000 and yields annually a profit of Rs. 3,00,000
after depreciation @ 12½% (straight line method) but before tax 50%. The first
step would be to calculate the cash inflow from this project. The cash inflow is
Rs. 4,00,000 calculated as follows : Rs. Profit before tax 3,00,000 Less : Tax @
50% 1,50,000 Profit after tax 1,50,000 Add : Depreciation written off 2,50,000
Total cash inflow 4,00,000 While calculating cash inflow, depreciation is added
back to profit after tax since it does not
6.11
Financial Management

result in cash outflow. The cash generated from a project therefore is equal to
profit after tax plus depreciation. Payback period =
Rs. 20,00,000 = 5 Years 4,00,000

Some Accountants calculate payback period after discounting the cash flows by a
predetermined rate and the payback period so calculated is called, ‘Discounted
payback period’. Payback Reciprocal : As the name indicates it is the reciprocal
of payback period. A major drawback of the payback period method of capital
budgeting is that it does not indicate any cut off period for the purpose of
investment decision. It is, however, argued that the reciprocal of the payback
would be a close approximation of the internal rate of return if the life of the
project is at least twice the payback period and the project generates equal
amount of the annual cash inflows. In practice, the payback reciprocal is a
helpful tool for quickly estimating the rate of return of a project provided its
life is at least twice the payback period. The payback reciprocal can be
calculated as follows:

Average annual cash in flow Initial investment


Illustration 4 Suppose a project requires an initial investment of Rs. 20,000 and
it would give annual cash inflow of Rs. 4,000. The useful life of the project is
estimated to be 5 years. In this example payback reciprocal will be :
Rs 4,000 × 100 = 20% Rs20,000

The above payback reciprocal provides a reasonable approximation of the internal


rate of return, i.e. 19% discussed later in this chapter. Accounting (Book) Rate
of Return: The accounting rate of return of an investment measures the average
annual net income of the project (incremental income) as a percentage of the
investment. Accounting rate of return =

Average annual net income Investment

The numerator is the average annual net income generated by the project over its
useful life. The denominator can be either the initial investment or the average
investment over the useful life of the project. Some organizations prefer the
initial investment because it is objectively determined and is not influenced by
either the choice of the depreciation method or the

6.12
Investment Decisions

estimation of the salvage value. Either of these amounts is used in practice but
it is important that the same method be used for all investments under
consideration. Advantages: The accounting rate of return technique uses readily
available data that is routinely generated for financial reports and does not
require any special procedures to generate data. This method may also mirror the
method used to evaluate performance on the operating results of an investment and
management performance. Using the same procedure in both decision-making and
performance evaluation ensures consistency. Lastly, the calculation of the
accounting rate of return method considers all net incomes over the entire life of
the project and provides a measure of the investment’s profitability. Limitations:
The accounting rate of return technique, like the payback period technique,
ignores the time value of money and considers the value of all cash flows to be
equal. Additionally, the technique uses accounting numbers that are dependent on
the organization’s choice of accounting procedures, and different accounting
procedures, e.g., depreciation methods, can lead to substantially different
amounts for an investment’s net income and book values. The method uses net income
rather than cash flows; while net income is a useful measure of profitability, the
net cash flow is a better measure of an investment’s performance. Furthermore,
inclusion of only the book value of the invested asset ignores the fact that a
project can require commitments of working capital and other outlays that are not
included in the book value of the project. Illustration 5 Suppose a project
requiring an investment of Rs. 10,00,000 yields profit after tax and depreciation
as follows: Profit after tax and depreciation Rs. 1. 50,000 2. 75,000 3. 1,25,000
4. 1,30,000 5. 80,000 Total 4,60,000 Suppose further that at the end of 5 years,
the plant and machinery of the project can be sold for Rs. 80,000. In this case
the rate of return can be calculated as follows. Years

Total Profit × 100 Net investment in the project × No. of years of profit

6.13
Financial Management

Rs 4,60,000 × 100 Rs 9,20,000 × 5 years

= 10%

This rate is compared with the rate expected on other projects, had the same funds
been invested alternatively in those projects. Sometimes, the management compares
this rate with the minimum rate (called-cut off rate) they may have in mind. For
example, management may decide that they will not undertake any project which has
an average annual yield after tax less than 15%. Any capital expenditure proposal
which has an average annual yield of less than 15% will be automatically rejected.
Net Present Value Technique: The net present value technique is a discounted cash
flow method that considers the time value of money in evaluating capital
investments. An investment has cash flows throughout its life, and it is assumed
that a rupee of cash flow in the early years of an investment is worth more than a
rupee of cash flow in a later year. The net present value method uses a specified
discount rate to bring all subsequent net cash inflows after the initial
investment to their present values (the time of the initial investment or year 0).
Theoretically, the discount rate or desired rate of return on an investment is the
rate of return the firm would have earned by investing the same funds in the best
available alternative investment that has the same risk. Determining the best
alternative opportunity available is difficult in practical terms so rather that
using the true opportunity cost, organizations often use an alternative measure
for the desired rate of return. An organization may establish a minimum rate of
return that all capital projects must meet; this minimum could be based on an
industry average or the cost of other investment opportunities. Many organizations
choose to use the cost of capital as the desired rate of return; the cost of
capital is the cost that an organization has incurred in raising funds or expects
to incur in raising the funds needed for an investment. The overall cost of
capital of a firm is a proportionate average of the costs of the various
components of the firm’s financing. A firm obtains funds by issuing preferred or
common stock; borrowing money using various forms of debt such a notes, loans, or
bonds; or retaining earnings. The costs to the firm are the returns demanded by
debt and equity investors through which the firm raises the funds. The net present
value of a project is the amount, in current rupees, the investment earns after
yielding the desired rate of return in each period. Net present value = Present
value of net cash flow - Total net initial investment The steps to calculating net
present value are (1) determine the net cash inflow in each year of the
investment, (2) select the desired rate of return, (3) find the discount factor
for each year based on the desired rate of return selected, (4) determine the
present values of the net

6.14
Investment Decisions

cash flows by multiplying the cash flows by the discount factors, (5) total the
amounts for all years in the life of the project, and (6) subtract the total net
initial investment. Illustration 6 Compute the net present value for a project
with a net investment of Rs. 1, 00,000 and the following cash flows if the
company’s cost of capital is 10%? Net cash flows for year one is Rs. 55,000; for
year two is Rs. 80,000 and for year three is Rs. 15,000. [PVIF @ 10% for three
years are 0.909, 0.826 and 0.751] Solution Year 1 2 3 Total Discounted Cash Flows
Less: Net Investment Net Present Value Net Cash Flows 55,000 80,000 15,000
1,27,340 1,00,000 27,340 PVIF @ 10% 0.909 0.826 0.751 Discounted Cash Flows 49,995
66,080 11,265 1,27,340

Recommendation: Since the net present value of the project is positive, the
company should accept the project. Illustration 7 ABC Ltd is a small company that
is currently analyzing capital expenditure proposals for the purchase of
equipment; the company uses the net present value technique to evaluate projects.
The capital budget is limited to 500,000 which ABC Ltd believes is the maximum
capital it can raise. The initial investment and projected net cash flows for each
project are shown below. The cost of capital of ABC Ltd is 12%. You are required
to compute the NPV of the different projects. Initial Investment Project Cash
Inflows Year 1 2 3 4 5 Project A 200,000 50,000 50,000 50,000 50,000 50,000
Project B 190,000 40,000 50,000 70,000 75,000 75,000 Project C 250,000 75,000
75,000 60,000 80,000 100,000 Project D 210,000 75,000 75,000 60,000 40,000 20,000

6.15
Financial Management

Solution Calculation of net present value: Present Period value factor 1 0.893 2
0.797 3 0.712 4 0.636 5 0.567 Present value of cash inflows Less: Initial
investment Net present value Advantages (i) (ii) NPV method takes into account the
time value of money. The whole stream of cash flows is considered. Project A
44,650 39,850 35,600 31,800 28,350 180,250 200,000 (19,750) Project B 35,720
39,850 49,840 47,700 42,525 215,635 190,000 25,635 Project C 66,975 59,775 42,720
50,880 56,700 277,050 250,000 27,050 Project D 66,975 59,775 42,720 25,440 11,340
206,250 210,000 (3,750)

(iii) The net present value can be seen as the addition to the wealth of share
holders. The criterion of NPV is thus in conformity with basic financial
objectives. (iv) The NPV uses the discounted cash flows i.e., expresses cash flows
in terms of current rupees. The NPVs of different projects therefore can be
compared. It implies that each project can be evaluated independent of others on
its own merit. Limitations (i) (ii) It involves difficult calculations. The
application of this method necessitates forecasting cash flows and the discount
rate. Thus accuracy of NPV depends on accurate estimation of these two factors
which may be quite difficult in practice.

(iii) The ranking of projects depends on the discount rate. Let us consider two
projects involving an initial outlay of Rs. 25 lakhs each with following inflow :
(Rs in lakhs) Project A Project B 1st year 50.0 12.5 2nd year 12.5 50.0

6.16
Investment Decisions

At discounted rate of 5% and 10% the NPV of Projects and their rankings at 5% and
10% are as follows: NPV @ 5% Project A Project B 33.94 32.25 Rank I II NPV @ 10%
30.78 27.66 Rank I II

The project ranking is same when the discount rate is changed from 5% to 10%.
However, the impact of the discounting becomes more severe for the later cash
flows. Naturally, higher the discount rate, higher would be the impact. In the
case of project B the larger cash flows come later in the project life, thus
decreasing the present value to a larger extent. (iv) The decision under NPV
method is based on absolute measure. It ignores the difference in initial
outflows, size of different proposals etc. while evaluating mutually exclusive
projects. Desirability Factor/Profitability Index: In above Illustration the
students may have seen how with the help of discounted cash flow technique, the
two alternative proposals for capital expenditure can be compared. In certain
cases we have to compare a number of proposals each involving different amounts of
cash inflows. One of the methods of comparing such proposals is to workout what is
known as the ‘Desirability factor’, or ‘Profitability index’. In general terms a
project is acceptable if its profitability index value is greater than 1.
Mathematically : The desirability factor is calculated as below :

Sum of discounted cash in flows Initial cash outlay/Total discounted cash outflow
(as the case may)
Illustration 8 Suppose we have three projects involving discounted cash outflow of
Rs.5,50,000, Rs75,000 and Rs.1,00,20,000 respectively. Suppose further that the
sum of discounted cash inflows for these projects are Rs. 6,50,000, Rs. 95,000 and
Rs 1,00,30,000 respectively. Calculate the desirability factors for the three
projects. Solution The desirability factors for the three projects would be as
follows: 1.
Rs. 6,50,000 = 1.18 Rs. 5,50,000

6.17
Financial Management

2. 3.

Rs. 95,000 = Rs. 75,000

1.27 1.001

Rs.1,00,30,000 = Rs.1,00,20,000

It would be seen that in absolute terms project 3 gives the highest cash inflows
yet its desirability factor is low. This is because the outflow is also very high.
The Desirability/ Profitability Index factor helps us in ranking various projects.
Advantages The method also uses the concept of time value of money and is a better
project evaluation technique than NPV. Limitations Profitability index fails as a
guide in resolving capital rationing (discussed later in this chapter) where
projects are indivisible. Once a single large project with high NPV is selected,
possibility of accepting several small projects which together may have higher NPV
than the single project is excluded. Also situations may arise where a project
with a lower profitability index selected may generate cash flows in such a way
that another project can be taken up one or two years later, the total NPV in such
case being more than the one with a project with highest Profitability Index. The
Profitability Index approach thus cannot be used indiscriminately but all other
type of alternatives of projects will have to be worked out. 8. CAPITAL RATIONING

Generally, firms fix up maximum amount that can be invested in capital projects,
during a given period of time, say a year. The firm then attempts to select a
combination of investment proposals that will be within the specific limits
providing maximum profitability and rank them in descending order according to
their rate of return; such a situation is of capital rationing. A firm should
accept all investment projects with positive NPV, with an objective to maximise
the wealth of shareholders. However, there may be resource constraints due to
which a firm may have to select from among various projects. Thus there may arise
a situation of capital rationing where there may be internal or external
constraints on procurement of necessary funds to invest in all investment
proposals with positive NPVs. Capital rationing can be experienced due to external
factors, mainly imperfections in capital markets which can be attributed to non-
availability of market information, investor attitude etc. Internal capital
rationing is due to the self-imposed restrictions imposed by management like not
to raise additional debt or laying down a specified minimum rate of return on each
project.

6.18
Investment Decisions

There are various ways of resorting to capital rationing. For instance, a firm may
effect capital rationing through budgets. It may also put up a ceiling when it has
been financing investment proposals only by way of retained earnings (ploughing
back of profits). Since the amount of capital expenditure in that situation cannot
exceed the amount of retained earnings, it is said to be an example of capital
rationing. Capital rationing may also be introduced by following the concept of
‘Responsibility Accounting’, whereby management may introduce capital rationing by
authorising a particular department to make investment only up to a specified
limit, beyond which the investment decisions are to be taken by higher-ups. The
selection of project under capital rationing involves two steps: (i) (ii) To
identify the projects which can be accepted by using the technique of evaluation
discussed above. To select the combination of projects.

In capital rationing it may also be more desirable to accept several small


investment proposals than a few large investment proposals so that there may be
full utilisation of budgeted amount. This may result in accepting relatively less
profitable investment proposals if full utilisation of budget is a primary
consideration. Similarly, capital rationing may also mean that the firm foregoes
the next most profitable investment following after the budget ceiling even though
it is estimated to yield a rate of return much higher than the required rate of
return. Thus capital rationing does not always lead to optimum results. The
following illustration shows how a firm may resort to capital rationing under
situation of resource constraints. Illustration 9 Alpha Limited is considering
five capital projects for the years 2000,2001,2002 and 2003. The company is
financed by equity entirely and its cost of capital is 12%. The expected cash
flows of the projects are as follows : Year and Cash flows (Rs. ‘000) Project A B
C D E 2000 (70) (40) (50) — (60) 2001 35 (30) (60) (90) 20 2002 35 45 70 55 40
2003 20 55 80 65 50

Note : Figures in brackets represent cash outflows.


6.19
Financial Management

All projects are divisible i.e. size of investment can be reduced, if necessary in
relation to availability of funds. None of the projects can be delayed or
undertaken more than once. Calculate which project Alpha Limited should undertake
if the capital available for investment is limited to Rs. 1,10,000 in year 2000
and with no limitation in subsequent years. For your analysis, use the following
present value factors: Year Discounting factor Solution Computation of Net Present
Value (NPV) & Profitability Index (PI) (Rs. ‘000) Project Discounted 2000 A B C D
E (70) (40) (50) — (60) Cash Flows 2001 31.15 (26.70) (53.40) (80.10) 17.80 2002
28 36 56 44 32 (Refer to working note) 2003 14.20 39.05 56.80 46.15 35.50 NPV 3.35
8.35 9.40 10.05 25.30 PI 1.048 1.125 1.091 1.125 1.422 2000 1.00 2001 0.89 2002
0.80 2003 0.71

Ranking of Projects in descending order of profitability index Rank Projects 1 E 2


D 3 B 4 C 5 A

Selection and Analysis: For Project ‘D’ there is no capital rationing but it
satisfies the criterion of required rate of return. Hence Project D may be
undertaken. For other projects the requirement is Rs. 2,20,000 in year 2000
whereas the capital available for investment is only Rs. 1,10,000. Based on the
ranking, the final selection from other projects which will yield maximum NPV will
be:

6.20
Investment Decisions

Project and Rank E(1) B(2) C(3) Ranking of Projects capital availability :
Projects Rank Working Note :

Amount of Initial Investment Rs. 60,000 40,000 10,000 (Restriction) excluding ‘D’
which is to start in 2001 when there is no limitation on E 1 B 2 C 3 A 4

Computation of Discounted Cash flows Year Present value factor Project A Cash
flows Discounted cash flows B Cash flows Discounted cash flows C Cash flows
Discounted cash flows D Cash flows Discounted cash flows E Cash flows Discounted
cash flows Illustration 10 2000 1.00 (70) (70) (40) (40) (50) (50) — — (60) (60)
2001 0.89 35 31.15 (30) (26.70) (60) (53.40) (90) (80.10) 20 17.80

(Rs. ‘000) 2002 0.80 35 28 45 36 70 56 55 44 40 32 2003 0.71 20 14.20 55 39.05 80


56.80 65 46.15 50 35.50

Note : Figures in brackets represent cash outflows. Venture Ltd. has Rs. 30 lakhs
available for investment in capital projects. It has the option of making
investment in projects 1,2,3 and 4. Each project is entirely independent and has a
useful life of 5 years. The expected present value of cash flows from the projects
is as follows:

6.21
Financial Management

Projects

Initial outlay Rs.

Present value of cash flowsf Rs. 10,00,000 19,00,000 11,40,000 20,00,000

1 2 3 4

8,00,000 15,00,000 7,00,000 13,00,000

Which of the above investment should be undertaken? Assume that the cost of
capital is 12% and risk free interest rate is 10% per annum. Given compounded sum
of Re. 1 at 10% in 5 years is Rs. 1.611 and discount factor of Re. 1 at 12% rate
for 5 years is 0.567. Solution This is a problem on capital rationing. The fund
available with the company is Rs. 30 lakhs. The company will adopt those projects
which will maximise the NPV. Statement showing NPV of projects Project Initial
outlay Present value of future cash flow Rs (i) 1 2 3 4 8,00,000 15,00,000
7,00,000 13,00,000 (ii) 10,00,000 19,00,000 11,40,000 20,00,000 NPV

Rs (iii)=(ii) – (i) 2,00,000 4,00,000 4,40,000 7,00,000

The NPV of the projects 1,2,3 is Rs. 10,40,000 (with full available amount
utilised). The NPV of the projects 1, 3 and 4 is Rs. 40,000 (with Rs. 28 lakhs
utilised, leaving Rs. 2,00,000 to be invested elsewhere). Now, Rs. 2,00,000 can be
invested for a period of 5 years @ 10%. It is given in the question that the
compounded value of Re. 1 @ 10% per annum for 5 years is Rs. 1.611. Therefore for
Rs. 2,00,000 invested now for 5 years @ 10% the amount to be received after 5
years will be Rs. 3,22,200 (Rs. 2,00,000 × 1.611).

6.22
Investment Decisions

The amount to be received at the end of 5th years would be Rs. 1,82,687.40 (Rs.
3,22,200 × 0.567). Since, the amount to be received 15,22,687 after 5 years by
making investment in projects 1, 3 and 4 & investing balance amount available @
10% is greater than the amount to be received of Rs. 10,40,000 by investing in
projects 1, 2 and 3. It is advisable that the venture Ltd. should make investment
in projects 1, 3 and 4. Internal Rate of Return Method: Like the net present value
method, the internal rate of return method considers the time value of money, the
initial cash investment, and all cash flows from the investment. Unlike the net
present value method, the internal rate of return method does not use the desired
rate of return but estimates the discount rate that makes the present value of
subsequent net cash flows equal to the initial investment. Using this estimated
rate of return, the net present value of the investment will be zero. This
estimated rate of return is then compared to a criterion rate of return that can
be the organization’s desired rate of return, the rate of return from the best
alternative investment, or another rate the organization chooses to use for
evaluating capital investments. The procedures for computing the internal rate of
return vary with the pattern of net cash flows over the useful life of an
investment. The first step is to determine the investment’s total net initial cash
disbursements and commitments and its net cash inflows in each year of the
investment. For an investment with uniform cash flows over its life, the following
equation is used: Total initial investment = Annual net cash flow x Annuity
discount factor of the discount rate for the number of periods of the investment’s
useful life If A is the annuity discount factor, then A=

Total initial cash disbursements and commitment s for the investment Annual
(equal) net cash flows from the investment

Once A has been calculated, the discount rate is the interest rate that has the
same discount factor as A in the annuity table along the row for the number of
periods of the useful life of the investment. This computed discount rate or the
internal rate of return will be compared to the criterion rate the organization
has selected to assess the investment’s desirability. When the net cash flows are
not uniform over the life of the investment, the determination of the discount
rate can involve trial and error and interpolation between interest rates. It
should be noted that there are several spreadsheet programs available for
computing both net present value and internal rate of return that facilitate the
capital budgeting process.

6.23
Financial Management

Illustration 11 Calculate the internal rate of return of an investment of Rs. 1,


36,000 which yields the following cash inflows: Year 1 2 3 4 5 Solution
Calculation of IRR Since the cash inflow is not uniform, the internal rate of
return will have to be calculated by the trial and error method. In order to have
an approximate idea about such rate, the ‘Factor’ must be found out. ‘The factor
reflects the same relationship of investment and cash inflows as in case of
payback calculations’: Cash Inflows (in Rs.) 30,000 40,000 60,000 30,000 20,000

F=
Where, F I C

I C
= = = Factor to be located Original Investment Average Cash inflow per year

For the project,

Factor =

1,36,000 36,000

= 3.78 The factor thus calculated will be located in the present value of Re.1
received annually for N year’s table corresponding to the estimated useful life of
the asset. This would give the expected rate of return to be applied for
discounting the cash inflows. In case of the project, the rate comes to 10%.

6.24
Investment Decisions

Year 1 2 3 4 5

Cash Inflows (Rs.) 30,000 40,000 60,000 30,000 20,000

Discounting factor at 10% 0.909 0.826 0.751 0.683

Present Value (Rs.) 27,270 33,040 45,060 20,490 12,420 1,38,280

0.621 Total present value

The present value at 10% comes to Rs. 1,38,280, which is more than the initial
investment. Therefore, a higher discount rate is suggested, say, 12%. Year 1 2 3 4
5 Cash Inflows (Rs.) 30,000 40,000 60,000 30,000 20,000 Discounting factor at 10%
0.893 0.797 0.712 0.636 0.567 Total present value Present Value (Rs.) 26,790
31,880 42,720 19,080 11,340 1,31,810

The internal rate of return is, thus, more than 10% but less than 12%. The exact
rate can be obtained by interpolation:

IRR =

⎡ ⎛ Rs.1,38,280 − Rs.1,36,000 ⎞⎤ ⎟⎥ ⎢10 + ⎜ ⎝ Rs.1,38,280 − Rs.1,31,810 ⎠⎦ x 2 ⎣ ⎛


2280 ⎞ x2 ⎟ ⎜ ⎝ 6470 ⎠ 10 +
10 + 0.7

= =

IRR = 10.7% Acceptance Rule The use of IRR, as a criterion to accept capital
investment decision involves a comparison of

6.25
Financial Management

IRR with the required rate of return known as cut off rate . The project should
the accepted if IRR is greater than cut-off rate. If IRR is equal to cut off rate
the firm is indifferent. If IRR less than cut off rate the project is rejected.
Internal rate of return and mutually exclusive projects Projects are called
mutually exclusive, when the selection of one precludes the selection of others
e.g. in case a company owns a piece of land which can be put to use for either of
the two different projects S or L, then such projects are mutually exclusive to
each other i.e. the selection of one project necessarily means the rejection of
the other. Refer to the figure below:
Net Present Value (Rs.) 400 Project L’s Net Present Value Profile 300

200 Cross overrabe = 7% Project S5 Net Present value profile 100 IRR = 14% 0

10 IRR = 12%

15

Cost of Capital %

As long as the cost of capital is greater than the crossover rate of 7 % , then
(1) NPV

is

larger than NPV L and (2) IRR S exceeds IRR L . Hence , if the cut off rate or the
cost of capital is greater than 7% , both the methods shall lead to selection of
project S. However, if the cost of capital is less than 7% , the NPV method ranks
Project L higher , but the IRR method indicates that the Project S is better. As
can be seen from the above discussion, mutually exclusive projects can create
problems with the IRR technique because IRR is expressed as a percentage and does
not take into account the scale of investment or the quantum of money earned. Let
us consider another example of two mutually exclusive projects A and B with the
following details,

6.26
Investment Decisions

Cash flows Year 0 Project A Project B (Rs 1,00,000) (Rs 5,00,000) Year 1 Rs
1,50,000 Rs 6,25,000 IRR 50% 25% NPV(10%) Rs 36,360 Rs 68,180

Project A earns a return of 50% which is more than what Project B earns; however
the NPV of Project B is greater than that of Project A . Acceptance of Project A
means that Project B must be rejected since the two Projects are mutually
exclusive. Acceptance of Project A also implies that the total investment will be
Rs 4,00,000 less than if Project B had been accepted, Rs 4,00,000 being the
difference between the initial investment of the two projects. Assuming that the
funds are freely available at 10% , the total capital expenditure of the company
should be ideally equal to the sum total of all outflows provided they earn more
than 10% along with the chosen project from amongst the mutually exclusive. Hence,
in case the smaller of the two Projects i.e. Project A is selected, the
implication will be of rejecting the investment of additional funds required by
the larger investment. This shall lead to a reduction in the shareholders wealth
and thus, such an action shall be against the very basic tenets of Financial
Management. In the above mentioned example the larger of the two projects had the
lower IRR , but never the less provided for the wealth maximising choice. However,
it is not safe to assume that a choice can be made between mutually exclusive
projects using IRR in cases where the larger project also happens to have the
higher IRR . Consider the following two Projects A and B with their relevant cash
flows; Year 0 1 2 3 4 A Rs (9,00,000) 7,00,000 6,00,000 4,00,000 50,000 B Rs
(8,00,000) 62,500 6,00,000 6,00,000 6,00,000

6.27
Financial Management

In this case Project A is the larger investment and also has t a higher IRR as
shown below, Year 0 1 2 3 4 IRR A = 46% IRR B = 35% However, in case the relevant
discounting factor is taken as 5% , the NPV of the two projects provides a
different picture as follows; Project A Year 0 1 2 3 4 (Rs) (9,00,000) 7,00,000
6,00,000 4,00,000 50,000 r= 5% 1.0 0.9524 0.9070 0.8638 0.8227 NPV PV(Rs)
(9,00,000) 6,66,680 5,44,200 3,45,520 41,135 6,97,535 (Rs) (8,00,000) 62,500
6,00,000 6,00,000 6,00,000 Project B r= 5% 1.0 0.9524 0.9070 0.8638 0.8227 PV(Rs)
(8,00,000) 59,525 5,44,200 5,18,280 4,93,630 8,15,635 (Rs) (9,00,000) 7,00,000
6,00,000 4,00,000 50,000 r=46% 1.0 0.6849 0.4691 0.3213 0.2201 PV(Rs) (9,00,000)
4,79, 430 2,81,460 1,28,520 11,005 (415) (Rs) (8,00,000) 62,500 6,00,000 6,00,000
6,00,000 R=35% 1.0 0.7407 0.5487 0.4064 0.3011 PV(Rs) (8,00,000) 46,294 3,29,220
2,43,840 1,80,660 14

As may be seen from the above, Project B should be the one to be selected even
though its IRR is lower than that of Project A. This decision shall need to be
taken in spite of the fact that Project A has a larger investment coupled with a
higher IRR as compared with Project B. This type of an anomalous situation arises
because of the reinvestment assumptions implicit in the two evaluation methods of
NPV and IRR. The Reinvestment assumption The Net Present Value technique assumes
that all cash flows can be reinvested at the discount rate used for calculating
the NPV. This is a logical assumption since the use of the NPV technique implies
that all projects which provide a higher return than the discounting factor are
accepted. In contrast, IRR technique assumes that all cash flows are reinvested at
the projects IRR. This assumption means that projects with heavy cash flows in the
early

6.28
Investment Decisions

years will be favoured by this method vis-à-vis projects which have got heavy cash
flows in the later years. This implicit reinvestment assumption means that
Projects like A , with cash flows concentrated in the earlier years of life will
be preferred by the method relative to Projects such as B. Projects with unequal
lives Let us consider two mutually exclusive projects ‘A’ and ‘B’ with the
following cash flows, Year Project A Project B 0 Rs (30,000) (30,000) 1 Rs 20,000
37,500 2 Rs 20,000

The calculation of NPV and IRR could help us evaluate the two projects; however,
it is also possible to equate the life span of the two for decision making
purposes. The two projects can be equated in terms of time span by assuming that
the company can reinvest in Project ‘B’ at the end of year 1. In such a case the
cash flows of Project B will appear to be as follows; Year Project ‘B’ Project B
reinvested Total cash flows (30,000) 0 Rs (30,000) 1 Rs 37,500 (30,000) 7,500
37,500 37,500 2 Rs

The NPVs and IRRs of both these projects under both the alternatives are shown
below NPV (r=10%) Rs Cash flows (Project A 2Years) Cash flows (Project B 1 Year)
Adjusted cash flows (Project A 2 Years) Adjusted cash flow (Project B 2 years) NPV
A = 4,711 22% 25% 22% 25% IRR

NPV B = 4,090 NPV A = 4,711

NPV B = 7,810

6.29
Financial Management

As may be seen from the above analysis, the ranking as per IRR makes Project B
superior to Project A, irrespective of the period over which the assumption is
made. However, when we consider NPV, the decision shall be favouring Project B; in
case the re investment assumption is taken into account. This is diametrically
opposite to the decision on the basis of NPV when re investment is not assumed. In
that case the NPV of Project A makes it the preferred project. Hence, in case it
is possible to re invest as shown in the example above, it is advisable to compare
and analyse alternative projects by considering equal lives, however, this process
cannot be generalised to be the best practice, as every case shall need to be
judged on its own distinctive merits. Comparisons over differing lives are
perfectly fine if there is no presumption that the company will be required or
shall decide to re invest in similar assets. Multiple Internal Rate of Return: In
cases where project cash flows change signs or reverse during the life of a
project e.g. an initial cash outflow is followed by cash inflows and subsequently
followed by a major cash outflow , there may be more than one IRR. The following
graph of discount rate versus NPV may be used as an illustration;
NPV

IRR

IRR 2

Discount Role

In such situations if the cost of capital is less than the two IRRs , a decision
can be made easily , however otherwise the IRR decision rule may turn out to be
misleading as the project should only be invested if the cost of capital is
between IRR1 and IRR2. To understand the concept of multiple IRRs it is necessary
to understand the implicit re investment assumption in both NPV and IRR
techniques. Advantages

6.30
Investment Decisions

(i) (ii)

This method makes use of the concept of time value of money. All the cash flows in
the project are considered.

6.31
Financial Management

(iii) IRR is easier to use as instantaneous understanding of desirability can be


determined by comparing it with the cost of capital (iv) IRR technique helps in
achieving the objective of minimisation of shareholders wealth. Limitations (i)
The calculation process is tedious if there are more than one cash outflows
interspersed between the cash inflows, there can be multiple IRRs, the
interpretation of which is difficult. The IRR approach creates a peculiar
situation if we compare two projects with different inflow/outflow patterns.

(ii)

(iii) It is assumed that under this method all the future cash inflows of a
proposal are reinvested at a rate equal to the IRR. It is ridiculous to imagine
that the same firm has a ability to reinvest the cash flows at a rate equal to
IRR. (iv) If mutually exclusive projects are considered as investment options
which have considerably different cash outlays. A project with a larger fund
commitment but lower IRR contributes more in terms of absolute NPV and increases
the shareholders’ wealth. In such situation decisions based only on IRR criterion
may not be correct. Modified Internal Rate of Return (MIRR): As mentioned earlier,
there are several limitations attached with the concept of the conventional
Internal Rate of Return. The MIRR addresses some of these deficiencies e.g, it
eliminates multiple IRR rates; it addresses the reinvestment rate issue and
produces results which are consistent with the Net Present Value method. Under
this method , all cash flows , apart from the initial investment , are brought to
the terminal value using an appropriate discount rate(usually the Cost of
Capital). This results in a single stream of cash inflow in the terminal year. The
MIRR is obtained by assuming a single outflow in the zeroth year and the terminal
cash in flow as mentioned above. The discount rate which equates the present value
of the terminal cash in flow to the zeroth year outflow is called the MIRR.
Illustration 12 Using details given in illustration 11, calculate MIRR considering
a 8% Cost of Capital .

6.32
Investment Decisions

Solution Year Cash flow Rs 0 1,36,000 The net cash flows from the investment shall
be compounded to the terminal year at 8% as follows, Year 1 2 3 4 5 Cash flow @8%
reinvestment rate factor 30,000 1.3605 40,000 60,000 30,000 20,000 1.2597 1.1664
1.0800 1.0000 Rs. 40,815 50,388 69,984 32,400 20,000

MIRR of the investment based on a single cash in flow of Rs 2,13,587 and a zeroth
year cash out flow of Rs 1,36,000 is 9.4% (approximately) Comparison of Net
Present Value and Internal Rate of Return Methods Both the net present value and
the internal rate of return methods are discounted cash flow methods which mean
that they consider the time value of money. The time value of money takes into
account that cash received today is worth more than cash received at any future
time because cash received today can be invested at a specified interest rate. The
time value of money is the opportunity cost (forgone interest) from not having the
cash today. Additionally, both these techniques consider all cash flows over the
expected useful life of the investment. Because of these two characteristics,
discounted cash flow techniques are considered to be the most theoretically sound
methods for evaluating capital investments. There are circumstances under which
the net present value method and the internal rate of return methods will reach
different conclusions. Results may vary significantly when capital investment
projects differ in (1) amount of initial investments, (2) net cash flow patterns,
or (3) length of useful lives. In addition, these two methods can yield different
conclusions in situations with (4) varying costs of capital over the life of a
project and (5) multiple investments. To use capital budgeting techniques
properly, these situations must be understood. (1) The net present value method
will favour a project with a large initial investment because the project is more
likely to generate large net cash inflows. Because the internal rate of return
method uses percentages to evaluate the relative profitability of an investment,
the

6.33
Financial Management

amount of the initial investment has no effect on the outcome. Therefore, the
internal rate of return method is more appropriate for assessing investments
requiring significantly different initial investments. (2) Differences in the
timing and amount of net cash inflows affect a project’s internal rate of return.
This results from the fact that the internal rate of return method assumes that
all net cash inflows from a project earn the same rate of return as the project’s
internal rate of return. In contrast, the net present value method assumes that
all net cash inflows from an investment earn the desired rate of return used in
the calculation. The desired rate of return used by the net present value method
is usually the organization’s weighted-average cost of capital, a more
conservative and more realistic expectation in most cases. (3) Both methods favour
projects with long useful lives as long as a project earns positive net cash
inflow during the extended years. As long as the net cash inflow in a year is
positive, no matter how small, the net present value increases, and the projects
desirability improves. Likewise, the internal rate of return method considers each
additional useful year of a project another year that its cumulative net cash
inflow will earn a return equal to the project’s internal rate of return. A
problem arises when an organization is forgoing more beneficial opportunities to
continue a project. For example, there might be uses of space or talent where the
organization would earn a higher return than the return from the continuation of
the project. (4) As an organization’s financial condition or operating environment
changes, its cost of capital could also change. A proper capital budgeting
procedure should incorporate changes in the organization’s cost of capital or
desired rate of return in evaluating capital investments. The net present value
method can accommodate different rates of return over the years by using the
appropriate discount rates for the net cash inflow of different periods. The
internal rate of return method calculates a single rate that reflects the return
of the project under consideration and cannot easily handle situations with
varying desired rates of return. (5) The net present value method evaluates
investment projects in cash amounts while the internal rate of return method
evaluates investment projects in percentages or rates. The net present values from
multiple projects can be added to arrive at a single total net present value for
all investments while the percentages or rates of return on multiple projects
cannot be added to determine an overall rate of return. A combination of projects
requires a recalculation of the internal rate of return. Illustration 13 CXC
Company is preparing the capital budget for the next fiscal year and has
identified the following capital investment projects: Project A: Redesign and
modification of an existing product that is current scheduled to be terminated.
The enhanced model would be sold for six more years.

6.34
Investment Decisions

Project B: Expansion of a product that has been produced on an experimental basis


for the past year. The expected life of the product line is eight years. Project
C: Reorganization of the plant’s distribution centre, including the installation
of computerized equipment for tracking inventory. Project D: Addition of a new
product. In addition to new manufacturing equipment, a significant amount of
introductory advertising would be required. If this project is implemented,
Project A would not be feasible due to limited capacity. Project E: Automation of
the Packaging Department that would result in cost savings over the next six
years. Project F: Construction of a building wing to accommodate offices presently
located in an area that could be used for manufacturing. This change would not add
capacity for new lines but would alleviate crowded conditions that currently
exist, making it possible to improve the productivity of two existing product
lines that have been unable to meet market demand. The cost of capital for CXC
Company is 12%, and it is assumed that any funds not invested in capital projects
and any funds released at the end of a project can be invested at this rate. As a
benchmark for the accounting (book) rate of return, CXC has traditionally used
10%. Further information about the projects is shown below.
Project A Capital Investment Net Present Value @12% Internal Rate of Return
Payback Period Accounting Rate of Return 106,000 69,683 35% 2.2 years 18% Project
B 200,000 23,773 15% 4.5 years 9% Project C 140,000 (10,228) 9% 3.9 years 6%
Project D 160,000 74,374 27% 4.3 years 21% Project E 144,000 6,027 14% 2.9 years
12% Project F 130,000 69,513 26% 3.3 years 18%

If CXC Company has no budget restrictions for capital expenditures and wishes to
maximize stakeholder value, the company would choose, based on the given
information, to proceed with Projects A or D (mutually exclusive projects), B, E,
and F. All of these projects have a positive net present value and an internal
rate of return that is greater that the hurdle rate or cost of capital.
Consequently, any one of these projects will enhance stakeholder value. Project C
is omitted because it has a negative net present value and the internal rate of
return is below the 12% cost of capital. With regard to the mutually exclusive
projects, the selection of Project A or Project D is dependent on the valuation
technique used for selection. If net present value is the only technique used, CXC
Company would select Projects B, D, E, and F with a combined net present value of
173,687, the maximum total available. If either the payback method or the

6.35
Financial Management

internal rate of return is used for selection, Projects A, B, E, and F would be


chosen as Project A has a considerably shorter payback period than Project D, and
Project A also has a higher internal rate of return that Project D. The accounting
rate of return for these two projects is quite similar and does not provide much
additional information to inform the company’s decision. The deciding factor for
CXC Company between Projects A and D could very well be the payback period and the
size of the initial investment; with Project A, the company would be putting fewer
funds at risk for a shorter period of time. If CXC Company were to use the
accounting rate of return as the sole measurement criteria for selecting projects,
Project B would not be selected. It is clear from the other measures that Project
B will increase stakeholder value and should be implemented if CXC has no budget
restrictions; this clearly illustrates the necessity that multiple measures be
used when selecting capital investment projects. Rather than an unrestricted
budget, let us assume that the CXC capital budget is limited to 4,50,000. In cases
where there are budget limitations (referred to as capital rationing), the use of
the net present value technique is generally recommended as the highest total net
present value of the group of projects that fits within the budget limitation will
provide the greatest increase in stakeholder value. The combination of Projects A,
B, and F will yield the highest net present value to CXC Company for an investment
of 436,000. Self Examination Questions A. 1. Objective Type Questions A capital
budgeting technique which does not require the computation of cost of capital for
decision making purposes is, (a) Net Present Value method (b) Internal Rate of
Return method (c) Modified Internal Rate of Return method (d) Pay back 2. If two
alternative proposals are such that the acceptance of one shall exclude the
possibility of the acceptance of another then such decision making will lead to,
(a) Mutually exclusive decisions (b) Accept reject decisions (c) Contingent
decisions (d) None of the above

6.36
Investment Decisions

3.

In case a company considers a discounting factor higher than the cost of capital
for arriving at present values, the present values of cash inflows will be (a)
Less than those computed on the basis of cost of capital (b) More than those
computed on the basis of cost of capital (c) Equal to those computed on the basis
of the cost of capital (d) None of the above

4.

The pay back technique is specially useful during times (a) When the value of
money is turbulent (b) When there is no inflation (c) When the economy is growing
at a steady rate coupled with minimal inflation. (d) None of the above

5.

While evaluating capital investment proposals, time value of money is used in


which of the following techniques, (a) Payback method (b) Accounting rate of
return (c) Net present value (d) None of the above

6.

IRR would favour project proposals which have, (a) Heavy cash inflows in the early
stages of the project. (b) Evenly distributed cash inflows throughout the project.
(c) Heavy cash inflows at the later stages of the project (d) None of the above.

7.

The re investment assumption in the case of the IRR technique assumes that, (a)
Cash flows can be re invested at the projects IRR (b) Cash flows can be re
invested at the weighted cost of capital (c) Cash flows can be re invested at the
marginal cost of capital (d) None of the above

8.

Multiple IRRs are obtained when, (a) Cash flows in the early stages of the project
exceed cash flows during the later stages.

6.37
Financial Management

(b) Cash flows reverse their signs during the project (c) Cash flows are un even
(d) None of the above. 9. Depreciation is included as a cost in which of the
following techniques, (a) Accounting rate of return (b) Net present value (c)
Internal rate of return (d) None of the above 10. Management is considering a Rs
1,00,000 investment in a project with a 5 year life and no residual value . If the
total income from the project is expected to be Rs 60,000 and recognition is given
to the effect of straight line depreciation on the investment, the average rate of
return is : (a) 12% (b) 24% (c) 60% (d) 75% 11. Assume cash outflow equals Rs
1,20,000 followed by cash inflows of Rs 25,000 per year for 8 years and a cost of
capital of 11%. What is the Net present value? (a) (Rs 38,214) (b) Rs 9,653 (c) Rs
8,653 (d) Rs 38,214 12. What is the Internal rate of return for a project having
cash flows of Rs 40,000 per year for 10 years and a cost of Rs 2,26,009? (a) 8%
(b) 9% (c) 10% (c) 12%

6.38
Investment Decisions

13. While evaluating investments, the release of working capital at the end of the
projects life should be considered as, (a) Cash in flow (b) Cash out flow (c)
Having no effect upon the capital budgeting decision (d) None of the above. 14.
Capital rationing refers to a situation where, (a) Funds are restricted and the
management has to choose from amongst available alternative investments. (b) Funds
are unlimited and the management has to decide how to allocate them to suitable
projects. (c) Very few feasible investment proposals are available with the
management. (d) None of the above 15. Capital budgeting is done for (a) Evaluating
short term investment decisions. (b) Evaluating medium term investment decisions.
(c) Evaluating long term investment decisions. (d) None of the above Answers to
Objective Type Questions 1. (d); 2. (a); 3. (a) ; 4. (a); 5. (c) ; 6. (a); 7. 10.
(b); 11. (c); 12. (d); 13. (a); 14. (a); 15. (c). B. 1. Short Answer Type
Questions Define the following terms: (a) Capital Budgeting (b) Regular payback
period and discounted payback period (c) Independent projects and mutually
exclusive projects. (d) Internal rate of return method and modified rate of return
method. (e) Net Present Value method. (f) Capital rationing. (a); 8. (b); 9. (a);

6.39
Financial Management

2. 3. 4. 5. 6. 7. 8. C. 1.

Why is discounted cash flow a superior method for capital budgeting? “The higher
the cut off rate, the more will be the company willing to pay for cost saving
equipment”. Discuss. Does the IRR model make significantly different decisions
than the NPV method? Discuss. Two projects have an identical Net Present Value of
Rs 50,000. Are both projects equal in desirability. What are the basic objections
to the use of Average Rate of Return concept for evaluating projects? Discuss the
principal limitations of the cash payback period for evaluating capital investment
proposals. Your boss has suggested that a one year payback means 100 % average
returns. Do you agree? Long Answer Type Questions Explain why, if two mutually
exclusive projects are being compared, the short term project might have the
higher ranking under NPV criteria if the cost of capital is high, but the long
term project will be deemed better if the cost of capital is low. Would changes in
the cost of capital ever cause a change in the IRR ranking of two such projects?
In what sense is a reinvestment rate assumption embodied in the NPV , IRR and MIRR
methods? What is the assumed reinvestment rate of each method? Discuss in detail
the ‘Capital Budgeting Process’ What are the various types of Capital Investment
decisions known to you? Discuss the basic principles for measuring project cash
flows. Practical Problems (a) You are required to calculate the total present
value of inflow at rate of discount of 12% of following data. Year end Cash
inflows Rs. 1 2 3 2,30,000 2,28,000 2,78,000

2. 3. 4. 5. D. 1.

6.40
Investment Decisions

4 5 6

2,83,000 2,73,000 80,000 (Scrap Value)

(b) Considering the data given in the above. Calculate the total present value of
inflows and outflows if the rate of discount is 10% assuming that Rs. 10,00,000 of
outflows would be spent as follows: Beginning of year 1 Beginning of year 2
Beginning of year 3 Beginning of year 4 2. Consider the following example of cash
flows from two projects. No. of years 1 2 3 4 5 6 7 8 Total Project A Nil Nil
5,000 20,000 50,000 1,50,000 50,000 40,000 3,15,000 Project B 40,000 50,000
1,20,000 10,000 10,000 Nil Nil Nil 2,30,000 Rs. 2,50,000 Rs. 2,50,000 Rs. 2,50,000
Rs. 2,50,000

Both projects cost Rs. 1,50,000 each . You are required to compute the payback
period for both projects. Which project will you prefer? 3. A company wants to
replace its old machine with a new automatic machine. Two models A and B are
available at the same cost of Rs. 5 lakhs each. Salvage value of the old machine
is Rs. 1 lakh. The utilities of the existing machine can be used if the company
purchases model A. Additional cost of utilities to be purchased in that case are
Rs. 1 lakh. If the company purchases model B then all the existing utilities will
have to be replaced with new utilities costing Rs. 2 lakhs. The salvage value of
the old utilities will

6.41
Financial Management

be Rs. 0.20 lakhs. The earnings after taxation are expected to be as follows :
(Cash inflows) Year/Model 1. 2. 3. 4. 5. Salvage value at the end of Year 5 A Rs.
1,00,000 1,50,000 1,80,000 2,00,000 1,70,000 50,000 B Rs. 2,00,000 2,10,000
1,80,000 1,70,000 40,000 60,000 P.V. Factor @ 15% 0.87 0.76 0.66 0.57 0.50

The targeted return on capital is 15% you are required to : (i) (ii) 4. Compute,
for the two machines separately, net present value, discounted payback period and
desirability factor and Advice which of the machines is to be selected ?

Suppose the management of a concern is considering two projects both involving Rs.
10,00,000 each and having profits after tax and depreciation as follows: Year 1 2
3 4 5 Total A Rs. 50,000 75,000 1,25,000 1,30,000 80,000 4,60,000 B Rs. Nil Nil
Nil 2,30,000 2,30,000 4,60,000

Suppose further that both projects can be sold for Rs. 80,000 each after 5 years.
You are required to compute the annual rate of return of both the projects. Will
you consider both projects to be equal?

6.42
Investment Decisions

5.

A product is currently manufactured on a plant which is not fully depreciated for


tax purposes and has a book value of Rs., 60,000 (it was bought for Rs. 1,20,000
six years ago). The cost of the product is as under: Unit cost Rs. 24.00 8.00
16.00 16.00 Rs. 64.00

Direct costs Indirect labour Other variable overheads Fixed overheads

10,000 units are normally produced. It is expected that the old machine can be
used, indefinitely into the future, after suitable repairs, estimated to cost Rs.
40,000 annually, are carried out. A manufacturer of machinery is offering a new
machine with the latest technology at Rs.3,00,000 after trading off the old plant
for Rs. 30,000. The projected cost of the product will then be : Per unit Rs.
Direct costs 14.00 Indirect labour 12.00 Other variable overheads 12.00 20.00
Fixed overheads 58.00 The fixed overheads are allocations from other departments
pls the depreciation of plant and machinery. The old machine can be sold for Rs.
40,000 in the open market. The new machine is expected to last for 10 years at the
end of which, its salvage value will be Rs. 20,000. Rate of corporate taxation is
50%. For tax purposes, the cost of the new machine and that of the old one may be
depreciated in 10 years. The minimum rate of return expected is 10%. It is also
anticipated that in future the demand for the products will stay to 10,000 units.
Advise whether the new machine can be purchased ignore capital gains taxes.
Present value of Re. 1 at 10% for years 1-10 are : .909, .826, .751, .683, .621,
.564, .513, .467, .424 and .383 respectively.

6.43
Financial Management

6.

Modern Enterprises Ltd. is considering the purchase of a new computer system for
its Research and Development Division, which would cost Rs. 35 lakhs. The
operation and maintenance costs (excluding depreciation) are expected to be Rs. 7
lakhs per annum. It is estimated that the useful life of the system would be 6
years, at the end of which the disposal value is expected to be Rs. 1 lakh. The
tangible benefits expected from the system in the form of reduction in design and
drafts-menship costs would be Rs. 12 lakhs per annum. Besides, the disposal of
used drawing, office equipment and furniture, initially, is anticipated to net Rs.
9 lakhs. Capital expenditure in research and development would attract 100% write-
off for tax purpose. The gains arising from disposal of used assets may be
considered tax-free. The company’s effective tax rate is 50%. The average cost of
capital to the company is 12%. The present value factors at 12% discount rate are
: Year 1 2 3 4 5 6 PVF 0.892 0.797 0.711 0.635 0.567 0.506

After appropriate analysis of cash flows, please advise the company of the
financial viability of the proposal. 7. A sole trader installs plant and machinery
in rented premises for the production of luxury article, the demand for which is
expected to last only 5 years. The total capital put in by the sole trader is as
under: Plant and Machinery Working Capital Rs. 2,70,500 Rs. 40,000 Rs. 3,10,500

The working capital will be fully realised at the end of the 5th year. The scrap
value of the plant expected to be realised at the end of the 5th year is only Rs.
5,500. The trader’s earnings are expected to be as under :

6.44
Investment Decisions

Years

Cash profits after depreciation and tax Rs.

Tax payable Rs. 20,000 30,000 40,000 26,000 5,000

1 2 3 4 5

90,000 1,30,000 1,70,000 1,16,000 19,500

Present value factors of various rates of interest are given below : Years 11%
0.9009 2 3 4 5 0.8116 0.7312 0.6587 0.5935 12% 0.8929 0.7972 0.7118 0.6355 0.5674
13% 0.8850 0.7831 0.6931 0.6133 0.5428 14% 0.8770 0.7695 0.6750 0.5921 0.5194 15%
0.8696 0.7561 0.6575 0.5718 0.4972

You are required to compute the present value of cash flows discounted at the
various rates of interests given above and state the return from the project.
(3,34,172; 3,25,996; 3,18,128; 3,10,543; 3,03,251 : Yield 14%) 8. The Alpha Co.
Ltd, is considering the purchase of a new machine. Two alternative machines (A &
B) have been suggested, each costing Rs. 4,00,000. Earnings after taxation but
before depreciation are expected to be as follows :

6.45
Financial Management

Year Machine A Rs. 1 2 3 4 5 Total 40,000 1,20,000 1,60,000 2,40,000 1,60,000


7,20,000

Cash Flows Machine B Rs. 1,20,000 1,60,000 2,00,000 1,20,000 80,000 6,80,000

The company has a target rate return on capital @ 10 percent and on this basis,
you are required : (a) Compare profitability of the machines and state which
alternative you consider financially preferable, (b) Compute the pay back period
for each project and, (c) Compute annual rate of return for each project. (Present
value of machine B is higher than that of machine A; Payback period machine A – 3
years 4 months, machine B 2 years 7.2 months ; Annual return machine A – 16%,
machine B – 14%) 9. Company X is forced to choose between two machines A and B.
The two machines are designed differently, but have identical capacity and do
exactly the same job. Machine A costs Rs. 1,50,000 and will last for 3 years. It
costs Rs. 40,000 per year to run. Machine B is an ‘economy’ model costing only Rs.
1,00,000, but will last only for 2 years, and costs Rs. 60,000 per year to run.
These are real cash flows. The costs are forecasted in rupees of constant
purchasing power. Ignore tax. Opportunity cost of capital is 10 percent. Which
machine company X should buy ?

10. S Engineering Company is considering replacing or repairing a particular


machine, which has just broken down. Last year this machine costed Rs. 20,000 to
run and maintain. These costs have been increasing in real terms in recent years
with the age of the machine. A further useful life of 5 years is expected, if
immediate repairs of Rs. 19,000 are carried out. If the machine is not repaired it
can be sold immediately to realise about Rs. 5,000 (Ignore loss/gain on such
disposal)

6.46
Investment Decisions

Alternatively, the company can buy a new machine for Rs. 49,000 with an expected
life of 10 years with no salvage value after providing depreciation on straight
line basis. In this case, running and maintenance costs will reduce to Rs. 14,000
each year and are not expected to increase much in real terms for a few years at
least. S Engineering Company regard a normal return of 10% p.a. after tax as a
minimum requirement on any new investment. Considering capital budgeting
techniques, which alternative will you choose? Take corporate tax rate of 50% and
assume that depreciation on straight line basis will be accepted for tax purposes
also. Given cumulative present value of Re. 1 p.a. at 10% for 5 years Rs. 3.791
and for 10 years Rs. 6.145.

6.47
CHAPTER

MANAGEMENT OF WORKING CAPITAL


UNIT – I : MEANING, CONCEPT AND POLICIES OF WORKING CAPITAL Learning Objectives
After studying this chapter, you will be able to understand ♦ The meaning and the
significance of working capital management; ♦ The concept of operating cycle and
the estimation of working capital needs; ♦ The need for investing in current
assets; ♦ The need for managing current assets and current liabilities; and ♦
Financing of working capital. 1.1 INTRODUCTION

Decisions relating to working capital and short term financing are referred to as
Working Capital Management. These involve managing the relationship between a
firm’s short-term assets and its short-term liabilities. The goal of working
capital management is to ensure that the firm is able to continue its operations
and that it has sufficient cash flow to satisfy both maturing short-term debt and
upcoming operational expenses. 1.2 MEANING AND CONCEPT OF WORKING CAPITAL

There are two concepts of working capital - gross and net. Gross working capital
refers to the firm’s investment in current assets. Current assets are those assets
which can be converted into cash within an accounting year. Net working capital
refers to the difference between current assets and current liabilities. Current
liabilities are those claims of outsiders which are expected to mature for payment
within an accounting year. Current Assets include: Stocks of raw materials, Work-
in-progress, Finished goods, Trade debtors, Prepayments, Cash balances etc.
Current Liabilities include: Trade creditors, Accruals, Taxation payable, Bills
Payables, Outstanding expenses, Dividends payable, short term loans.
Financial Management

Working capital is also known as operating capital. A most important value, it


represents the amount of day-to-day operating liquidity available to a business. A
company can be endowed with assets and profitability, but short of liquidity if
these assets cannot readily be converted into cash. A positive working capital
means that the company is able to payoff its short-term liabilities. A negative
working capital means that the company currently is unable to meet its short-term
liabilities. From the point of view of time, the term working capital can be
divided into two categories viz., Permanent and temporary. Permanent working
capital refers to the hard core working capital. It is that minimum level of
investment in the current assets that is carried by the business at all times to
carry out minimum level of its activities. Temporary working capital refers to
that part of total working capital, which is required by a business over and above
permanent working capital. It is also called variable working capital. Since the
volume of temporary working capital keeps on fluctuating from time to time
according to the business activities it may be financed from short-term sources.
The following diagrams shows Permanent and Temporary or Fluctuating or variable
working capital

7.2
Management of Working Capital

Both kinds of working capital i.e. permanent and fluctuating (temporary) are
necessary to facilitate production and sales through the operating cycle. 1.2.1
Importance of Adequate Working Capital: The importance of adequate working capital
in commercial undertakings can be judged from the fact that a concern needs funds
for its day-to-day running. Adequacy or inadequacy of these funds would determine
the efficiency with which the daily business may be carried on. Management of
working capital is an essential task of the finance manager. He has to ensure that
the amount of working capital available with his concern is neither too large nor
too small for its requirements. A large amount of working capital would mean that
the company has idle funds. Since funds have a cost, the company has to pay huge
amount as interest on such funds. The various studies conducted by the Bureau of
Public Enterprises have shown that one of the reason for the poor performance of
public sector undertakings in our country has been the large amount of funds
locked up in working capital This results in over capitalization. Over
capitalization implies that a company has too large funds for its requirements,
resulting in a low rate of return a situation which implies a less than optimal
use of resources. A firm has, therefore, to be very careful in estimating its
working capital requirements. If the firm has inadequate working capital, it is
said to be under-capitalised. Such a firm runs the risk of insolvency. This is
because, paucity of working capital may lead to a situation where the firm may not
be able to meet its liabilities. It is interesting to note that many firms which
are otherwise prosperous (having good demand for their products and enjoying
profitable marketing conditions) may fail because of lack of liquid resources. If
a firm has insufficient working capital and tries to increase sales, it can easily
over-stretch the financial resources of the business. This is called overtrading.
Early warning signs of over trading include: ♦ Pressure on existing cash. ♦
Exceptional cash generating activities e.g., offering high discounts for early
cash payment. ♦ Bank overdraft exceeds authorized limit. ♦ Seeking greater
overdrafts or lines of credit. ♦ Part-paying suppliers or other creditors. ♦
Paying bills in cash to secure additional supplies. ♦ Management pre-occupation
with surviving rather than managing. ♦ Frequent short-term emergency requests to
the bank (to help pay wages, pending receipt of a cheque).

7.3
Financial Management

Every business needs adequate liquid resources in order to maintain day-to-day


cash flow. It needs enough cash to pay wages and salaries as they fall due and to
pay creditors if it is to keep its workforce engaged and ensure its supplies.
Maintaining adequate working capital is not just important in the short-term.
Sufficient liquidity must be maintained in order to ensure the survival of the
business in the long-term as well. Even a profitable business may fail if it does
not have adequate cash flow to meet its liabilities as they fall due. Therefore,
when business make investment decisions they must not only consider the financial
outlay involved with acquiring the new machine or the new building, etc., but must
also take account of the additional current assets that are usually required with
any expansion of activity. Increased production leads to hold additional stocks of
raw materials and work in progress. Increased sales usually means that the level
of debtors will increase. A general increase in the firm’s scale of operations
tends to imply a need for greater levels of working capital. A question then
arises what is an optimum amount of working capital for a firm? We can say that a
firm should neither have too high an amount of working capital nor should the same
be too low. It is the job of the finance manager to estimate the requirements of
working capital carefully and determine the optimum level of investment in working
capital. 1.2.2 Optimum Working Capital: If a company’s current assets do not
exceed its current liabilities, then it may run into trouble with creditors that
want their money quickly. The working capital ratio, which measures this ability
to pay back can be calculated as current assets divided by current liabilities.
Current ratio (current assets/current liabilities) (along with acid test ratio to
supplement it) has traditionally been considered the best indicator of the working
capital situation. It is understood that a current ratio of 2 (two) for a
manufacturing firm implies that the firm has an optimum amount of working capital.
This is supplemented by Acid Test Ratio (Quick assets/Current liabilities) which
should be at least 1 (one). Thus it is considered that there is a comfortable
liquidity position if liquid current assets are equal to current liabilities.
Bankers, financial institutions, financial analysts, investors and other people
interested in financial statements have, for years, considered the current ratio
at, ‘two’ and the acid test ratio at, ‘one’ as indicators of a good working
capital situation. As a thumb rule, this may be quite adequate. However, it should
be remembered that optimum working capital can be determined only with reference
to the particular circumstances of a specific situation. Thus, in a company where
the inventories are easily saleable and the sundry debtors are as good as liquid
cash, the current ratio may be lower than 2 and yet firm may be sound. An optimum
working capital ratio is dependent upon the business situation as such and the
nature and composition of various current assets. A company having short
conversion cycle (from cash to cash) my have a lower current ratio.

7.4
Management of Working Capital

In nutshell, a firm needs to maintain a sound working capital position. It should


have adequate working capital to run its business operations. Both excessive as
well as inadequate working capital positions are dangerous. Excessive working
capital means holding costs and idle funds which earn no profits for the firm.
Paucity of working capital not only impairs the firm’s profitability but also
results in production interruptions, inefficiencies and sales disruptions. The
management should therefore maintain the right amount of working capital
continuously. 1.3 MANAGEMENT OF WORKING CAPITAL Working capital management is the
functional area of finance that covers all the current accounts of its firm. It is
concerned with management of the level of individual current assets and the
current liabilities or in other words the management of total working capital.
Managing Working Capital is a matter of balance. A firm must have sufficient cash
on hand to meet its immediate needs while ensuring that idle cash is invested to
the organizations best possible advantage. To avoid the difficulties, it is
necessary to have clear and accurate reports on each of the components of working
capital and an awareness of the potential impact of likely influences. Sound
financial and statistical techniques, supported by judgement should be used to
predict the quantum of working capital required at different times. Adequate
provisions of working capital mitigates risk. Working capital management entails
short-term decisions generally, relating to its next one year period which are
“reversible”. Management will use a combination of policies and techniques for the
management of working capital. These require managing the current assets –
generally cash and cash equivalents, inventories and debtors. There are also a
variety of short term financing options which are considered. The various steps in
the management of working capital involve: ♦ Cash management – Identify the cash
balance which allows for the business to meet day to day expenses, but reduces
cash holding costs. ♦ Inventory management – Identify the level of inventory which
allows for uninterrupted production but reduces the investment in raw materials
and hence increases cash flow; The techniques like Just In Time (JIT) and Economic
order quantity (EOQ) are used for this. ♦ Debtors management – Identify the
appropriate credit policy, i.e., credit terms which will attract customers, such
that any impact on cash flows and the cash conversion cycle will be offset by
increased revenue and hence Return on Capital (or vice versa). The tools like
Discounts and allowances are used for this. ♦ Short term financing – Inventory is
ideally financed by credit granted by the supplier; dependent on the cash
conversion cycle, it may however, be necessary to utilize a bank loan (or
overdraft), or to “convert debtors to cash” through “factoring” in order to
finance
7.5
Financial Management

working capital requirements. There are, however, certain constraints in the


management of working capital such as: (i) (ii) Non-realisation of the importance
of working capital. Continuous inflation in the economy.

(iii) The existence of seller’s market or monopoly conditions; and (iv) High
profitability. 1.3.1 Determinants of Working Capital: The following factors will
generally influence the working capital requirements of the firm: (i) (ii) Nature
of Business. Market and demand conditions.

(iii) Technology and manufacturing Policies. (iv) Credit Policy of the firm. (v)
Availability of credit from suppliers. (vi) Operating efficiency. (vii) Price
Level Changes. 1.4 ISSUES IN THE WORKING CAPITAL MANAGEMENT Working capital
management entails the control and monitoring of all components of working capital
i.e. cash, marketable securities, debtors (receivables) and stocks (inventories)
and creditors (payables). The finance manager has to determine the levels and
composition of current assets. He has to ensure a right mix of different current
assets and that current liabilities are paid in time. There are many aspects of
working capital management which makes it important function of financial
management. ♦ Time: Working capital management requires much of the finance
manager’s time. ♦ Investment: Working capital represents a large portion of the
total investment in assets. ♦ Credibility: Working capital management has great
significance for all firms but it is very critical for small firms. ♦ Growth: The
need for working capital is directly related to the firm’s growth. It is advisable
that the finance manager should take precautionary measures for effective and
efficient management of working capital. He has to pay particular attention to the
levels of

7.6
Management of Working Capital

current assets and their financing. To decide the levels and financing of current
assets, the risk return trade off must be taken into account. 1.4.1 Current Assets
to Fixed Assets Ratio: The finance manager is required to determine the optimum
level of current assets so that the shareholders value is maximized. A firm needs
fixed and current assets to support a particular level of output. However, to
support the same level of output, the firm can have different levels of current
assets. As the firm’s output and sales increases, the need for current assets also
increases. Generally, current assets do not increase in direct proportion to
output, current assets may increase at a decreasing rate with output. This
relationship is based upon the notion that it takes a greater proportional
investment in current assets when only a few units of output are produced than it
does later on when the firm can use its current assets more efficiently. The level
of the current assets can be measured by creating a relationship between current
assets and fixed assets. Dividing current assets by fixed assets gives current
assets/fixed assets ratio. Assuming a constant level of fixed assets, a higher
current assets/fixed assets ratio indicates a conservative current assets policy
and a lower current assets/fixed assets ratio means an aggressive current assets
policy assuming all factors to be constant. A conservative policy implies greater
liquidity and lower risk whereas an aggressive policy indicates higher risk and
poor liquidity. Moderate current assets policy will fall in the middle of
conservative and aggressive policies. The current assets policy of most of the
firms may fall between these two extreme policies. The following diagram shows
alternative current assets policies:

7.7
Financial Management

1.4.2 Liquidity versus Profitability: Risk return trade off − A firm may follow a
conservative, aggressive or moderate policy as discussed above. However, these
policies involve risk, return trade off. A conservative policy means lower return
and risk. While an aggressive policy produces higher return and risk. The two
important aims of the working capital management are profitability and solvency. A
liquid firm has less risk of insolvency that is, it will hardly experience a cash
shortage or a stock out situation. However, there is a cost associated with
maintaining a sound liquidity position. However, to have higher profitability the
firm may have to sacrifice solvency and maintain a relatively low level of current
assets. This will improve firm’s profitability as fewer funds will be tied up in
idle current assets, but its solvency would be threatened and exposed to greater
risk of cash shortage and stock outs. The following illustration explains the
risk-return trade off of various working capital management policies, viz.,
conservative, aggressive and moderate etc. Illustration 1 A firm has the following
data for the year ending 31st March, 2006: Rs. Sales (1,00,000 @ Rs.20/-) Earning
before Interest and Taxes 20,00,000 2,00,000

Fixed Assets 5,00,000 The three possible current assets holdings of the firm are
Rs.5,00,000/-, Rs.4,00,000/- and Rs. 3,00,000. It is assumed that fixed assets
level is constant and profits do not vary with current assets levels. The effect
of the three alternative current assets policies is as follows: Effect of
alternative Working Capital Policies (Amount in Rs.) Working Capital Policy Sales
Earnings before Interest and Taxes (EBIT) Current Assets Fixed Assets Total Assets
Return on Total Assets (EBIT/Total Assets) Current Assets/Fixed Assets
7.8

Conservative 20,00,000 2,00,000 5,00,000 5,00,000 10,00,000 20% 1.00

Moderate 20,00,000 2,00,000 4,00,000 5,00,000 9,00,000 22.22% 0.80

Aggressive 20,00,000 2,00,000 3,00,000 5,00,000 8,00,000 25% 0.60


Management of Working Capital

The aforesaid calculations shows that the conservative policy provides greater
liquidity (solvency) to the firm, but lower return on total assets. On the other
hand, the aggressive policy gives higher return, but low liquidity and thus is
very risky. The moderate policy generates return higher than Conservative policy
but lower than aggressive policy. This is less risky than Aggressive policy but
more risky than conservative policy. In determining the optimum level of current
assets, the firm should balance the profitability – Solvency tangle by minimizing
total costs. Cost of liquidity and cost of illiquidity. 1.5 ESTIMATING WORKING
CAPITAL NEEDS Operating cycle is one of the most reliable method of Computation of
Working Capital. However, other methods like ratio of sales and ratio of fixed
investment may also be used to determine the Working Capital requirements. These
methods are briefly explained as follows: (i) Current assets holding period: To
estimate working capital needs based on the average holding period of current
assets and relating them to costs based on the company’s experience in the
previous year. This method is essentially based on the Operating Cycle Concept.

(ii) Ratio of sales: To estimate working capital needs as a ratio of sales on the
assumption that current assets change with changes in sales. (iii) Ratio of fixed
investments: To estimate Working Capital requirements as a percentage of fixed
investments. A number of factors will, however, be impacting the choice of method
of estimating Working Capital. Factors such as seasonal fluctuations, accurate
sales forecast, investment cost and variability in sales price would generally be
considered. The production cycle and credit and collection policies of the firm
will have an impact on Working Capital requirements. Therefore, they should be
given due weightage in projecting Working Capital requirements. 1.6 OPERATING OR
WORKING CAPITAL CYCLE A useful tool for managing working capital is the operating
cycle. The operating cycle analyzes the accounts receivable, inventory and
accounts payable cycles in terms of number of days. In other words, accounts
receivable are analyzed by the average number of days it takes to collect an
account. Inventory is analyzed by the average number of days it takes to turn over
the sale of a product (from the point it comes in the store to the point it is
converted to cash or an account receivable). Accounts payable are analyzed by the
average number of days it takes to pay a supplier invoice. Most businesses cannot
finance the operating cycle (accounts receivable days + inventory days) with
accounts payable financing alone. Consequently, working capital financing is
needed. This shortfall is typically covered by the net profits generated
internally or by externally borrowed funds or by a combination of the two.
7.9
Financial Management

Most businesses need short-term working capital at some point in their operations.
For instance, retailers must find working capital to fund seasonal inventory
build-up. But even a business that is not seasonal occasionally experiences peak
months when orders are unusually high. This creates a need for working capital to
fund the resulting inventory and accounts receivable build-up. Some small
businesses have enough cash reserves to fund seasonal working capital needs.
However, this is very rare for a new business. If your new venture experiences a
need for short-term working capital during its first few years of operation, you
will have several potential sources of funding. The important thing is to plan
ahead. If you get caught off guard, you might miss out on the one big order. Cash
flows in a cycle into, around and out of a business. It is the business’s life
blood and every manager’s primary task is to help keep it flowing and to use the
cashflow to generate profits. If a business is operating profitably, then it
should, in theory, generate cash surpluses. If it doesn’t generate surplus, the
business will eventually run out of cash. The faster a business expands, the more
cash it will need for working capital and investment. The cheapest and best
sources of cash exist as working capital right within business. Good management of
working capital will generate cash which will help improve profits and reduce
risks. Bear in mind that the cost of providing credit to customers and holding
stocks can represent a substantial proportion of a firm’s total profits. There are
two elements in the business cycle that absorb cash – Inventory (stocks and
workin-progress) and Receivables (debtors owing you money). The main sources of
cash are Payables (your creditors) and Equity and Loans. Working Capital Cycle
CASH DEBTORS STOCK WIP RAW MATERIAL LABOUR OVERHEAD

Each component of working capital (namely inventory, receivables and payables) has
two dimensions ……TIME ………and MONEY, when it comes to managing working capital then
time is money. If you can get money to move faster around the cycle (e.g. collect
monies due from debtors more quickly) or reduce the amount of money tied up (e.g.
reduce inventory

7.10
Management of Working Capital

levels relative to sales), the business will generate more cash or it will need to
borrow less money to fund working capital. As a consequence, you could reduce the
cost of bank interest or you will have additional free money available to support
additional sales growth or investment. Similarly, if you can negotiate improved
terms with suppliers e.g. get longer credit or an increased credit limit, you are
effectively creating free finance to help fund future sales. If you……………… Collect
receivables (debtors) faster Collect receivables (debtors) slower Get better
credit (in terms of duration or amount) from suppliers. Shift inventory (stocks)
faster Move inventory (stocks) slower. Then …………………. You release cash from the
cycle Your receivables soak up cash. You increase your cash resources. You free up
cash. You consume more cash.

Working capital cycle indicates the length of time between a company’s paying for
materials, entering into stock and receiving the cash from sales of finished
goods. It can be determined by adding the number of days required for each stage
in the cycle. For example, a company holds raw materials on an average for 60
days, it gets credit from the supplier for 15 days, production process needs 15
days, finished goods are held for 30 days and 30 days credit is extended to
debtors. The total of all these, 120 days, i.e., 60 – 15 + 15 + 30 + 30 days is
the total working capital cycle. The determination of working capital cycle helps
in the forecast, control and management of working capital. It indicates the total
time lag and the relative significance of its constituent parts. The duration of
working capital cycle may vary depending on the nature of the business. In the
form of an equation, the operating cycle process can be expressed as follows:
Operating Cycle Where, R= W= F= D= C= Raw material storage period Work-in-progress
holding period Finished goods storage period Debtors collection period. Credit
period availed.
7.11

R+W+F+D–C
Financial Management

The various components of operating cycle may be calculated as shown below: (1)
(2)
(3) (4) (5)
Raw material storage period = Average stock of raw material Average cost of raw
material consumption per day

Work - in - progress holding period =


Finished goods storage period = Debtors collection period = Credit period availed
=

Average work - in - progress inventory Average cost of production per day

Average stock of finished goods Average cost of goods sold per day

Average book debts Average Credit Sales per day

Average trade creditors Average credit purchases per day

1.6.1 Working Capital Based on Operating Cycle: One of the method for forecasting
working capital requirement is based on the concept of operating cycle. The
calculation of operating cycle and the formula for estimating working capital on
its basis has been demonstrated with the help of following illustration:
Illustration 2 From the following information of XYZ Ltd., you are required to
calculate : (a) Net operating cycle period. (b) Number of operating cycles in a
year. (i) (ii) (iii) (iv) (v) (vi) (vii) (viii) (ix) Raw material inventory
consumed during the year Average stock of raw material Work-in-progress inventory
Average work-in-progress inventory Finished goods inventory Average finished goods
stock held Average collection period from debtors Average credit period availed
No. of days in a year Rs. 6,00,000 50,000 5,00,000 30,000 8,00,000 40,000 45 days
30 days 360 days

7.12
Management of Working Capital

The various components of operating cycle may be calculated as shown below: (1)
(2)

Raw material storage period =

Average stock of raw material Average cost of raw material consumption per day

Work - in - progress holding period =

Average work - in - progress inventory Average cost of production per day

(3) (4) (5)

Finished goods storage period = Debtors collection period = Credit period availed
=

Average stock of finished goods Average cost of goods sold per day

Average book debts Average Credit Sales per day

Average trade creditors Average credit purchases per day

1.6.1 Working Capital Based on Operating Cycle: One of the method for forecasting
working capital requirement is based on the concept of operating cycle. The
calculation of operating cycle and the formula for estimating working capital on
its basis has been demonstrated with the help of following illustration:
Illustration 2

From the following information of XYZ Ltd., you are required to calculate : (a)
Net operating cycle period. (b) Number of operating cycles in a year. (i) (ii)
(iii) (iv) (v) (vi) (vii) (viii) (ix) Raw material inventory consumed during the
year Average stock of raw material Work-in-progress inventory Average work-in-
progress inventory Finished goods inventory Average finished goods stock held
Average collection period from debtors Average credit period availed No. of days
in a year
Rs. 6,00,000 50,000 5,00,000 30,000 8,00,000 40,000 45 days 30 days 360 days

7.13
Financial Management

Solution Calculation of Net Operating Cycle period of XYZ Ltd. Raw material
storage period: (a) Days 30

⎛ ⎞ Average stock of raw material ⎜ ⎜ Average cost of raw material consumption per
day ⎟ ⎟ ⎝ ⎠
(Rs. 50,000 / 1667*) *(Rs. 6,00,000 / 360 days) W.I.P. holding period : (b) 22

⎛ Average work − in − progress inventory ⎞ ⎜ ⎜ Average cost of production per day


⎟ ⎟ ⎝ ⎠
Rs. 30,000 / 1,388)** **(Rs. 5,00,000 / 360 days) Finished goods storage period :
(c) 18

⎛ Average stock of finished goods ⎜ ⎜ Average cost of goods sold per day ⎝
(Rs.40,000 / 2,222)***
***(Rs. 8,00,000 / 360 days) Debtors collection period: (d) Total operating cycle
period: [(a) + (b) + (c) + (d)] Less: Average credit period availed

⎞ ⎟ ⎟ ⎠

45 115 30 85 4.2 (360 days / 85 days)

(i) (ii)

Net operating cycle period Number of operating cycles in a year

The net operating cycle represents the net time gap between investment of cash and
its recovery of sales revenue. If depreciation is excluded from expenses in the
computation of operating cycle, the net operating cycle also represents the cash
conversion cycle. It is the

7.14
Management of Working Capital

net time interval between cash collections from sales of product and cash payments
for the resources acquired by the firm. The Finance Manager is required to manage
the operating cycle effectively and efficiently. The length of operating cycle is
the indicator of performance of management. The net operating cycle represents the
time interval for which the firm has to negotiate for Working Capital from its
Bankers. It enables to determine accurately the amount of working capital needed
for the continuous operation of business activities. The operating cycle calls for
proper monitoring of external environment of the business, changes in government
policies like taxation, import policies, credit policy of Reserve Bank of India,
price trend, technological advancement etc. They have since their own impact on
the length of operating cycle.
1.6.2 Estimate of amount of Working Capital based on Current Assets and Current
Liabilities

The estimate of working capital can be projected if the amount of current assets
and current liabilities can be estimated as follows: The various constituents of
current assets and current liabilities have a direct bearing on the computation of
working capital and the operating cycle. The holding period of various
constituents of operating cycle may either contract or expand the net operating
cycle period. Shorter the operating cycle period, lower will be the requirement of
working capital and viceversa.
Estimation of Current Assets

The estimates of various components of working capital may be made as follows:


(i) Raw materials inventory: The funds to be invested in raw materials inventory
may be estimated on the basis of production budget, the estimated cost per unit
and average holding period of raw material inventory by using the following
formula:

⎧ Estimated production × Estimated cost of raw material ⎫ ⎪ (in units) ⎪ Average


raw material holding period per unit ⎪ ⎪ ⎨ ⎬× (in months / in days) 12 months /
360 days ⎪ ⎪ ⎪ ⎪ ⎩ ⎭
Note: 360 days in a year are generally assumed to facilitate calculation. (ii)
Work-in-progress inventory: The funds to be invested in work-in-progress can be
estimated by the following formula:

7.15
Financial Management

⎧ Estimated production × Estimated work − in − process ⎪ (in units) cost per unit
⎪ ⎨ 12 months / 360 days ⎪ ⎪ ⎩

⎫ ⎪ Average holding period of ⎪ ⎬× W.I.P. (months / days) ) ⎪ ⎪ ⎭

(iii) Finished Goods: The funds to be invested in finished goods inventory can be
estimated with the help of following formula:

⎧ Estimated production × Cost of production (Per unit ⎫ ⎪ (in units) excluding


depreciation ⎪ Average holding period of finished ⎪ ⎪ ⎨ ⎬× 12 months / 360 days ⎪
⎪ goods inventory (months / days) ⎪ ⎪ ⎩ ⎭
(iv) Debtors: Funds to be invested in trade debtors may be estimated with the help
of following formula:

⎧ Estimated credit sales × Cost of sales (Per unit ⎫ ⎪ ( in units) excluding


depreciation ⎪ Average debtors collection ⎪ ⎪ ⎨ ⎬× period (months/days) 12
months/360 days ⎪ ⎪ ⎪ ⎪ ⎩ ⎭ (v) Minimum desired Cash and Bank balances to be
maintained by the firm has to be added in the current assets for the computation
of working capital.
Estimation of Current Liabilities

Current liabilities generally affect computation of working capital. Hence, the


amount of working capital is lowered to the extent of current liabilities (other
than bank credit) arising in the normal course of business. The important current
liabilities like trade creditors, wages and overheads can be estimated as follows:
(i)
Trade creditors:

× Raw material requirements ⎫ ⎧ Estimated yearly ⎪ production (in units) ⎪ Credit


period granted by per unit ⎪ ⎪ ⎨ ⎬× 12 months/360 days ⎪ ⎪ suppliers (months/days)
⎪ ⎪ ⎩ ⎭

7.16
Management of Working Capital

(ii)

Direct Wages:

⎧ Estimated production × Direct labour cost ⎫ ⎪ (in units) per unit ⎪ Average time
lag in payment ⎪ ⎪ ⎨ ⎬x of wages (months/days) 12 months/360 days ⎪ ⎪ ⎪ ⎪ ⎩ ⎭
(iii) Overheads (other than depreciation and amortization): × Overhead cost ⎫ ⎧
Estimatd yearly ⎪ production (in units) ⎪ Average time lag in payment per unit ⎪ ⎪
⎨ ⎬× 12 months / 360 days ⎪ ⎪ of overheads (months / days) ⎪ ⎪ ⎩ ⎭
Note:The amount of overheads may be separately calculated for different types of
overheads. In the case of selling overheads, the relevant item would be sales
volume instead of production volume.

The following illustration shows the process of working capital estimation:


Illustration 3

On 1st January, the Managing Director of Naureen Ltd. wishes to know the amount of
working capital that will be required during the year. From the following
information prepare the working capital requirements forecast. Production during
the previous year was 60,000 units. It is planned that this level of activity
would be maintained during the present year. The expected ratios of the cost to
selling prices are Raw materials 60%, Direct wages 10% and Overheads 20%. Raw
materials are expected to remain in store for an average of 2 months before issue
to production. Each unit is expected to be in process for one month, the raw
materials being fed into the pipeline immediately and the labour and overhead
costs accruing evenly during the month. Finished goods will stay in the warehouse
awaiting dispatch to customers for approximately 3 months. Credit allowed by
creditors is 2 months from the date of delivery of raw material. Credit allowed to
debtors is 3 months from the date of dispatch. Selling price is Rs.5 per unit.
There is a regular production and sales cycle. Wages and overheads are paid on the
1st of each month for the previous month. The company normally keeps cash in hand
to the extent of Rs.20,000.
Solution Working Notes: 1. Raw material inventory: The cost of materials for the
whole year is 60% of the Sales value.

7.17
Financial Management

Hence it is 60,000 units x Rs.5 x

60 = Rs.1,80,000 . The monthly consumption of raw 100 material would be Rs.15,000.


Raw material requirements would be for two months; hence raw materials in stock
would be Rs.30,000.

2. 3.

Debtors: The average sales would be Rs.25,000 p.m. Therefore, a sum of


Rs.75,000/would be the amount of sundry debtors. Work in process: (Students may
give special attention to this point). It is stated that each unit of production
is expected to be in process for one month). Rs.

(a) (b)

Raw materials in work-in-process (being one month’s raw material requirements)


Labour costs in work-in-process (It is stated that it accrues evenly during the
month. Thus, on the first day of each month it would be zero and on the last day
of month the work-in-process would include one month’s labour costs. On an average
therefore, it would be equivalent to ½ of the month’s labour costs) Overheads (For
½ month as explained above) Total work-inprocess
Finished goods inventory:

15,000 1,250

(c)

_2,500 18,750

4.

5. 6.

45,000 (3 month’s costs of production) 7,500 Raw materials Labour 15,000 Overheads
67,500 Creditors: Suppliers allow a two months’ credit period. Hence, the average
amount of creditors would be Rs.30,000 being two months’ purchase of raw
materials.
Direct Wages payable: The direct wages for the whole year is 60,000 units × Rs.5 x
10% = Rs.30,000. The monthly direct wages would be Rs.2,500 (Rs. 30,000 ÷12).
Hence, wages payable would be Rs.2,500. Overheads Payable: The overheads for the
whole year is 60,000 units × Rs.5 x 20% = Rs.60,000. The monthly overheads will be
Rs.5,000 (Rs.60,000 ÷ 12). Hence overheads payable would be Rs.5,000 p.m.

7.

7.18
Management of Working Capital

Statement of Working Capital required: Current Assets: Rs. Rs.

Raw materials inventory (Refer to working note 1) Debtors (Refer to working note
2) Working–in-process (Refer to working note 3) Finished goods inventory (Refer to
working note 4) Cash
Current Liabilities

30,000 75,000 18,750 67,500 20,000 30,000 2,500 5,000 37,500 1,73,750 2,11,250

Creditors (Refer to working note 5) Direct wages payable (Refer to working note 6)
Overheads payable (Refer to working note 7) Estimated working capital requirements

1.6.3 Working capital requirement estimation based on cash cost: We have already
seen that working capital is the difference between current assets and current
liabilities. To estimate requirements of working capital, we have to forecast the
amount required for each item of current assets and current liabilities. However,
in practice another approach may also be useful in estimating working capital
requirements. This approach is based on the fact that in the case of current
assets, like sundry debtors and finished goods, etc., the exact amount of funds
blocked is less than the amount of such current assets. Thus, if we have sundry
debtors worth Rs.1 lakh and our cost of production is Rs.75,000, the actual amount
of funds blocked in sundry debtors is Rs.75,000 the cost of sundry debtors, the
rest (Rs.25,000) is profit. Again some of the cost items also are non-cash costs;
depreciation is a non-cash cost item. Suppose out of Rs.75,000, Rs.5,000 is
depreciation; then it is obvious that the actual funds blocked in terms of sundry
debtors totaling Rs.1 lakh is only Rs.70,000. In other words, Rs.70,000 is the
amount of funds required to finance sundry debtors worth Rs.1 lakh. Similarly, in
the case of finished goods which are valued at cost, non-cash costs may be
excluded to work out the amount of funds blocked. Many experts, therefore,
calculate the working capital requirements by working out the cash costs of
finished goods and sundry debtors. Under this approach, the debtors are calculated
not as a percentage of sales value but as a percentage of cash costs. Similarly,
finished goods are valued according to cash costs.

7.19
Financial Management

Illustration 4

The following annual figures relate to XYZ Co.,


Rs.

Sales (at two months’ credit) Materials consumed (suppliers extend two months’
credit) Wages paid (monthly in arrear) Manufacturing expenses outstanding at the
end of the year (Cash expenses are paid one month in arrear) Total administrative
expenses, paid as above

36,00,000 9,00,000 7,20,000 80,000 2,40,000

Sales promotion expenses, paid quarterly in advance 1,20,000 The company sells its
products on gross profit of 25% counting depreciation as part of the cost of
production. It keeps one months’ stock each of raw materials and finished goods,
and a cash balance of Rs.1,00,000. Assuming a 20% safety margin, work out the
working capital requirements of the company on cash cost basis. Ignore work-in-
process.
Solution Statement of Working Capital requirements (cash cost basis)

A. Current Asset Materials Finished Goods Debtors Cash Prepaid expenses (Sales
promotion) B. Current Liabilities: Creditors for materials Wages outstanding
Manufacturing expenses Administrative expenses Net working capital (A-B)

Rs

Rs

(Rs. 9,00,000 ÷12) (Rs. 25,80,000 ÷12) (Rs.29,40,000÷6) (Rs. 1,20,000÷4)


(Rs.9,00,000÷6) (Rs.7,20,000÷ 12) (Rs.2,40,000÷12)

75,000 2,15,000 4,90,000 1,00,000 30,000 1,50,000 60,000 80,000 20,000 3,10,000
6,00,000 9,10,000

7.20
Management of Working Capital

Add safety margin 20% Total working capital requirements Working Notes: (i)
Computation of Annual Cash cost of Production Material consumed Wages
Manufacturing expenses (Rs.80,000 x 12) Total cash cost of production Computation
of Annual Cash cost of sales: Cash cost of production as in (i) above
Administrative Expenses Sales promotion expenses Total cash cost of sales

1,20,000 7,20,000
Rs. 9,00,000 7,20,000 _9,60,000 25,80,000 Rs. 25,80,000 2,40,000 _1,20,000
29,40,000

(ii)

Illustration 5

PQ Ltd., a company newly commencing business in 2005 has the undermentioned


projected Profit and Loss Account:
Rs.

Sales Cost of goods sold Gross Profit Administrative Expenses Selling Expenses
Profit before tax Provision for taxation Profit after tax The cost of goods sold
has been arrived at as under: Materials used Wages and manufacturing Expenses
Depreciation
Less: Stock of Finished goods (10% of goods produced not yet sold)

Rs. 2,10,000 1,53,000 57,000

14,000 13,000

27,000 30,000 10,000 20,000

84,000 62,500 _23,500 1,70,000 __17,000 1,53,000

7.21
Financial Management

The figure given above relate only to finished goods and not to work-in-progress.
Goods equal to 15% of the year’s production (in terms of physical units) will be
in process on the average requiring full materials but only 40% of the other
expenses. The company believes in keeping materials equal to two months’
consumption in stock. All expenses will be paid one month in advance. Suppliers of
materials will extend 1-1/2 months credit. Sales will be 20% for cash and the rest
at two months’ credit. 70% of the Income tax will be paid in advance in quarterly
instalments. The company wishes to keep Rs.8,000 in cash. Prepare an estimate of
(i) working capital, and (ii) cash cost of working capital.
Note: All workings should form part of the answer. Solution (i) Estimate of
Working Capital requirements Rs. Current Assets Finished stock: Rs. Current
Liabilities Sundry Creditors:

Purchases Provision for taxation

14,088 3,000 --------17,088

Raw materials Wages Depreciation Work-in-progress

8,400 6,250 2,350 17,000

Balance being working capital required, (say Rs.77,500)

77,543

Materials Wages Depreciation Raw Material Sundry Debtors: Materials Wages


Depreciation Adm. & expenses Profit Selling

12,600 3,750 _1,410

17,760 16,100

10,080 7,500 2,820 3,600 4,000 28,000

7.22
Management of Working Capital

Prepaid Expenses: Wages Admn. & expenses Cash in hand 94,631


(ii) Estimate of Cash Cost of Working Capital

Selling

5,521 2,250

7,771 _8,000 94,631


Rs.

Working capital as per statement given above


Less:

77,543 10,580 66,963


Rs.

Profit and Depreciation for which funds are not needed (see working note (vii)

Cash Cost of working capital required, say Rs.67,000 Working Notes: (i) Work-in-
progress 15% of material consumed for finished goods 15% of 40% of wages and
expenses 15% of 40% of Depreciation (ii)

12,600 3,750

_1,410 17,760 th of total materials consumed i.e., Rs.84,000 for finished goods
Raw materials will be 1/6 plus Rs.12,600 for work-in-progress, i.e. Rs.16,100.

(iii) Sundry Debtors: 80% of two months’ Sales, i.e., Rs.2,10,000 ×

80 2 × = Rs.28,000 100 12

Individual items have been computed on that basis. (iv) Creditors for raw
materials on the basis of total purchases during the year (Rs.84,000 +
Rs.12,600+Rs.16,100) × 1½ /12) = Rs.14,088. (v) Wages paid in advance: (Rs.62,500
+ Rs.3,750) × 1/12 = Rs.5,521. (vi) Administrative & Selling expenses paid in
advance Rs.27,000 ×1/12 = Rs.2,250/-

7.23
Financial Management

(vii) Depreciation and profit included in the cost of current assets:


Rs.

Depreciation: Finished goods Work-in-progress Debtors Profit included in Debtors


(including income-tax, i.e., Rs.1,333) 2,350 1,410 2,820 6,580 4,000 10,580
Illustration 6

M.A. Limited is commencing a new project for manufacture of a plastic component.


The following cost information has been ascertained for annual production of
12,000 units which is the full capacity:
Costs per unit (Rs.)

Materials Direct labour and variable expenses Fixed manufacturing expenses


Depreciation Fixed administration expenses

40 20 6 10 _4 80

The selling price per unit is expected to be Rs.96 and the selling expenses Rs.5
per unit. 80% of which is variable. In the first two years of operations,
production and sales are expected to be as follows:
Year Production (No. of units) Sales (No.of units)

1 2.

6,000 9,000

5,000 8,500

To assess the working capital requirements, the following additional information


is available:
7.24
Management of Working Capital

(a) (b) (c) (d) (e) (f)

Stock of materials Work-in-process Debtors Cash balance Creditors for supply of


materials Creditors for expenses

2.25 months’ average consumption Nil 1 month’s average sales. Rs.10,000 1 month’s
average purchase during the year. 1 month’s average of all expenses during the
year.

Prepare, for the two years: (i) (ii) (i)


M.A. Limited Projected Statement of Profit / Loss (Ignoring Taxation) Year 1 Year
2

A projected statement of Profit/Loss (Ignoring taxation); and A projected


statement of working capital requirements.

Solution

Production (Units) Sales (Units) Sales revenue @ Rs. 96 per unit: (A)
Cost of production:

6,000 5,000
Rs.

9,000 8,500
Rs.

4,80,000 2,40,000 1,20,000 72,000 1,20,000 48,000

8,16,000 3,60,000 1,80,000 72,000 1,20,000 48,000

Materials @ Rs. 40 per unit Direct labour and variable expenses @ Rs.20 per unit
Fixed manufacturing expenses (Production Capacity: 12,000 units @ Rs.6)
Depreciation (Production Capacity : 12,000 units @ Rs.10) Fixed administration
expenses (Production Capacity : 12,000 units @ Rs.4)

7.25
Financial Management

Total costs of production


Add: Opening stock of finished goods

_6,00,000

7,80,000 1,00,000

(Year 1 : Nil; Year 2 : 1,000 units) Cost of goods available (Year 1: 6,000 units;
Year 2: 10,000 units)
Less: Closing stock of finished goods at average cost (year 1: 1000 units, year 2
: 1500 units)

6,00,000 1,00,000 5,00,000 20,000 12,000 5,32,000 (-) 52,000

8,80,000 1,32,000 7,48,000 34,000 12,000 7,94,000 (+) 22,000

Cost of goods sold


Add: Selling expenses – Variable @ 4 per unit Fixed (12,000 × Re.1)

Cost of Sales : (B) Profit (+) / Loss (-): (A-B)


Working Notes:

1.

Calculation of creditors for supply of materials:


Year 1 Rs. Year 2 Rs.

Materials consumed during the year


Add: Closing stock (2.25 month’s average consumption) Less: Opening Stock

2,40,000 45,000 2,85,000

3,60,000 67,500 4,27,500 45,000 3,82,500 31,875


Year 2 Rs.

Purchases during the year Average purchases per month (Creditors)

2,85,000 23,750
Year 1 Rs.

2.

Creditors for expenses: Total direct labour, manufacturing, administration and


selling expenses for the year Average per month 2,72,000 22,667 3,46,000 28,833

7.26
Management of Working Capital

(ii) Projected statement of working capital requirements Year 1 Rs. Current


Assets: Year 2 Rs.

Stock of materials (2.25 month’s average consumption) Finished goods Debtors (1


month’s average sales) Cash Total Current Assets (A)
Current Liabilities:

45,000 1,00,000 40,000 _10,000 1,95,000 23,750 22,667 46,417 1,48,583

67,500 1,32,000 68,000 _10,000 2,77,500 31,875 28,833 60,708 2,16,792

Creditors for supply of materials


Refer to working note 1)

Creditors for expenses


(Refer to working note 2)

Total Current Liabilities: (B) Estimated Working Capital Requirements: (A-B)

1.6.4 Effect of Double Shift Working on Working Capital requirements: Increase in


the number of hours of production has an effect on the working capital
requirements. The greatest economy in introducing double shift is the greater use
of fixed assets-little or marginal funds may be required for additional assets.

It is obvious that in double shift working, an increase in stocks will be required


as the production rises. However, it is quite possible that the increase may not
be proportionate to the rise in production since the minimum level of stocks may
not be very much higher. Thus, it is quite likely that the level of stocks may not
be required to be doubled as the production goes up two-fold. The amount of
materials in process will not change due to double shift working since work
started in the first shift will be completed in the second; hence, capital tied up
in materials in process will be the same as with single shift working. As such the
cost of work-in-process, will not change unless the second shift’s workers are
paid at a higher rate. Fixed overheads will remain fixed whereas variable
overheads will increase in proportion to the increased production. Semi-variable
overheads will increase according to the variable element in them.

7.27
Financial Management

However, in examinations the students may increase the amount of stocks of raw
materials proportionately unless instructions are to the contrary.
Illustration 7

Samreen Enterprises has been operating its manufacturing facilities till 31.3.2006
on a single shift working with the following cost structure:
Per Unit Rs.

Cost of Materials Wages (out of which 40% fixed) Overheads (out of which 80%
fixed) Profit Selling Price Sales during 2005-06 – Rs.4,32,000. As at 31.3.2006
the company held:

6.00 5.00 5.00 2.00 18.00


Rs.

Stock of raw materials (at cost) Work-in-progress (valued at prime cost) Finished
goods (valued at total cost) Sundry debtors

36,000 22,000 72,000 1,08,000

In view of increased market demand, it is proposed to double production by working


an extra shift. It is expected that a 10% discount will be available from
suppliers of raw materials in view of increased volume of business. Selling price
will remain the same. The credit period allowed to customers will remain
unaltered. Credit availed of from suppliers will continue to remain at the present
level i.e., 2 months. Lag in payment of wages and expenses will continue to remain
half a month. You are required to assess the additional working capital
requirements, if the policy to increase output is implemented.

7.28
Management of Working Capital

Solution Statement of cost at single shift and double shift working 24,000 units
Per Unit Rs. Total Rs. 48,000 Units Per unit Rs. Total Rs.

Raw materials Wages Fixed Overheads - Variable Fixed Total cost Profit Variable

6 3 2 1 4 16 2 18

1,44,000 72,000 48,000 24,000 96,000 3,84,000 48,000 4,32,000

5.40 3.00 1.00 1.00 2.00 12.40 5.60 18.00

2,59,200 1,44,000 48,000 48,000 96,000 5,95,200 2,68,800 8,64,000

Sales in units 2005-06

Sales Rs.4,32,000 = = 24,000 units Unit selling price Rs.18

Stock of Raw Materials in units on 31.3.2006 =


Value of stock Rs.36,000 = 6,000 units = Cost per unit 6

Stock of work-in-progress in units on 31.3.2006 =

Value of work − in − progress Rs.22,000 = = 2,000 units Cost per unit (Rs.6 +
Rs.5)
Value of stock Rs.72,000 = = 4,500 units. Cost per unit Rs.16

Stock of finished goods in units 2005-06


=

7.29
Financial Management

Comparative Statement of Working Capital Requirement Single Shift Unit Rate Rs.
Amount Rs. Double Shift Unit Rate Rs. Amount Rs.

Current Assets Inventories Raw Materials Work-in-Progress Finished Goods Sundry


Debtors Total Current Assets: (A) Current Liabilities Creditors for Materials
Creditors for Wages Creditors for Expenses Total Current Liabilities: (B) Working
Capital: (A) – (B) Less: Profit included in Debtors 6000 2 4000 1000 1000 6 5 5
24,000 5,000 5,000 34,000 2,04,000 8000 2000 2000 5.40 4.00 3.00 43,200 8,000
_6,000 57,200 3,54,000 6000 2000 4500 6000 6 11 16 18 36,00 22,000 72,000 1,08,000
2,38,000 12000 2000 9000 12000 5.40 9.40 12.40 18.00 64,800 18,800 1,11,600
2,16,000 4,11,200

_12,000 12,000 5.60 _67,200 1,92,000 2,86,800 Increase in Working Capital


requirement is (Rs.2,86,800 – Rs.1,92,000) or Rs.94,800
Notes:

(i) (ii)

The quantity of material in process will not change due to double shift working
since work started in the first shift will be completed in the second shift. The
valuation of work-in-progress based on prime cost as per the policy of the company
is as under.
Single shift Rs. Double shift Rs.

Materials Wages – Variable Fixed

6.00 3.00 _2.00 11.00

5.40 3.00 1.00 9.40

7.30
Management of Working Capital

UNIT – II : TREASURY AND CASH MANAGEMENT 2.1 TREASURY MANAGEMENT: MEANING

Tight money, escalating interest rates and economic volatility have called for a
specialised skills called Treasury Management. Until recently, no major efforts
were made to manage cash. In the wake of the competitive business environment
resulting from the liberalization of the economy, there is a pressure to manage
cash. The demand for funds for expansions coupled with high interest rates,
foreign exchange volatility and the growing volume of financial transactions have
necessitated efficient management of money. Treasury management is defined as ‘the
corporate handling of all financial matters, the generation of external and
internal funds for business, the management of currencies and cash flows and the
complex, strategies, policies and procedures of corporate finance.’ The treasury
management mainly deals with working capital management and financial risk
management. The former constitutes cash management and decides the asset liability
mix. Financial risk management includes forex and interest rate management. The
key goal of treasury management is planning, organizing and controlling cash
assets to satisfy the financial objectives of the organization. The goal may be to
maximize the return on the available cash, or minimize interest cost or mobilise
as much cash as possible for corporate ventures. Dealing in forex, money and
commodity markets involves complex risks of fluctuating exchange rates, interest
rates and prices which can affect the profitability of the organization. Treasury
managers try to minimize lapses by adopting risk transfer and hedging techniques
that suit the internal policies of the organisation. Options, futures and swaps
are a few of the major derivative instruments, the Treasury Managers use for
hedging their risk.
2.2 FUNCTIONS OF TREASURY DEPARTMENT

1. Cash Management: The efficient collection and payment of cash both inside the
organisation and to third parties is the function of the treasury department. The
involvement of the department with the details of receivables and payables will be
a matter of policy. There may be complete centralization within a group treasury
or the treasury may simply advise subsidiaries and divisions on policy matter
viz., collection/payment periods, discounts, etc. Any position between these two
extremes would be possible. Treasury will normally manage surplus funds in an
investment portfolio. Investment policy will consider future needs for liquid
funds and acceptable levels of risk as determined by company policy.

7.31
Financial Management

2. Currency Management: The treasury department manages the foreign currency risk
exposure of the company. In a large multinational company (MNC) the first step
will usually be to set off intra-group indebtedness. The use of matching receipts
and payments in the same currency will save transaction costs. Treasury might
advise on the currency to be used when invoicing overseas sales.

The treasury will manage any net exchange exposures in accordance with company
policy. If risks are to be minimized then forward contracts can be used either to
buy or sell currency forward.
3. Funding Management: Treasury department is responsible for planning and
sourcing the company’s short, medium and long-term cash needs. Treasury department
will also participate in the decision on capital structure and forecast future
interest and foreign currency rates. 4. Banking: It is important that a company
maintains a good relationship with its bankers. Treasury department carry out
negotiations with bankers and act as the initial point of contact with them.
Short-term finance can come in the form of bank loans or through the sale of
commercial paper in the money market. 5. Corporate Finance: Treasury department is
involved with both acquisition and divestment activities within the group. In
addition it will often have responsibility for investor relations. The latter
activity has assumed increased importance in markets where share-price performance
is regarded as crucial and may affect the company’s ability to undertake
acquisition activity or, if the price falls drastically, render it vulnerable to a
hostile bid. 2.3 MANAGEMENT OF CASH

Management of cash is an important function of the finance manager. The Finance


Manager has to provide adequate cash to each of the units. For the survival of the
business it is absolutely essential that there should be adequate cash. It is the
duty of finance manger to have liquidity at all parts of the organization while
managing cash. On the other hand, he has also to ensure that there are no funds
blocked in idle cash. Idle cash resources entail a great deal of cost in terms of
interest charges and in terms of opportunities costs. Hence, the question of costs
of idle cash must also be kept in mind by the finance manager. A cash management
scheme therefore, is a delicate balance between the twin objectives of liquidity
and costs.
2.3.1 The Need for Cash: The following are three basic considerations in
determining the amount of cash or liquidity as have been outlined by Lord Keynes:

♦ Transaction need: Cash facilitates the meeting of the day-to-day expenses and
other debt payments. Normally, inflows of cash from operations should be
sufficient for this purpose. But sometimes this inflow may be temporarily blocked.
In such cases, it is only the
7.32
Management of Working Capital

reserve cash balance that can enable the firm to make its payments in time. ♦
Speculative needs: Cash may be held in order to take advantage of profitable
opportunities that may present themselves and which may be lost for want of ready
cash/settlement. ♦ Precautionary needs: Cash may be held to act as for providing
safety against unexpected events. Safety as is explained by the saying that a man
has only three friends an old wife, an old dog and money at bank.
Facets of Cash Management: Cash management is concerned with the managing of (i)
Cash flows into and out of the firm; (ii) Cash flows within the firm; and (iii)
Cash balances held by the firm at a point of time by financing deficit or
investing surplus cash. It is generally represented by a cash management cycle.
Sales generates cash which has to be disbursed out.

In recent years, a number of innovations have been made in cash management


techniques. An obvious aim of the firm these days is to mange its cash affairs in
such a way as to maintain a minimum balance of cash and to invest the surplus
immediately in profitable investment opportunities. In order to synchronise the
cash receipt and payments. A firm need to develop appropriate strategies for cash
management viz:
(i) Cash Planning: The pattern of cash inflows and outflows should be properly
predicted in advance. Cash budget is a tool to achieve this objective.

(ii) Managing the cash flows: The cash inflows should be accelerated, while as far
as possible, the outflows should be decelerated. (iii) Optimum cash level: In
deciding about the appropriate level of cash balances, the cost of idle cash and
danger of shortage should be taken into consideration. (iv) Investing surplus
cash: The surplus cash should be properly invested to earn profits. The firm
should decide about the division of such cash balance between various alternative
short term investment opportunities such as, bank deposits, marketable securities,
inter-corporate lending.

The ideal cash management system will depend upon various factors viz., product,
organization structure, competition, culture and options available. The task is
really complex. The exact nature of a cash management system would depend upon the
organizational structure of an enterprise. In a highly centralized organization
the system would be such that the central or head office controls the inflows and
outflows of cash on a routine and daily basis. In a decentralized form of
organisation, where the divisions have compelete

7.33
Financial Management

responsibility of conducting their affairs, it may not be possible and advisable


for the central office to exercise a detailed control over cash inflows and
outflows.
2.3.2 Cash Planning: Cash Planning is a technique to plan and control the use of
cash. This protects the financial conditions of the firm by developing a projected
cash statement from a forecast of expected cash inflows and outflows for a given
period. This may be done periodically either on daily, weekly or monthly basis.
The period and frequency of cash planning generally depends upon the size of the
firm and philosophy of management. As firms grows and business operations become
complex, cash planning becomes inevitable for continuing success.

The very first step in this direction is to estimate the requirement of cash. For
this purpose cash flow statements and cash budget are required to be prepared. The
technique of preparing cash flow and funds flow statements have been discussed in
this book. The preparation of cash budget has however, been demonstrated here.
2.3.3 Cash Budget: Cash Budget is the most significant device to plan for and
control cash receipts and payments. This represents cash requirements of business
during the budget period.

One of the significant advantage of cash budget is to determine the net cash
inflow or outflow so that the firm is enabled to arrange finances. However, the
firm’s decision for appropriate sources of financing should depend upon factors
such as cost and risk. Cash Budget helps a firm to manage its cash position. It
also helps to utilise funds in better ways. On the basis of cash budget, the firm
can decide to invest surplus cash in marketable securities and earn profits. The
cash budget is prepared on the basis of receipts and payments method and offers
following benefits: (i) (ii) (i) (ii) It provides a complete picture of all items
of expected cash flows. It is a sound tool of managing daily cash operations. Its
reliability is reduced because of the uncertainty of cash forecasts. For example,
collections may be delayed, or unanticipated demands may cause large
disbursements. It fails to highlight the significant movements in the Working
Capital items.

This method, however, suffers from the following limitations:

In order to maintain an optimum cash balance, what is required is (i) a complete


and accurate forecast of net cash flows over the planning horizon and (ii) perfect
synchronization of cash receipts and disbursements. Thus, implementation of an
efficient cash management system starts with the preparation of a plan of firm’s
operations for a period in future. This plan will help in preparation of a
statement of receipts and disbursements expected at different point of

7.34
Management of Working Capital

time of that period. It will enable the management to pin point the time of
excessive cash or shortage of cash. This will also help to find out whether there
is any expected surplus cash still unutilized or shortage of cash which is yet to
be arranged for. In order to take care of all these considerations, the firm
should prepare a cash budget. The following figure highlights the cash surplus and
cash shortage position over the period of cash budget for preplanning to take
corrective and necessary steps.

2.4

METHODS OF CASH FLOW BUDGETING

Cash flow budget is a detailed budget of income and cash expenditures


incorporating both revenue and capital items. The cash flow budget should be
prepared in the same format in which the actual position is to be presented. The
year’s budget is usually phased into shorter periods for control e.g., monthly or
quarterly. Cash budget is concerned with liquidity must reflect changes between
opening and closing debtor balances and between opening and closing creditor
balances as well as focusing attention on other inflows and outflows of cash. The
cash budget shows the cash flows arising from the operational budgets and the
profit and assets structure. A cash budget can be prepared in the following ways:
7.35
Financial Management

1. Receipts and Payments Method: In this method all the expected receipts and
payments for budget period are considered. All the cash inflow and outflow of all
functional budgets including capital expenditure budgets are considered. Accruals
and adjustments in accounts will not affect the cash flow budget. Anticipated cash
inflow is added to the opening balance of cash and all cash payments are deducted
from this to arrive at the closing balance of cash. This method is commonly used
in business organizations. 2. Adjusted Income Method: In this method the annual
cash flows are calculated by adjusting the sales revenues and cost figures for
delays in receipts and payments (change in debtors and creditors) and eliminating
non-cash items such as depreciation. 3. Adjusted Balance Sheet Method: In this
method, the budgeted balance sheet is predicted by expressing each type of asset
and short-term liabilities as percentage of the expected sales. The profit is also
calculated as a percentage of sales, so that the increase in owners equity can be
forecasted. Known adjustments, may be made to long-term liabilities and the
balance sheet will then show if additional finance is needed.

It is important to note that the capital budget will also be considered in the
preparation of cash flow budget because the annual budget may disclose a need for
new capital investments and also, the costs and revenues of any new projects
coming on stream will need to be incorporated in the short-term budgets. A number
of additional financial statements, such as sources and application of funds
statement or schedules or loan service payments or capital raising schedules may
be produced. The Cash Budget can be prepared for short period or for long period.
Cash budget for short period: Preparation of cash budget month by month would
require the following estimates:

(a) As regards receipts: 1. 2. 3. 1. 2. 3. Receipts from debtors; Cash Sales; and


Any other source of receipts of cash (say, dividend from a subsidiary company)
Payments to be made for purchases; Payments to be made for expenses; Payments that
are made periodically but not every month; (i) (ii) debenture interest; income tax
paid in advance;

(b) As regards payments:

7.36
Management of Working Capital

(iii) sales tax etc. 4. Special payments to be made in a particular month, for
example, dividends to shareholders, redemption of debentures, repayments of loan,
payment of assets acquired, etc.
Co. Ltd. Cash Budget Period……………… Month 1 Receipts: Month 2 Month 3 Month 12

2.4.1 Format of Cash Budget

1. Opening balance 2. Collection from debtors 3. Cash sales 4. Loans from banks 5.
Share capital 6. Miscellaneous receipts 7. Other items
Total Payments:

1. Payments to creditors 2. Wages 3. Overheads (a) (b) (c) 4. Interest 5. Dividend


6. Corporate tax
7.37
Financial Management

7. Capital expenditure 8. Other items


Total

Closing balance [Surplus (+)/Shortfall (-)] Students are required to do good


practice in preparing the cash budgets. The following illustration will show how
short term cash budgets can be prepared.
Illustration 1

Prepare monthly cash budget for six months beginning from April 2006 on the basis
of the following information:(i) Estimated monthly sales are as follows:Rs. Rs.

January February March April May (ii)

1,00,000 June 1,20,000 July 1,40,000 August 80,000 September 60,000 October

80,000 1,00,000 80,000 60,000 1,00,000

Wages and salaries are estimated to be payable as follows:Rs. Rs.

April May June

9,000 July 8,000 August 10,000 September

10,000 9,000 9,000

(iii) Of the sales, 80% is on credit and 20% for cash. 75% of the credit sales are
collected within one month and the balance in two months. There are no bad debt
losses. (iv) Purchases amount to 80% of sales and are made and paid for in the
month preceding the sales. (v) The firm has 10% debentures of Rs.1,20,000.
Interest on these has to be paid quarterly in January, April and so on. (vi) The
firm is to make an advance payment of tax of Rs.5,000 in July, 2006.
7.38
Management of Working Capital

(vii) The firm had a cash balance of Rs.20,000 on April 1, 2006, which is the
minimum desired level of cash balance. Any cash surplus/deficit above/below this
level is made up by temporary investments/liquidation of temporary investments or
temporary borrowings at the end of each month (interest on these to be ignored).
Solution

Workings: Collection from debtors:


(Amount in Rs.)
February Total sales Credit sales (80% of total sales) Collections: One month Two
months Total collections 72,000 84,000 24,000 1,08,000 48,000 28,000 76,000 36,000
16,000 52,000 48,000 12,000 60,000 60,000 16,000 76,000 48,000 20,000 68,000
1,20,000 March 1,40,000 April 80,000 May 60,000 June 80,000 July 1,00,000 August
80,000 September 60,000

96,000

1,12,000

64,000

48,000

64,000

80,000

64,000

48,000

Monthly Cash Budget for Six months, April to September, 2006


(Amount in Rs.)
Receipts: April Opening balance Cash sales Collection from debtors Total cash
available (A) 20,000 16,000 1,08,000 1,44,000 May 20,000 12,000 76,000 1,08,000
June 20,000 16,000 52,000 88,000 July 20,000 20,000 60,000 1,00,000 August 20,000
16,000 76,000 1,12,000 September 20,000 12,000 68,000 1,00,000

7.39
Financial Management
Payments: Purchases Wages & salaries Interest on debentures Tax payment Total
payments (B) Minimum desired cash balance 20,000 80,000 64,000 (64,000) 20,000
92,000 16,000 (16,000) 20,000 1,10,000 (22,000) ---20,000 1,02,000 (2,000) 20,000
77,000 35,000 (35,000) 20,000 1,09,000 (9,000) ----48,000 9,000 3,000 --60,000
64,000 8,000 ----72,000 80,000 10,000 ------90,000 64,000 10,000 3,000 5,000
82,000 48,000 9,000 -----57,000 80,000 9,000 ------89,000

Total cash needed (C) Surplus deficit (A-C) Investment/financing Temporary


Investments Liquidation of investments or borrowings temporary temporary

---of (64,000) 20,000

---(16,000) 20,000

22,000 22,000 20,000

2,000 2,000 20,000

---(35,000) 20,000

9,000 9,000 20,000

Total effect investment/financing (D)

Closing cash balance (A+DB)

Illustration 2

From the following information relating to a departmental store, you are required
to prepare for the three months ending 31st March, 2006:(a) Monthwise cash budget
on receipts and payments basis; and (b) Statement of Sources and uses of funds for
the three months period. It is anticipated that the working capital at 1st
January, 2006 will be as follows:Rs. in ‘000’s

Cash in hand and at bank Short term investments Debtors

545 300 2,570

7.40
Management of Working Capital

Stock Trade creditors Other creditors Dividends payable Tax due Plant Budgeted
Profit Statement: January Sales Cost of sales Gross Profit Administrative, Selling
and Distribution Expenses 315 Net Profit before tax Budgeted balances at the end
of each months: 31st Short term investments Debtors Stock Trade creditors Other
creditors Dividends payable Tax due Plant (depreciation ignored) Jan. 700 2,600
1,200 2,000 200 485 320 800 150 270 125
Rs. in ‘000’s

1,300 2,110 200 485 320 800


Rs.in ‘000’s

February 1,800 1,405 395

March 1,700 1,330 370 255 115 31st March 200 2,350 1,000 1,900 200 -320 1,550

2,100 1,635 465

29th Feb. --2,500 1,100 1,950 200 -320 1,600

Depreciation amount to Rs.60,000 is included in the budgeted expenditure for each


month.

7.41
Financial Management

Solution

Workings: (1) Payments to creditors: Cost of Sales Add Closing Stocks Less:
Opening Stocks Purchases Add: Trade Creditors, Opening balance Less: Trade
Creditors, closing balance Payment (2) Receipts from debtors: Debtors, Opening
balances Add Sales Less Debtors, closing balance Receipt
CASH BUDGET

Rs. in ‘000’

Jan. 2006 1,635 1,200 2,835 1,300 1,535 2,110 3,645 2,000 1,645 2,570 2,100 4,670
2,600 2,070

Feb.2006

March, 2006

1,405 1,100 2,505 1,200 1,305 2,000 3,305 1,950 1,355 2,600 1,800 4,400 2,500
1,900

1,330 1,000 2,330 1,100 1,230 1,950 3,180 1,900 1,280 2,500 1,700 4,200 2,350
1,850

(a) 3 months ending 31st March, 2006


(Rs, in 000’s) January, 2006 Feb. 2006 March, 2006

Opening cash balances Add Receipts: From Debtors Sale of Investments Sale of Plant
Total (A)

545 2,070 ----2,615

315 1,900 700 --2,915

65 1,850 ---50 1,965

7.42
Management of Working Capital

Deduct Payments Creditors Expenses Capital Expenditure Payment of dividend


Purchase of investments Total payments (B) Closing cash balance (A - B) (b)
Sources:

1,645 255 ----400 2,300 315

1,355 210 800 485 --2,850 65

1,280 195 ----200 1,675 290

Statement of Sources and uses of Funds for the Three Month Period Ending 31st
March, 2006 Rs.’000 Rs.’000

Funds from operation: Net profit Add Depreciation Sale of plant Decrease in
Working Capital Total
Uses:

390 180 570 50 620 665 1,285 800 485 1,285


Statement of Changes in Working Capital January,06 Rs.000 March, 06 Rs.000
Increase Rs.000 Decrease Rs.000

Purchase of plant Payment by dividends Total

Current Assets

Cash in hand and at Bank Short term Investments

545 300

290 200

255 100

7.43
Financial Management

Debtors Stock
Current Liabilities

2,570 1,300 4,715

2,350 1,000 3,840 1,900 200 320 2,420 1,420 665 665 210 -----

220 300

Trade Creditors Other Creditors Tax Due Working Capital Decrease

2,110 200 320 2,630 2,085

-------

2,085 2,085 875 875 2.4.2 Cash Budget for long period: Long-range cash forecast
often resemble the projected sources and application of funds statement. The
following procedure may be adopted to prepare long-range cash forecasts: (i) (ii)
Take the cash at bank and in the beginning of the year:
Add:

(a) Trading profit (before tax) expected to be earned; (b) Depreciation and other
development expenses incurred to be written off; (c) Sale proceeds of assets’; (d)
Proceeds of fresh issue of shares or debentures; and (e) Reduction in working
capital that is current assets (except cash) less current liabilities. (iii)
Deduct: (a) Dividends to be paid. (b) Cost of assets to be purchased. (c) Taxes to
be paid. (d) Debentures or shares to be redeemed. (e) Increase in working capital.

7.44
Management of Working Capital

Illustration 3

You are given below the Profit & Loss Accounts for two years for a company:
Profit and Loss Account
Year 1 Rs. To Opening stock To Raw materials To Stores To Manufacturing Expenses
To Other Expenses To Depreciation To Net Profit 80,00,000 3,00,00,000 1,00,00,000
1,00,00,000 1,00,00,000 1,00,00,000 1,30,00,000 9,10,00,000 Year 2 Rs. 1,00,00,000
4,00,00,000 1,20,00,000 1,60,00,000 1,00,00,000 1,00,00,000 1,80,00,000
11,60,00,000 9,10,00,000 11,60,00,000 By Sales By Closing stock By Misc. Income
Year 1 Rs. 8,00,00,000 1,00,00,000 10,00,000 Year 2 Rs. 10,00,00,000 1,50,00,000
10,00,000

Sales are expected to be Rs.12,00,00,000 in year 3. As a result, other expenses


will increase by Rs.50,00,000 besides other charges. Only raw materials are in
stock. Assume sales and purchases are in cash terms and the closing stock is
expected to go up by the same amount as between year 1 and 2. You may assume that
no dividend is being paid. The Company can use 75% of the cash generated to
service a loan. How much cash from operations will be available in year 3 for the
purpose? Ignore income tax.
Solution Projected Profit and Loss Account for the year 3 Year 2 Year 3 Actual
Projected (Rs. in (Rs. in lakhs) lakhs) Year 2 Actual (Rs. in lakhs) Year 3
Projected (Rs. in lakhs)

To Materials consumed To Stores

350 120

420 By Sales 144 By Misc. Income

1,000 10

1,200 10

7.45
Financial Management

To Mfg. Expenses To Other expenses To Depreciation To Net profit


Cash Flow:

160 100 100 180 1,010

192 150 100 204 1,210 1,010 1,210

(Rs. in lakhs)

Profit
Add: Depreciation Less: Cash required for increase in stock

204 100 304 50 254

Net cash inflow Available for servicing the loan: 75% of Rs.2,54,00,000 or
Rs.1,90,50,000
Working Notes:

(i)

Material consumed in year 2: 35% of sales. Likely consumption in year 3 : Rs.1200


×

35 or Rs. 420 (lakhs) 100

(ii)

Stores are 12% of sales, as in year 2.

(iii) Manufacturing expenses are 16% of sales.


Note: The above also shows how a projected profit and loss account is prepared.
2.4.3 Managing Cash Collection and Disbursements: The finance manager must control
the levels of cash balance at various points in the organization. This task
assumes special importance on account of the fact that there is generally a
tendency amongst divisional managers to keep cash balance in excess of their
needs. Hence, the finance manager must devise a system whereby each division of an
organization retains enough cash to meet its day-to-day requirements without
having surplus balance on hand. For this, methods have to be employed to:

(a) Speed up the mailing time of payments from customers; (b) Reduce the time
during which payments received by the firm remain uncollected and speed up the
movement of funds to disbursement banks.

7.46
Management of Working Capital

Having prepared the cash budget, the finance manager should ensure that there does
not exists a significant deviation between projected cash flows and actual cash
flows. To achieve this cash management efficiency will have to be improved through
a proper control of cash collection and disbursement. The twin objectives in
managing the cash flows should be to accelerate cash collections as much as
possible and to decelerate or delay cash disbursements.
2.4.4 Accelerating Cash Collections: A firm can conserve cash and reduce its
requirements for cash balances if it can speed up its cash collections by issuing
invoices quickly and taking other necessary steps for cash collection. It can be
accelerated by reducing the time lag between a customer pays bill and the cheque
is collected and funds become available for the firm’s use. A firm can
decentralized collection system known as concentration banking and lock box system
to speed up cash collection and reduce float time. (i) Concentration Banking: In
concentration banking the company establishes a number of strategic collection
centres in different regions instead of a single collection centre at the head
office. This system reduces the period between the time a customer mails in his
remittances and the time when they become spendable funds with the company.
Payments received by the different collection centers are deposited with their
respective local banks which in turn transfer all surplus funds to the
concentration bank of head office. The concentration bank with which the company
has its major bank account is generally located at the headquarters. Concentration
banking is one important and popular way of reducing the size of the float. (ii)
Lock Box System: Another means to accelerate the flow of funds is a lock box
system. While concentration banking, remittances are received by a collection
centre and deposited in the bank after processing. The purpose of lock box system
is to eliminate the time between the receipt of remittances by the company and
deposited in the bank. A lock box arrangement usually is on regional basis which a
company chooses according to its billing patterns.

Under this arrangement, the company rents the local post-office box and authorizes
its bank at each of the locations to pick up remittances in the boxes. Customers
are billed with instructions to mail their remittances to the lock boxes. The bank
picks up the mail several times a day and deposits the cheques in the company’s
account. The cheques may be microfilmed for record purposes and cleared for
collection. The company receives a deposit slip and lists all payments together
with any other material in the envelope. This procedure frees the company from
handling and depositing the cheques. The main advantage of lock box system is that
cheques are deposited with the banks sooner and become collected funds sooner than
if they were processed by the company prior to deposit. In other words lag between
the time cheques are received by the company and the time they are actually
deposited in the bank is eliminated. The main drawback of lock box system is the
cost of its operation. The bank provides a number of services in addition to usual
clearing of cheques

7.47
Financial Management

and requires compensation for them. Since the cost is almost directly proportional
to the number of cheques deposited. Lock box arrangements are usually not
profitable if the average remittance is small. The appropriate rule for deciding
whether or not to use a lock box system or for that matter, concentration banking,
is simply to compare the added cost of the most efficient system with the marginal
income that can be generated from the released funds. If costs are less than
income, the system is profitable, if the system is not profitable, it is not worth
undertaking.
(iii) Playing the float: Besides accelerating collections, an effective control
over payments can also cause faster turnover of cash. This is possible only by
making payments on the due date, making excessive use of draft (bill of exchange)
instead of cheques. Availability of cash can be maximized by playing the float. In
this, a firm estimates accurately the time when the cheques issued will be
presented for encashment and thus utilizes the float period to its advantage by
issuing more cheques but having in the bank account only so much cash balance as
will be sufficient to honour those cheques which are actually expected to be
presented on a particular date. 2.4.5 Different Kinds of Float with reference to
Management of Cash: The term float is used to refer to the periods that affect
cash as it moves through the different stages of the collection process. Four
kinds of float with reference to management of cash are:

♦ Billing float: An invoice is the formal document that a seller prepares and
sends to the purchaser as the payment request for goods sold or services provided.
The time between the sale and the mailing of the invoice is the billing float. ♦
Mail float: This is the time when a cheque is being processed by post office,
messenger service or other means of delivery. ♦ Cheque processing float: This is
the time required for the seller to sort, record and deposit the cheque after it
has been received by the company. ♦ Banking processing float: This is the time
from the deposit of the cheque to the crediting of funds in the sellers account.
2.4.6 Delaying Payments: A firm can increase its net float by speeding up
collections. It can also increase the net float by delayed disbursement of funds
from the bank by increasing the mail time. A company may make payment to its
outstation suppliers by a cheque and send it through mail. The delay in transit
and collection of the cheque, will be used to increase the float. 2.4.7
Controlling Disbursements: The effective control of disbursement can also help the
firm in conserving cash and reducing the financial requirements. Disbursement
arise due to trade credit, which is a spontaneous, source of funds. The firm
should make payments using credit terms to the fullest extent.

7.48
Management of Working Capital

2.4.8 Determining The Optimum Cash Balance: A firm should maintain optimum cash
balance to cater to the day-to-day operations. It may also carry additional cash
as a buffer or safety stock. The amount of cash balance will depend on the risk-
return trade off. The firm should maintain optimum - just enough, neither too much
nor too little cash balance. This, however, poses a question. How to determine the
optimum cash balance if cash flows are predictable and if they are not
predictable? 2.5 CASH MANAGEMENT MODELS

In recent years several types of mathematical models have been developed which
helps to determine the optimum cash balance to be carried by a business
organization. The purpose of all these models is to ensure that cash does not
remain idle unnecessarily and at the same time the firm is not confronted with a
situation of cash shortage. All these models can be put in two categories-
inventory type models and stochastic models. Inventory type models have been
constructed to aid the finance manager to determine optimum cash balance of his
firm. William J. Baumol’s economic order quantity model applies equally to cash
management problems under conditions of certainty or where the cash flows are
predictable. However, in a situation where the EOQ Model is not applicable,
stochastic model of cash management helps in determining the optimum level of cash
balance. It happens when the demand for cash is stochastic and not known in
advance.
2.5.1 William J. Baumol’s Economic Order Quantity Model, (1952): According to this
model, optimum cash level is that level of cash where the carrying costs and
transactions costs are the minimum. The carrying costs refers to the cost of
holding cash, namely, the interest foregone on marketable securities. The
transaction costs refers to the cost involved in getting the marketable securities
converted into cash. This happens when the firm falls short of cash and has to
sell the securities resulting in clerical, brokerage, registration and other
costs.

The optimum cash balance according to this model will be that point where these
two costs are minimum. The formula for determining optimum cash balance is: C=
Where, 2U × P S C= U= P= S= Optimum cash balance Annual (or monthly) cash
disbursement Fixed cost per transaction. Opportunity cost of one rupee p.a. (or
p.m.)

7.49
Financial Management

Illustration 4

A firm maintains a separate account for cash disbursement. Total disbursement are
Rs.1,05,000 per month or Rs.12,60,000 per year. Administrative and transaction
cost of transferring cash to disbursement account is Rs.20 per transfer.
Marketable securities yield is 8% per annum. Determine the optimum cash balance
according to William J. Baumol model.
Solution

The optimum cash balance C =

2 × Rs.12,60,000 × Rs. 20 = Rs.25,100 0.08

The limitation of the Baumol’s model is that it does not allow the cash flows to
fluctuate. Firms in practice do not use their cash balance uniformly nor they are
able to predict daily cash inflows and outflows. The Miller-Orr (MO) model
overcomes this shortcoming and allows for daily cash flow variation.
2.5.2 Miller-Orr Cash Management Model (1966): According to this model the net
cash flow is completely stochastic. When changes in cash balance occur randomly
the application of control theory serves a useful purpose. The Miller-Orr model is
one of such control limit models. This model is designed to determine the time and
size of transfers between an investment account and cash account. In this model
control limits are set for cash balances. These limits may consist of h as upper
limit, z as the return point; and zero as the lower limit. When the cash balance
reaches the upper limit, the transfer of cash equal to h – z is invested in
marketable securities account. When it touches the lower limit, a transfer from
marketable
7.50
Management of Working Capital

securities account to cash account is made. During the period when cash balance
stays between (h, z) and (z, 0) i.e. high and low limits no transactions between
cash and marketable securities account is made. The high and low limits of cash
balance are set up on the basis of fixed cost associated with the securities
transactions, the opportunity cost of holding cash and the degree of likely
fluctuations in cash balances. These limits satisfy the demands for cash at the
lowest possible total costs. The following diagram illustrates the Miller-Orr
model.

h Upper control limit

Cash Balance (Rs.)

Return point

0 Time Lower control limit

The MO Model is more realistic since it allows variations in cash balance within
lower and upper limits. The finance manager can set the limits according to the
firm’s liquidity requirements i.e., maintaining minimum and maximum cash balance.
2.6 RECENT DEVELOPMENTS IN CASH MANAGEMENT

Now-a-days, electronic delivery and payment system are becoming increasingly


important because of increased competition and the demand for more efficient and
convenient capabilities. A considerable number of transactions and amounts of
funds can be moved electronically from one place to another almost
instantaneously. Consequently, the opportunities presented are not only beneficial
but can create a risk to a business firm. Such threats include internal and
external fraud, theft and unauthorized manipulation of financial data. Therefore,
we can easily observe the rapid transition from the most basic and traditional
principles to now complex strategies dominated by the technology and
globalisation, but the

7.51
Financial Management

basic goal is same i.e., the efficient utilisation of cash in a way which is
consistent with the overall strategic objectives of a business unit.
2.6.1 Electronic Fund Transfer: With the developments which took place in the
Information technology, the present banking system is switching over to the
computerisation of banks branches to offer efficient banking services and cash
management services to their customers. The network will be linked to the
different branches, banks. This will help the customers in the following ways:

♦ Instant updation of accounts. ♦ The quick transfer of funds. ♦ Instant


information about foreign exchange rates.
2.6.2 Zero Balance Account: For efficient cash management some firms employ an
extensive policy of substituting marketable securities for cash by the use of zero
balance accounts. Every day the firm totals the cheques presented for payment
against the account. The firm transfers the balance amount of cash in the account
if any, for buying marketable securities. In case of shortage of cash the firm
sells the marketable securities. 2.6.3 Money Market Operations: One of the tasks
of ‘treasury function’ of larger companies is the investment of surplus funds in
the money market. The chief characteristic of money market banking is one of size.
Banks obtain funds by competing in the money market for the deposits by the
companies, public authorities, High Networth Investors (HNI), and other banks.
Deposits are made for specific periods ranging from overnight to one year, a
highly competitive rates which reflect supply and demand on a daily, even hourly
basis are quoted. Consequently, the rates can fluctuate quite dramatically,
especially for the shorter-term deposits. Surplus funds can thus be invested in
money market easily. 2.6.4 Petty Cash Imprest System: For better control on cash,
generally the companies use petty cash imprest system wherein the day-to-day petty
expenses are estimated taking into account past experience and future needs and
generally a week’s requirement of cash will be kept separate for making petty
expenses. Again, the next week will commence with the predetermined balance. This
will reduce the strain of the management in managing petty cash expenses and help
in the managing cash efficiently. 2.6.5 Management of Temporary Cash Surplus

Temporary cash surpluses can be profitably invested in the following: ♦ Short-term


deposits in Banks and financial institutions. ♦ Short-term debt market
instruments. ♦ Long-term debt instruments.

7.52
Management of Working Capital

♦ Shares of Blue chip listed companies.


2.6.6 Electronic Cash Management System: Most of the cash management systems nowa-
days are electronically based, since ‘speed’ is the essense of any cash management
system. Electronically, transfer of data as well as funds play a key role in any
cash management system. Various elements in the process of cash management are
linked through a satellite. Various places that are interlinked may be the place
where the instrument is collected, the place where cash is to be transferred in
company’s account, the place where the payment is to be transferred etc.

Certain networked cash management system may also provide a very limited access to
third parties like parties having very regular dealings of receipts and payments
with the company etc. A finance company accepting deposits from public through
sub-brokers may give a limited access to sub-brokers to verify the collections
made through him for determination of his commission among other things.
Benefits: Good cash management is a conscious process of knowing:

♦ When, where and how a company’s cash needs will arise. ♦ Knowing what are the
best sources of meeting at a short notice additional cash requirement. ♦
Maintaining good and cordial relations with bankers and other creditors.
Scientific cash management results in:

♦ Significant saving in time. ♦ Decrease in interest costs. ♦ Less paper work. ♦


Greater accounting accuracy. ♦ More control over time and funds. ♦ Supports
electronic payments. ♦ Faster transfer of funds from one location to another,
where required. ♦ Speedy conversion of various instruments into cash. ♦ Making
available funds wherever required, whenever required. ♦ Reduction in the amount of
‘idle float’ to the maximum possible extent. ♦ Ensures no idle funds are placed at
any place in the organization. ♦ It makes inter-bank balancing of funds much
easier.

7.53
Financial Management

♦ It is a true form of centralised ‘Cash Management’. ♦ Produces faster electronic


reconciliation. ♦ Allows for detection of book-keeping errors. ♦ Reduces the
number of cheques issued. ♦ Earns interest income or reduce interest expense. Even
a multinational organization having subsidiaries worldwide, can pool everything
internationally so that the company offset the debts with the surplus monies from
various subsidiaries. It will result in transformation of treasury function into a
profit-centre by optimizing cash and putting it on profitable use. Creative and
pro-active cash management solutions can contribute dramatically to a company’s
profitability and to its competitive edge. The ultimate purpose of scientific cash
management is to ensure solvency, liquidity and profitability of the organization
as a whole.
2.6.7 Virtual Banking: The practice of banking has undergone a significant change
in the nineties. While banks are striving to strengthen customer base and
relationship and move towards relationship banking, customers are increasingly
moving away from the confines of traditional branch banking and are seeking the
convenience of remote electronic banking services. And even within the broad
spectrum of electronic banking the virtual banking has gained prominence

Broadly virtual banking denotes the provision of banking and related services
through extensive use of information technology without direct recourse to the
bank by the customer. The origin of virtual banking in the developed countries can
be traced back to the seventies with the installation of Automated Teller Machines
(ATMs). Subsequently, driven by the competitive market environment as well as
various technological and customer pressures, other types of virtual banking
services have grown in prominence throughout the world. The Reserve Bank of India
has been taking a number of initiatives, which will facilitate the active
involvement of commercial banks in the sophisticated cash management system. One
of the pre-requisites to ensure faster and reliable mobility of funds in a country
is to have an efficient payment system. Considering the importance of speed in
payment system to the economy, the RBI has taken numerous measures since mid
Eighties to strengthen the payments mechanism in the country. Introduction of
computerized settlement of clearing transactions, use of Magnetic Ink Character
Recognition (MICR) technology, provision of inter-city clearing facilities and
high value clearing facilities, Electronic Clearing Service Scheme (ECSS),
Electronic Funds Transfer (EFT) scheme, Delivery vs. Payment (DVP) for Government
securities transactions, setting up of Indian Financial Network (INFINET) are some
of the significant developments.

7.54
Management of Working Capital

Introduction of Centralised Funds Management System (CFMS), Securities Services


System (SSS), Real Time Gross Settlement System (RTGS) and Structured Financial
Messaging System (SFMS) are the other top priority items on the agenda to
transform the existing system into a state of the art payment infrastructure in
India. The current vision envisaged for the payment systems reforms is one, which
contemplates linking up of at least all important bank branches with the domestic
payment systems network thereby facilitating cross border connectivity. With the
help of the systems already put in place in India and which are coming into being,
both banks and corporates can exercise effective control over the cash management.
Advantages

The advantages of virtual banking services are as follows: ♦ Lower cost of


handling a transaction. ♦ The increased speed of response to customer
requirements. ♦ The lower cost of operating branch network along with reduced
staff costs leads to cost efficiency. ♦ Virtual banking allows the possibility of
improved and a range of services being made available to the customer rapidly,
accurately and at his convenience. The popularity which virtual banking services
have won among customers, is due to the speed, convenience and round the clock
access they offer.
2.7 CASH MANAGEMENT SERVICES – THE ICICI BANK WAY

ICICI Bank offers receivables and payable management solutions to companies under
its Cash Management Services. Local cheque collection is offered from more than
630 centres and up country cheques payable at more than 4,500 centres. The cheques
can be collected and credited to the centralized account. Companies can plan their
expenditure/investments as the credit is provided on a guaranteed day arrangement
basis. The vast reach is offered in combination with advanced technology, which
enables the companies to receive customized MIS through e-mail and web to take
care of reconciliation. Companies can also avail of ICICI Bank’s payment products
viz, issue of bulk demand drafts/pay orders, cheque writing, RTGS, ECS and
dividend/interest warrants. These products help the companies to reduce
administrative cost and improve efficiency. Companies can avoid transit delays and
courier costs by using the remote printing (pay orders) facility.
(Source: The Economic Times, New Delhi May 2, 2006)

7.55
Financial Management

2.8

MANAGEMENT OF MARKETABLE SECURITIES

Management of marketable securities is an integral part of investment of cash as


this may serve both the purposes of liquidity and cash, provided choice of
investment is made correctly. As the working capital needs are fluctuating, it is
possible to park excess funds in some short term securities, which can be
liquidated when need for cash is felt. The selection of securities should be
guided by three principles. ♦ Safety: Return and risks go hand in hand. As the
objective in this investment is ensuring liquidity, minimum risk is the criterion
of selection. ♦ Maturity: Matching of maturity and forecasted cash needs is
essential. Prices of long term securities fluctuate more with changes in interest
rates and are therefore, more risky. ♦ Marketability: It refers to the
convenience, speed and cost at which a security can be converted into cash. If the
security can be sold quickly without loss of time and price it is highly liquid or
marketable. The choice of marketable securities is mainly limited to Government
treasury bills, Deposits with banks and Intercorporate deposits. Units of Unit
Trust of India and commercial papers of corporates are other attractive means of
parking surplus funds for companies along with deposits with sister concerns or
associate companies. Besides this Money Market Mutual Funds (MMMFs) have also
emerged as one of the avenues of short-term investment. They focus on short-term
marketable securities such as Treasury bills, commercial papers certificate of
deposits or call money market. There is a lock in period of 30 days after which
the investment may be converted into cash. They offer attractive yields, and are
popular with institutional investors and some big companies.

7.56
Management of Working Capital

UNIT – III : MANAGEMENT OF INVENTORY

3.1

INVENTORY MANAGEMENT

Inventories constitute a major element of working capital. It is, therefore,


important that investment in inventory is property controlled. The objectives of
inventory management are, to a great extent, similar to the objectives of cash
management. Inventory management covers a large number of problems including
fixation of minimum and maximum levels, determining the size of inventory to be
carried, deciding about the issues, receipts and inspection procedures,
determining the economic order quantity, proper storage facilities, keeping check
over obsolescence and ensuring control over movement of inventories. The aspects
concerning control over inventories have been discussed in Chapters 1 and 2 of
Cost Accounting under Section A of this book.

7.57
Financial Management

UNIT – IV : MANAGEMENT OF RECEIVABLES

4.1

INTRODUCTION

A firm needs to offer its goods and services on credit to customers as a Business
strategy to boost the sales. This represents a considerable investment of funds so
the management of this asset can have significant effect on the profit performance
of the company. The basic objective of management of sundry debtors is to optimise
the return on investment on this assets known as receivables. Large amounts are
tied up in sundry debtors, there are chances of bad debts and there will be cost
of collection of debts. On the contrary, if the investment in sundry debtors is
low, the sales may be restricted, since the competitors may offer more liberal
terms. Therefore, management of sundry debtors is an important issue and requires
proper policies and their implementation. While studying management of accounts
receivable, we focus on its importance, what determines the investment in it, what
are the decision variables involved and how do we determine them. Investment in
accounts receivables constitute a substantial portion of a firms assets. Moreover,
since cash flows from a sale cannot be invested until the accounts receivable are
collected their control warrants added importance, efficient collection will lead
to both profitability and liquidity of the firm.
4.2 ROLE TO BE PLAYED BY THE FINANCE MANAGER:

It is possible that the finance manager can affect the volume of credit sales and
collection period and consequently, the investment in receivables. That is through
the changes in credit policy. The term credit policy is used to refer to the
combination of three decisions variables: (i) credit standards; (ii) Credit terms;
and (iii) Collection efforts. Credit standards refer to the criteria’s to decide
the types of customers to whom goods could be sold on credit. Whereas credit terms
specify the duration of credit and terms of payments by customers. Collection
efforts determine the actual collection period. The lower the collection period,
the lower the investment in accounts receivables.
4.3 ASPECTS OF MANAGEMENT OF DEBTORS

There are basically three aspects of management of sundry debtors.

7.58
Management of Working Capital

1. Credit policy: The credit policy is to be determined. It involves a trade off


between the profits on additional sales that arise due to credit being extended on
the one hand and the cost of carrying those debtors and bad debt losses on the
other. This seeks to decide credit period, cash discount and other relevant
matters. The credit period is generally stated in terms of net days. For example
if the firm’s credit terms are “net 50”. It is expected that customers will repay
credit obligations not later than 50 days.

Further, the cash discount policy of the firm specifies: (a) The rate of cash
discount. (b) The cash discount period; and (c) The net credit period. For
example, the credit terms may be expressed as “3/15 net 60”. This means that a 3%
discount will be granted if the customer pays within 15 days; if he does not avail
the offer he must make payment within 60 days.
2. Credit Analysis: This require the finance manager to determine as to how risky
it is to advance credit to a particular party. 3. Control of receivable: This
requires finance manager to follow up debtors and decide about a suitable credit
collection policy. It involves both laying down of credit policies and execution
of such policies.

There is always cost of maintaining receivables which comprises of following


costs: (i) (ii) The company requires additional funds as resources are blocked in
receivables which involves a cost in the form of interest (loan funds) or
opportunity cost (own funds) Administrative costs which include record keeping,
investigation of credit worthiness etc.

(iii) Collection costs. (iv) Defaulting costs.


4.4 FACTORS DETERMINING CREDIT POLICY

The credit policy is an important factor determining both the quantity and the
quality of accounts receivables. Various factors determine the size of the
investment a company makes in accounts receivables. They are, for instance: (i)
(ii) The effect of credit on the volume of sales; Credit terms;

(iii) Cash discount; (iv) Policies and practices of the firm for selecting credit
customers.

7.59
Financial Management

(v) Paying practices and habits of the customers. (vi) The firm’s policy and
practice of collection. (vii) The degree of operating efficiency in the billing,
record keeping and adjustment function, other costs such as interest, collection
costs and bad debts etc., would also have an impact on the size of the investment
in receivables. The rising trend in these costs would depress the size of
investment in receivables. The firm may follow a lenient or a stringent credit
policy. The firm which follows a lenient credit policy sells on credit to
customers on very liberal terms and standards. On the contrary a firm following a
stringent credit policy sells on credit on a highly selective basis only to those
customers who have proper credit worthiness and who are financially sound. Any
increase in accounts receivables that is, additional extension of trade credit not
only results in higher sales but also requires additional financing to support the
increased investment in accounts receivables. The costs of credit investigations
and collection efforts and the chances of bad debts are also increased.
4.5 FACTORS UNDER THE CONTROL OF THE FINANCE MANAGER

The finance manager has operating responsibility for the management of the
investment in receivables. In addition to his role of supervising the
administration of credit, the finance manager is in a particularly strategic
position to contribute to top management decisions relating to the best credit
policies of the firm. In the beginning, the finance manager plays a very important
role in the determination of credit period and deciding the criteria for selection
of credit applications. Once it has been done and the management has determined
the role of credit in the package of goods and services offered, the finance
manager has relatively little impact upon the level of receivables. He may,
however, limit the amount of receivables by rejecting occasionally credit
applications or he may speed up the conversion of receivables into cash by
aggressive collection policy. But these activities have smaller effect upon the
level of receivables than the initial and fundamental decision regarding the terms
of credit and the overall credit standards laid down by the firm. The basic
objective of receivables management should be to maximize return on total
investment. The policies which stress short credit terms, stringent credit
standards, and aggressive collection policies would, no doubt, reduce the size of
investment in receivables and also minimize bad debt losses, but such policies
would also restrict sales and profit margins. As a consequence, despite the low
investment in receivables, the rate of return on total investment of the firm
would be lower than that attainable with higher levels of sales, profits and
receivables. The finance manager has to strike a balance between the cost of
increased investment in receivables and profits from the higher levels of sales.
7.60
Management of Working Capital

Illustration 1

A trader whose current sales are in the region of Rs.6 lakhs per annum and an
average collection period of 30 days wants to pursue a more liberal policy to
improve sales. A study made by a management consultant reveals the following
information:Credit Policy Increase in collection period Increase in sales Present
default anticipated

A B C D

10 days 20 days 30 days 45 days

Rs.30,000 Rs.48,000 Rs.75,000 Rs.90,000

1.5% 2% 3% 4%

The selling price per unit is Rs.3. Average cost per unit is Rs.2.25 and variable
costs per unit are Rs.2. The current bad debt loss is 1%. Required return on
additional investment is 20%. Assume a 360 days year. Which of the above policies
would you recommend for adoption?
Solution Evaluation of Credit Policies Part I Credit Policy Exiting A B C D

Credit Period (Days) Expected (Rs.) additional sales

30

40 30,000 10,000

50 48,000 16,000

60 75,000 25,000

75 90,000 30,000

Contribution of additional sales (one-third of selling price) Bad debs (Expected


Sales × Default percentage) Additional bad debts 6,000 --

9,450 3,450

12,960 6,960

20,250 14,250

27,600 21,600

7.61
Financial Management

Contribution of additional sales less additional bad debts (A)


Part II

__

6,550

9,040

10,750

8,400

Expected sales (Rs.) Receivables turnover ratio Average receivables Investment in


receivables (Receivables × Variable cost i.e, two-thirds of sales price i.e.
Rs.50,000 × 2/3 = Rs.33,333 and so on) Additional investment receivables in

6,00,000 12 50,000

6,30,000 9 70,000

6,48,000 7.2 90,000

6,75,000 6 1,12,500

6,90,000 4.8 1,43,750

33,333 __ __

46,667 13,334 2,667

60,000 26,667 5,333

75,000 41,667 8,333

95,833 62,500 12,500

Required return on additional investment at 20% (B) Excess of additional


contribution over required return on additional investment in receivables (A)-(B)

__

3,883

3,707

2,417

(4,100)

The additional contribution over required return on additional investment in


receivables is the maximum under Credit Policy A. Hence, Policy A is recommended
for adoption followed by B and C. Policy D cannot be adopted because it would
result in the reduction of the existing profits.
Illustration 2
XYZ Corporation is considering relaxing its present credit policy and is in the
process of evaluating two proposed policies. Currently, the firm has annual credit
sales of Rs.50 lakhs and accounts receivable turnover ratio of 4 times a year. The
current level of loss due to bad debts is Rs.1,50,000. The firm is required to
give a return of 25% on the investment in new accounts receivables. The company’s
variable costs are 70% of the selling price. Given the following information,
which is the better option?

7.62
Management of Working Capital

(Amount in Rs.) Present Policy Policy Option I Policy Option I

Annual credit sales Accounts receivable turnover ratio Bad debt losses
Solution

50,00,000 4 times 1,50,000

60,00,000 3 times 3,00,000

67,50,000 2.4 times 4,50,000

XYZ CORPORATION Evaluation of Credit Policies (Amount in Rs.) Present Policy


Policy Option I Policy Option II

Annual credit sales Accounts receivable turnover Average collection period Average
level of accounts receivable Marginal increase in investment in receivable less
profit margin Marginal increase in sales Profit on marginal increase in sales
(30%) Marginal increase in bad debt losses Profit on marginal increase in sales
less marginal bad debts loss Required return on marginal investment @ 25% Surplus
(loss) after required rate of return

50,00,000 4 times 3 months 12,50,000 ___ ___ ___ ___ ___ ___ ___ ___

60,00,000 3 times 4 months 20,00,000 5,25,000 10,00,000 3,00,000 1,50,000 1,50,000


1,31,250 18,750

67,50,000 2.4 times 5 months 28,12,500 5,68,750 7,50,000 2,25,000 1,50,000 75,000
1,42,188 (67,188)

7.63
Financial Management

It is clear from the above that the policy option I has a surplus of Rs.18,750/-
whereas option II shows a deficit of Rs.67,188/- on the basis of 25% return. Hence
policy option I is better.
Illustration 3

As a part of the strategy to increase sales and profits, the sales manager of a
company proposes to sell goods to a group of new customers with 10% risk of non-
payment. This group would require one and a half months credit and is likely to
increase sales by Rs.1,00,000 p.a. Production and Selling expenses amount to 80%
of sales and the income-tax rate is 50%. The company’s minimum required rate of
return (after tax) is 25%. Should the sales manager’s proposal be accepted? Also
find the degree of risk of non-payment that the company should be willing to
assume if the required rate of return (after tax) were (i) 30%, (ii) 40% and (iii)
60%.
Solution Extension of credit to a group of new customers:

Profitability of additional sales: Increase in sales per annum Less Bad debt
losses (10%) of sales Net sales revenue Less Production and selling expenses (80%
of sales) Profit before tax Less Income tax (50%) Profit after tax
Average investment in additional receivables.

Rs.

1,00,000 10,000 90,000 80,000 10,000 5,000 5,000

Period of credit: 1 1 2 months Receivables turnover: Average amount of


receivables: Average investment in receivables:

12 =8 1 12 Rs.1,00,000 = Rs.12,500 8
Rs.12,500 × 80% = Rs.10,000

7.64
Management of Working Capital

The available rate of return:

Rs. 5,000 × 100 = 50% Rs.10,000

Since the available rate of return is 50%, which is higher than the required rate
of return of 25%, the Sales Manager’s proposal should be accepted. (i) Acceptable
degree of risk of non-payment if the required rate of return (after tax is 30%)
Required amount of profit after tax on investment: Rs.10,000 × 30% = Rs.3,000
Required amount of profit before tax at this level:

Rs.3,000×100 = Rs.6,000 50
Net sales revenue required: Rs.80,000 + Rs.6,000 = Rs.86,000 Acceptable amount of
bad debt losses: Rs.1,00,000 – Rs.86,000 = Rs.14,000 Acceptable degree of risk of
non-payment:
Rs.14,000 x 100 = 14% Rs.1,00,000

(ii)

Acceptable degree of risk of non-payment if the required rate of return (after


tax) is 40%: Required amount of profit after tax on investment: Rs.10,000 × 40% =
Rs.4,000 Required amount of profit before tax

Rs. 4,000 x 100 = Rs.8,000 50


Net sales revenue required: Rs.80,000 + Rs.8,000 = Rs.88,000 Acceptable amount of
bad debt losses: Rs.1,00,000 – Rs.88,000 = Rs.12,000 Acceptable degree of risk of
non-payment:

7.65
Financial Management

Rs.12,000 ×100 = 12% 1,00,000

(iii) Acceptable degree of risk of non-payment of the required rate of return


(after tax) is 60%: Required amount of profit after tax on investment: Rs.10,000 ×
60% = Rs.6,000 Required amount of profit before tax:

Rs. 6,000×100 = Rs.12,000 50


Net sales revenue required: Rs.80,000 + Rs.12,000 = Rs.92,000 Acceptable amount of
bad debt losses: Rs.1,00,000 – Rs.92,000 = Rs.8,000 Acceptable degree of risk of
non-payment:
Rs.8,000 ×100 = 8% Rs.1,00,000

Illustration 4

Slow Payers are regular customers of Goods Dealers Ltd., Calcutta and have
approached the sellers for extension of a credit facility for enabling them to
purchase goods from Goods Dealers Ltd. On an analysis of past performance and on
the basis of information supplied, the following pattern of payment schedule
emerges in regard to Slow Payers: Schedule At the end of 30 days At the end of 60
days At the end of 90 days At the end of 100 days Pattern 15% of the bill 34% of
the bill. 30% of the bill. 20% of the bill.

Non-recovery 1% of the bill. Slow Payers want to enter into a firm commitment for
purchase of goods of Rs.15 lakhs in 2005, deliveries to be made in equal
quantities on the first day of each quarter in the calendar year. The price per
unit of commodity is Rs.150 on which a profit of Rs.5 per unit is expected to be
made. It is anticipated by Goods Dealers Ltd., that taking up of this contract
would mean
7.66
Management of Working Capital

an extra recurring expenditure of Rs.5,000 per annum. If the opportunity cost of


funds in the hands of Goods Dealers is 24% per annum, would you as the finance
manager of the seller recommend the grant of credit to Slow Payers? Workings
should form part of your answer. Assume year of 360 days.
Solution Evaluation of Extension of Credit Facility to Slow Payers:

(i) (ii)

Anticipated Return on the Contract

Rs.

⎛ Rs.15,00,000 ⎞ ×5 ⎟ Margin return: ⎜ 150 ⎝ ⎠


Less: Recurring annual costs

50,000 5,000 ______ 45,000 1st January, 3,75,000


Period (Days) Products for each quarter

Net anticipated return (ii) Quarterly sales value of the goods to be delivered on
⎛ Rs.15,00,000 ⎞ 1st April, 1st July nd 1st October: ⎜ ⎟ 4 ⎝ ⎠
Amount due for each quarter

(iii)

Opportunity Cost (Interest Cost) of Funds to be Locked up:

Rs.56,250 (15% of Rs.3,75,000) Rs.1,27,500 (34% of Rs.3,75,000). Rs.1,12,500 (30%


of Rs.3,75,000) Rs.75,000 (20% of Rs.3,75,000) Rs.3,750 (1% of Rs.3,75,000)

30 days 60 days 90 days 100 days Non recovery (See Note 1) Total Products

16,87,500 76,50,000 1,01,25,000 75,00,000

2,69,62,500

Amount of interest cost for the year @ 24% p.a.: ⎛ 2,69,62,500 24 ⎞ × ×4⎟ ⎜ 360
100 ⎠ ⎝

71,900

7.67
Financial Management

(iv) (v)

Total Non-recovery of Bad Debts for the year: (Rs.3,750 × 4) Profitability of


Proposed Grant of Credit Facility: Net anticipated return from sales

15,000

45,000

Less: Interest cost on funds :Locked up 71,900 86,900 …….. Bad debts 15,000
(41,900) Profits (Loss) In the light of the above the finance manager will not
recommend the grant of credit facility to Slow Payers as it is not profitable.
Note:(i) Interest cost could be calculated on the amount of bad debts also.

(ii) Interest cost could be calculated on the amount of cost of sales instead of
sales value.
4.6 FINANCING RECEIVABLES

Pledging of accounts receivables and Factoring have emerged as the important


sources of financing of accounts receivables now a days.
(i) Pledging: This refers to the use of a firm’s receivable to secure a short term
loan. A firm’s receivables can be termed as its most liquid assets and this serve
as prime collateral for a secured loan. The lender scrutinizes the quality of the
accounts receivables, selects acceptable accounts, creates a lien on the
collateral and fixes the percentage of financing receivables which ranges around
50 to 90%. The major advantage of pledging accounts receivables is the ease and
flexibility it provides to the borrower. Moreover, financing is done regularly.
This, however, suffers on account of high cost of financing. (ii) Factoring:
Factoring is a new concept in financing of accounts receivables. This refers to
out right sale of accounts receivables to a factor or a financial agency. A factor
is a firm that acquires the receivables of other firms. The factoring lays down
the conditions of the sale in a factoring agreement. The factoring agency bears
the right of collection and services the accounts for a fee.

Normally, factoring is the arrangement on a non-recourse basis where in the event


of default the loss is borne by this factor. However, in a factoring arrangement
with recourse, in such situation, the accounts receivables will be turned back to
the firm by the factor for resolution. There are a number of financial
distributors providing factoring services in India. Some

7.68
Management of Working Capital

commercial banks and other financial agencies provide this service. The biggest
advantages of factoring are the immediate conversion of receivables into cash and
predicted pattern of cash flows. Financing receivables with the help of factoring
can help a company having liquidity without creating a net liability on its
financial condition. Besides, factoring is a flexible financial tool providing
timely funds, efficient record keepings and effective management of the collection
process. This is not considered to be as a loan. There is no debt repayment, no
compromise to balance sheet, no long term agreements or delays associated with
other methods of raising capital. Factoring allows the firm to use cash for the
growth needs of business.
Illustration 5

A Factoring firm has credit sales of Rs.360 lakhs and its average collection
period is 30 days. The financial controller estimates, bad debt losses are around
2% of credit sales. The firm spends Rs.1,40,000 annually on debtors
administration. This cost comprises of telephonic and fax bills along with
salaries of staff members. These are the avoidable costs. A Factoring firm has
offered to buy the firm’s receivables. The factor will charge 1% commission and
will pay an advance against receivables on an interest @15% p.a. after withholding
10% as reserve. What should the firm do? Assume 360 days in a year.
Solution

Average level of receivables = Rs.360 lakhs ×

30 = 30 lakhs 360
Rs.30,000 Rs.3,00,000 Rs.3,30,000 Rs.30,00,000 Rs. 3,30,000 Rs.26,70,000 Rs.
33,375 Rs.26,36,625 = =

Factoring Commission = 1% of Rs.30,00,000 = Reserve = 10% of Rs.30,00,000 Total


(i) Thus, the amount available for advance is Average level of receivables
Less: Total (i) from above

(ii)
Less: Interest @ 15% p.a. for 30 days

Net Amount of Advance available.

7.69
Financial Management

Evaluation of Factoring Proposal

Cost to the Firm Factoring Commission = Rs.30,00,000 × Interest charges = Rs.


33,375 × Savings to the firm
Rs.

360 = Rs. 3,60,000 30

360 Rs.4,00,500 = 30 Rs.7,60,500

Cost of credit administration Cost of bad-debt losses, 0.02 × 360 lakhs


∴ The Net benefit to the firm

1,40,000 7,20,000 8,60,000


Rs.

Savings to the firm - Cost to the firm Net Savings

8,60,000 7,60,500 99,500

Conclusion: Since the savings to the firm exceeds the cost to the firm on account
of factoring, ∴The proposal is acceptable. 4.7 INNOVATIONS IN RECEIVABLE
MANAGEMENT

During the recent years, a number of tools, techniques, practices and measures
have been invented to increase effectiveness in accounts receivable management.
Following are the major determinants for significant innovations in accounts
receivable management and process efficiency.
1. Re-engineering Receivable Process: In some of the organizations real cost
reductions and performance improvements have been achieved by re-engineering in
accounts receivable process. Re-engineering is a fundamental re-think and re-
design of business processes by incorporating modern business approaches. The
nature of accounts receivables is such that decisions made elsewhere in the
organization are likely to affect the level of resources that are expended on the
management of accounts receivables.

The following aspects provides an opportunity to improve the management of


accounts receivables.
7.70
Management of Working Capital

(a) Centralisation: Centralisation of high nature transactions of accounts


receivables and payable is one of the practice for better efficiency. This focuses
attention on specialized groups for speedy recovery. (b) Alternative Payment
Strategies: Alternative payment strategies in addition to traditional practices,
result into efficiencies in the management of accounts receivables. It is observed
that payment of accounts outstanding is likely to be quicker where a number of
payment alternatives are made available to customers. Besides, this convenient
payment methods is a marketing tool that is of benefit in attracting and retaining
customers. The following alternative modes of payment may also be used alongwith
traditional methods like Cheque Book etc., for making timely payment, added
customer service, reducing remittance processing costs and improved cash flows and
better debtor turnover.

(i) (ii)

Direct debit: I.e., authorization for the transfer of funds from the purchasers
bank account. Integrated Voice Response: This system uses human operators and a
computer based system to allow customers to make payment over phone, generally by
credit card. This system has proved to be beneficial in the orgnisations
processing a large number of payments regularly.

(iii) Collection by a third party: The payment can be collected by an authorized


external firm. The payments can be made by cash, cheque, credit card or Electronic
fund transfer. Banks may also be acting as collecting agents of their customers
and directly depositing the collections in customers bank accounts. (iv) Lock Box
Processing: Under this system an outsourced partner captures cheques and invoice
data and transmits the file to the client firm for processing in that firm’s
systems. (v) Payments via Internet.
(c) Customer Orientation: Where individual customers or a group of customers have
some strategic importance to the firm a case study approach may be followed to
develop good customer relations. A critical study of this group may lead to
formation of a strategy for prompt settlement of debt. 2. Evaluation of Risk: Risk
evaluation is a major component in the establishment of an effective control
mechanism. Once risks have been properly assessed controls can be introduced to
either contain the risk to an acceptable level or to eliminate them entirely. This
also provides an opportunity for removing inefficient practices. This involves a
rethink of processes and questioning the way that tasks are performed. This also
opens

7.71
Financial Management

the way for efficiency and effectiveness benefits in the management of accounts
receivables.
3. Use of Latest Technology: Technological developments now-a-days provides an
opportunity for improvement in accounts receivables process. The major innovations
available are the integration of systems used in the management of accounts
receivables, the automation and the use of e-commerce. (a) E-commerce refer to the
use of computer and electronic telecommunication technologies, particularly on an
inter-organisational level, to support trading in goods and services. It uses
technologies such as Electronic Data Inter-change (EDI), Electronic Mail,
Electronic Funds Transfer (EFT) and Electronic Catalogue Systems to allow the
buyer and seller to transact business by exchange of information between computer
application systems. (b) Accounts Receivable Systems: Now-a-days all the big
companies develop and maintain automated receivable management systems. Manual
systems of recording the transactions and managing receivables is not only
cumbersome but ultimately costly also. These integrated systems automatically
update all the accounting records affected by a transaction. For example, if a
transaction of credit sale is to be recorded, the system increases the amount the
customer owes to the firm, reduces the inventory for the item purchased, and
records the sale. This system of a company allows the application and tracking of
receivables and collections, using the automated receivables system allows the
company to store important information for an unlimited number of customers and
transactions, and accommodate efficient processing of customer payments and
adjustments. 4. Receivable Collection Practices: The aim of debtors collection
should be to reduce, monitor and control the accounts receivable at the same time
maintain customer goodwill. The fundamental rule of sound receivable management
should be to reduce the time lag between the sale and collection. Any delays that
lengthen this span causes receivables to unnecessary build up and increase the
risk of bad debts. This is equally true for the delays caused by billing and
collection procedures as it is for delays caused by the customer.

The following are major receivable collection procedures and practices: (i) (ii)
Issue of Invoice. Open account or open-end credit.

(iii) Credit terms or time limits. (iv) Periodic statements. (v) Use of payment
incentives and penalties.
7.72
Management of Working Capital

(vi) Record keeping and Continuous Audit. (vii) Export Factoring: Factors provide
comprehensive credit management, loss protection collection services and provision
of working capital to the firms exporting internationally. (viii) Business Process
Outsourcing: This refers to a strategic business tool whereby an outside agency
takes over the entire responsibility for managing a business process.
5. Use of Financial tools/techniques: The finance manager while managing accounts
receivables uses a number of financial tools and techniques. Some of them have
been described hereby as follows: (i) Credit analysis: While determining the
credit terms, the firm has to evaluate individual customers in respect of their
credit worthiness and the possibility of bad debts. For this purpose, the firm has
to ascertain credit rating of prospective customers. Credit rating: An important
task for the finance manager is to rate the various debtors who seek credit
facility. This involves decisions regarding individual parties so as to ascertain
how much credit can be extended and for how long. In foreign countries specialized
agencies are engaged in the task of providing rating information regarding
individual parties. Dun and Broadstreet is one such source.

The finance manager has to look into the credit-worthiness of a party and sanction
credit limit only after he is convinced that the party is sound. This would
involve an analysis of the financial status of the party, its reputation and
previous record of meeting commitments. The credit manager here has to employ a
number of sources to obtain credit information. The following are the important
sources: Trade references; Bank references; Credit bureau reports; Past
experience; Published financial statements; and Salesman’s interview and reports.
Once the credit-worthiness of a client is ascertained, the next question is to set
a limit of the credit. In all such enquiries, the credit manager must be discreet
and should always have the interest of high sales in view.
(ii) Decision tree analysis of granting credit: The decision whether to grant
credit or not is a decision involving costs and benefits. When a customer pays,
the seller makes profit but when he fails to pay the amount of cost going into the
product is also gone. If the relative chances of recovering the dues can be
decided it can form a probability distribution of payment or non-payment. If the
chances of recovery are 9 out of 10 then probability of recovery is 0.9 and that
of default is 0.1.

7.73
Financial Management

Credit evaluation of a customer shows that the probability of recovery is 0.9 and
that of default is 0.1. the revenue from the order is Rs.5 lakhs and cost is Rs.4
lakhs. The decision is whether credit should be granted or not. The analysis is
presented in the following diagram.
Rs.1,00,000.00 Grant

Rs.4,00,000.00

Do not grant

The weighted net benefit is Rs.[1,00,000 × 0.9 i.e. 90,000 – 0.1 × 4,00,000 i.e.
40,000] = 50,000. So credit should be granted.
(iii) Control of receivables: Another aspect of management of debtors is the
control of receivables. Merely setting of standards and framing a credit policy is
not sufficient; it is, equally important to control receivables. (iv) Collection
policy: Efficient and timely collection of debtors ensure that the bad debt losses
are reduced to the minimum and the average collection period is shorter. If a firm
spends more resources on collection of debts, it is likely to have smaller bad
debts. Thus, a firm must work out the optimum amount that it should spend on
collection of debtors. This involves a trade off between the level of expenditure
on the one hand and decrease in bad debt losses and investment in debtors on the
other.

The collection cell of a firm has to work in a manner that it does not create too
much resentment amongst the customers. On the other hand, it has to keep the
amount of the outstandings in check. Hence, it has to work in a very smoothen
manner and diplomatically. It is important that clear-cut procedures regarding
credit collection are set up. procedures must answer questions like the following:
Such

(a) How long should a debtor balance be allowed to exist before collection process
is started. (b) What should be the procedure of follow up with defaulting
customer? How reminders are to be sent and how should each successive reminder be
drafted? (c) Should there be a collection machinery whereby personal calls by
company’s representatives are made?

7.74
Management of Working Capital

(d) What should be the procedure for dealing with doubtful accounts? Is legal
action to be instituted? How should account be handled?
4.8 (i) MONITORING OF RECEIVABLES Computation of average age of receivables: It
involves computation of average collection period.

(ii) Ageing Schedule: When receivables are analysed according to their age, the
process is known as preparing the ageing schedules of receivables. The computation
of average age of receivables is a quick and effective method of comparing the
liquidity of receivables with the liquidity of receivables in the past and also
comparing liquidity of one firm with the liquidity of the other competitive firm.
It also helps the firm to predict collection pattern of receivables in future.
This comparison can be made periodically. The purpose of classifying receivables
by age groups is to have a closer control over the quality of individual accounts.
It requires going back to the receivables ledger where the dates of each
customer’s purchases and payments are available. The ageing schedule, by
indicating a tendency for old accounts to accumulate, provides a useful supplement
to average collection period of receivables/sales analysis. Because an analysis of
receivables in terms of associated dates of sales enables the firm to recognise
the recent increases, and slumps in sales. To ascertain the condition of
receivables for control purposes, it may be considered desirable to compare the
current ageing schedule with an earlier ageing schedule in the same firm and also
to compare this information with the experience of other firms. The following is
an illustration of the ageing schedule of receivables:Ageing Schedule Age Classes
(Days) As on 30th June, 2005 Month of Sale Balance of Percentage Receivables to
total (Rs.) 41,500 11.9 74,200 21.4 1,85,600 53.4 35,300 10.2 10,800 3.1 As on
30th September, 2005 Month Sale of Balance of Percentage Receivables to total
(Rs.) 1,00,000 22.7 2,50,000 56.8 48,000 10.9 40,000 9.1 2,000 0.5

1-30 31-60 61-90 91-120 121 and more

June May April March Earlier

September August July June Earlier

_______ 3,47,400

___ 100

_______ 4,40,000

___ 100

7.75
Financial Management

The above ageing schedule shows a substantial improvement in the liquidity of


receivables for the quarter ending September, 2005 as compared with the liquidity
of receivables for the quarter ending June, 2005. It could be possible due to
greater collection efforts of the firm.
(iii) Collection Programme:

(a) Monitoring the state of receivables. (b) Intimation to customers when due date
approaches. (c) Telegraphic and telephonic advice to customers on the due date.
(d) Threat of legal action on overdue A/cs. (e) Legal action on overdue A/cs. The
following diagram shows the relationship between collection expenses and bad debt
losses which has to be established as initial increase in collection expenses may
have only a small impact on bad debt losses.

7.76
Management of Working Capital

UNIT – V : FINANCING OF WORKING CAPITAL

5.1

INTRODUCTION

After determining the amount of working capital required, the next step to be
taken by the finance Manager is to arrange the funds. As discussed earlier, it is
advisable that the finance manager bifurcates the working capital requirements
between the permanent working capital and temporary working capital. The permanent
working capital is always needed irrespective of sales fluctuations, hence should
be financed by the long-term sources such as debt and equity. On the contrary the
temporary working capital may be financed by the short-term sources of finance.
The short-term sources of finance, which are generally expected to be matured
within the same operating cycle or say within the same accounting year or at the
most in next year, finance a major portion of total current assets. This requires
a number of decisions to be taken by the finance manager with regard to the Cash
Balance and the timing of cash to be maintained, investment in short-term
securities, when the payment to creditors is to be made, when and how much funds
are to be raised by borrowings. Most of these sources are not often close
substitute for one another because each source has unique characteristics,
advantages and disadvantages. The present unit, focuses on (i) the different
sources of financing working capital requirements as well as recent developments.
Broadly speaking, the working capital finance may be classified between the two
categories: (i) (ii) Spontaneous sources. Negotiable sources.

The finance manager has to be very careful while selecting a particular source, or
a combination thereof for financing of working capital. Generally, the following
parameters will guide his decisions in this respect: (i) (ii) Cost factor Impact
on credit rating

(iii) Feasibility (iv) Reliability (v) Restrictions (vi) Hedging approach or


matching approach i.e., Financing of assets with the same maturity as of assets.
7.77
Financial Management

The spontaneous sources of finance are those which naturally arise in the course
of business operations. Trade credit, credit from employees, credit from suppliers
of services, etc. Are some of the examples which may be quoted in this respect. On
the other hand the negotiated sources, as the name implies, are those which have
to be specifically negotiated with lenders say, commercial banks, financial
institutions, general public etc.
5.2 SOURCES OF FINANCE

5.2.1 As outlined above trade credit is a spontaneous source of finance which is


normally extended to the purchaser organization by the sellers or services
providers. This source of financing working capital is more important since it
contributes to about one-third of the total short-term requirements. The
dependence on this source is higher due to lesser cost of finance as compared with
other sources. Trade credit is guaranteed when a company acquires supplies,
merchandise or materials and does not pay immediately. If a buyer is able to get
the credit with out completing much formalities, it is termed as ‘open account
trade credit.’

On the other hand in the case of “Bills Payable” the purchaser will have to give a
written promise to pay the amount of the bill/invoice either on demand or at a
fixed future date to the seller or the bearer of the note. Due to its simplicity,
easy availability and lesser explicit cost, the dependence on this source is much
more in all small or big organizations. Especially, for small enterprises this
form of credit is more helpful to small and medium enterprises. The amount of such
financing depends on the volume of purchases and the payment timing. Another
spontaneous source of short-term financing is the accrued expenses or the
outstanding expenses liabilities. The accrued expenses refer to the services
availed by the firm, but the payment for which has yet to be made. It is a built
in and an automatic source of finance as most of the services like wages,
salaries, taxes, duties etc., are paid at the end of the period. The accrued
expenses represent an interest free source of finance. There is no explicit or
implicit cost associated with the accrued expenses and the firm can ensure
liquidity by accruing these expenses.
5.2.2 Inter-corporate Loans and Deposits: Sometime, organizations having surplus
funds invest for shot-term period with other organizations. The rate of interest
will be higher than the bank rate of interest and depending on the financial
soundness of the borrower company. This source of finance reduces dependence on
bank financing. 5.2.3 Commercial Papers: Commercial Paper (CP) is an unsecured
promissory note issued by a firm to raise funds for a short period. This is an
instrument that enables highly rated corporate borrowers for short-term borrowings
and provides an additional financial instrument

7.78
Management of Working Capital

to investors with a freely negotiable interest rate. The maturity period ranges
from minimum 7 days to less than 1 year.
5.2.4 Commercial Papers in India: Since the CP represents an unsecured borrowing
in the money market, the regulation of CP comes under the purview of the Reserve
Bank of India which issued guidelines in 1990 on the basis of the recommendations
of the Vaghul Working Group. These guidelines were aimed at:

(i) (ii)

Enabling the highly rated corporate borrowers to diversify their sources of short
term borrowings, and To provide an additional instrument to the short term
investors.

These guidelines have stipulated certain conditions meant primarily to ensure that
only financially strong companies come forward to issue the CP. Subsequently,
these guidelines have been modified. The main features of the guidelines relating
to issue of CP in India may be summarized as follows: (i) CP should be in the form
of usance promissory note negotiable by endorsement and delivery. It can be issued
at such discount to the face value as may be decided by the issuing company. CP is
subject to payment of stamp duty. The aggregate amount that can be raised by
commercial papers is not restricted any longer to the company’s cash credit
component of the Maximum Permissible Bank Finance.

(ii)

(iii) CP is issued in the denomination of Rs.5,00,000, but the maximum lot or


investment is Rs.25,00,000 per investor. The secondary market transactions can be
of Rs.5,00,000 or multiples thereof. The total amount proposed to be issued should
be raised within two weeks from the date on which the proposal is taken on record
by the bank. (iv) CP should be issued for a minimum period of 7 days and a maximum
of less than 1 year. No grace period is allowed for repayment and if the maturity
date falls on a holiday, then it should be paid on the previous working day. Each
issue of CP is treated as a fresh issue. (v) Commercial papers can be issued by a
company whose (i) tangible net worth is not less than Rs.5 crores, (ii) funds
based working capital limit is not less than 4 crores, (iii) shares are listed on
a stock exchange, (iv) specified credit rating of P2 is obtained from CRISIL or A2
from ICRA, and (v) the current ratio is 1.33:1. (vi) The issue expenses consisting
of dealers fees, credit rating agency fees and other relevant expenses should be
borne by the issuing company. (vii) CP may be issued to any person, banks,
companies. The issue of CP to NRIs can only be on a non-repatriable basis and is
not transferable.

7.79
Financial Management

(viii) CP can be issued up to 100% of the fund based working capital loan limit.
The working capital limit is reduced accordingly on issuance of CP. (ix) Deposits
by the issue of CP have been exempted from the provisions of section 58A of the
Companies Act, 1956. Any company proposing to issue CP has to submit an
application to the bank which provide working capital limit to it, along with the
credit rating of the firm. The issue has to be privately placed within two weeks
by the company or through a merchant banker. The initial investor pays the
discounted value of the CP to the firm. Thus, CP is issued only through the bank
who has sanctioned the working capital limit to the company. It is counted as a
part of the total working capital limit and it does not increase the working
capital resources of the firm. Annual Financing Cost = Where FV SP MP = = =

FV − SP 360 x MP SP
Face value of CP Issue price of CP Maturity period of CP.

For example, a CP of the face value of Rs.6,00,000 is issued at Rs.5,80,000 for a


maturity period of 120 days. The annual financing cost of the CP is: Annual
Financing Cost =

Rs. 6,00,000 − Rs. 5,80,000 360 x Rs. 5,80,000 120

= 10.34% In the same case, if the maturity period is 180 days, then the annual
financing cost is: Annual Financing Cost =

Rs. 6,00,000 − Rs. 5,80,000 360 x Rs. 5,80,000 180

= 6.90% For the same maturity periods of 120 days and 180 days, if the issue price
is taken at Rs.5,60,000, then the annual financing cost comes to 21.42% and 14.28%
respectively. So, it can be seen that the cost of CP varies inversely to the issue
price as well as the maturity period.
5.2.5 CP as a Source of Financing: From the point of the issuing company, CP
provides the following benefits:

(a) CP is sold on an unsecured basis and does not contain any restrictive
conditions.

7.80
Management of Working Capital

(b) Maturing CP can be repaid by selling new CP and thus can provide a continuous
source of funds. (c) Maturity of CP can be tailored to suit the requirement of the
issuing firm. (d) CP can be issued as a source of fund even when money market is
tight. (e) Generally, the cost of CP to the issuing firm is lower than the cost of
commercial bank loans. However, CP as a source of financing has its own
limitations: (i) (ii) Only highly credit rating firms can use it. New and
moderately rated firm generally are not in a position to issue CP. CP can neither
be redeemed before maturity nor can be extended beyond maturity.

5.2.6 Funds Generated from Operations: Funds generated from operations, during an
accounting period, increase working capital by an equivalent amount. The two main
components of funds generated from operations are profit and depreciation. Working
capital will increase by the extent of funds generated from operations. Students
may refer to funds flow statement given earlier in this chapter. 5.2.7 Public
Deposits: Deposits from the public is one of the important source of finance
particularly for well established big companies with huge capital base for short
and mediumterm. 5.2.8 Bills Discounting: Bill discounting is recognized as an
important short term Financial Instrument and it is widely used method of short
term financing. In a process of bill discounting, the supplier of goods draws a
bill of exchange with direction to the buyer to pay a certain amount of money
after a certain period, and gets its acceptance from the buyer or drawee of the
bill. 5.2.9 Bill Rediscounting Scheme: The bill rediscounting Scheme was
introduced by Reserve Bank of India with effect from 1st November, 1970 in order
to extend the use of the bill of exchange as an instrument for providing credit
and the creation of a bill market in India with a facility for the rediscounting
of eligible bills by banks. Under the bills rediscounting scheme, all licensed
scheduled banks are eligible to offer bills of exchange to the Reserve Bank for
rediscount. 5.2.10 Factoring: Students may refer to the unit on Receivable
Management wherein the concept of factoring has been discussed. Factoring is a
method of financing whereby a firm sells its trade debts at a discount to a
financial institution. In other words, factoring is a continuous arrangement
between a financial institution, (namely the factor) and a firm (namely the
client) which sells goods and services to trade customers on credit. As per this
arrangement, the factor purchases the client’s trade debts including accounts
receivables

7.81
Financial Management

either with or without recourse to the client, and thus, exercises control over
the credit extended to the customers and administers the sales ledger of his
client. To put it in a layman’s language, a factor is an agent who collects the
dues of his client for a certain fee. The differences between Factoring and Bills
discounting are as follows: (i) (ii) Factoring is called as ‘Invoice factoring’
whereas bills discounting is known as “Invoice discounting”. In factoring the
parties are known as client, factor and debtor whereas in bills discounting they
are known as Drawer, Drawee and Payee.

(iii) Factoring is a sort of management of book debts whereas bills discounting is


a sort of borrowing from commercial banks. (iv) For factoring there is no specific
Act; whereas in the case of bills discounting, the Negotiable Instrument Act is
applicable.
5.3 WORKING CAPITAL FINANCE FROM BANKS

Banks in India today constitute the major suppliers of working capital credit to
any business activity. Recently, some term lending financial institutions have
also announced schemes for working capital financing. The two committees viz.,
Tandon Committee and Chore Committee have evolved definite guidelines and
parameters in working capital financing, which have laid the foundations for
development and innovation in the area.
5.3.1 Instructions on Working Capital Finance by Banks Assessment of Working
Capital

Reserve Bank of India has withdrawn the prescription, in regard to assessment of


working capital needs, based on the concept of Maximum Permissible Bank Finance,
in April 1997. Banks are now free to evolve, with the approval of their Boards,
methods for assessing the working capital requirements of borrowers, within the
prudential guidelines and exposure norms prescribed. Banks, however, have to take
into account Reserve Bank’s instructions relating to directed credit (such as
priority sector, export, etc.), and prohibition of credit (such as bridge finance,
rediscounting of bills earlier discounted by NBFCs) while formulating their
lending policies. With the above liberalizations, all the instructions relating to
MPBF issued by RBI from time to time stand withdrawn. Further, various
instructions/guidelines issued to banks with objective of ensuring lending
discipline in appraisal, sanction, monitoring and utilization of bank finance
cease to be mandatory. However, banks have the option of incorporating such of the
instructions/guidelines as are considered necessary in their lending
policies/procedures.

7.82
Management of Working Capital

5.4

FACTORS DETERMINING CREDIT POLICY Cash Credit: This facility will be given by the
banker to the customers by giving certain amount of credit facility on continuous
basis. The borrower will not be allowed to exceed the limits sanctioned by the
bank. Bank Overdraft: It is a short-term borrowing facility made available to the
companies in case of urgent need of funds. The banks will impose limits on the
amount they can lend. When the borrowed funds are no longer required they can
quickly and easily be repaid. The banks issue overdrafts with a right to call them
in at short notice. Bills Discounting: The company which sells goods on credit,
will normally draw a bill on the buyer who will accept it and sends it to the
seller of goods. The seller, in turn discounts the bill with his banker. The
banker will generally earmarks the discounting bill limit. Bills Acceptance: To
obtain finance under this type of arrangement a company draws a bill of exchange
on bank. The bank accepts the bill thereby promising to pay out the amount of the
bill at some specified future date. Line of Credit: Line of Credit is a commitment
by a bank to lend a certain amount of funds on demand specifying the maximum
amount. Letter of Credit: It is an arrangement by which the issuing bank on the
instructions of a customer or on its own behalf undertakes to pay or accept or
negotiate or authorizes another bank to do so against stipulated documents subject
to compliance with specified terms and conditions. Bank Guarantees: Bank guarantee
is one of the facilities that the commercial banks extend on behalf of their
clients in favour of third parties who will be the beneficiaries of the
guarantees.

The bank credit will generally be in the following forms: •

• •

Self Examination Questions A. Objective Type Questions

1.

The credit terms may be expressed as “3/15 net 60”. This means that a 3% discount
will be granted if the customer pays within 15 days, if he does not avail the
offer he must make payment within 60 days. (a) I agree with the statement (b) I do
not agree with the statement (c) I cannot say.

7.83
Financial Management

2.

The term ‘net 50’ implies that the customer will make payment. (a) Exactly on 50th
day (b) Before 50th day (c) Not later than 50th day (iv) None of the above.

3.

Trade credit is a source of : (a) Long-term finance (b) Medium term finance (c)
Spontaneous source of finance (d) None of the above.

4.

The term float is used in (a) Inventory Management (b) Receivable Management (c)
Cash Management (d) Marketable securities.

5.

William J Baumol’s model of Cash Management determines optimum cash level where
the carrying cost and transaction cost are: (a) Maximum (b) Minimum (c) Medium (d)
None of the above.

6.

In Miller – ORR Model of Cash Management: (a) The lower, upper limit, and return
point of Cash Balances are set out (b) Only upper limit and return point are
decided (c) Only lower limit and return point are decided (d) None of the above
are decided.

7.

Working Capital is defined as (a) Excess of current assets over current


liabilities (b) Excess of current liabilities over current assets

7.84
Management of Working Capital

(c) Excess of Fixed Assets over long-term liabilities (d) None of the above. 8.
Working Capital is also known as “Circulating Capital, fluctuating Capital and
revolving capital”. The aforesaid statement is; (a) Correct (b) Incorrect (c)
Cannot say. 9. The basic objectives of Working Capital Management are: (a) Optimum
utilization of resources for profitability (b) To meet day-to-day current
obligations (c) Ensuring marginal return on current assets is always more than
cost of capital (d) Select any one of the above statement. 10. The term Gross
Working Capital is known as: (a) The investment in current liabilities (b) The
investment in long-term liability (c) The investment in current assets (d) None of
the above. 11. The term net working capital refers to the difference between the
current assets minus current liabilities. (a) The statement is correct (b) The
statement is incorrect (c) I cannot say. 12. The term “Core current assets’ was
coined by (a) Chore Committee (b) Tandon Committee (c) Jilani Committee (d) None
of the above. 13. The concept operating cycle refers to the average time which
elapses between the acquisition of raw materials and the final cash realization.
This statement is

7.85
Financial Management

(a) Correct (b) Incorrect (c) Partially True (d) I cannot say. 14. As a matter of
self-imposed financial discipline can there be a situation of zero working capital
now-a-days in some of the professionally managed organizations. (a) Yes (b) No (c)
Impossible (d) Cannot say. 15. Over trading arises when a business expands beyond
the level of funds available. The statement is (a) Incorrect (b) Correct (c)
Partially correct (d) I cannot say. 16. A Conservative Working Capital strategy
calls for high levels of current assets in relation to sales. (a) I agree (b) Do
not agree (c) I cannot say. 17. The term Working Capital leverage refer to the
impact of level of working capital on company’s profitability. This measures the
responsiveness of ROCE for changes in current assets. (a) I agree (b) Do not agree
(c) The statement is partially true. 18. The term spontaneous source of finance
refers to the finance which naturally arise in the course of business operations.
The statement is (a) Correct

7.86
Management of Working Capital

(b) Incorrect (c) Partially Correct (d) I cannot say. 19. Under hedging approach
to financing of working capital requirements of a firm, each asset in the balance
sheet assets side would be offset with a financing instrument of the same
approximate maturity. This statement is (a) Incorrect (b) Correct (c) Partially
correct (d) I cannot say. 20. Trade credit is a (a) Negotiated source of finance
(b) Hybrid source of finance (c) Spontaneous source of finance (d) None of the
above. 21. Factoring is a method of financing whereby a firm sells its trade debts
at a discount to a financial institution. The statement is (a) Correct (b)
Incorrect (c) Partially correct (d) I cannot say. 22. A factoring arrangement can
be both with recourse as well as without recourse: (a) True (b) False (c)
Partially correct (d) Cannot say. 23. The Bank financing of working capital will
generally be in the following form. Cash Credit, Overdraft, bills discounting,
bills acceptance, line of credit; Letter of credit and bank guarantee. (a) I agree

7.87
Financial Management

(b) I do not agree (c) I cannot say. 24. When the items of inventory are
classified according to value of usage, the technique is known as: (a) XYZ
Analysis (b) ABC Analysis (c) DEF Analysis (d) None of the above. 25. When a firm
advises its customers to mail their payments to special Post Office collection
centers, the system is known as. (a) Concentration banking (b) Lock Box system (c)
Playing the float (d) None of the above.
Answer to Objective Type Questions 1. (a); 2.(c); 3. (c); 4. (c); 5. (b); 6. (a);
7. (a); 8. (a); 9. (b); 10. (c); 11. (a); 12. (b); 13. (a); 14. (a); 15. (b); 16.
(a); 17. (a); 18. (a); 19. (b); 20. (c); 21. (a); 22. (a); 23. (a); 24. (b); 25.
(b). B. Practical Problems

1.

Foods Ltd. is presently operating at 60% level producing 36,000 packets of snack
foods and proposes to increase capacity utilization in the coming year by 33 %
over the existing level of production. The following data has been supplied: (i)
Unit cost structure of the product at current level: Raw Material Wages (Variable)
Overheads (Variable) Fixed Overheads Profit Selling Price
Rs.

4 2 2 1 3 12

7.88
Management of Working Capital

(ii)

Raw materials will remain in stores for 1 month before being issued for
production. Material will remain in process for further 1 month. Suppliers grant 3
months credit to the company.

(iii) Finished goods remain in godown for 1 month. (iv) Debtors are allowed credit
for 2 months. (v) Lag in wages and overhead payments is 1 month and these expenses
accrue evenly throughout the production cycle. (vi) No increase either in cost of
inputs or selling price is envisaged. Prepare a projected profitability statement
and the working capital requirement at the new level, assuming that a minimum cash
balance of Rs.19,500 has to be maintained. 2. A newly formed company has applied
to the commercial bank for the first time for financing its working capital
requirements. The following information is available about the projections for the
current year: Estimated level of activity: 1,04,000 completed units of production
plus 4,000 units of work-in-progress. Based on the above activity estimated cost
per unit is:
(Rs. per unit)

Raw material Direct wages Overheads (exclusive of depreciation) Total cost

80 30 60 170

Selling price 200 Raw materials in stock: average 4 weeks consumption, work-in-
progress (assume 50% completion stage in respect of conversion cost) (materials
issued at the start of the processing). Finished goods in stock Credit allowed by
suppliers Credit allowed to debtors/receivables Lag in payment of wages 8,000
units Average 4 weeks Average 8 weeks Average 1½ weeks

Cash at banks (for smooth operation) is expected to be Rs.25,000 Assume that


production is carried on evenly throughout the year (52 weeks) and wages and
overheads accrue similarly. All sales are on credit basis only. Find out: the net
working capital required.

7.89
Financial Management

3.

The following is the projected Balance Sheet of Excel Limited as on 31-3-2004. The
company wants to increase the fund-based limits from the Zonal Bank from Rs.100
lakhs to Rs.300 lakhs:
Balance Sheet as on 31-3-2004 (Rs. lakhs) Liabilities Rs. Assets Rs.

Share Capital Reserves & Surplus Secured Loans Unsecured Loans Current Liabilities

100 Fixed Assets 150 Current Assets 450 Miscellaneous Expenditure 1,050 200

800 1,000 150

1,950 The following are the other information points to be considered:

1,950

(1) Secured loans include instalments payable to financial institutions before 31-
3-2004 Rs.100 lakhs. (2) Secured loans include working capital facilities expected
from Zonal Bank Rs.300 lakhs. (3) Unsecured loans include fixed deposits from
public amounting to Rs.400 lakhs out of which Rs.100 lakhs are due for repayment
before 31-3-2004. (4) Unsecured loans include Rs.600 lakhs of zero interest fully
convertible debentures due for conversion on 30-9-2003. (5) Current assets include
deferred receivables due for payment after 31-3-2004 Rs.40 lakhs. (6) The company
has introduced a voluntary retirement scheme for workers costing Rs.40 lakhs
payable on 31-3-2008 and this amount is included in current liabilities: (i) You
are required to calculate from the above information the maximum permissible bank
finance by all the three methods for working capital as per Tandon Committee
norms. For your exercise, assume that core current assets constitute 25% of the
current assets. Also compute the Current Ratio for all the three methods.

(ii) 4.

A company newly commencing business in 2003 has the under mentioned Projected
Profit and Loss Account:

7.90
Management of Working Capital

Sales Cost of goods sold


Gross Profit

42,00,000 30,60,000 11,40,000 2,80,000 2,60,000 5,40,000 6,00,000 2,00,000


4,00,000 (Rs) 16,80,000 12,50,000 4,70,000 34,00,000 3,40,000

Administrative expenses Selling expenses Profit before tax Provision for taxation
Profit after tax The cost of goods sold has been arrived at as under

Material used Wages and manufacturing expenses Depreciation


Less: Stock of finished goods (10% of goods produced not yet sold)

30,60,000 The figures given above relate only to finished goods and not to work-
in-progress. Goods equal to 15% of the year’s production (in terms of physical
units) will be in process on the average requiring full materials but only 40% of
the other expenses. The company believes in keeping material equal to two months
consumption in stock. All expenses will be paid one month in arrear. Suppliers of
material will extend 1½ month’s credit; Sales will be 20% for cash and the rest at
two months’ credit; 90% of the Income-tax will be paid in advance in quarterly
instalments. The company wishes to keep Rs.1,00,000 in cash. Prepare an estimate
of the requirement of (i) Working Capital; and (ii) Cash Cost of Working Capital.
5. A company is considering its working capital investment and financial policies
for the next year. Estimated fixed assets and current liabilities for the next
year are Rs.2.60 crore and Rs.2.34 crore respectively. Estimated Sales and EBIT
depend on current assets investment, particularly inventories and book-debts. The
Financial Controller of the company is examining the following alternative Working
Capital Policies:
(Rs. Crores) Working Capital Policy Investment in Estimated EBIT
7.91
Financial Management

Current Assets

Sales

Conservative Moderate

4.50 3.90

12.30 11.50

1.23 1.15

Aggressive 2.60 10.00 1.00 After evaluating the working capital policy, the
Financial Controller has advised the adoption of the moderate working capital
policy. The company is now examining the use of long-term and short-term
borrowings for financing its assets. The company will use Rs.2.50 crore of the
equity funds. The corporate tax rate is 35%. The company is considering the
following debt alternatives:
Financing Policy Short-term Debt Long-term Debt

Conservative Moderate Aggressive

0.54 1.00 1.50

1.12 0.66 0.16 16%

Interest rate- Average 12% You are required to calculate the following:

(a) Working Capital Investment for each Policy: (a) Net Working Capital position;
(b) Rate of Return; (c) Current ratio. (b) Financing for each policy; (a) Net
Working Capital; (b) Rate of Return of Shareholders equity; (c) Current ratio. 6.
The turnover of R Ltd. is Rs.60 lakhs of which 80% is on credit. Debtors are
allowed one month to clear off the dues. A factor is willing to advance 90% of the
bills raised on credit for a fee of 2% a month plus a commission of 4% on the
total amount of debts. R Ltd. as a result of this arrangement is likely to save
Rs.21,600 annually in management costs and avoid bad debts at 1% on the credit
sales. A scheduled bank has come forward to make an advance equal to 90% of the
debts at an interest rate of 18% p.a. However, its processing fee will be at 2% on
the debts. Would you accept factoring or the offer from the bank? 7. A Bank is
analyzing the receivables of Jackson Company in order to identify acceptable
collateral for a short-term loan. The company’s credit policy is 2/10 net 30. the
bank lends 80 per cent on accounts where customers are not currently overdue and
where the average payment period does not exceed 10 days past the net period. A
schedule of Jackson’s receivables has been prepared. How much will the bank lend
on a pledge of receivables, if the bank uses a 10 per cent allowance for cash
discount and returns?
Account Amount Days Outstanding Average Payment

7.92
Management of Working Capital

Rs.

In days

Period historically

74 91 107 108 114 116 8.

25,000 9,000 11,500 2,300 18,000 29,000

15 45 22 9 50 16

20 60 24 10 45 10

123 14,000 27 48 A Ltd. has a total sales of Rs.3.2 crores and its average
collection period is 90 days. The past experience indicates that bad debt losses
are 1.5% on Sales. The expenditure incurred by the firm in administering its
receivable collection efforts are Rs.5,00,000. A factor is prepared to buy the
firm’s receivables by charging 2% commission. The factor will pay advance on
receivables to the firm at an interest rate of 18% p.a. after withholding 10% as
reserve. Calculate the effective cost of factoring to the Firm. Explain briefly
some of the techniques of inventory control used in manufacturing organization.

9.

10. Ten items kept in inventory by the School of Management Studies at State
University are listed below. Which items should be classified as ‘A’ items, ‘B’
items and ‘C’ items? What percentage of items is in each class? What percentage of
total annual value is in each class?
Item Annual Usage Value per unit (Rs.) 1 200 40.00 2 100 360.00 3 2,000 0.20 4 400
20.00 5 6,000 0.04 6 1,200 0.80 7 120 100.00 8 2,000 0.70 9 1,000 1.00 10 80
400.00 Economic Enterprises require 90,000 units of a certain item annually. The
cost per unit is Rs.3, the cost per purchase order is Rs.300 and the Inventory
carrying cost Rs.6 per unit per year.

11

7.93
Financial Management

(i) (ii)

What is the Economic Order Quantity. What should the firm do if the supplier
offers discounts as below, viz.,
Discounts%

Order quantity

4,500 – 5,999 6,000 and above

2 3

12. The annual demand for an item of raw material is 4,000 units and the purchase
price is expected to be Rs.90 per unit. The incremental cost processing an order
is Rs.135 and the cost of storage is estimated to be Rs.12 per unit. What is the
optimal order quantity and total relevant cost of this order quantity? Suppose
that Rs.135 as estimated to be the incremental cost of processing an order is
incorrect and should have been Rs.80. All other estimates are correct. What is the
difference in cost on account of this error? Assume at the commencement of the
period that a supplier offers 4,000 units at a price of Rs.86. The materials will
be delivered immediately and placed in the stores. Assume that the incremental
cost of placing the order is zero and original estimate of Rs.135 for placing an
order for the economic batch is correct. Should the order be accepted? 13. (a) The
following details are available in respect of a firm: (i) (ii) (iii) Annual
requirement of inventory Cost per unit (other than carrying and ordering cost)
Carrying costs are likely to be 40,000 units Rs.16 15% per year Rs.480 per order

(iv) Cost of placing order Determine the economic ordering quantity. (b) The
experience of the firm being out of stock is summarized below: (1)
Stock out (No. of units)

No. of times

500 400 250 100 50 0

1 (1) 2 (2) 3 (3) 4 (4) 10 (10) 80 (80)

7.94
Management of Working Capital

Figures in brackets indicate percentage of time the firm has been out of stock.
(2) Stock out costs are Rs.40 per unit. (3) Carrying cost of inventory per unit is
Rs.20. Determine the optimal level of stock out inventory. (c) A firm has 5
different levels in its inventory. The relevant details are given. Suggest a
breakdown of the items into A, B and C classifications:
Item No. Avg. No. of units inventory Avg. Cost per unit (Rs.)

1 2 3 4 5

20,000 10,000 32,000 28,000 60,000

60 100 11 10 3.40

14. A firm is engaged in the manufacture of two products A and B. Product A uses
one unit of Component P and two units of Components Q. Products B uses two units
of Component P, one unit of Component Q and two units of Component R. Component R
which is assembled in the factory uses one unit of Component Q. Components P and Q
are purchased from the market. The firm has prepared the following forecast for
sales and inventory for the next year:
Products

A Sales Inventories: At the end of the year At the beginning of the year Units
Units 1,000 3,000 Units 8,000

B 15,000 2,000 5,000

The production of both the products and the assembling of the component R will be
spread out uniformly throughout the year. The firm at present orders its inventory
of components P and Q in quantities equivalent to 3 months consumption. The firm
has been advised that savings in the provisioning of components can arise by
changing over to the ordering system based on economic
7.95
Financial Management

ordering quantities. Components:


Particulars

The firm has compiled the following data relating to the two (Rs.)
P Q

Component usage per annum Price per unit Order placing costs per order Carrying
costs per annum Required:

30,000 2.00 15.00 20%

48,000 0.80 15.00 20%

(a) Prepare a budget of production and requirements of components for the next
year. (b) Find the economic order quantity. (c) Based on the economic order
quantity calculated in (b) above, calculate the savings arising from switching
over to the new ordering system both in terms of cost and reduction in working
capital. 15. Radiance Garments Ltd. manufactures readymade garments and sells them
on credit basis through a network of dealers. Its present sale is Rs.60 lakh per
annum with 20 days credit period. The company is contemplating an increase in the
credit period with a view to increasing sales. Present variable costs are 70% of
sales and the total fixed costs Rs.8 lakh per annum. The company expects pre-tax
return on investment @ 25%. Some other details are given as under:
Proposed Credit Policy Average Collection Period (days) Expected Annual Sales
(Rs.lakh)

I II III IV

30 40 50 60

65 70 74 75

Required: When credit policy should the company adopt? Present your answer in a
tabular form. Assume 360 days a year. Calculation should be made upto two digits
after decimal. 16. H. Ltd. has a present annual sales level of 10,000 units at
Rs.300 per unit. The variable cost is Rs.200 per unit and the fixed costs amount
to Rs.3,00,000 per annum. The
7.96
Management of Working Capital

present credit period allowed by the company is 1 month. The company is


considering a proposal to increase the credit periods to 2 months and 3 months and
has made the following estimates:
Existing Proposal

Credit Policy Increase in Sales % of Bad Debts

1 month 1%

2 months 15% 3%

3 months 30% 5%

There will be increase in fixed cost by Rs.50,000 on account of increase of sales


beyond 25% of present level. The company plans on a pretax return of 20% on
investment in receivables. You are required to calculate the most paying credit
policy for the company. 17. Star Limited manufacturers of Colour T.V. sets, are
considering the liberalization of existing credit terms to three of their large
Customers A,B and C. The credit period and likely quantity of TV sets that will be
lifted by the customers are as follows:
Quantity Lifted (No. of TV Sets) Credit Period (Days) A B C

0 30 60 90

1,000 1,000 1,000 1,000

1,000 1,500 2,000 2,500

− − 1,000 1,500

The selling price per TV set is Rs.9,000. The expected contribution is 20% of the
selling price. The cost of carrying debtors averages 20% per annum. You are
required: (a) Determine the credit period to be allowed to each customer. (Assume
360 day in a year for calculation purposes). (b) What other problems the company
might face in allowing the credit period as determined in (a) above?

7.97
Financial Management

18. The present credit terms of P Company are 1/10 net 30. Its annual sales are
Rs.80 lakhs, its average collection period is 20 days. Its variable costs and
average total costs to sales are 0.85 and 0.95 respectively and its cost of
capital is 10 per cent. The proportion of sales on which customers currently take
discount is 0.5. P Company is considering relaxing its discount terms to 2/10 net
30. Such relaxation is expected to increase sales by Rs.5 lakhs, reduce the
average collection period to 14 days and increase the proportion of discount to
sales to 0.8. What will be the effect of relaxing the discount policy on company’s
profit? Take year as 360 days. 19. The credit manager of XYZ Ltd. is reappraising
the company’s policy. The company sells its products on terms of net 30. Cost of
goods sold is 85% of sales and fixed costs are further 5% of sales. XYZ classifies
its customers on a scale of 1 to 4. During the past five years, the experience was
as under:
Classification Default as a percentage of sales Average collection period-indays
for non-defaulting accounts 45 42 40 80

1 2 3 4

0 2 10 20

The average rate of interest is 15%. What conclusions do you draw about the
Company’s Credit Policy? What other factors should be taken into account before
changing the present policy? Discuss. 20. Easy Limited specializes in the
manufacture of a computer component. The component is currently sold for Rs.1,000
and its variable cost is Rs.800. For the year ended 31-32006 the company sold on
an average 400 components per month. At present the company grants one month
credit to its customers. The company is thinking of extending the same to two
months on account of which the following is expected: Increase in Sales Increase
in Stock Increase in Creditors You are required: To advise the company on whether
or not to extend the credit terms if: (a) all customers avail the extended credit
period of two months; and 25% Rs.2,00,000 Rs.1,00,000

7.98
Management of Working Capital

(b) existing customers do not avail the credit terms but only the new customers
avail the same. Assume in this case the entire increase in sales is attributable
to the new customers. 21 Star Limited is manufacturer of various electronic
gadgets. The annual turnover for the year 2006 was Rs.730 lakhs. The company has a
wide network of sales outlets all over the country. The turnover is spread evenly
for each of the 50 weeks of the working year. All sales are for credit and sales
within the week are also spread evenly over each of the five working days. All
invoicing of credit sales is carried out at the Head Office in Bombay. Sales
documentation is sent by post daily from each location to the Head Office for the
past two years. Delays in preparing and dispatching invoices were noticed. As a
result, only some of the invoices were dispatched in the same week and the
remainder the following week. An analysis of the delay in invoicing (being the
interval between the date of sale and the date of despatch of the invoice
indicated the following pattern: No. of days of delay in invoicing % of weeks
sales 3 20 4 10 5 40 6 30

A further analysis indicated that the debtors take on an average 36 days of credit
before paying. This period is measured from the day of despatch of the invoice
rather than the date of sale. It is proposed to hire an agency for undertaking the
invoicing work at various locations. The agency has assured that the maximum delay
would be reduced to three days under the following pattern: No. of days of delay
in invoicing % of weeks sales 0 40 1 40 2 20

The agency has also offered additionally to monitor the collections which will
reduce the credit period to 30 days. Star Limited expects to save Rs.4,000 per
month in postage costs. All working funds are borrowed from a local bank at simple
interest rate of 20% p.a. The agency has quoted a fee of Rs.2,00,000 p.a. for the
invoicing work and Rs.2,50,000 p.a. for monitoring collections and is willing to
offer a discount of Rs.50,000 provided both the works are given. You are required
to advise Star Limited about the acceptance of agency’s proposal. Working should
form part of the answer.
7.99
Financial Management

22. Pollock Co. Pvt. Ltd., which is operating for the last 5 years, has approached
Sudershan Industries for grant of credit limit on account of goods bought from the
latter, annexing Balance Sheet and Income Statement for the last 2 years as below:
Pullock Co. Pvt. Ltd. – Balance Sheet (Rs. ‘000) Liabilities Current year Last
year Assets Current year Last year

Share Capital Equity (Rs.10) Share Premium Retained Earnings Total Equity First
Mortgage Second Mortgage Bonds Long-term Liabilities Account Payable Notes Payable
Secured Liabilities Total Liabilities Current

600 400 900 1,900 200 -300 500 300 600 100 1,000 3,400

Plant & Equipment 600 (Less Depreciation) 400 Land 700 1,700 Total Fixed Assets
300 Inventories 200 Account Receivable 300 Marketable Securities 800 Cash 60 Total
Current Assets 220 70 350 2,850

1,500 750 2,250 580 350 120 100 1,150

1,400 750 2,150 300 200 120 80 700

3,400

2,850

7.100
Management of Working Capital

Pollock Co. Pvt. Ltd. – Income Statement (Rs.’000) Particulars Current Year Last
Year

Sales Income from investments Opening inventory Total Manufacturing Costs Ending
Inventory General and Admn. Expenses Operating Income Interest Expenses Earnings
before Taxes Income-tax Net Income after Taxes Dividend declared and paid

5,980 20 300 4,200 (580) 3,920 2,080 950 1,130 60 1,070 480 590 6,000

5,780 20 400 3,200 (300) 3,300 2,500 750 1,750 62 1,688 674 1,014 250 5,800

Sudershan Industries has established the following broad guidelines for granting
credit limits to its customers: (i) (ii) Limit credit limit to 10% of net worth
and 20% of the net working capital. Not to give credit in excess of Rs.1,00,000 to
any single customer.

You are required to detail the steps required for establishing credit limits to
Pollock Co. Pvt. Ltd. In this case what you consider to be reasonable credit
limit. 23 The annual cash requirement of A Ltd. is Rs.10 lakhs. The company has
marketable securities in lot sizes of Rs.50,000, Rs.1,00,000, Rs.2,00,000,
Rs.2,50,000 and Rs.5,00,000. Cost of conversion of marketable securities per lot
is Rs.1,000. The company can earn 5% annual yield on its securities. You are
required to prepare a table indicating which lot size will have to be sold by the
company. Also show that the economic lot size can be obtained by the Baumol Model.
24. JPL has two dates when it receives its cash inflows, i.e., Feb. 15 and Aug.
15. On each of these dates, it expects to receive Rs.15 crore. Cash expenditure
are expected to be steady throughout the subsequent 6 month period. Presently, the
ROI in marketable

7.101
Financial Management

securities is 8% per annum, and the cost of transfer from securities to cash is
Rs.125 each time a transfer occurs. (i) (ii) What is the optimal transfer size
using the EOQ model? What is the average cash balance? What would be your answer
to part (i), if the ROI were 12% per annum and the transfer costs were Rs.75/-?
Why do they differ from those in part (i)?

25. Beta Limited has an annual turnover of Rs.84 crores and the same is spread
over evenly each of the 50 weeks of the working year. However, the pattern within
each week is that the daily rate of receipts on Mondays and Tuesdays is twice that
experienced on the other three days of the week. The cost of banking per day is
estimated at Rs.2,500. It is suggested that banking should be done daily or twice
a week. Tuesdays and Fridays as compared to the current practice of banking only
on Fridays. Beta Limited always operates on bank overdraft and the current rate of
interest is 15% per annum. This interest charge is applied by the bank on a simple
daily basis. Ignoring taxation, advise Beta Limited the best course of banking.
For your exercise, use 360 days a year for computational purposes. 26. The
following is the Balance Sheet of Amar Industries Ltd. as at 31st March, 2006:
(Rs. lakhs) Liabilities

Capital and Reserves 12% Debentures Creditors for purchases Creditors for expenses
Provision for bonus Provision for tax Proposed dividends
Assets

1,650 900 600 70 30 100 50 3,400

Fixed Assets at cost


Less: Depreciation

1,300 400 900 700 1,200

Sundry debtors Stocks and stores


7.102
Management of Working Capital

Loans and advances Cash and bank balances

500 100 3,400

The projected Profit and Loss Account for the first four months in 2006-2007 shows
the following:
(Rs.lakhs)

Particulars Sales Excise duty recoveries (a) Materials: Opening Stock


Add: Purchases Less: Closing Stock

April 800 80 880 1200 600 1200 600 180 80 (b) 860 20 (a)-(b)

May 800 80 880 1200 660 1260 600 180 84 864 16

June 900 90 990 1260 720 1320 660 200 88 948 42

July 900 90 990 1320 720 1320 720 200 92 1012 (22)

Net Expenses Excise duty Profi/(Loss)t

The following are relevant additional information: (i) (ii) 10% of sales are for
cash and the balance on 30 days’ credit. Creditors for purchases are paid in 30
days. (a) interest payable at the end of each quarter. (b) depreciation of Rs.10
lakhs per month; (c) provision for bonus to workmen of Rs.5 lakhs per month,
payable only in October, 2006. (d) one-half of rest of the expenses payable in the
month following.

(iii) Expenses include:

7.103
Financial Management

(iv) Rs.250 lakhs of debentures are redeemable by 30th June. (v) Provision for
taxation includes Rs.70 lakhs of surplus provision carried forward from earlier
years besides the balance for the year 2005-2006 payable before 30th June, 2006.
(vi) Annual General Meeting is to be held on 31st May, 2006. (vii) Over-draft is
permissible interest on which may be ignored. You are required to prepare the cash
budgets for the months of April to July, 2006 on a monthly basis. 27. The
following information is available in respect of ABC Ltd: (1) Materials are
purchased and received one month before being used and payment is made to
suppliers two months after receipt of materials. (2) Cash is received from
customers three months after finished goods are sold and delivered to them. (3) No
time lag applies to payment of wages and expenses. (4) The following figures apply
to recent and future months:
Month Materials received Sales Wages and Expenses

January Febuary March April May June July August

20,000 22,000 24,000 26,000 28,000 30,000 32,000 34,000

30,000 33,000 36,000 39,000 42,000 45,000 48,000 51,000

9,500 10,000 10,500 11,000 11,500 12,000 12,500 13,000

(5) Cash balance at the beginning of April is Rs.10,000. (6) all products are sold
immediately they have been made and that materials used and sums spent on wages
and expenses during any particular month relate strictly to the sales made during
that month. Prepare cash flow forecast month by month from April to July, profit
and loss forecast for four months (April-July) and a movement of funds statement
for the four-month period (April-July).

7.104
Management of Working Capital

28. Fixed overheads excluding bank interest amount to Rs.6,00,000 per annum spread
out evenly throughout the year. Sales forecast is as under:
Product July August Sept. Oct. Nov. 2005

L B

4,200 2,100

4,600 2,300

3,600 1,800

4,000 2,000

4,500 1,900

Production: 75% of each months’ sales will be produced in the month of sale and
25% in the previous months. Sales Pattern: L: -One-third of sales will be on cash
basis on which cash discount of 2% is allowed. -One third will be on documents
against payment basis. The documents will be discounted by the bank in the month
of sales itself. -Balance of one-third will be on documents against acceptance
basis. The payment under this scheme will be received in the third month. For e.g.
for sales made in September, payment will be received in November. B: 80% of the
sales will be against cash to be received in the month of sales and the balance
20% will be received next following month.

Direct Materials: 50% of the direct materials required for each month’s production
will be purchased in the previous month and the balance in the month of production
itself. The payment will be made in the month next following the purchase. Direct
Wages: 80% of the direct wages will be paid in the month of use of direct labour
for production and the balance in the next following month. Variable Overheads:
50% to be paid in the month of incurrence and the balance in the next following
month. Fixed Overheads: 40% will be paid in the month of incurrence and the other
40% in the next following month. The balance of 20% represents depreciation.

The bill discounting charges payable to the bank in the month in which the bills
are discounted amount to 50 paise per Rs.100 of bills discounted. A cash balance
of Rs.1,00,000 will be maintained on 1st July, 2006. Prepare a cash budget
monthwise for July, August and September, 2006.
7.105
Financial Management

29. A new manufacturing company is to be incorporated from January 1, 2003. Its


authorised capital will be Rs.2 crore divided into 20 lakh equity shares of Rs.10
each. It intends to raise capital by issuing equity shares of Rs.1 crore (fully
paid) on 1st January. Besides, a loan of Rs.13 lakh @12% per annum will be
obtained from a financial institution on 1st January and further borrowings will
be made at the same rate of interest on the first day of the month in which
borrowing is required. All borrowings will be repaid alongwith interest on the
expiry of one year. The company will make payment for the following assets in
January:
(Rs. lakhs)

Plant and Machinery Land and Building Furniture Motor Vehicles Stock of Raw
Materials The following further details are available: (i) Projected Sales
(January-June):
(Rs. lakhs)

20 40 10 10 10

(Rs. lakhs)

January February March (ii) Gross Profit will be 25% on sales.

30 35 35

April May June

40 40 45

(iii) The company will make credit sales only and these will be collected in the
second month following sales. (iv) Creditors will be paid in the first month
following credit purchases. There will be credit purchases only. (v) The company
will keep minimum stock of raw materials of Rs.10 lakhs. (vi) Depreciation will be
charged @10% per annum on cost on all fixed assets. (vii) Payment of preliminary
expenses of Rs.1 lakh will be made in January.

7.106
Management of Working Capital

(viii) Wages and salaries will be Rs.2 lakhs each month and will be paid on the
first day of the next month. (ix) Administrative expenses of Rs.1 lakh per month
will be paid in the month of their incurrence. Assume no minimum required cash
balance. You are required to prepare the monthly cash budget (January-June), the
projected Income Statement for the 6 month period and the projected Balance Sheet
as on 30th June, 2003. 30. Dyer Ltd. manufactures a variety of products using a
standardized process, which takes one month to complete. Each production batch is
started at the beginning of a month and is transferred to finished goods at the
beginning of the next month. The cost structure, based on current selling price
is: % Sale Price Variable Costs Raw Materials Other Variable Costs Total Variable
Cost – used for Stock Valuation Contribution 30 40 70 30 100

Activity levels are constant throughout the year and annual sales, all of which
are made on credit are Rs.24,00,000. Dyer Ltd. is now planning to increase sales
volume by 50% and unit sales price by 10%, such expansion would not alter the
fixed costs of Rs.50,000 per month, which includes monthly depreciation of plant
of Rs.10,000. Similarly raw material and other variable costs per unit will not
alter as a result of the price rise. In order to facilitate the envisaged
increases several changes would be required in the long run. The relevant changes
are:(i) (ii) The average credit period allowed to customers will increase to 70
days; Suppliers will continue to be paid on strictly monthly terms;

(iii) Raw material stocks held will continue to be sufficient for one month’s
production; (iv) Stocks of finished goods held will increase to one month’s
output;

7.107
Financial Management

(v) There will be no change in the production period and other variable costs will
continue to be paid for in the month of production; (vi) The current end-of-month
working capital position is:
(Rs.’000)

Raw Materials WIP Finished Goods Debtors Creditors Net Working Capital – Excluding
Cash

60 140 70 270 200 470 60 410

Compliance with the long-term changes required by the expansion will be spread
over several months. The relevant points concerning the transitional arrangements
are: (i) (ii) The cash balance anticipated at the end of the May is Rs.80,000.
Upto and including June all sales will be made on one month’s credit. From July
all sales will be on the transitional credit terms which will mean: 60% of sales
will take 2 months’ credit. 40% of sales will take 3 months’ credit. (iii) Sale
price increase will occur with effect from sales in the month of August. (iv)
Production will increase by 50% with effect from the month of July. Raw material
purchases made in June will reflect this. (v) Sales volume will increase by 50%
from sales made in October. Required: (a) Show the long-term increase in annual
profit and long-term working capital requirements as a result of the plans for
expansion and a price increase. (Costs of financing the extra working capital
requirements may be ignored). (b) Produce a monthly cash forecast for the period
from June to December, the first seven months of the transitional period. Prepare
also a working capital position at the end of December.

7.108
Management of Working Capital

(c) Using your findings for (a) and (b) above, make brief comments to the
management of Dyer Ltd. on the major factors concerning the financial aspects of
the expansion which should be brought to their attention. Assume that there are
360 days in a year and each month contains 30 days.

7.109

Вам также может понравиться